Unit 2 pt 2

Lakukan tugas rumah & ujian kamu dengan baik sekarang menggunakan Quizwiz!

Sydenham's chorea, RF clinical manifestation

-CNS manifestation -delayed sign, several months after initial infection -involuntary movements of face and limbs -muscle weakness -disturbances of speech and gait

Describe Premature Atrial Contraction (PAC)

Contraction originating from ectopic focus in atrium in location other than SA node Travels across atria by abnormal pathway, creating distorted P wave May be stopped, delayed, or conducted normally at the AV node

Coronary Revascularization

Percutaneous coronary intervention (PCI) Balloon angioplasty Stent

Heparin

The ICU nurse is caring for a patient with septic shock. Which IV infusion order for this patient does the nurse question?

Endotoxins

The clinical manifestations in the first phase of sepsis-induced distributive shock results from the body's reaction to which factor?

Which assessment data would the nurse expect to auscultate in the client diagnosed with mitral valve insufficiency? 2. A holosystolic murmur heard best at cardiac apex.

The murmur associated with mitral valve insufficiency is loud, high-pitched, rumbling, and holosystolic (occurring throughout systole) and is heard best at the cardiac apex.

ANS: B Administration of oxygen for any type of shock is appropriate to help reduce potential damage from tissue hypoxia. The other interventions should be completed after oxygen is administered.

The nurse is assessing a client at risk for shock. The client's systolic blood pressure is 20 mm Hg lower than baseline. Which intervention does the nurse perform first? a. Increase the IV fluid rate. b. Administer oxygen. c. Notify the health care provider. d. Place the client in high Fowler's position.

ANS: A During the hyperdynamic phase of septic shock, because of alterations in the clotting cascade, clients begin to form numerous small clots. Heparin is administered to limit clotting and prevent consumption of clotting factors. The other medications would not be prescribed during the hyperdynamic phase of septic shock.

The nurse is caring for a client in the hyperdynamic phase of septic shock. Which medication does the nurse expect to be prescribed? a. Heparin sodium b. Vitamin K c. Corticosteroids d. Hetastarch (Hespan)

A rapid, weak, & thready pulse

The nurse is caring for a post op patient who had major abdominal surgery. Which assessment finding is consistent with hypovolemic shock?

10 The formula to calculate the minute ventilation (VE) is: VE = VTf. Therefore, VE = (20) (0.5) = 10 L/min. Text Reference - p. 1626

The nurse is performing a weaning assessment on a patient receiving mechanical ventilation. What is the value of minute ventilation (VE) if the patient's spontaneous respiratory rate (f) is 20 breaths/minute and spontaneous tidal volume (VT) is 0.5 L? Record your answer using a whole number. ___________________L/min

What are some precipitating factors of Anaphylactic shock?

_Insect bites, vaccines, allergens

Distributive shock has three sub-categories_

_septic, anaphylactic, and neurogenic shock

• Septic shock has three major pathophysiologic effects: _

_vasodilation, maldistribution of blood flow, and myocardial depression.

The cornerstone of therapy for septic, hypovolemic, and anaphylactic shock is _

_volume expansion with the administration of the appropriate fluid.

Artifact is

a distortion of the baseline and waveforms seen on the ECG.

first system during shock to show signs of critical dysfunction

pulmonary

what else can cause neurogenic shock

spinal anesthesia

how does body respond to volume loss

sympathetic reaction

•Second-degree AV block

, type II (Mobitz II heart block), involves a P wave that is nonconducted without progressive antecedent PR lengthening. This almost always occurs when a block in one of the bundle branches is present.

The nurse is caring for a client diagnosed with a myocardial infarction who is experiencing chest pain. Which interventions should the nurse implement? Select all that apply.

- Administer an aspirin orally. - Apply oxygen via a nasal cannula. PAY ATTENTION THE ROUTES! - MORPHINE ONLY IV - NITROGLYCERIN ONLY SUBLINGUAL IN THIS SITUATION

Patients in septic shock require-

-large amounts of fluid replacement.

Etiology Infective endocarditis

-most commonly Staph. aureus or viridian's -Fungi and viruses -Rheumatic heart disease used to be leading cause

Physiological Integrity 9. A patient with dilated cardiomyopathy has new onset atrial fibrillation that has been unresponsive to drug therapy for several days. The priority teaching needed for this patient would include information about a. anticoagulant therapy. b. permanent pacemakers. c. electrical cardioversion. d. IV adenosine (Adenocard).

A Atrial fibrillation therapy that has persisted for more than 48 hours requires anticoagulant treatment for 3 weeks before attempting cardioversion. This is done to prevent embolization of clots from the atria. Cardioversion may be done after several weeks of anticoagulation therapy. Adenosine is not used to treat atrial fibrillation. Pacemakers are routinely used for patients with bradydysrhythmias. Information does not indicate that the patient has a slow heart rate. DIF: Cognitive Level: Apply (application) REF: 796 OBJ: Special Questions: Prioritization TOP: Nursing Process: Planning MSC:

tricuspid and pulmonic valve disease

...

CK-MB indicate MI not heart failure

A positive D-dimer indicate pulmonary embolism

Circulatory Shock

An acute failure of the circulatory system to supply the peripheral tissues & organs of the body with an adequate blood supply, resulting in cellular hypoxia

Pathophysiology of CAD

Atherosclerosis is the major cause

Second-Degree AV block Type 1 (Mobitz I, Wanckebach) cause

Drugs or CAD

The client is employed in a job that requires extensive standing. Which intervention should the nurse include when discussing how to prevent varicose veins? 4. Wear graduated compression hose.

Graduated compression hose help decrease edema and increase the circulation back to the heart; this helps prevent varicose veins.

Which action should the nurse include in the plan of care when caring for a patient admitted with acute decompensated heart failure (ADHF) who is receiving nesiritide (Natrecor)? a. Monitor blood pressure frequently.

Nesiritide is a potent arterial and venous dilator, and the major adverse effect is hypotension.

Implantable Cardioverter Defibrillators

Take care of problems with conduction & fibrillation

hemorrhage. Severe bleeding, or hemorrhage, is the major cause of shock (hypoperfusion).

The major cause of shock is:

Restlessness

The nurse is caring for an older adult patient at risk for shock. What's an early sign of shock in this patient?

ANS: A A decrease in urine output is a sensitive indicator of early shock. In severe shock, urine output is decreased (compared with fluid intake) or even absent. Alterations in temperature, irregular rhythms, and changes in bowel movements are not early signs of shock.

The nurse is planning discharge education for a client who had an exploratory laparotomy. Which nursing statement is appropriate when teaching the client to monitor for early signs of shock? a. "Monitor how much urine you void and report a decrease in the amount." b. "Take your temperature daily and report any below-normal body temperatures." c. "Assess your radial pulse every day and report an irregular rhythm." d. "Monitor your bowel movements and report ongoing constipation or diarrhea."

A stable patient with acute decompensated heart failure (ADHF) suddenly becomes dyspneic. Before positioning the patient on the bedside, what should the nurse assess first? D Blood pressure

The nurse should evaluate the blood pressure before dangling the patient on the bedside because the blood pressure can decrease as blood pools in the periphery and preload decreases.

A patient admitted with acute dyspnea is newly diagnosed with dilated cardiomyopathy. Which information will the nurse plan to teach the patient about managing this disorder? d. Notify the doctor about any symptoms of heart failure such as shortness of breath.

The patient should be instructed to notify the health care provider about any worsening of heart failure symptoms.

Systemic inflammatory response syndrome (SIRS) is always present in a patient with "sepsis". (T/F)

True

What causes sudden cardiac death?

V. tach V. fib

o

VT can be stable (patient has a pulse) or unstable (patient is pulseless).

The home health nurse is admitting a client diagnosed with a DVT. Which action by the client warrants immediate intervention by the nurse? 3. The client takes vitamin E over-the-counter medications.

Vitamin E can affect the action of warfarin. The nurse should explain to the client that these and other medications could potentiate the action of warfarin.

Renin-Angiotensin-Aldosterone Mechanism

Will be activated and as a result we will have more sodium and water retention

• Hypovolemic shock occurs when_

_there is a loss of intravascular fluid volume.

Cardiogenic shock -• Definitive measures include _

_thrombolytic therapy, angioplasty with stenting, emergency revascularization, and valve replacement.

Match the mechanism that can trigger SIRS with the injury/event to which relates (answers may be used more than once). 1. Mechanical tissue trauma 2. Global perfusion defect 3. Ischemic/necrotic tissue 4. Microbial invasion 5. Endotoxin release a. Burns b. Fungi c. Gram-negative bacteria d. MI e. Pancreatitis f. Post-cardiac resuscitation g. surgical procedure h. Viruses

a. Burns: 1. Mechanical tissue trauma b. Fungi: 4. Microbial invasion c. Gram-negative bacteria: 5. Endotoxin release d. MI: 3. Ischemic/necrotic tissue e. Pancreatitis: 3. Ischemic/necrotic tissue f. Post-cardiac resuscitation: 2. Global perfusion defect g. surgical procedure: 1. Mechanical tissue trauma h. Viruses: 4. Microbial invasion

Appropriate treatment modalities for the management of cardiogenic shock include *(select all that apply)*: a. dobutamine to increase myocardial contractility b. vasopressors to increase systemic vascular resistance. c. circulatory devices such as an IABP d. corticosteroids to stabilize the cell wall in the infarcted myocardium. e. trendelenburg positioning to facilitate venous return and increase preload

a. dobutamine to increase myocardial contractility c. circulatory devices such as an IABP

The hemodynamic changes the nurse expects to find after successful initiation of IABP therapy in a patient with cardiogenic shock include *(select all that apply)*: a. decreased SV b. decreased SVR c. decreased PAWP d. increased diastolic BP e. a decreased myocardial oxygen consumption

b. decreased SVR c. decreased PAWP d. increased diastolic BP e. a decreased myocardial oxygen consumption

The impulse started by the SA node or an ectopic focus must be

conducted to the entire heart.

accurate interpretation of an ECG depends on the

correct placement of the leads on the patient.

absolute and relative refractory periods

correlated with the cardiac muscle's action potential

For a patient in late irreversible shock with massive thermal burns, the nurse would expect the patient's laboratory results to reveal a. respiratory alkalosis b. decreased potassium c. increased blood glucose d. increased ammonia levels

d. increased ammonia levels In late irreversible shock, progressive cellular destruction causes changes in lab findings that indicate organ damage. Increasing ammonia levels indicate impaired liver function. metabolic acidosis is usually in severe as cells continue anaerobic metabolism, and the respiratory alkalosis that may occur in the progressive stage has failed to compensate for the acidosis. Potassium levels increase and glucose levels decrease.

The health care provider orders the following interventions for a 67-kg patient who has septic shock with a BP of 70/42 mm Hg and oxygen saturation of 90% on room air. In which order will the nurse implement the actions? (Put a comma and a space between each answer choice [A, B, C, D, E].) a. Obtain blood and urine cultures. b. Give vancomycin (Vancocin) 1 g IV. c. Start norepinephrine (Levophed) 0.5 mcg/min. d. Infuse normal saline 2000 mL over 30 minutes. e. Titrate oxygen administration to keep O2 saturation >95%.

e d c a b

simpler process and less time consuming (for reg rhythm)

every 3 seconds a marker appears on the ECG paper . Count the number of R-R intervals in 6 seconds and multiply that number by 10. (An R wave is the first upward [or positive] wave of the QRS complex.) This is the estimated number of beats per minute

The relative refractory period occurs slightly later in the cycle, and

excitability is more likely.

The absolute refractory phase or period occurs when

excitability is zero and the heart cannot be stimulated.

The client had open-heart surgery to replace the mitral valve. Which intervention should the intensive care unit nurse implement? 1. Restrict the client's fluids as ordered.

fluid intake may be restricted to reduce the cardiac workload and pressures within the heart and pulmonary circuit.

vital signs, cerebral and abdominal perfusion pressures, capillary refill, skin temp and urine output determine

fluid responsiveness

increase O2 supply by doing what 3 things

giving fluids, giving blood (more hemoglobin) and giving O2

electrophysiologic study (EPS)

identifies different mechanisms of tachydysrhythmias, heart blocks, bradydysrhythmias, and causes of syncope.

Treatments for PSVT

includes vagal stimulation and IV adenosine.

Increased angiotensin 2 => Venous and arterial vasoconstriction => ______ and ______.

increase venous return to heart; increase in BP

Premature atrial contraction (PAC)

is a contraction originating from an ectopic focus in the atrium in a location other than the sinus node. In healthy persons, isolated PACs are not significant. In persons with heart disease, frequent PACs may warn of or initiate more serious dysrhythmias.

Paroxysmal supraventricular tachycardia (PSVT)

is a dysrhythmia originating in an ectopic focus anywhere above the bifurcation of the bundle of His.

Cardiac resynchronization therapy (CRT)

is a pacing technique that resynchronizes the cardiac cycle by pacing both ventricles, thus promoting improvement in ventricular function. Devices often combine CRT with an ICD for maximum therapy.

Atrial fibrillation

is characterized by a total disorganization of atrial electrical activity caused by multiple ectopic foci resulting in loss of effective atrial contraction.

Radiofrequency catheter ablation therapy

is used to "burn" or ablate accessory pathways or ectopic sites in the atria, AV node, and ventricles.Care of the patient following ablation therapy is similar to that of a patient undergoing cardiac catheterization.

Type I AV block

is usually a result of myocardial ischemia or infarction. It is almost always transient and is usually well tolerated. However, it may be a warning signal of a more serious AV conduction disturbance.

The signal-averaged ECG identifies

late potentials strongly suggesting that the patient may be at risk for developing serious ventricular dysrhythmias.

secondary firing mechanisms...

may start from the AV node at a rate of 40 to 60 times per minute or the His-Purkinje system at a rate of 20 to 40 times per minute. (these fire if their firing rate is higher than the SA node) or if malfunction of heart secondary pacemakers can start is when they fire more rapidly than the normal pacemaker of the SA node.

Wireless ECG Monitoring can...

monitoring systems continuously monitor and interpret the findings, sending an alert when patient rhythm and/or measurements fall outside of set parameters. • Early detection of abnormal heart rhythms allows you time to assess the patient for signs of hemodynamic instability (e.g., chest pain, hypotension, palpitations, dyspnea) and determine the need to intervene (e.g., call the rapid response team). • These systems can automatically save the pre-event portion of the ECG while continuing to record the post-event portion and send all of the information to the health care provider. • Computerized monitoring systems are not fail-proof. Frequently assess all monitored patients for any signs of hemodynamic instability.

A 72-year-old man with a history of aortic stenosis is admitted to the emergency department. He reports severe left-sided chest pressure radiating to the jaw. Which medication, if ordered by the health care provider, should the nurse question? C. Nitroglycerin

nitroglycerin should be used cautiously or avoided in patients with aortic stenosis as a significant reduction in blood pressure may occur. Chest pain can worsen because of a drop in blood pressure.

Type II AV block

often progresses to third-degree AV block and is associated with a poor prognosis.

A modified chest lead (MCL1) is used when

only three leads are available for monitoring

•In third-degree AV block

or complete heart block, no impulses from the atria are conducted to the ventricles.It almost always results in reduced CO with subsequent ischemia, HF, and shock.

Exercise treadmill testing evaluates the

patient's heart rhythm during exercise. If exercise-induced dysrhythmias or ECG changes occur, they are analyzed and drug therapy evaluated.

Event monitors has improved the evaluation of dysrhythmias in outpatients. Event monitors are

recorders that the patient activates only when he or she experiences symptoms.

Junctional dysrhythmias

refer to dysrhythmias that originate in the area of the AV node, in which the AV node becomes the pacemaker of the heart. Junctional dysrhythmias include junctional premature beats, junctional escape rhythm, accelerated junctional rhythm, and junctional tachycardia. These dysrhythmias are treated according to the type, patient's tolerance of the rhythm, and patient's clinical condition.

•Sudden cardiac death (SCD)

refers to death from a cardiac cause. The majority of SCDs result from ventricular dysrhythmias, specifically ventricular tachycardia or fibrillation.

when fluid volume moves out of vascular space and moves into extravascular space/third space

relative hypovolemia

Increased ADH => ______ => Increased blood volume.

renal water reabsorption

characterized by persistent vasodilation despite adequate fluid resuscitation

septic shock

coagulation increases and fibrinolysis decreases

septic shock

myocardial O2 metabolism and coronary artery perfusion is not primarily altered in

septic shock

respiratory failure is common with this shock as the patient initially hyperventilates then develops respiratory alkalosis and once they can no longer compensate respiratory acidosis develops

septic shock

The cardiac impulse leaves the SA nodes and...

spreads over the atrial myocardium via interatrial and internodal pathways, causing atrial contraction.

What is the priority assessment by the nurse caring for a patient receiving IV nesiritide (Natrecor) to treat heart failure? C Blood pressure

the priority assessment would be monitoring for hypotension, the main adverse effect of nesiritide.

A-fib occurrence

usually occurs in the patient with underlying heart disease, such as coronary artery disease (CAD), rheumatic heart disease, and cardiomyopathy.

Causes of Left-Sided Heart Failure

~ Acute myocardial infarction ~ Hypertension ~ Cardiomyopathy ~ Stenosis & regurgitation of aortic valve & mitral valve

Complications of Cardiogenic Shock

~ Acute respiratory distress syndrome ~ Acute renal failure ~ Gastrointestinal complications ~ Disseminated intravascular coagulation (DIC) - large number of blood vessels that there will be activation of coagulation, thrombosis formation, very dangerous, can be fatal ~ Multiple organ dysfunction syndrome - can affect the kidneys, liver, & brain

Causes of Right-Sided Heart Failure

~ Conditions that restrict blood flow into the lungs ~ Stenosis or regurgitation of the tricuspid or pulmonic valves ~ Right ventricular infarction ~ Cardiomyopathy ~ Persistent lift-sided failure ~ Acute or chronic pulmonary disease (cor pulmonale)

When teaching the patient about the long-term consequences of rheumatic fever, the nurse should discuss the possibility of:

A. valvular heart disease

8. Which action is a priority for the nurse to take when the low pressure alarm sounds for a patient who has an arterial line in the left radial artery? a. Fast flush the arterial line. b. Check the left hand for pallor. c. Assess for cardiac dysrhythmias. d. Rezero the monitoring equipment.

ANS: C The low pressure alarm indicates a drop in the patient's blood pressure, which may be caused by cardiac dysrhythmias. There is no indication to rezero the equipment. Pallor of the left hand would be caused by occlusion of the radial artery by the arterial catheter, not by low pressure. There is no indication of a need for flushing the line.

Human serum albumin (5%), plasma protein fraction (5% albumin in 500 mL NSS) are used to treat which types of shock?

All types except cardiogenic and neurogenic shock

Blood/Blood products are use for what type of shock?

All types of shock

Physiological Integrity 16. A patient has ST segment changes that support an acute inferior wall myocardial infarction. Which lead would be best for monitoring the patient? a. I b. II c. V2 d. V6

B Leads II, III, and AVF reflect the inferior area of the heart and the ST segment changes. Lead II will best capture any electrocardiographic (ECG) changes that indicate further damage to the myocardium. The other leads do not reflect the inferior part of the myocardial wall and will not provide data about further ischemic changes in that area. DIF: Cognitive Level: Analyze (analysis) REF: 806 TOP: Nursing Process: Implementation MSC:

Safe and Effective Care Environment 13. Which action should the nurse perform when preparing a patient with supraventricular tachycardia for cardioversion who is alert and has a blood pressure of 110/66 mm Hg? a. Turn the synchronizer switch to the "off" position. b. Give a sedative before cardioversion is implemented. c. Set the defibrillator/cardioverter energy to 360 joules. d. Provide assisted ventilations with a bag-valve-mask device.

B When a patient has a nonemergency cardioversion, sedation is used just before the procedure. The synchronizer switch is turned "on" for cardioversion. The initial level of joules for cardioversion is low (e.g., 50). Assisted ventilations are not indicated for this patient. DIF: Cognitive Level: Apply (application) REF: 802 TOP: Nursing Process: Implementation MSC:

Mitral valve prolapse

-abnormality of mitral valve leaflet and papillary muscles or cordae -leaflets proverbs back into left atrium during systole

Mitral valve regurgitation, pulmonary effects

-acute MR, increase in pressure affects pulmonary bed -pulmonary edema -if untreated, cariogenic shock

Rheumatic Fever etiology

-delayed result 2-3 weeks of group A strep pharyngitis -abnormal immunologic response to stream antigens -affects heart, skin, joints, CNS

Dilated cardiomyopathy, patho

-diffuse inflammation -rapid degeneration of myocardial fibers -ventricular dilation -impaired systolic function -atrial enlargement -stasis of blood in left ventricle -cardiomegaly -contractile dysfintion -walls of ventricles do not hypertrophy

Myocarditis clinical manifestations

-fever, fatigue, malaise, myalgias, pharyngitis, dyspnea, lymphadenopathy, nausea, vomitting= systemic viral illness -cardiac signs= 7-10 days after viral infection, pleuritic chest pain, pericardial rub and effusion -late signs= HF, S3 heart sounds, crackles, JVD, syncope, peripheral edema, angina

Cardiomyopathy (CMP)

-group of diseases that a affect myocardial structure or function -classified as primary or secondary

Acute infective endocarditis

-healthy valves, rapid progressive illness

Drug therapy bacterial endocarditis

-identify organism -long term treatment necessary to kill format bacteria in valvular vegetations -takes weeks, relapses common -IV antibiotics during acute phase

Primary Cardiomyopathy

-idiopathic etiology -heart muscle is only part of heart involved -other structure unaffected

Pericarditis

-inflammation of pericardial sac

Mono arthritis or polyarthitis, RF clinical manifestation

-most common RF finding -inflammatory process affecting synovial membrane of joints -swelling, heat, redness, tenderness, limited mobility -affects larger joins: knee, ankle, elbow, wrist

Dilated cardiomyopathy, etiology

-most common type of CMP -follows infections myocarditis -autoimmune process -alcoholism -genetic

Pericarditis Etiology

-most common, idiopathic -viral causes -coxackie B virus -MI

Aortic valve stenosis, patho

-obstruction of flow from left ventricle to aorta during systole -left ventricular hypertrophy, increased myocardial O2 consumption -decreased CO, tissue perfusion, -causes pulmonary hypertension, HF

diagnostic criteria infective endocarditis

-postive blood cultures -new or changed heart murmur -intracardiac mass or vegetation on ECHO

vegetation, infective endocarditis

-primary lesions -consist of fibrin, leukocytes, platelets, and microbes that stick to valve surface or endocardium -loss of parts of left-sided vegetations results in emboli -right-sided causes PE

Therapy fungal and prosthetic valvue endocarditis

-respond poorly to just antibiotics -valve replacement -6 weeks or more antibiotics

Pericardial friction rub, Pericarditis

-scratching, grating, high-pitched sound -friction btw roughened pericardial and epicardial surfaces -stethoscope placed at lower left sternal border of chest with pt leaning forward -differentiate btw pleural friction rub, have pt hold breath

What is the primary precipitating factor for neurogenic shock?

-trauma to spinal cord.

Mitral valve prolapse, etiology

-unknown -increased familial incidence -genetic autosomal dominant

Myocarditis etiology

-viruses, bacteria, fungi, radiation therapy, chemical factors -Coxackie A and B most common etiological agents -autoimmune disorders -idiopathic -associated with acute pericarditis

Pericardium anatomy

-visceral pericardium=inner serous membrane -parietal pericardium= outer fibrous layer -pericardial space= cavity in between 2 layers, contains 10-15 mL serous fluid

Myocarditis patho

-when myocardium is infection, causative agent invades myocytes -cellular damage and necrosis -immune response, cytokines ROS are released -progression of infection triggers autoimmune response= further destruction of myocytes

Normal QRS interval

0.04-0.10 The book says <0.12

Normal P wave

0.06-0.12

What is the normal ST segment?

0.12

Normal PR interval

0.12-0.20

What is a normal T wave?

0.16

What is the normal QT interval?

0.34-0.43

The client is one (1) day postoperative coronary artery bypass surgery. The client complains of chest pain. Which intervention should the nurse implement first?

Assess the client's chest dressing and vital signs. - if the chest pain is expected postop or if it is a complication

Which intervention should the nurse implement when administering a loop diuretic to a client diagnosed with coronary artery disease?

Assess the client's serum potassium level. - assess apical pulse not radial pulse

The client had an abdominal aortic aneurysm repair two (2) days ago. Which intervention should the nurse implement first? 1. Assess the client's bowel sounds.

Assessment is the first part of the nursing process and is the first intervention the nurse should implement.

Causes for First-Degree Block

Associated with disease and certain drugs

Junctional dysrthymia causes

Associated with disease, certain drugs

o

Atrial fibrillation often results in a decrease in CO and poses an increased risk of cardiovascular accident (CVA) because of clot formation, necessitating anticoagulation therapy.

impaled object in the abdomen.

All of the following are indications for the use of PASG EXCEPT:

A 19-year-old patient with massive trauma and possible spinal cord injury is admitted to the emergency department (ED). Which assessment finding by the nurse will help confirm a diagnosis of neurogenic shock? a. Inspiratory crackles. b. Cool, clammy extremities. c. Apical heart rate 45 beats/min. d. Temperature 101.2° F (38.4° C).

ANS: C Neurogenic shock is characterized by hypotension and bradycardia. The other findings would be more consistent with other types of shock

The nurse is assessing the client diagnosed with congestive heart failure. Which signs/symptoms would indicate that the medical treatment has been effective?

An elevated B-type natriuretic peptide (BNP). specific test, higher the number worse the condition

Why does dextran have limited use?

Because of side effects including reducing platelet adhesion, diluting clotting factors.

Which action by the nurse will determine if the therapies ordered for a patient with chronic constrictive pericarditis are effective? c. Assess for the presence of jugular venous distention (JVD).

Because the most common finding on physical examination for a patient with chronic constrictive pericarditis is jugular venous distention, a decrease in JVD indicates improvement.

Beyond the first year after a heart transplant, the nurse knows that what is a major cause of death? D Cardiac vasculopathy

Beyond the first year after a heart transplant, malignancy (especially lymphoma) and cardiac vasculopathy (accelerated CAD) are the major causes of death.

Beyond the first year after a heart transplant, the nurse knows that what is a major cause of death? d. Cardiac vasculopathy

Beyond the first year after a heart transplant, malignancy (especially lymphoma) and cardiac vasculopathy (accelerated CAD) are the major causes of death.

decreased blood pressure.

It is important to recognize and treat shock early. A late sign of shock is:

When teaching the patient with newly diagnosed heart failure about a 2000-mg sodium diet, the nurse explains that foods to be restricted include d. milk, yogurt, and other milk products.

Milk and yogurt naturally contain a significant amount of sodium, and intake of these should be limited for patients on a diet that limits sodium to 2000 mg daily.

At a clinic visit, the nurse provides dietary teaching for a 56-year-old woman who was recently hospitalized with an exacerbation of chronic heart failure. The nurse determines that teaching is successful if the patient makes which statement? A "I will limit the amount of milk and cheese in my diet."

Milk products should be limited to 2 cups per day for a 2500-mg sodium-restricted diet.

PAC Treatment

Monitor for more serious dysrhythmias Withhold sources of stimulation β-adrenergic blockers

Nursing Implications for use of NSS & LR

Monitor pt closely for circulatory overload. Do not use LR in pts w/liver failure. LR may be used if hypercholoremic acidosis develops from use of NSS in fluid resuscitation.

The client diagnosed with congestive heart failure is complaining of leg cramps at night. Which nursing interventions should be implemented?

Monitor the client's potassium level and assess the client's intake of bananas and orange juice. - most probable cause of the pain is hypokalemia - peripheral edema may cause calf tightness not cramping

Atrial flutter treatment

Radiofrequency catheter ablation is the treatment of choice.

The wife of a client with arterial occlusive disease tells the nurse, "My husband says he is having rest pain. What does that mean?" Which statement by the nurse would be most appropriate? 2. "His legs are deprived of oxygen during periods of inactivity."

Rest pain indicates a worsening of the arterial occlusive disease; the muscles of the legs are not getting enough oxygen when the client is resting to prevent muscle ischemia.

Clinical Manifestations MI

Result of sustained ischemia (>20 minutes), causing irreversible myocardial cell death (necrosis), Necrosis of entire thickness of myocardium takes 4 to 6 hours.

The client is being evaluated for valvular heart disease. Which information would be most significant? 4. The client has a history of rheumatic heart disease.

Rheumatic heart disease is the most common cause of valvular heart disease.

normal pacemaker of the heart is the...

SA node at 60-100 per minute. (HR)

Stimulation of the sympathetic nerves increases

SA node firing, AV node impulse conduction, and cardiac contractility.

The heart has specialized cells in the

SA node, atria, AV node, and bundle of His and Purkinje fibers (His-Purkinje system), which can fire (discharge) spontaneously.

Sympathetic tone

State of partial blood vessel constriction

The nurse is discussing angina with a client who is diagnosed with coronary artery disease. Which action should the client take first when experiencing angina?

Stop the activity immediately and rest. - that is first action , then nitroglycerine

Causes for PACs (8)

Stress Fatigue Caffeine Tobacco Alcohol Hypoxia Electrolyte imbalance Disease states

The health-care provider has ordered an angiotensin-converting enzyme (ACE) inhibitor for the client diagnosed with congestive heart failure. Which discharge instructions should the nurse include?

Teach the client how to prevent orthostatic hypotension. - if cough stop the medication - may cause hyperkalemia - take 1 hour before meals or 2 hours after meals

The charge nurse is making assignments for clients on a cardiac unit. Which client should the charge nurse assign to a new graduate nurse?

The 75-year-old client scheduled for a cardiac catheterization. - not a client diagnosed with MI - not a client with unstable angina - not a client who complaining of chest pain

Which statement by the client diagnosed with coronary artery disease indicates that the client understands the discharge teaching concerning diet? 2. "I should bake or grill any meats I eat."

The American Heart Association recommendS a low-fat, low-cholesterol diet for a client with coronary artery disease. The client should avoid any fried foods, especially meats, and bake, boil, or grill any meat.

3 Certification as an advance care nurse practitioner (ACNP) is available through the American Association of Critical Care Nurses (AACN). The National League of Nurses (NLN), the American Association of Nurse Practitioners (AANP), and the National Council of State Boards of Nursing (NCSBN) are all nursing organizations but they do not offer certification to the advance practice registered nurse (APRN) as an ACNP. Text Reference - p. 1600

The advance practice registered nurse (APRN) is seeking certification as an advance care nurse practitioner (ACNP). Which professional organization can grant this certification to the APRN? 1 National League of Nurses (NLN) 2 American Association of Nurse Practitioners (AANP) 3 American Association of Critical Care Nurses (AACN) 4 National Council of State Boards of Nursing (NCSBN

The nurse and an unlicensed nursing assistant are bathing a bedfast client. Which action by the assistant warrants immediate intervention? 2. The assistant begins to massage and rub lotion into the client's calf.

The assistant could dislodge a blood clot in the leg when massaging the calf. The assistant can apply lotion gently, being sure not to massage the leg.

The nurse on the telemetry unit has just received the a.m. shift report. Which client should the nurse assess first?

The client diagnosed with myocardial infarction who has an audible S3 heart sounds. - S3 is ventricular failure it is an emergency

Which client problem would be priority in a client diagnosed with arterial occlusive disease who is admitted to the hospital with a foot ulcer? 1. Impaired skin integrity.

The client has a foot ulcer, therefore the protective lining of the body—the skin— has been impaired.

The client with a mechanical valve replacement asks the nurse, "Why do I have to take antibiotics before getting my teeth cleaned?" Which response by the nurse is most appropriate? 1. "You are at risk of developing an infection in your heart."

The client is at risk for developing endocarditis and should take prophylactic antibiotics before any invasive procedure.

The client is exhibiting sinus bradycardia, is complaining of syncope and weakness, and has a BP of 98/60. Which collaborative treatment should the nurse anticipate being implemented? 3. Prepare for an insertion of a pacemaker.

The client is symptomatic and will require a pacemaker.

The client tells the nurse that his cholesterol level is 240 mg/dL. Which action should the nurse implement? 2. Explain that the client needs to lower the cholesterol level.

The client needs to be taught ways to lower the cholesterol level.

The nurse is assessing the client diagnosed with long-term arterial occlusive disease. Which assessment data support the diagnosis? 1. Hairless skin on the legs.

The decreased oxygen over time causes the loss of hair on top of feet and ascends both legs.

The patient with chronic heart failure is being discharged from the hospital. What information should the nurse emphasize in the patient's discharge teaching to prevent progression of the disease to ADHF? A Take medications as prescribed.

The goal for the patient with chronic HF is to avoid exacerbations and hospitalization.

4 Pulse oximetry is a noninvasive and continuous method of determining the oxygen saturation of SpO2. Monitoring SpO2 may reduce the frequency of arterial blood gas (ABG) sampling. SpO2 is normally 95% to 100%. A value less than that may indicate hypoperfusion. Text Reference - p. 1609

The health care provider requests constant hemoglobin (SpO2) monitoring for a patient. What is the normal range of SpO2, which indicates that the saturation pressure of oxygen in this patient is adequate? 1 80-85% 2 85-90% 3 90-95% 4 95-100%

2 Normal central venous oxygen saturation or ScvO2/SvO2 is 60% to 80%. It denotes normal oxygen supply and metabolic demand. Any reading out of the normal range can cause danger to the patient. More than 80% denotes increased oxygen supply and decreased oxygen demand. Less than 60% denotes decreased oxygen supply and increased demand. Text Reference - p. 1609

The nurse is monitoring a patient in ICU. Which ScvO2/SvO2reading is a cause of concern in the patient? 1. 68% 2. 54% 3. 72% 4. 78%

The nurse is administering a beta blocker to the client diagnosed with essential hypertension. Which intervention should the nurse implement? 2. Question administering the medication if the blood pressure is 90/60 mmHg.

The nurse should question administering the beta blocker if the B/P is low because this medication will cause the blood pressure to drop even lower, leading to hypotension.

The 80-year-old client is being discharged home after having surgery to debride a chronic venous ulcer on the right ankle. Which referral would be most appropriate for the client? Social worker.

The social worker would assess the client to determine if home health care services or financial interventions were appropriate for the client. The client is elderly, immobility is a concern, and wound care must be a concern when the client is discharged home.

The client comes to the clinic complaining of muscle cramping and pain in both legs when walking for short periods of time. Which medical term would the nurse document in the client's record? 2. Intermittent claudication.

This is the classic symptom of arterial occlusive disease.

The client asks the nurse, "My doctor just told me that atherosclerosis is why my legs hurt when I walk. What does that mean?" Which response by the nurse would be the best response? 4. "You have a hardening of your arteries that decreases the oxygen to your legs."

This response explains in plain terms why the client's legs hurt from atherosclerosis.

Is V. tach considered life threatening?

Yes because of decreased CO and the possibility of deterioration to v. fib Hypotension Pulmonary edema Decreased cerebral blood flow Cardiopulmonary arrest

• Systemic inflammatory response syndrome (SIRS) is_

_a systemic inflammatory response to a variety of insults, including infection (referred to as "sepsis"), ischemia, infarction, and injury.

• Anaphylactic shock is _

_an acute and life-threatening hypersensitivity (allergic) reaction to a sensitizing substance (e.g., drug, chemical, vaccine, food, insect venom).

• Clinical manifestations of anaphylaxis can include _

_anxiety, confusion, dizziness, chest pain, incontinence, swelling of the lips and tongue, wheezing, stridor, flushing, pruritus, urticaria, and angioedema.

• The progressive stage of shock begins _

_as compensatory mechanisms fail and aggressive interventions are necessary to prevent the development of multiple-organ dysfunction system (MODS).

• Neurogenic shock is a hemodynamic phenomenon that _

_can occur within 30 minutes of a spinal cord injury at the fifth thoracic (T5) vertebra or above and last up to 6 weeks, or in response to spinal anesthesia.

Collaborative Care: Septic shock_• Intravenous corticosteroids are only recommended for patients who _

_cannot maintain an adequate blood pressure (BP) with vasopressor therapy, despite fluid resuscitation.

• The four main categories of shock are _

_cardiogenic, hypovolemic, distributive and obstructive.

• In the irreversible stage,

_decreased perfusion from peripheral vasoconstriction and decreased CO exacerbate anaerobic metabolism.

• Shock is a syndrome characterized by_

_decreased tissue perfusion and impaired cellular metabolism resulting in an imbalance between the supply of and demand for oxygen and nutrients.

Collaborative care: Neurogenic shock_• The treatment of neurogenic shock is _

_dependent on the cause. In spinal cord injury, general measures to promote spinal stability are initially used.

Collaborative care: Obstructive shock_• The primary strategy in treating obstructive shock is _

_early recognition and treatment to relieve or manage the obstruction.

• Cardiogenic shock occurs when _

_either systolic or diastolic dysfunction of the pumping action of the heart results in reduced cardiac output (CO).

Collaborative care: Anaphylactic shock_• Maintaining the airway is critical; _

_endotracheal intubation or cricothyroidotomy may be necessary.

• Clinical manifestations depend on the _

_extent of injury or insult, age, and general state of health and may include anxiety; an increase in heart rate, CO, and respiratory rate and depth; and a decrease in stroke volume, pulmonary artery wedge pressure (PAWP), and urine output.

ABSOLUTE hypovolemia results when_

_fluid is lost through hemorrhage, GI loss (vomiting/diarrhea), fistula drainage, diabetes insipidus, or diuresis.

RELATIVE hypovolemia results when_

_fluid volume moves out of the vascular space into the extravascular space. (Third spacing)

• The prognosis for the patient with MODS is poor; _

_the most important goal is to prevent the progression of SIRS to MODS.

One or more ECG leads can be used to continuously....

monitor a patient.

patient with neurogenic shock is at risk for

hypothermia

Telemetry monitoring

involves the observation of a patient's HR and rhythm to help rapidly diagnose dysrhythmias, ischemia, or infarction.

Ventricular tachycardia (VT)

is a run of three or more PVCs. It occurs when an ectopic focus or foci fire repetitively and the ventricle takes control as the pacemaker.

•Ventricular fibrillation (VF)

is characterized on ECG by irregular undulations of varying shapes and amplitude. VF results in an unresponsive, pulseless, and apneic state. If not rapidly treated, the patient will die. Treatment consists of immediate initiation of CPR and advanced cardiac life support (ACLS) measures with the use of defibrillation and definitive drug therapy.

A temporary pacemaker

is one that has the power source outside the body. There are three types of temporary pacemakers: transvenous, epicardial, and transcutaneous pacemakers.

You will see artifact on the monitor when

leads and electrodes are not secure, or when there is muscle activity (e.g., shivering) or electrical interference.

The client is exhibiting ventricular tachycardia. Which intervention should the nurse implement first?

left off at 81 page at question 55

release of TNF and IL-1 are thought to have a role in sepsis-induced

myocardial dysfunction

pulsus paradoxus (decrease in SBP during inspiration) and JVD

obstructive shock

Normal sinus rhythm

originates in the SA node and follows the normal conduction pattern of the cardiac cycle.

LAB- Hypovolemic Shock

pH (arterial)- 7.35-7.45- Decreased: insufficient tissue oxygenation causing anaerobic metabolism & acidosis. Pao2- 80-100 mm Hg- Decreased: anaerobic metabolism Paco2- 35-45 mm Hg- Increased; anaerobic metabolism Lactic Acid (arterial) 3-7 mg/dL (0.3-0.8 mmol/L) Increased: anaerobic metabolism with build up of metabolites Hematocrit- Females- 37%-47%- Males- 12-52%- Increased: fluid shift; dehydration; Decreased: hemorrhage Hemoglobin- Female- 12-16 g/dL- Male- 14-18 g/dL- Increased fluid shift; dehydration; Decreased: hemorrhage Potassium- 3.5-5.0 mEq/L- Increased: dehydration; acidosis

autonomic nervous system

plays an important role in the rate of impulse formation, the speed of conduction, and the strength of cardiac contraction.

inside of the cell, when at rest, is the .......state.

polarized state, is negative compared with the outside.

Four properties of cardiac cells enable the conduction system to

start an electrical impulse, send it through the cardiac tissue, and stimulate muscle contraction

The 12-lead ECG may show changes suggesting....

structural changes, conduction disturbances, damage (e.g., ischemia, infarction), electrolyte imbalance, or drug toxicity. Also useful in assessing dysrhythmias.

A synchronized shock that occurs on the R wave of the QRS complex of the monitor must be in ____________ mode

sync

two pulmonary symptoms of cardiogenic shock

tachypnea and crackles

The level of excitability is determined by

the length of time after depolarization that the tissues can be restimulated.

To correctly interpret an ECG, you measure

time and voltage on the ECG paper.

p wave

time for the passage of the electrical impulse through the atrium causing atrial depolarization (contraction). Should be upright. Time: 0.06-0.12 Disturbance in conduction within atria

ischemia

tissue per fusion impaired in organ (heart)

The most accurate method for calculating heart rate from an ECG... (for reg rhythm)

to count the number of QRS complexes in 1 minute (300 large sqares).

BP helps determine if CO is adequate but assessment of end organ perfusion provides more relevant date; give 3 examples of this

urine output, neurologic function, peripheral pulses

Acute Pulmonary Edema

~ Capillary fluid moves into the alveoli, causes lung stiffness, makes lung expansion more difficult, and impairs the gas exchange function of the lung ~ With the decreased ability of the lungs to oxygenate the blood, the hemoglobin leaves the pulmonary circulation without being fully oxygenated - Results in shortness of breath & cyanosis

Diastolic Dysfunction

~ Heart failure due to diastolic dysfunction ~ Abnormal filling of ventricles affect cardiac output, leads to pulmonary & systemic venous congestion ~ Higher in women, people w/ hypertension, & atrial fibrillation ~ Problems that alter expansion of ventricles (cardial effusion), increase thickness of ventricular wall (hypertrophy) - the chamber of the ventricles will not receive enough blood volume ~ Cardiac output will decrease & stroke volume will decrease

Obstructive Circulatory Shock

~ Inability of the heart to fill properly ~ Obstruction of outflow from the heart ~ Due to pulmonary embolus or dissecting aneurysm (when you have a tear in a blood vessel)

Distributive Circulatory Shock

~ Loss of sympathetic vasomotor tone ~ Presence of a vasodilating substance in the blood ~ Presence of inflammatory mediators

Hypovolemic Circulatory Shock

~ Loss of whole blood ~ Loss of plasma ~ Loss of extracellular fluid ~ Could be due to injury

Dysrhythmias: Ongoing monitoring

• Monitor vital signs, level of consciousness, O2 saturation, and cardiac rhythm. • Anticipate need for administration of antidysrhythmia drugs and analgesics. • Anticipate need for intubation if respiratory distress is evident. • Prepare to initiate advanced cardiac life support (e.g., CPR, defibrillation, transcutaneous pacing).

Myocarditis Treatment

-ACE inhibitors -Beta blockers -diuretics, reduce preload -Digoxin improve contractility, reduce HR -immunosuppresive agents if autoimmune

Drug Therapy

-IV antibiotics with known activity against gram-negative bacteria are given before organisms are identified, preferably within 1 hour of sepsis diagnosis. -Septic Shock- vancomycin, aminoglycosides, systemic penicillin, or cephalosporins, macrolides & quinolones.

for the patient and in counseling other family members. The patient should be counseled against the use of stimulant drugs, but the limited past history indicates that the patient is not at current risk for cocaine use. Viral infections and CAD are risk factors for dilated cardiomyopathy, but not for HC.

...

The intensive care unit nurse is caring for a patient who is ventilated mechanically. To prevent sepsis in this patient, which nursing intervention does the nurse include in the plan of care? 1 Provide oral care every two to four hours. 2 Turn patient from side to side every eight hours. 3 Position patient in a supine position every two hours. 4 Use clean gloves when suctioning the endotracheal tube.

1 Providing oral care every two to four hours is correct, because research has found that the oral flora of critically ill patients are predominately gram-negative organisms that can potentially cause ventilator-associated pneumonia. Oral care will help reduce the organisms. Turning the patient from side to side every eight hours is incorrect, because patients need to be turned at least every two hours to prevent accumulation of mucus, which could lead to pneumonia. Positioning the patient in a supine position is incorrect, because patients should have the head of the bed elevated during mechanical ventilation. The nurse should use sterile gloves when conducting endotracheal suctioning of the patient. Test-Taking Tip: Never leave a question unanswered. Even if answering is no more than an educated guess on your part, go ahead and mark an answer. You might be right, but if you leave it blank, you will certainly be wrong and lose precious points. Text Reference - p. 1648

A patient is showing signs of anaphylactic shock from an insect sting. Which primary health care provider's prescription does the nurse implement first? 1 Epinephrine 1:1000, 0.5 mg subcutaneous (SQ) 2 Normal saline intravenous (IV) to run at 150 mL/hr 3 Diphenhydramine 50 mg IV 4 Oxygen via nasal cannula at 3 L

1 The patient in anaphylaxis experiences bronchial spasm and constriction. The administration of epinephrine is necessary to reverse this process and facilitate an open airway. Although administering normal saline, diphenhydramine, and oxygen are appropriate, they must be done after an airway has been established. Text Reference - p. 1646

From the data above, identify six clinical manifestations that may be evident in a patient in the compensated stage of shock.

1) Increase HR (b-Adrenergic stimulation) 2) cool, pale skin (a-Adrenergic stimulation) 3) thirst (with fluid shift to intravascular space) 4) decrease urine output (ADH and Aldosterone) 5) Fluctuating BP 6) decreased bowel sounds (a-adrenergic stimulation)

Risk factors infective endocarditis

1) aging 2) IVDA 3) prosthetic valves 4) intravascular devices with nosocomial infection 5) renal dialysis

systematic approach to assessing a heart rhythm... pwave

1. Look for the P wave. Is it upright/inverted? Is there one/more than one for every QRS complex? Are atrial fibrillatory/flutter waves present? 2. Evaluate the atrial rhythm. Is it regular or irregular? 3. Calculate the atrial rate. 4. Measure the duration of the PR interval. Is it normal duration/prolonged? 5. Evaluate the ventricular rhythm. Is it regular or irregular? 6. Calculate the ventricular rate. 7. Measure the duration of the QRS complex. Normal duration/prolonged? 8. Assess the ST segment. Is it isoelectric (flat), elevated, or depressed? 9. Measure the duration of the QT interval. Is it normal duration or prolonged? 10. Note the T wave. Is it upright or inverted? A. What is the dominant or underlying rhythm and/or dysrhythmia? B. What is the clinical significance of your findings? C. What is the treatment for the particular rhythm?

The nurse is caring for a 65-year-old man with acute respiratory distress syndrome (ARDS) who is on pressure support ventilation (PSV), fraction of inspired oxygen (FIO2) at 80%, and positive end-expiratory pressure (PEEP) at 15 cm H2O. The patient weighs 72 kg. What finding would indicate that treatment is effective? a. PaO2 of 60 mm Hg b. Tidal volume of 700 mL c. Cardiac output of 2.7 L/minute d. Inspiration to expiration ratio of 1:2

13. A Severe hypoxemia (PaO2 less than 40 mm Hg) occurs with ARDS, and PEEP is increased to improve oxygenation and prevent oxygen toxicity by reducing FIO2. A PaO2 level of 60 mm Hg indicates that treatment is effective and oxygenation status has improved. Decreased cardiac output is a complication of PEEP. Normal cardiac output is 4 to 8 L/minute. Normal tidal volume is 6 to 10 mL/kg. PSV delivers a preset pressure but the tidal volume varies with each breath. I:E ratio is usually set at 1:2 to 1:1.5 and does not indicate patient improvement.

What is a manifestation of the irreversible stage of shock? 1 Delirium 2 Areflexia 3 Restlessness 4 Alterations in the level of consciousness

2 Areflexia or loss of reflexes is a manifestation of the irreversible stage of shock. The progressive state of shock is associated with delirium. Restlessness and altered levels of consciousness indicate that the patient is in the compensatory stage of shock. Test-Taking Tip: Identifying content and what is being asked about that content is critical to your choosing the correct response. Be alert for words in the stem of the item that are the same or similar in nature to those in one or two of the options. Text Reference - p. 1639

When managing a patient with shock, which appropriate actions should the nurse take as part of nutritional therapy? Select all that apply. 1 Plan enteral feeding to meet at least 50 percent of calorie requirements. 2 Start enteral nutrition within the first 24 hours. 3 Wait until the patient recovers to start with enteral nutrition. 4 Start a slow continuous drip of small amounts of enteral feedings. 5 Start parenteral nutrition if enteral feedings are contraindicated.

2, 4, 5 Enteral nutrition should ideally begin within the first 24 hours. It is important because it enhances the perfusion of the (gastrointestinal) GI tract and helps to maintain the integrity of the gastrointestinal mucosa. Enteral feeding should be started with a slow continuous drip of small amounts of enteral feedings. If enteral feedings are contraindicated, parenteral feedings can be started. Enteral feeding should be planned to meet at least 80 percent of the total calorie requirements, but if it is not feasible, parenteral feeding should be started. Nutritional therapy should start as early as possible. The nurse should not wait for the patient to recover before starting nutritional therapy. Text Reference - p. 1644

The nurse recalls that cardiogenic shock is differentiated from other forms of shock because the patient with cardiogenic shock typically experiences: 1 Hypotension 2 Dysrhythmias 3 Volume excess 4 Volume depletion

3 In cardiogenic shock the heart fails as a pump. This usually results in fluid retention and poor perfusion of organs, including the kidneys, which adds to the fluid volume excess. As cardiogenic shock progresses, hypotension will develop, but it is not the cause of cardiogenic shock. Dysrhythmias may or may not be present with cardiogenic shock. Volume depletion is not generally seen with cardiogenic shock. It is, however, seen more with hypovolemic or distributive forms of shock. Text Reference - p. 1633

What causes dysrhythmias in patients with systemic inflammatory response syndrome (SIRS) and multiple organ dysfunction syndrome (MODS)? 1 Metabolic acidosis resulting in increase in lactate levels 2 Increased capillary permeability in the cardiovascular system 3 Hypokalemia due to activities of aldosterone and catecholamines 4 Aldosterone-mediated sodium and water reabsorption in kidneys

3 In systemic inflammatory response syndrome (SIRS) and multiple organ dysfunction syndrome (MODS), there are hormonal and metabolic changes and fluid shifts, including the release of aldosterone and catecholamines. Aldosterone increases urinary potassium loss and catecholamines cause potassium to move into the cell, resulting in hypokalemia, which causes dysrhythmias. Dysrhythmias are not associated with an increased lactate level caused by metabolic acidosis. Increased capillary permeability causes tachycardia rather than dysrhythmias. The increase in sodium reabsorption raises the serum osmolality and stimulates the release of the antidiuretic hormone (ADH). Increased sodium is not related to the development of dysrhythmias. Text Reference - p. 1649

32. When planning care for the patient with hypertrophic CMP, what should the nurse include? a. Ventricular pacing b. Administration of vasodilators c. Teach the patient to avoid strenuous activity and dehydration d. Surgery for cardiac transplantation will need to be done soon

32. c. Nursing interventions for the patient with hypertrophic CMP are to improve ventricular filling by reducing ventricular contractility and relieving left ventricular outflow obstruction to relieve symptoms and prevent complications. Strenuous activity and dehydration will increase systemic vascular resistance and should be avoided. Atrioventricular pacing will allow the septum to move away from the left ventricular wall and reduce the degree of outflow obstruction. Vasodilators may decrease venous return and further increase obstruction of blood flow from the heart. The surgery that could be done involves cutting into the thickened septal wall and removing some of the ventricular muscle.

To establish hemodynamic monitoring for a patient, the nurse zeros the a. CO monitoring system to the level of the LV b. pressure monitoring system to the level of the catheter tip located in the patient c. pressure monitoring system to the level of the atrium, identified as the phlebostatic axis d. pressure monitoring system to the level of the atrium, identified as the midclavicular line

4. Correct answer: c Rationale: Referencing means positioning the transducer so that the zero reference point is at the level of the atria of the heart. The stopcock nearest the transducer is usually the zero reference for the transducer. To place this reference level with the atria, use an external landmark: the phlebostatic axis. The phlebostatic axis is the intersection between the fourth intercostal space at the sternum and the midpoint between the anterior and posterior aspects of the chest wall. Position the port of the stopcock nearest the transducer level with the phlebostatic axis.

1 The distal lumen port (catheter tip), labeled A in the image, is within the pulmonary artery. This port is used to monitor pulmonary artery (PA) pressures. Choice B is the port used for infusions. Choice C is the port used for injecting medications. Choice D is the port used to inflate the balloon. Text Reference - p. 1607

A patient has the following device. Which port should be used to measure pulmonary artery pressure? 1. A 2. B 3. C 4. D

2 The patient with a ventricular assist device (VAD) may be mobile and require an activity plan such as progressive ambulation. Complete bedrest is not required. The patient will be permitted to do more than move from the bed to a chair twice a day. Activity will be greater than bed rest with bathroom privileges. Text Reference - p. 1613

A patient is recovering from the implantation of a ventricular assist device (VAD). What should the nurse anticipate being prescribed for this patient's activity status? 1 Complete bedrest 2 Progressive ambulation 3 Moving out of bed to a chair twice a day 4 Bedrest with bathroom privileges

1 The patient has a urinary tract infection, which can lead to sepsis. In sepsis, oxygen is not extracted properly at the tissue level, resulting in increased central venous oxygen saturation (ScvO2) mixed venous oxygen saturation (SvO2) measurements. Central venous oxygen saturation (ScvO2) mixed venous oxygen saturation (SvO2) measurements would be low if the patient was experiencing decreased cardiac output or increased oxygen demand. Central venous oxygen saturation (ScvO2) mixed venous oxygen saturation (SvO2) measurements would be between 60% and 80% if the oxygen supply and demand was balanced. Text Reference - p. 1609

A patient with a pulmonary arterial catheter for systolic heart failure is diagnosed with a urinary tract infection (UTI). The last central venous oxygen saturation (ScvO2) mixed venous oxygen saturation (SvO2) measurement was 89%. What should the nurse suspect is occurring with this patient? 1 Sepsis 2 Decreased cardiac output 3 Increased oxygen demand 4 Balanced oxygen supply and deman

1 The high- and low-pressure alarms are set based on the patient's current status. Since the patient's lowest auscultated systolic blood pressure was 118 mm Hg, the best setting to use would be systolic 100. Because the patient's lowest diastolic blood pressure was 78, the best setting to use would be diastolic 60. The setting of systolic 120 and diastolic 80 may cause the low pressure alarm to go off frequently. The settings of systolic 140, diastolic 80 and systolic 150, diastolic 90 would not be appropriate for low-pressure settings. Text Reference - p. 1606

A patient with an arterial invasive device has the following auscultated blood pressures. What setting should the nurse use for this patient's low pressure alarms? 1 Systolic 100; Diastolic 60 2 Systolic 120; Diastolic 80 3 Systolic 140; Diastolic 80 4 Systolic 150; Diastolic 90

The client diagnosed with pericarditis is experiencing cardiac tamponade. Which collaborative intervention should the nurse anticipate for this client? 1. Prepare for a pericardiocentesis.

A pericardiocentesis removes fluid from the pericardial sac and is the emergency treatment for cardiac tamponade.

The nurse conducts a complete physical assessment on a patient admitted with infective endocarditis. Which finding is significant? B. Regurgitant murmur at the mitral valve area

A regurgitant murmur of the aortic or mitral valves would indicate valvular disease, which is a complication of endocarditis.

Which assessment data would support the diagnosis of abdominal aortic aneurysm (AAA)? 2. Abdominal bruit.

A systolic bruit over the abdomen is a diagnostic indication of an AAA.

Appropriate treatment modalities for the mgmt. of carcinogenic shock include (select all) A) dobutamine to increase myocardial contractility B) Vasopressors to increase systemic vascular resistance C) circulatory assist devices such as an intraaortic balloon pump D) corticosteroids to stabilize the cell wall in the infarcted myocardium E) Trendelenburg positioning to facilitate venous return and increase preload

A) dobutamine to increase myocardial contractility C) circulatory assist devices such as an intraaortic balloon pump

Following an acute myocardial infarction, a previously healthy 63-year-old develops clinical manifestations of heart failure. The nurse anticipates discharge teaching will include information about d. angiotensin-converting enzyme (ACE) inhibitors.

ACE inhibitor therapy is currently recommended to prevent the development of heart failure in patients who have had a myocardial infarction and as a first-line therapy for patients with chronic heart failure.

The following therapies are prescribed by the health care provider for a patient who has respiratory distress and syncope after a bee sting. Which will the nurse administer first? a. normal saline infusion b. epinephrine (Adrenalin) c. dexamethasone (Decadron) d. diphenhydramine (Benadryl)

ANS: B Epinephrine rapidly causes peripheral vasoconstriction, dilates the bronchi, and blocks the effects of histamine and reverses the vasodilation, bronchoconstriction, and histamine release that cause the symptoms of anaphylaxis. The other interventions also are appropriate but would not be the first ones administered. DIF: Cognitive Level: Application REF: 1736-1737 OBJ: Special Questions: Prioritization TOP: Nursing Process: Implementation MSC: NCLEX: Physiological Integrity

The following interventions are ordered by the health care provider for a patient who has respiratory distress and syncope after eating strawberries. Which will the nurse complete first? a. Start a normal saline infusion. b. Give epinephrine (Adrenalin). c. Start continuous ECG monitoring. d. Give diphenhydramine (Benadryl).

ANS: B Epinephrine rapidly causes peripheral vasoconstriction, dilates the bronchi, and blocks the effects of histamine and reverses the vasodilation, bronchoconstriction, and histamine release that cause the symptoms of anaphylaxis. The other interventions are also appropriate but would not be the first ones completed.

A patient with cardiogenic shock has the following vital signs: BP 102/50, pulse 128, respirations 28. The pulmonary artery wedge pressure (PAWP) is increased and cardiac output is low. The nurse will anticipate an order for which medication? a. 5% human albumin b. Furosemide (Lasix) IV c. Epinephrine (Adrenalin) drip d. Hydrocortisone (Solu-Cortef)

ANS: B The PAWP indicates that the patient's preload is elevated, and furosemide is indicated to reduce the preload and improve cardiac output. Epinephrine would further increase heart rate and myocardial oxygen demand. 5% human albumin would also increase the PAWP. Hydrocortisone might be considered for septic or anaphylactic shock

A patient who has chronic heart failure tells the nurse, "I was fine when I went to bed, but I woke up in the middle of the night feeling like I was suffocating!" The nurse will document this assessment finding as a. orthopnea. b. pulsus alternans. c. paroxysmal nocturnal dyspnea. d. acute bilateral pleural effusion.

ANS: C Paroxysmal nocturnal dyspnea is caused by the reabsorption of fluid from dependent body areas when the patient is sleeping and is characterized by waking up suddenly with the feeling of suffocation. Pulsus alternans is the alternation of strong and weak peripheral pulses during palpation. Orthopnea indicates that the patient is unable to lie flat because of dyspnea. Pleural effusions develop over a longer time period.

25. When caring for the patient with a pulmonary artery (PA) pressure catheter, the nurse observes that the PA waveform indicates that the catheter is in the wedged position. Which action should the nurse take next? a. Zero balance the transducer. b. Activate the fast flush system. c. Notify the health care provider. d. Deflate and reinflate the PA balloon.

ANS: D When the catheter is in the wedge position, blood flow past the catheter is obstructed, placing the patient at risk for pulmonary infarction. A health care provider or advanced practice nurse should be called to reposition the catheter. The other actions will not correct the wedging of the PA catheter. DIF: Cognitive Level: Apply (application) REF: 1608 TOP: Nursing Process: Implementation MSC: NCLEX: Physiological Integrity

Hespan is used in which type of shock?

All types except cardiogenic and neurogenic shock

Anaphylaxis

Allergic reactions that result in widespread loss of blood vessel tone & decreased cardiac output

Vasodilators

Allow normal blood flow & delivery of oxygen

Oher tests (besides ECG) can assess dysrhythmias and the effectiveness of antidysrhythmia drug therapy...

An electrophysiologic study, Holter monitoring, event monitoring (or loop recorder), exercise treadmill testing, a nd signal-averaged ECG can be performed on an inpatient/outpatient

The nurse is assessing the client diagnosed with congestive heart failure. Which signs/ symptoms would indicate that the medical treatment has been effective? 3. The client is able to perform activities of daily living without dyspnea.

Being able to perform activities of daily living (ADLs) without shortness of breath (dyspnea) would indicate the client's condition is improving. The client's heart is a more effective pump and can oxygenate the body better without increasing fluid in the lungs.

A 72-year-old man with a history of aortic stenosis is admitted to the emergency department. He reports severe left-sided chest pressure radiating to the jaw. Which medication, if ordered by the health care provider, should the nurse question? A. Aspirin B. Oxygen C. Nitroglycerin D. Morphine sulfate

C. Nitroglycerin Aspirin, oxygen, nitroglycerin, and morphine sulfate are all commonly used to treat acute chest pain suspected to be caused by myocardial ischemia. However, nitroglycerin should be used cautiously or avoided in patients with aortic stenosis as a significant reduction in blood pressure may occur. Chest pain can worsen because of a drop in blood pressure.

The client is diagnosed with a myocardial infarction. Which referral would be most appropriate for the client? 3. Cardiac rehabilitation.

Cardiac rehabilitation is the most appropriate referral. The client can start rehabilitation in the hospital and then attend an outpatient cardiac rehabilitation, which includes progressive exercise, diet teaching, and classes on modifying risk factors.

A 55-year-old female patient develops acute pericarditis after a myocardial infarction. It is most important for the nurse to assess for which clinical manifestation of a possible complication? D. Decreased blood pressure with tachycardia

Cardiac tamponade is a serious complication of acute pericarditis. Signs and symptoms indicating cardiac tamponade include narrowed pulse pressure, tachypnea, tachycardia, a decreased cardiac output, and decreased blood pressure.

Physical Assessment

Cardiovascular- differ in different stages. Cardiac output & BP are lower (hypodynamic) in early sepsis & septic shock. In severe septic; cardiac output is higher & HR & BP (hyperdynamic); Increased CO reflected by tachycardia, increased SV, normal systolic BP, & normal CVP. Respiratory- as tissue hypoxia becomes more profound & metabolic acidosis is present, the depth of respiration increases; ARDs- acute respiratory distress syndome- Skin Changes- hypodynamic stage- blood is shunted away from skin, & pallor, cyanosis or mottling may be present. Hyperdynamic- skin is warm & no cyanosis evident. Cool & clammy with pallow, mottling or cyanosis. Kidney/Urinary- low urine output compared with fluid intake indicates shock

The nurse is caring for a 78-year-old patient with aortic stenosis. Which assessment data obtained by the nurse would be most important to report to the health care provider? a. The patient complains of chest pressure when ambulating.

Chest pressure (or pain) occurring with aortic stenosis is caused by cardiac ischemia, and reporting this information would be a priority

A pt. has a spinal cord injury at T4. VS include a falling BP with bradycardia. The nurse recognizes that the pt. is experiencing A) a relative hypervolemia B) an absolute Hypovolemia C) Neurogenic shock from low blood flow D) Neurogenic shock from massive vasodilation

D) Neurogenic shock from massive vasodilation

The client diagnosed with essential hypertension asks the nurse, "I don't know why the doctor is worried about my blood pressure. I feel just great." Which statement by the nurse would be the most appropriate response? 1. "Damage can be occurring to your heart and kidneys even if you feel great."

Even if the client feels great, the blood pressure can be elevated, causing damage to the heart, kidney, and blood vessels.

The female client is diagnosed with rheumatic fever and prescribed penicillin, an antibiotic. Which statement indicates the client needs more teaching concerning the discharge teaching? 3. "I will have no problems as long as I take my medication."

Even with antibiotic treatment for rheumatic fever, the client may experience bacterial endocarditis in later years and should know this may occur.

Colloid solution

Hemorrhagic shock Restore osmotic pressure Carries oxygen to peripheral tissues Substitute for blood Hetastarch, Packed red cells, fresh frozen plasma (FFP)

How can atrial flutter lead to heart failure?

High ventricular rate and loss of atrial "kick" --> decreased CO --> HF

What are the 6Hs of pulseless electrical activity (PEA)

Hyperkalemia Hypoxia Hpothermia Hydrogen ion excess Hypovolemia Hypoglycemia

Colloid Dextran (dextran 40, dextran 70) - Mechanism of Action

Hyperosmotic glucose polymer Similar degrees of volume expansion w/dextran 40 and dextran 70 Longer duration of action w/dextran 70

The client who has just had a percutaneous balloon valvuloplasty is in the recovery room. Which intervention should the recovery room nurse implement? 4. Keep the client's affected leg straight

In this invasive procedure, performed in a cardiac catheterization laboratory, the client has a catheter inserted into the femoral artery. Therefore, the client must keep the leg straight to prevent hemorrhaging at the insertion site.

A 53-year-old patient with Stage D heart failure and type 2 diabetes asks the nurse whether heart transplant is a possible therapy. Which response by the nurse is most appropriate? b. "The choice of a patient for a heart transplant depends on many different factors."

Indications for a heart transplant include end-stage heart failure (Stage D), but other factors such as coping skills, family support, and patient motivation to follow the rigorous posttransplant regimen are also considered.

1.8%, 3%, 5% NaCl is used in which type of shock?

Initial volume expansion in hypovolemic shock

Nursing Implications for use of 1.8%, 3%, 5% NaCl

Monitor pt closely for signs of hypernatremia (e.g., disorientation, convulsions). Central line preferred for infusing 3% or greater NaCl as these solns are caustic.

The nurse is assessing a patient with myocarditis before administering the scheduled dose of digoxin (Lanoxin). Which finding is most important for the nurse to communicate to the health care provider? b. Irregular pulse

Myocarditis predisposes the heart to digoxin-associated dysrhythmias and toxicity. T

First-Degree AV Block symptoms?

Nope - asymptomatic

DISTRIBUTIVE SHOCK

Occurs when blood volume is not lost from the body but is distributed to the interstitial tissues where it can't circulate & deliver oxygen. Can be caused by loss of sympathetic tone, blood vessel dilation, pooling of blood in venous & capillary beds, & increased capillary leak.

Distributive-neurogenic

Older adult who has a spinal cord reunion

Obstructive

Patient with cardiac tamponade Patient with tension pneumothorax

BNP

Peptide that causes natriuresis. elevation helps distinguish cardiac vs resp cause of dyspnea

The client is exhibiting sinus bradycardia, is complaining of syncope and weakness, and has a BP of 98/60. Which collaborative treatment should the nurse anticipate being implemented?

Prepare for insertion of a pacemaker. - client is symptomatic and will need a pacemaker - PAY ATTENTION IT SAYS COLLABORATIVE, NOT INDEPENDENT! - assessment is independent action

The client with varicose veins is six (6) hours post-operative vein ligation. Which nursing intervention should the nurse implement first? 2. Assess and maintain pressure bandages on the affected leg.

Pressure bandages are applied for up to six (6) weeks after vein ligation to help prevent bleeding and to help venous return from the lower extremities when in the standing or sitting position.

Calcium channel blockers

Prevent Ca entry into vascular smooth muscle cells and myocytes. May prevent or control coronary vasospasm. Promote coronary and peripheral vasodialtion. Reduce HR, BP, and contractility,

Symmetric Hypertrophy

Proportionate increase in the thickness and in the length of muscle fibers (as seen in athletes)

A 25-year-old patient with a group A streptococcal pharyngitis does not want to take the antibiotics prescribed. What should the nurse tell the patient to encourage the patient to take the medications and avoid complications of the infection? C. "Without treatment, you could get rheumatic fever, which can lead to rheumatic heart disease."

Rheumatic fever (RF) is not common because of effective use of antibiotics to treat streptococcal infections.

o

Temporary treatment before the insertion of a permanent pacemaker may be necessary if the patient becomes symptomatic and involves the use of a temporary transvenous or transcutaneous pacemaker.

1, 2, 3 ICU patients often require intensive and complicated nursing support related to the use of IV polypharmacy and advanced technology. Prescriptions that are classified as IV polypharmacy include sedation, thrombolytics, and vasopressor titration. Mechanical ventilation and hemodynamic monitoring are advanced technology prescriptions. Text Reference - p. 1600

The intensive care unit (ICU) nurse is providing care to a patient requiring IV polypharmacy. Which prescriptions is this patient receiving? Select all that apply. 1 Sedation 2 Thrombolytics 3 Vasopressor titration 4 Mechanical ventilation 5 Hemodynamic monitoring

The charge nurse is making shift assignments for the medical floor. Which client should be assigned to the most experienced registered nurse? 3. The client with an apical pulse rate of 116, a respiratory rate of 26, and a blood pressure of 94/62.

This client is exhibiting signs/symptoms of shock, which makes this client the most unstable. An experienced nurse should care for this client.

A patient with a history of chronic heart failure is admitted to the emergency department (ED) with severe dyspnea and a dry, hacking cough. Which action should the nurse do first? c. Auscultate the breath sounds.

This patient's severe dyspnea and cough indicate that acute decompensated heart failure (ADHF) is occurring.

MI pain

Total occlusion → Anaerobic metabolism and lactic acid accumulation → Severe, immobilizing chest pain not relieved by rest, position change, or nitrate administration

V. tach treatment (consider stable vs. unstable)

Treat the cause With pulse = antidysrhythmics or cardioversion Pulseless = CPR and rapid defibrillation

When planning care for a patient hospitalized with a streptococcal infective endocarditis (IE), which intervention is a priority for the nurse to include? b. Arrange for placement of a long-term IV catheter.

Treatment for IE involves 4 to 6 weeks of IV antibiotic therapy in order to eradicate the bacteria, which will require a long-term IV catheter such as a peripherally inserted central catheter (PICC) line.

The client diagnosed with pericarditis is being discharged home. Which intervention should the nurse include in the discharge teaching? 1. Be sure to allow for uninterrupted rest and sleep.

Uninterrupted rest and sleep help decrease the workload of the heart and help ensure the restoration of physical and emotional health.

o

VT is a life-threatening dysrhythmia because of decreased CO and the possibility of deterioration to ventricular fibrillation, which is a lethal dysrhythmia.

2 Positive end-expiratory pressure (PEEP) therapy is given to patients with pulmonary edema to provide a counter pressure opposing fluid extravasation. PEEP is not used for patients with hypovolemia, low cardiac output and unilateral or nonuniform lung disease because in those patients, the adverse effects of PEEP may outweigh any benefits. Text Reference - p. 1621

What condition would the nurse infer that a patient undergoing positive end-expiratory pressure (PEEP) therapy has? 1 Hypovolemia 2 Pulmonary edema 3 Low cardiac output 4 Unilateral lung disease

What are two common characteristics of NEUROGENIC shock?

_Bradycardia and hypotension

Which intervention may prevent GI bacterial and endotoxin translocation in a critically ill patient with SIRS? a. early enteral feedings b. surgical removal of necrotic tissue c. aggressive multiple antibiotic therapy d. strict aseptic technique in all procedures

a. early enteral feedings Early enteral feeding in the patient with shock are believed to increase the blood supply to the GI tract and help prevent translocation of the GI bacteria and endotoxins into the blood, preventing initial or additional infection in patients in shock. Surgical removal of necrotic tissue, especially from burns and strict aseptic technique in all procedures eliminates a source of infection. Known infections are treated with specific agents, and broad-spectrum agents are used only until organisms are identified.

2 other methods (for reg rhythm)

count the number of small squares between one R-R interval. Divide this number into 1500 to get the HR. count the number of large squares between one R-R interval and divide this number into 300 to get the HR

signs of peripheral hypoperfusion (6)

cyanosis, pallor, diaphoresis, weak peripheral pulses, cool and clammy skin, delayed capillary refill

Following coronary artery bypass graft surgery a patient has postoperative bleeding that requires returning to surgery to repair the leak. During surgery, the patient has a myocardial infarction (MI). After restoring the patient's body temperature to normal, which patient assessment is the most important for planning nursing care? a Cardiac index (CI) 5 L/min/m2 b Central venous pressure 8 mm Hg c Mean arterial pressure (MAP) 86 mm Hg d Pulmonary artery pressure (PAP) 28/14 mm Hg

d Pulmonary artery pressure (PAP) 28/14 mm Hg Pulmonary hypertension as indicated by an elevated PAP indicates impaired forward flow of blood because of left ventricular dysfunction or hypoxemia. Both can be due to the MI. The CI, CVP, and MAP readings are normal.

Beyond the first year after a heart transplant, the nurse knows that what is a major cause of death? a. Infection b. Acute rejection c. Immunosuppression d. Cardiac vasculopathy

d. Cardiac vasculopathy Beyond the first year after a heart transplant, malignancy (especially lymphoma) and cardiac vasculopathy (accelerated CAD) are the major causes of death. During the first year after transplant, infection and acute rejection are the major causes of death. Immunosuppressive therapy will be used for posttransplant management to prevent rejection and increases the patient's risk of an infection.

The electrocardiogram (ECG) is a

graphic tracing of the electrical impulses produced in the heart.

Primary Reason Chronic Stable Angina

insufficient blood flow in narrowed coronary arteries due to atherosclerosis

Catheter ablation

is the nonpharmacologic treatment of choice for atrial dysrhythmias resulting in rapid ventricular rates and AV nodal reentrant tachycardia refractory to drug therapy.

Premature ventricular contraction (PVC)

is the premature occurrence of a QRS complex, which is wide and distorted in shape compared with a QRS complex initiated from the normal conduction pathway.

While assessing a 68-year-old with ascites, the nurse also notes jugular venous distention (JVD) with the head of the patient's bed elevated 45 degrees. The nurse knows this finding indicates c. increased right atrial pressure.

jugular veins empty into the superior vena cava and then into the right atrium, so JVD with the patient sitting at a 45-degree angle reflects increased right atrial pressure.

Decreased kidney perfusion => ______ => Increased angiotensin 1.

renin released

Manual defibrillators

require health care providers to interpret cardiac rhythms, determine the need for a shock, and deliver a shock.

centralized telemetry monitoring consists of

requires you or a telemetry technician to continuously observe a group of patients' ECGs at a central location.

`The membrane of a cardiac cell is

semipermeable

The recovery period after stimulation is

the refractory phase or period.

ST segment represents

the time between ventricular depolarization and repolarization. This segment should be flat or isoelectric and represents the absence of any electrical activity between these two events.

The heart's conduction system consists of specialized neuromuscular tissue located

throughout the heart

Noninvasive diagnostic tests

used to evaluate cardiac dysrhythmias and the effectiveness of antidysrhythmia drug therapy include Holter monitoring, event monitoring (or loop recorder), exercise treadmill testing, and signal-averaged ECG.

Compensatory or Adaptive Mechanisms in Maintenance of Cardiac Reserve in Heart Failure

~ Frank-Starling Mechanism ~ Activation of Neurohumoral ~ Renin-angiotensin-aldosteron Mechanism ~ Natriuretic Peptides, ANP (atrial natriuretic peptide) ~ Locally produced vasoactive substances ~ Myocardial hypertrophy & remodeling ~ Symmetric Hypertrophy ~ Concentric Hypertrophy ~ Eccentric Hypertrophy

Signs & Symptoms of Left-Sided Heart Failure

~ Pulmonary congestion & edema ~ Decreased cardiac output & tissue perfusion ~ Cyanosis & hypoxia ~ Cough with sputum (secretions coming out like phlegm)

Electrocardiogram Monitoring

•The electrocardiogram (ECG) is a graphic tracing of the electrical impulses produced in the heart. •ECG waveforms are produced by the movement of charged ions across the semipermeable membranes of myocardial cells.

The client diagnosed with a myocardial infarction asks the nurse, "Why do I have to rest and take it easy? My chest doesn't hurt anymore." Which statement would be the nurse's best response?

"Your heart is damaged and needs about four (4) to six (6) weeks to heal."

If the patient is symptomatic

, atropine is used to increase HR, or a temporary pacemaker may be needed.

Ventricular assist devices are useful for-

-cardiogenic shock.

Myocarditis

-focal or diffuse inflammation of myocardium

Clinical manifestations of MODS include symptoms of-

-respiratory distress, signs and symptoms of decreased renal perfusion, decreased serum albumin and prealbumin, decreased GI motility, acute neurologic changes, myocardial dysfunction, disseminated intravascular coagulation (DIC), and changes in glucose metabolism.

Five steps of ECG Analysis

1. Determine the heart rate 2. Determine the heart rhythm 3. Analyze the P waves 4. Measure the PR interval 5. Measure the QRS Duration

Five indicators of tissue perfusion that should be monitored in critically ill patients are:

1. Vital signs, including spO2 2. LOC 3. skin (color, temperature, moisture) 4. urine output 5. peripheral pulses with capillary refill

An ECG has.......leads

12 recording leads.

The intensive care department nurse is calculating the total intake for a client diagnosed with hypertensive crisis. The client has received 880 mL of D5W, IVPB of 100 mL of 0.9% NS, 8 ounces of water, 4 ounces of milk, and 6 ounces of chicken broth. The client has had a urinary output of 1480 mL. What is the total intake for this client?_______

1520 mL

Which medication would the nurse administer to treat renal complications associated with systemic inflammatory response syndrome (SIRS)? 1 Antacids 2 Sucralfate 3 Furosemide 4 Omeprazole

3 A SIRS-affected (systemic inflammatory response syndrome) patient is administered a loop diuretic such as furosemide to address renal complications related to hypoperfusion and a decreased glomerular filtration rate. Antacids, sucralfate, and omeprazole act as a stress ulcer prophylaxis for the gastrointestinal system. Text Reference - p. 1651

A patient experiences a myocardial infarction (MI). The nurse closely monitors the patient for complications and recognizes that hypotension is a warning sign of: 1 A secondary MI 2 Pulmonary edema 3 Cardiogenic shock 4 Fatal dysrhythmias

3 One of the initial cardinal signs of cardiogenic shock after a myocardial infarction (MI) is a slow, steady drop in blood pressure. Hypotension after an MI may be an indirect sign of a secondary MI or a fatal dysrhythmia. Depending on the origin of pulmonary edema, patients may experience hypotension or hypertension. Text Reference - p. 1633

The critical care nurse recognizes that an ideal plan for CG involvement includes a. a CG at the bedside at all times b. allowing CG at the bedside at preset, brief intervals c. an individually devised plan to involve CG with care and comfort measures d. restriction of visiting in the ICU because the environment is overwhelming to CG

3. Correct answer: c Rationale: An individualized plan of care should be developed for each patient and the caregivers. Caregivers should be allowed to assist with care and comfort measures in the ICU.

The client diagnosed with a myocardial infarction is six (6) hours post-right femoral percutaneous transluminal coronary angioplasty (PTCA), also known as balloon surgery. Which assessment data would require immediate intervention by the nurse?

3. The client is complaining of numbness in the right foot. - indicate decreased blood supply to the right foot - do neurovascular assessment after PTCA - keep legs straight for 6-8 hours

3. Which manifestation of infective endocarditis is a result of fragmentation and microembolization of vegetative lesions? a. Petechiae b. Roth's spots c. Osler's nodes d. Splinter hemorrhages

3. a. Petechiae are seen as small hemorrhages in the conjunctiva, lips, and buccal mucosa and over the ankles, feet, and antecubital and popliteal areas. Roth's spots are hemorrhagic retinal lesions seen with funduscopic examination. Osler's nodes are lesions on the fingertips or toes. The cause of Roth's spots and Osler's nodes is not clear. Splinter hemorrhages are black longitudinal streaks that occur on nail beds. They may be caused by vessel damage from vasculitis or microemboli.

Which laboratory finding in a patient with multiple organ dysfunction syndrome (MODS) suggests prerenal manifestations of renal dysfunction? 1 Urine Na+ is 22 mEq/L. 2 Urine specific gravity is 1.010. 3 Urine osmolality is decreased. 4 Urine specific gravity is increased.

4 An increase in urine specific gravity suggests prerenal manifestations in patients with MODS. This occurs due to renal hypoperfusion. A urine Na+ level above 20 mEq/L is a sign of intrarenal manifestations in MODS. It occurs due to acute tubular necrosis. Other intrarenal manifestations of MODS are urine specific gravity of around 1.010 and a decrease of the urine osmolality.

4. What describes Janeway's lesions that are manifestations of infective endocarditis? a. Hemorrhagic retinal lesions b. Black longitudinal streaks in nail beds c. Painful red or purple lesions on fingers or toes d. Flat, red, painless spots on the palms of hands and soles of feet

4. d. Janeway's lesions are flat, painless, small red spots found on the palms of the hands and the soles of the feet. Black streaks on the nails are splinter hemorrhages. Hemorrhagic retinal lesions are Roth's spots. Painful lesions on the fingers and toes are Osler's nodes.

Physiological Integrity COMPLETION 1. When analyzing an electrocardiographic (ECG) rhythm strip of a patient with a regular heart rhythm, the nurse counts 30 small blocks from one R wave to the next. The nurse calculates the patient's heart rate as ____.

50 There are 1500 small blocks in a minute, and the nurse will divide 1500 by 30. DIF: Cognitive Level: Remember (knowledge) REF: 789-790 TOP: Nursing Process: Assessment MSC:

can compensate for a total blood loss of

750 ml (15%)

Physiological Integrity 18. A patient's cardiac monitor shows a pattern of undulations of varying contours and amplitude with no measurable ECG pattern. The patient is unconscious and pulseless. Which action should the nurse take first? a. Perform immediate defibrillation. b. Give epinephrine (Adrenalin) IV. c. Prepare for endotracheal intubation. d. Give ventilations with a bag-valve-mask device.

A The patient's rhythm and assessment indicate ventricular fibrillation and cardiac arrest; the initial action should be to defibrillate. If a defibrillator is not immediately available or is unsuccessful in converting the patient to a better rhythm, the other actions may be appropriate. DIF: Cognitive Level: Apply (application) REF: 801 OBJ: Special Questions: Prioritization TOP: Nursing Process: Implementation MSC:

Which assessment finding in a patient admitted with acute decompensated heart failure (ADHF) requires the most immediate action by the nurse? a. Oxygen saturation of 88%

A decrease in oxygen saturation to less than 92% indicates hypoxemia.

The nurse is teaching a class on coronary artery disease. Which modifiable risk factors should the nurse discuss when teaching about atherosclerosis? 1. Stress.

A modifiable risk factor is a risk factor that can possibly be altered by modifying or changing behavior, such as developing new ways to deal with stress.

The nurse is discussing the pathophysiology of atherosclerosis with a client who has a normal high-density lipoprotein (HDL) level. Which information should the nurse discuss with the client concerning HDL? A normal HDL is good because it has a protective action in the body.

A normal HDL level is good because HDL transports cholesterol away from the tissues and cells of the arterial wall to the liver for excretion. This helps decrease the development of atherosclerosis.

Decreased Cardiac Output The primary problem for the client in shock is decreased cardiac output. All of the diagnoses are applicable, but decreased cardiac output is the priority.

A nurse caring for a client in shock establishes which of the following as the primary nursing diagnosis?

2 Mechanical ventilation can cause pneumothorax as a result of excessive pressure applied to lung tissue. Hypertension is not a direct complication; however, a patient undergoing mechanical ventilation may be anxious and fearful, resulting in high blood pressure; sedation should be considered in this event. Electrolyte imbalance is not a related complication. Mechanical ventilation does increase intrathoracic pressure, which may then increase cardiac output, causing a beneficial secondary effect. Text Reference - p. 1615

A nurse should monitor a patient undergoing mechanical ventilation for which common complication? 1 Hypertension 2 Pneumothorax 3 Electrolyte imbalance 4 Increased cardiac output

When caring for a patient with infective endocarditis, the nurse will assess the patient for which vascular manifestations (select all that apply)?

A. Osler's nodes B. Janeway's lesions C. Splinter hemorrhages

The nurse is teaching a community group about preventing rheumatic fever. What information should the nurse include?

A. Prompt recognition and treatment of streptococcal pharyngitis

The nurse is teaching a community group about preventing rheumatic fever. What information should the nurse include? A. Prompt recognition and treatment of streptococcal pharyngitis B. Completion of 4 to 6 days of antibiotic therapy for infective endocarditis of respiratory infections in children born with heart defects C. Avoidance of respiratory infections in children who have rheumatoid arthritis D. Requesting antibiotics before dental surgery for individuals with rheumatoid arthritis

A. Prompt recognition and treatment of streptococcal pharyngitis The nurse should emphasize the need for prompt and adequate treatment of streptococcal pharyngitis infection, which can lead to the complication of rheumatic fever.

While admitting a patient with pericarditis, the nurse will assess for what manifestations of this disorder? A. Pulsus paradoxus B. Prolonged PR intervals C. Widened pulse pressure D. Clubbing of the fingers

A. Pulsus paradoxus Pericarditis can lead to cardiac tamponade, an emergency situation. Pulsus paradoxus greater than 10 mm Hg is a sign of cardiac tamponade that should be assessed at least every 4 hours in a patient with pericarditis. Prolonged PR intervals occur with first-degree AV block. Widened pulse pressure occurs with valvular heart disease. Clubbing of fingers may occur in subacute forms of infective endocarditis and valvular heart disease.

The health care provider orders the following interventions for a 67-kg patient who has septic shock with a BP of 70/42 mm Hg and oxygen saturation of 90% on room air. In which order will the nurse implement the actions? (Put a comma and a space between each answer choice [A, B, C, D, E].) a. Obtain blood and urine cultures. b. Give vancomycin (Vancocin) 1 g IV. c. Start norepinephrine (Levophed) 0.5 mcg/min. d. Infuse normal saline 2000 mL over 30 minutes. e. Titrate oxygen administration to keep O2 saturation >95%.

ANS: E, D, C, A, B The initial action for this hypotensive and hypoxemic patient should be to improve the oxygen saturation, followed by infusion of IV fluids and vasopressors to improve perfusion. Cultures should be obtained before administration of antibiotics

Which assessment finding in a patient admitted with acute decompensated heart failure (ADHF) requires the most immediate action by the nurse? a. Oxygen saturation of 88% b. Weight gain of 1 kg (2.2 lb) c. Heart rate of 106 beats/minute d. Urine output of 50 mL over 2 hours

ANS: A A decrease in oxygen saturation to less than 92% indicates hypoxemia. The nurse should administer supplemental oxygen immediately to the patient. An increase in apical pulse rate, 1-kg weight gain, and decreases in urine output also indicate worsening heart failure and require nursing actions, but the low oxygen saturation rate requires the most immediate nursing action.

A patient is admitted to the hospital with possible acute pericarditis. The nurse should plan to teach the patient about the purpose of a. echocardiography. b. daily blood cultures. c. cardiac catheterization. d. 24-hour Holter monitor.

ANS: A Echocardiograms are useful in detecting the presence of the pericardial effusions associated with pericarditis. Blood cultures are not indicated unless the patient has evidence of sepsis. Cardiac catheterization and 24-hour Holter monitor is not a diagnostic procedure for pericarditis.

During change-of-shift report, the nurse is told that a patient has been admitted with dehydration and hypotension after having vomiting and diarrhea for 4 days. Which finding is most important for the nurse to report to the health care provider? a. New onset of confusion b. Heart rate 112 beats/minute c. Decreased bowel sounds d. Pale, cool, and dry extremities

ANS: A The changes in mental status are indicative that the patient is in the progressive stage of shock and that rapid intervention is needed to prevent further deterioration. The other information is consistent with compensatory shock

33. A patient who is orally intubated and receiving mechanical ventilation is anxious and is "fighting" the ventilator. Which action should the nurse take next? a. Verbally coach the patient to breathe with the ventilator. b. Sedate the patient with the ordered PRN lorazepam (Ativan). c. Manually ventilate the patient with a bag-valve-mask device. d. Increase the rate for the ordered propofol (Diprivan) infusion.

ANS: A The initial response by the nurse should be to try to decrease the patient's anxiety by coaching the patient about how to coordinate respirations with the ventilator. The other actions may also be helpful if the verbal coaching is ineffective in reducing the patient's anxiety. DIF: Cognitive Level: Apply (application) REF: 1623 OBJ: Special Questions: Prioritization TOP: Nursing Process: Implementation MSC: NCLEX: Physiological Integrity

After receiving change-of-shift report on a heart failure unit, which patient should the nurse assess first? a. A patient who is cool and clammy, with new-onset confusion and restlessness b. A patient who has crackles bilaterally in the lung bases and is receiving oxygen. c. A patient who had dizziness after receiving the first dose of captopril (Capoten) d. A patient who is receiving IV nesiritide (Natrecor) and has a blood pressure of 100/62

ANS: A The patient who has "wet-cold" clinical manifestations of heart failure is perfusing inadequately and needs rapid assessment and changes in management. The other patients also should be assessed as quickly as possible but do not have indications of severe decreases in tissue perfusion.

The female patient presents to the emergency department just after a syncope episode. What should you assess first? A. History of syncope B. Capillary glucose level C. Last menstrual period D. Allergies

ANS: B A change in the level of consciousness should always have glucose and oxygen (and cardiac) assessed first. Hypoglycemia is a noncardiovascular cause that can be easily treated. It takes priority over the other assessments. Although those who have syncope have a 30% chance of recurrence, ruling out a simple treatable cause should be done first. Reference: 839

Which admission order written by the health care provider for a patient admitted with infective endocarditis (IE) and a fever would be a priority for the nurse to implement? a. Administer ceftriaxone (Rocephin) 1 g IV. b. Order blood cultures drawn from two sites. c. Give acetaminophen (Tylenol) PRN for fever. d. Arrange for a transesophageal echocardiogram.

ANS: B Treatment of the IE with antibiotics should be started as quickly as possible, but it is essential to obtain blood cultures before initiating antibiotic therapy to obtain accurate sensitivity results. The echocardiogram and acetaminophen administration also should be implemented rapidly, but the blood cultures (and then administration of the antibiotic) have the highest priority.

What term is applied to a pacemaker that is implanted for the purpose of terminating atrial tachycardias? A. Temporary pacemaker B. Antitachycardia pacing C. Overdriving pacing D. Cardiac resynchronization therapy

ANS: C Overdrive pacing involves pacing the atrium at a rate of 200 to 500 impulses per minute in an attempt to terminate atrial tachycardias (e.g., atrial flutter with a rapid ventricular response). A temporary pacemaker is a category of pacemakers used temporarily with the power source outside the body. Antitachycardia pacing delivers a stimulus to the ventricle to terminate tachydysrhythmias. Cardiac resynchronization therapy is a technique to resynchronize the cardiac cycle by pacing both ventricles. Reference: 835

What is the purpose of the head-up tilt test? A. To determine whether a fluid volume deficit exists B. To assess for dysrhythmias when under stress C. To determine whether there is positional decreased venous return to the heart D. To evaluate for peripheral vascular disease

ANS: C The head-up tilt test is used to see whether there is cardioneurogenic syncope with increased venous pooling that occurs in the upright position. This reduces the venous return to the heart. Reference: 839

5. When caring for a patient with pulmonary hypertension, which parameter is most appropriate for the nurse to monitor to evaluate the effectiveness of the treatment? a. Central venous pressure (CVP) b. Systemic vascular resistance (SVR) c. Pulmonary vascular resistance (PVR) d. Pulmonary artery wedge pressure (PAWP)

ANS: C PVR is a major contributor to pulmonary hypertension, and a decrease would indicate that pulmonary hypertension was improving. The other parameters also may be monitored but do not directly assess for pulmonary hypertension.

Which intervention will the nurse include in the plan of care for a patient who has cardiogenic shock? a. Avoid elevating head of bed. b. Check temperature every 2 hours. c. Monitor breath sounds frequently. d. Assess skin for flushing and itching.

ANS: C Since pulmonary congestion and dyspnea are characteristics of cardiogenic shock, the nurse should assess the breath sounds frequently. The head of the bed is usually elevated to decrease dyspnea. Elevated temperature and flushing or itching of the skin are not typical of cardiogenic shock. DIF: Cognitive Level: Application REF: 1721 TOP: Nursing Process: Implementation MSC: NCLEX: Physiological Integrity

What does prolonged PR interval indicate?

AV heart block

The client is diagnosed with acute pericarditis. Which sign/symptom warrants immediate attention by the nurse? 1. Muffled heart sounds.

Acute pericardial effusion interferes with normal cardiac filling and pumping, causing venous congestion and decreased cardiac output. Muffled heart sounds, indicative of acute pericarditis, must be reported to the health-care provider.

What cause asystole?

Advanced cardiac disease Severe conduction distrubance End-stage HF

A patient who is receiving dobutamine (Dobutrex) for the treatment of acute decompensated heart failure (ADHF) has the following nursing interventions included in the plan of care. Which action will be most appropriate for the registered nurse (RN) to delegate to an experienced licensed practical/vocational nurse (LPN/LVN)? c. Monitor the patient's blood pressure and heart rate every hour.

An experienced LPN/LVN would be able to monitor BP and heart rate and would know to report significant changes to the RN.

Which medication should the nurse expect the health-care provider to order for a client diagnosed with arterial occlusive disease? 3. An antiplatelet medication

Anti-platelet medications inhibit platelet aggregations in the arterial blood, such as aspirin or clopidogrel (Plavix).

During discharge teaching with a 68-year-old patient who had a mitral valve replacement with a mechanical valve, the nurse instructs the patient on the c. need for frequent laboratory blood testing.

Anticoagulation with warfarin (Coumadin) is needed for a patient with mechanical valves to prevent clotting on the valve.

What do you need too assess with PVCs

Apical-radial pulse

A 78 yo man has confusion and Temp of 104 F. He is a diabetic with purulent drainage from his right great toe. His assessment findings are BP 84/80, HR 110, RR 42 and shallow, CO 8L/min, and PAWP 4 mmHg. The pts sxs are most likely indicative of A) sepsis B) Septic shock C) Multiple organ dysfunction syndrome D) Systemic inflammatory response syndrome

B) Septic shock

The nurse provides discharge instructions for a 40-year-old woman who is newly diagnosed with cardiomyopathy. Which statement, if made by the patient, indicates that further teaching is necessary?

B. "I can drink alcohol in moderation."

The nurse performs discharge teaching for a 68-year-old man who is newly diagnosed with infective endocarditis with a history of IV substance abuse. Which statement by the patient indicates to the nurse that teaching was successful?

B. "I will inform my dentist about my hospitalization for infective endocarditis."

The nurse is assessing the client diagnosed with congestive heart failure. Which laboratory data would indicate that the client is in severe congestive heart failure? 1. An elevated B-type natriuretic peptide (BNP).

BNP is a specific diagnostic test. Levels higher than normal indicate congestive heart failure, with the higher the number, the more severe the CHF.

Which potential complication should the nurse assess for in the client with infective endocarditis who has embolization of vegetative lesions from the mitral valve? 2. Cerebrovascular accident.

Bacteria enter the bloodstream from invasive procedures and sterile platelet-fibrin vegetation forms on heart valves. The mitral valve is on the left side of the heart and, if the vegetation breaks off, it will go through the left ventricle into the systemic circulation and may lodge in the brain, kidneys, or peripheral tissues.

LDl

Bad cholesterol

Other drugs for chronic stable angina

Beta-Adrenergic blockers, calcium channel blockers, antiplatelet therapy

The client with infective endocarditis is admitted to the medical department. Which health-care provider's order should be implemented first?. 2. Obtain blood cultures times two (2).

Blood cultures must be done before administering antibiotics so that an adequate number of organisms can be obtained to culture and identify.

A 25-year-old patient with a group A streptococcal pharyngitis does not want to take the antibiotics prescribed. What should the nurse tell the patient to encourage the patient to take the medications and avoid complications of the infection?

C. "Without treatment, you could get rheumatic fever, which can lead to rheumatic heart disease."

What nursing action should the nurse prioritize during the care of a patient who has recently recovered from rheumatic fever?

C. Teach the patient about the need for continuous antibiotic prophylaxis.

PVCs cause... (3)

CO reduction Angina HF

cardiovascular causes of syncope

Common include vasovagal syncope and primary cardiac dysrhythmias.

The most accurate assessment parameters for the nurse to use to determine adequate tissue perfusion in the pt. with MODS are A) BP, HR, RR B) LS, BP, temp C) pulse pressure, LOC, and papillary response D) LOC, urine output, and skin color and temp

D) LOC, urine output, and skin color and temp

A 55-year-old female patient develops acute pericarditis after a myocardial infarction. It is most important for the nurse to assess for which clinical manifestation of a possible complication?

D. Decreased blood pressure with tachycardia

A massive gastrointestinal bleed has resulted in hypovolemic shock in an elderly patient. Which of the following is a priority nursing diagnosis? A. Acute pain B. Impaired tissue integrity C. Decreased cardiac output D. Ineffective tissue perfusion

D. Ineffective tissue perfusion Rationale: The many deleterious effects of shock are all related to inadequate perfusion and oxygenation of every body system. This nursing diagnosis supersedes the other diagnoses.

The patient with pericarditis is complaining of chest pain. After assessment, which intervention should the nurse expect to implement to provide pain relief?

D. Nonsteroidal antiinflammatory drugs

The patient with pericarditis is complaining of chest pain. After assessment, which intervention should the nurse expect to implement to provide pain relief? A. Corticosteroids B. Morphine sulfate C. Proton pump inhibitor D. Nonsteroidal antiinflammatory drugs

D. Nonsteroidal antiinflammatory drugs Nonsteroidal antiinflammatory drugs (NSAIDs) will control pain and inflammation. Corticosteroids are reserved for patients already taking corticosteroids for autoimmune conditions or those who do not respond to NSAIDs. Morphine is not necessary. Proton pump inhibitors are used to decrease stomach acid to avoid the risk of GI bleeding from the NSAIDs.

Vasocontriction

Decrease in diameter by contracting the muscle

MI pain characteristics

Described as heaviness, constriction, tightness, burning, pressure, or crushing

The nurse is caring for the client with chronic venous insufficiency. Which statement indicates that the client understands the discharge teaching? 2. "I need to elevate the foot of my bed while sleeping."

Elevating the foot of the bed while sleeping helps the venous blood return to the heart and decreases pressure in the lower extremity.

When caring for a patient with infective endocarditis of the tricuspid valve, the nurse should monitor the patient for the development of c. shortness of breath.

Embolization from the tricuspid valve would cause symptoms of pulmonary embolus.

STEMI Treatment

Emergent reperfusion therapy (PCI, Thrombolytic therapy, CABG surgery )

QRS COMPLEX: Q

First negative (downward) deflection after the P wave, short and narrow, not present in several leads. Time: <0.03 MI may result in development of a pathologic Q wave that is wide ( ≥0.03 sec) and deep ( ≥25% of the height of the R wave)

The nurse is developing a nursing care plan for a client diagnosed with congestive heart failure. A nursing diagnosis of "decreased cardiac output related to inability of the heart to pump effectively" is written. Which short-term goal would be best for the client? The client will:

Have an audible S1 and S2 with no S3 heard by end of shift.

The client diagnosed with rule-out myocardial infarction is experiencing chest pain while walking to the bathroom. Which action should the nurse implement first?

Have the client sit down immediately. - decreases the need for oxygen, may help decrease the chest pain

V. tach is caused by what? (4)

Heart disease Electrolyte imbalances Drugs CNS disorder

Second-Degree AV block Type 2 (Mobitz II) cause

Heart disease and drug toxicity

Etiology

Hypovolemic shock occurs when too little circulating blood volume causes a MAP decrease that prevents total body oxygenation. -Problems: hemorrhage (external or internal) & dehydrration

Ventricular Assist Device (VAD)

Implant that pumps into the ventricle to help with contraction

Dilation

Increase in diameter of vessel walls by relaxing the smooth muscle.

Increased renal sodium reabsorption => ______ => ______.

Increase serum osmo; Release of ADH

The client is experiencing multifocal premature ventricular contractions. Which antidysrhythmic medication would the nurse expect the health-care provider to order for this client? 1. Lidocaine.

Lidocaine suppresses ventricular ectopy and is the drug of choice for ventricular dysrhythmias.

most common cause of systolic dysfunction

MI

What causes V. fib?

MI Ischemia Disease states Procedures

A patient admitted with heart failure appears very anxious and complains of shortness of breath. Which nursing actions would be appropriate to alleviate this patient's anxiety (select all that apply)? a. Administer ordered morphine sulfate. b. Position patient in a semi-Fowler's position. d. Instruct patient on the use of relaxation techniques. e. Use a calm, reassuring approach while talking to patient.

Morphine sulfate reduces anxiety and may assist in reducing dyspnea. The patient should be positioned in semi-Fowler's position to improve ventilation that will reduce anxiety. Relaxation techniques and a calm reassuring approach will also serve to reduce anxiety.

What does elevated T wave indicate?

Myocardial injury Hyperkalemia

What does a depressed T wave indicate?

Myocardial ischemia

During the assessment of a 25-year-old patient with infective endocarditis (IE), the nurse would expect to find a new regurgitant murmur.

New regurgitant murmurs occur in IE because vegetations on the valves prevent valve closure.

Modifiable Risk factors for CAD

Obesity, tobacco use, hypertension, serum lipids, physical inactivity

Exogenous

Originate outside the body

Patients with temporary or permanent pacemakers will be ECG monitored to evaluate the status of the pacemaker.

o

Precipitating causes must be identified and treated (e.g., electrolyte imbalances, ischemia).

Holter monitoring

Recording of ECG rhythm for 24-48 hours and then correlating rhythm changes with symptoms and activities recorded in a diary

Reducing risk factors for CAD (hypertension)

Reduce salt in diet, monitor BP, take prescribed drugs, control or reduce wt.

A. fib increases risk for _______

Stroke

Atrial flutter increases risk of __________

Stroke

Endogenous

Substances normally found in the body

The nurse is preparing to administer a beta blocker to the client diagnosed with coronary artery disease. Which assessment data would cause the nurse to question administering the medication?

The client has an apical pulse of 56.

The nurse is preparing to administer digoxin to a patient with heart failure. In preparation, laboratory results are reviewed with the following findings: sodium 139 mEq/L, potassium 5.6 mEq/L, chloride 103 mEq/L, and glucose 106 mg/dL. What should the nurse do next? B Withhold the dose and report the potassium level.

The normal potassium level is 3.5 to 5.0 mEq/L.

The nurse is preparing to administer digoxin to a patient with heart failure. In preparation, laboratory results are reviewed with the following findings: sodium 139 mEq/L, potassium 5.6 mEq/L, chloride 103 mEq/L, and glucose 106 mg/dL. What should the nurse do next? b. Withhold the dose and report the potassium level.

The normal potassium level is 3.5 to 5.0 mEq/L.

Laceration A contusion is a superficial tissue injury resulting from blunt trauma, which causes the breakage of small blood vessels and bleeding into the surrounding tissues. An abrasion is a partial-thickness denudation of an area of integument, generally resulting from falls or scrapes. A puncture wound occurs when sharp or blunt objects penetrate the integument. Lacerations are open wounds resulting from sharp cutting or tearing.

The nurse documenting trauma injuries for a client with open wounds resulting from a lawnmower blade cut will document this integumentary injury as which of the following?

-Elevate the feet with the head flat or elevated 30 degrees. -Monitor vital signs every 5 minutes until they are stable -Provide oxygen therapy -Ensure IV access -Notify the Rapid Response Team

The nurse identifies signs & symptoms of internal hemorrhage in a post op patient. What's included in the care of this patient in hypovolemic shock?

Determine & treat the cause of the shock

The nurse is caring for a patient with cardiogenic shock. What's the priority for managing this patient?

Drop in BP

The nurse is caring for a patient with sepsis. What's a late clinical manifestation of shock?

Provide pain relief

The nurse, establishing a plan of care for a client with cardiogenic shock, includes which of the following in the plan of care?

Left Ventricular Remodeling

To minimize hypertrophy, will take care of problems associated with hypertrophy

While caring for a 23-year-old patient with mitral valve prolapse (MVP) without valvular regurgitation, the nurse determines that discharge teaching has been effective when the patient states that it will be necessary to d. avoid use of over-the-counter (OTC) medications that contain stimulant drugs.

Use of stimulant medications should be avoided by patients with MVP because these may exacerbate symptoms.

Polyunsaturated Fats

Use primarily. Nuts, vegetable oil, seeds, some fish oil

o

VT is associated with myocardial infarction (MI), CAD, significant electrolyte imbalances, cardiomyopathy, mitral valve prolapse, long QT syndrome, digitalis toxicity, and central nervous system disorders.

3 Negative inspiratory force is used to assess the muscle strength in a patient with positive pressure ventilation. Vital capacity, minute ventilation and rapid shallow breathing index are used to assess the muscle endurance. Test-Taking Tip: Multiple-choice questions can be challenging because students think that they will recognize the right answer when they see it or that the right answer will somehow stand out from the other choices. This is a dangerous misconception. The more carefully the question is constructed, the more each of the choices will seem like the correct response. Text Reference - p. 1626

What measurement is used to assess muscle strength in a patient with positive pressure ventilation? 1 Vital capacity 2 Minute ventilation 3 Negative inspiratory force 4 Rapid shallow breathing index

Pallor or cyanosis in the mucous membranes

Which change in the skin is an early indication of hypovolemic shock?

Patient with severe vomiting & large watery diarrheal stools.

Which patient is most likely to show elevated hemoglobin & hematocrit during shock?

Patient with a recent MI Patient with a history of CHF

Which patients are at risk for cardiogenic shock?

-Shock is a whole body response to body tissues not receiving enough oxygen. -Shock is widespread abnormal cellular metabolism. -Shock may occur in older adults in response to UTIs

Which statements about shock are true?

• Clinical manifestations of neurogenic shock include _

_hypotension, bradycardia, temperature dysregulation (resulting in heat loss), dry skin, and poikilothermia.

• Clinical manifestations of cardiogenic shock include _

_tachycardia, hypotension, a narrowed pulse pressure, tachypnea, pulmonary congestion, cyanosis, pallor, cool and clammy skin, diaphoresis, decreased capillary refill time, anxiety, confusion, and agitation.

ECG (i.e., rhythm strip) provides

a record of the patient's rhythm allowing for measurement of complexes and intervals and for assessment of dysrhythmias.

when increased abdominal pressures compress the inferior vena cava

abdominal compartment syndrome

called dysrhythmias

abnormal cardiac rhythms

Isoelectric (flat) line or baseline represents the

absence of electrical activity in the cardiac cells.

An EPS study can also locate

accessory pathways and determine the effectiveness of antidysrhythmia drugs.

The spontaneously firing (discharge) of electrical impulse...

automaticity.

Decreased BP => Activation of sympathetic nervous system => ______ coronary vasodilation and ______.

b-Adrenergic stimulation; Increase cardiac output

A 78 y.o. man has confusion and a temp of 104 F. He is a diabetic with purulent drainage from his right heel. After an infusion of 3L of normal saline solution, his assessment findings are BP 84/40, HR 110, RR 42/shallow, CO 8L/min and PAWP 4. This patient's symptoms are most likely indicative ofL a. sepsis b. septic shock c. MODS d. SIRS

b. septic shock (remember - a high CO is indicative of Sepsis, and a low PAWP is indicative of hypovolemia)

The nurse is caring for a 34-year-old woman with acute decompensated heart failure who has a pulmonary artery catheter. Which assessment best indicates that the patient's condition is improving? a Cardiac output (CO) is 3.5 L/minute. b Central venous pressure (CVP) is 10 mm Hg. c Pulmonary artery wedge pressure (PAWP) is 10 mm Hg. d Systemic vascular resistance (SVR) is 1500 dynes/sec/cm-5.

c Pulmonary artery wedge pressure (PAWP) is 10 mm Hg. PAWP is the most sensitive indicator of cardiac function and fluid volume status. Normal range for PAWP is 6 to 12 mm Hg. PAWP is increased in heart failure. Normal range for CVP is 2 to 8 mm Hg. An elevated CVP indicates right-sided heart failure or volume overload. Normal cardiac output is 4 to 8 L/minute. CO is decreased in heart failure. SVR is increased in left-sided heart failure. Normal SVR is 800 to 1200 dynes/sec/cm-5.

After having an MI, the nurse notes the patient has jugular venous distention, gained weight, developed peripheral edema, and has a heart rate of 108/minute. What should the nurse suspect is happening? a. ADHF b. Chronic HF c. Left-sided HF d. Right-sided HF

d. Right-sided HF An MI is a primary cause of heart failure. The jugular venous distention, weight gain, peripheral edema, and increased heart rate are manifestations of right-sided heart failure.

With ventricular tachycardia _____________ _______ take over as pacemaker

ectopic focii

Short acting nitrates

first line of defense for chronic stable angina

•Sinus tachycardia

has a normal sinus rhythm, but the SA node fires at a rate 101-200 beats/min because of vagal inhibition or sympathetic stimulation.

Outside the cell a

high concentration of sodium and a low concentration of potassium exist.

common lab finding in shock

lactate elevation

Telemetry monitoring is the

observation of a patient's HR and rhythm at a site distant from the patient.

QT interval

represents the total time for depolarization and repolarization of the ventricles.

Goals of Treatment for Chronic Heart Failure

~ Relieving the symptoms & improving the quality of life ~ Long-term goal: slowing, halting, or reversing the cardiac dysfunction

ACE Inhibitors (Angiotensin Converting Enzyme)

Block the conversion of angiotensin 1, will be converted into angiotensin 2 & the conversion is with angiotensin converting enzyme (ACE), will contribute to edema - will stop the build up

Colloid Human serum albumin (5%), plasma protein fraction (5% albumin in 500 mL NSS) - Mechanism of Action

Can increase plasma colloid osmotic pressure Rapid volume expander

Which client would the nurse suspect of having a mitral valve prolapse? 2. A 23-year-old male with Marfan syndrome.

Clients with Marfan syndrome have lifethreatening cardiovascular problems, including mitral valve prolapse,

While doing an admission assessment, the nurse notes clubbing of the patient's fingers. Based on this finding, the nurse will question the patient about which disease process? A. Endocarditis

Clubbing of the fingers is a loss of the normal angle between the base of the nail and the skin. This finding can be found in endocarditis, congenital defects, and/or prolonged oxygen deficiency.

Four properties of cardiac cells

(automaticity, excitability, conductivity, and contractility) enable the conduction system to initiate an electrical impulse, transmit it through the cardiac tissue, and stimulate the myocardial tissue to contract.

Surgical Management

-Vascular repair, surgical hemostasis of major wounds, closure of bleeding ulcers, & chemical scarring (chemosclerosis) of varicosities.

What is the cornerstone of therapy for septic shock?

-Volume expansion with the administration of intravenous fluids.

Septic shock is characterized by-

-a decreased circulating blood volume.

Nitrates and β-adrenergic blockers are most often used in the treatment of -

-patients in cardiogenic shock.

Valvular heart disease, diagnostic stiudies

...

What causes gut bacteria to move into circulation in patients with systemic inflammatory response syndrome (SIRS)? 1 Toxic effects of medicines 2 Effect of inflammatory mediators 3 Decreased gastrointestinal motility 4 Decreased perfusion of gut mucosa

4 Decreased perfusion in the gastrointestinal (GI) tract leads to a breakdown of the normally protective mucosal barrier, which causes the bacterial movement from the GI tract into circulation. In order to control these bacteria, antibiotics are administered. Antibiotics are nephrotoxic medicines that can cause acute kidney injury. The breakdown of the mucosal barrier is the direct effect of hypoperfusion rather than the inflammatory mediators. In critical illnesses, GI motility is often decreased causing abdominal distension and paralytic ileus. Test-Taking Tip: Make certain that the answer you select is reasonable and obtainable under ordinary circumstances and that the action can be carried out in the given situation. Text Reference - p. 1649

A massive gastrointestinal bleed has resulted in hypovolemic shock in an older patient. What is a priority nursing diagnosis? 1 Acute pain 2 Impaired tissue integrity 3 Decreased cardiac output 4 Ineffective tissue perfusion

4 The many deleterious effects of shock are all related to inadequate perfusion and oxygenation of every body system. Ineffective tissue perfusion supersedes acute pain, impaired tissue integrity, and decreased cardiac output, because circulation is a priority. Acute pain may occur, but is not a priority at this time. Tissue integrity is not impaired. Text Reference - p. 1633

LDL

<100 mg/dL

HDL Men

>40 mg/dL

HDL women

>50 mg/dL

1. To determine whether there is a delay in impulse conduction through the atria, the nurse will measure the duration of the patient's a. P wave. b. Q wave. c. P-R interval. d. QRS complex.

A The P wave represents the depolarization of the atria. The P-R interval represents depolarization of the atria, atrioventricular (AV) node, bundle of His, bundle branches, and the Purkinje fibers. The QRS represents ventricular depolarization. The Q wave is the first negative deflection following the P wave and should be narrow and short. DIF: Cognitive Level: Understand (comprehension) REF: 791 TOP: Nursing Process: Assessment MSC:

The nurse is preparing to administer a beta blocker to the client diagnosed with coronary artery disease. Which assessment data would cause the nurse to question administering the medication? 2. The client has an apical pulse of 56.

A beta blocker decreases sympathetic stimulation to the heart, thereby decreasing the heart rate. An apical rate less than 60 indicates lower-than-normal heart rate and should make the nurse question administering this medication because it will further decrease the heart rate.

Increase in HR Increase in respiratory rate

A patient is brought to the ED with a gunshot wound. The nurse closely monitors the patient for early signs of hypovolemic shock. What early sign does the nurse look for?

A patient is admitted to the emergency department (ED) for shock of unknown etiology. The first action by the nurse should be to a. administer oxygen. b. obtain a 12-lead electrocardiogram (ECG). c. obtain the blood pressure. d. check the level of consciousness.

ANS: A The initial actions of the nurse are focused on the ABCs—airway, breathing, and circulation—and administration of oxygen should be done first. The other actions should be accomplished as rapidly as possible after oxygen administration.

Increased angiotensin 2 => ______ => Increased renal sodium reabsorption.

Aldosterone secretion

What can cause a possible variation in P wave?

Disturbance in conduction within atria

Premature ventricular contractions (PVCs) caused by

Stimulants Electrolyte imbalances Hypoxia Heart disease

ANS: A, B, D Heart and respiratory rates increased from the client's baseline level and a slight increase in diastolic blood pressure may be the only objective manifestations of this early stage of shock. pg 813

The nurse is assessing a client who is in early stages of hypovolemic shock. Which manifestations does the nurse expect? (Select all that apply.) a. Elevated heart rate b. Elevated diastolic blood pressure c. Decreased body temperature d. Elevated respiratory rate e. Decreased pulse rate

A patient recovering from heart surgery develops pericarditis and complains of level 6 (0 to 10 scale) chest pain with deep breathing. Which ordered PRN medication will be the most appropriate for the nurse to give? c. Oral ibuprofen (Motrin) 600 mg

The pain associated with pericarditis is caused by inflammation, so nonsteroidal antiinflammatory drugs (NSAIDs) (e.g., ibuprofen) are most effective.

Severe sepsis is _

_sepsis complicated by organ dysfunction.

six leads measures forces in the frontal plane and are

bipolar and unipolar.

Atherosclerosis

lipid deposits within the intima of the artery. Endothelial injury and inflammation play a central role in the development (tobacco use, hypertension, diabetes)

The elderly client has coronary artery disease. Which question should the nurse ask the client during the client teaching?

"Are you sexually active?" - sexual activity is a risk factor for angina from coronary disease BUT client's being elderly should not affect teaching about sexual activity

Complications of invasive temporary

(i.e., transvenous) or permanent pacemaker insertion include infection and hematoma formation at the site of insertion of the pacemaker power source or leads, pneumothorax, failure to sense or capture with possible symptomatic bradycardia, perforation of the atrial or ventricular septum by the pacing lead, and appearance of "end-of-life" battery parameters on testing the pacemaker.

Aortic valve stenosis, etiology

-found in childhood or young adulthood -older adults= RF, calcification of valve leaflets, accompanied by mitral valve disease

What happens in neurogenic shock?

-spinal cord injury leads to loss of sympathetic tone, resulting in massive vasodilation.

Certification in critical care nursing by the American Association of Critical Care Nurses indicates that the nurse a. is an advanced practice nurse who cares for acutely and critically ill patients. b. may practice independently to provide symptoms management for the critically ill c. has earned a master's degree in the field of advanced acute and critical care nursing d. has practiced in critical care and successfully completed a test critical care knowledge

1. Correct answer: d Rationale: Certification in critical care nursing (CCRN) by the American Association of Critical-Care Nurses requires registered nurse licensure, practice experience in critical or progressive care nursing, and successful completion of a written test.

2. A patient has an admitting diagnosis of acute left-sided infective endocarditis. What is the best test to confirm this diagnosis? a. Blood cultures b. Complete blood count c. Cardiac catheterization d. Transesophageal echocardiogram

2. a. Blood cultures are the primary diagnostic tool for infective endocarditis. Although a complete blood count (CBC) will reveal a mild leukocytosis, this is a nonspecific finding. Transesophageal echocardiograms can identify vegetations on valves but are used when blood cultures are negative. Cardiac catheterizations are used when surgical intervention is being considered.

0.9% sodium chloride

A patient comes to the ED with severe injury & significant blood loss. The nurse anticipates that resuscitation will begin with which fluid?

Increased serum lactate level & rising band neutrophils

A patient has a systemic infection with a fever, increased respiratory rate, & change in mental status. Which lab values does the nurse seek out that are considered "hallmark" of sepsis?

Anaphylactic

A patient has cardiac dysrhythmias & pulmonary problems as a result of receiving an IV antibiotic. What type of shock does this represent?

Assess BP at least every 15 minutes because systemic vasodilation can cause hypotension.

A patient in hypovolemic shock is receiving sodium nitroprusside (Nitropress) to enhance myocardial perfusion. What's an important nursing implication for administering this drug?

The patient had myocarditis and is now experiencing fatigue, weakness, palpitations, and dyspnea at rest. The nurse assesses pulmonary crackles, edema, and weak peripheral pulses. Sinoatrial tachycardia is evident on the cardiac monitor. The Doppler echocardiography shows dilated cardiomyopathy. What collaborative and nursing care of this patient should be done to improve cardiac output and the quality of life? (Select all that apply.)

A. Decrease preload and afterload. C. Control heart failure by enhancing myocardial contractility.

While doing an admission assessment, the nurse notes clubbing of the patient's fingers. Based on this finding, the nurse will question the patient about which disease process? A. Endocarditis B. Acute kidney injury C. Myocardial infarction D. Chronic thrombophlebitis

A. Endocarditis Clubbing of the fingers is a loss of the normal angle between the base of the nail and the skin. This finding can be found in endocarditis, congenital defects, and/or prolonged oxygen deficiency. Clinical manifestations of acute kidney injury, myocardial infarction, and chronic thrombophlebitis will not include clubbing of the fingers.

When caring for a patient with infective endocarditis, the nurse will assess the patient for which vascular manifestations (select all that apply)? A. Osler's nodes B. Janeway's lesions C. Splinter hemorrhages D. Subcutaneous nodules E. Erythema marginatum lesions

A. Osler's nodes B. Janeway's lesions C. Splinter hemorrhages Osler's nodes, Janeway's lesions, and splinter hemorrhages are all vascular manifestations of infective endocarditis. Subcutaneous nodules and erythema marginatum lesions occur with rheumatic fever.

While admitting a patient with pericarditis, the nurse will assess for what manifestations of this disorder?

A. Pulsus paradoxus

1. A patient's vital signs are pulse 87, respirations 24, and BP of 128/64 mm Hg and cardiac output is 4.7 L/min. The patient's stroke volume is _____ mL. (Round to the nearest whole number.)

ANS: 54 Stroke volume = cardiac output/heart rate DIF: Cognitive Level: Understand (comprehension) REF: 1603 TOP: Nursing Process: Assessment MSC: NCLEX: Physiological Integrity

Which action should the nurse include in the plan of care when caring for a patient admitted with acute decompensated heart failure (ADHF) who is receiving nesiritide (Natrecor)? a. Monitor blood pressure frequently. b. Encourage patient to ambulate in room. c. Titrate nesiritide slowly before stopping. d. Teach patient about home use of the drug.

ANS: A Nesiritide is a potent arterial and venous dilator, and the major adverse effect is hypotension. Because the patient is likely to have orthostatic hypotension, the patient should not be encouraged to ambulate. Nesiritide does not require titration and is used for ADHF but not in a home setting.

The nurse suspects cardiac tamponade in a patient who has acute pericarditis. To assess for the presence of pulsus paradoxus, the nurse should a. note when Korotkoff sounds are auscultated during both inspiration and expiration. b. subtract the diastolic blood pressure (DBP) from the systolic blood pressure (SBP). c. check the electrocardiogram (ECG) for variations in rate during the respiratory cycle. d. listen for a pericardial friction rub that persists when the patient is instructed to stop breathing.

ANS: A Pulsus paradoxus exists when there is a gap of greater than 10 mm Hg between when Korotkoff sounds can be heard during only expiration and when they can be heard throughout the respiratory cycle. The other methods described would not be useful in determining the presence of pulsus paradoxus.

Which data collected by the nurse caring for a patient who has cardiogenic shock indicate that the patient may be developing multiple organ dysfunction syndrome (MODS)? a. The patient's serum creatinine level is elevated. b. The patient complains of intermittent chest pressure. c. The patient's extremities are cool and pulses are weak. d. The patient has bilateral crackles throughout lung fields.

ANS: A The elevated serum creatinine level indicates that the patient has renal failure as well as heart failure. The crackles, chest pressure, and cool extremities are all consistent with the patient's diagnosis of cardiogenic shock

Which data collected by the nurse caring for a patient who has cardiogenic shock indicate that the patient may be developing multiple organ dysfunction syndrome (MODS)? a. The patient's serum creatinine level is elevated. b. The patient complains of intermittent chest pressure. c. The patient's extremities are cool and pulses are weak. d. The patient has bilateral crackles throughout lung fields.

ANS: A The elevated serum creatinine level indicates that the patient has renal failure as well as heart failure. The crackles, chest pressure, and cool extremities are all consistent with the patient's diagnosis of cardiogenic shock.

A nurse is caring for a patient with shock of unknown etiology whose hemodynamic monitoring indicates BP 92/54, pulse 64, and an elevated pulmonary artery wedge pressure. Which collaborative intervention ordered by the health care provider should the nurse question? a. Infuse normal saline at 250 mL/hr. b. Keep head of bed elevated to 30 degrees. c. Hold nitroprusside (Nipride) if systolic BP <90 mm Hg. d. Titrate dobutamine (Dobutrex) to keep systolic BP >90 mm Hg.

ANS: A The patient's elevated pulmonary artery wedge pressure indicates volume excess. A saline infusion at 250 mL/hr will exacerbate the volume excess. The other actions are appropriate for the patient

A patient with suspected neurogenic shock after a diving accident has arrived in the emergency department. A cervical collar is in place. Which actions should the nurse take (select all that apply)? a. Prepare to administer atropine IV. b. Obtain baseline body temperature. c. Infuse large volumes of lactated Ringer's solution. d. Provide high-flow oxygen (100%) by non-rebreather mask. e. Prepare for emergent intubation and mechanical ventilation.

ANS: A, B, D, E All of the actions are appropriate except to give large volumes of lactated Ringer's solution. The patient with neurogenic shock usually has a normal blood volume, and it is important not to volume overload the patient. In addition, lactated Ringer's solution is used cautiously in all shock situations because the failing liver cannot convert lactate to bicarbonate.

The patient has a pacemaker set for 70 beats/minute. When taking the patient's pulse, you obtain a heart rate of 60 beats/minute. What is the best interpretation of this finding? A. The patient's heart has become more effective. B. The pacemaker is not working properly. C. The patient is tolerating a lower heart rate now. D. The pacemaker is sensing a ventricular rhythm.

ANS: B If the pacemaker is set for 70 beats/minute, a slower rate means that the pacemaker is not working properly and the patient's spontaneous rate has taken over. This situation needs to be evaluated. Reference: 836-837

The emergency department patient is in paroxysmal supraventricular tachycardia (PSVT) at a rate of 170 beats/minute. Which treatment do you anticipate first? A. Sotalol (Betapace) by slow IVP B. Adenosine (Adenocard) by fast IVP C. Defibrillation D. Digoxin (Lanoxin)

ANS: B PSVT is a dysrhythmia originating in an ectopic focus anywhere above the bifurcation of the bundle of His. Treatment includes vagal stimulation (e.g., Valsalva maneuver, coughing) and intravenous (IV) adenosine as the first drug of choice. The drug has a short half-life and is given rapid IVP. Other drugs are β-adrenergic blockers, calcium channel blockers, and amiodarone. Defibrillation is used if the vagal stimulation and drug therapy are ineffective and the patient becomes hemodynamically unstable. Digoxin is not used for this dysrhythmia but typically is used for atrial fibrillation. Reference: 826

The patient has a permanent cardiac pacemaker. On the electrocardiographic tracing, you notice a spike before each P wave. What action should you take? A. Assess the patient for syncope. B. Document the findings. C. Notify the physician. D. Take blood pressure in both arms.

ANS: B These pacer spikes show that the pacemaker is firing and the atrium is responding to the impulse. It is a normal, expected finding in this situation. Reference: 834-835

Which of these findings is the best indicator that the fluid resuscitation for a patient with hypovolemic shock has been successful? a. Hemoglobin is within normal limits. b. Urine output is 60 mL over the last hour. c. Pulmonary artery wedge pressure (PAWP) is normal. d. Mean arterial pressure (MAP) is 65 mm Hg.

ANS: B Assessment of end organ perfusion, such as an adequate urine output, is the best indicator that fluid resuscitation has been successful. The hemoglobin level, PAWP, and MAP are useful in determining the effects of fluid administration, but they are not as useful as data indicating good organ perfusion. DIF: Cognitive Level: Application REF: 1733-1735 TOP: Nursing Process: Evaluation MSC: NCLEX: Physiological Integrity

The nurse is caring for a patient who has septic shock. Which assessment finding is most important for the nurse to report to the health care provider? a. Blood pressure (BP) 92/56 mm Hg b. Skin cool and clammy c. Oxygen saturation 92% d. Heart rate 118 beats/minute

ANS: B Because patients in the early stage of septic shock have warm and dry skin, the patient's cool and clammy skin indicates that shock is progressing. The other information will also be reported, but does not indicate deterioration of the patient's status

The nurse is caring for a patient who has septic shock. Which assessment finding is most important for the nurse to report to the health care provider? a. Blood pressure (BP) 92/56 mm Hg b. Skin cool and clammy c. Oxygen saturation 92% d. Heart rate 118 beats/minute

ANS: B Because patients in the early stage of septic shock have warm and dry skin, the patient's cool and clammy skin indicates that shock is progressing. The other information will also be reported, but does not indicate deterioration of the patient's status.

The nurse is caring for a patient who has septic shock. Which assessment finding is most important for the nurse to report to the health care provider? a. Blood pressure (BP) 92/56 mm Hg b. Skin cool and clammy c. Oxygen saturation 92% d. Heart rate 118 beats/minute

ANS: B Because patients in the early stage of septic shock have warm and dry skin, the patient's cool and clammy skin indicates that shock is progressing. The other information will also be reported, but does not indicate deterioration of the patient's status.

Which information about a patient who is receiving vasopressin (Pitressin) to treat septic shock is most important for the nurse to communicate to the heath care provider? a. The patient's heart rate is 108 beats/min. b. The patient is complaining of chest pain. c. The patient's peripheral pulses are weak. d. The patient's urine output is 15 mL/hr.

ANS: B Because vasopressin is a potent vasoconstrictor, it may decrease coronary artery perfusion. The other information is consistent with the patient's diagnosis and should be reported to the health care provider but does not indicate a need for a change in therapy. DIF: Cognitive Level: Application REF: 1735-1736 OBJ: Special Questions: Prioritization TOP: Nursing Process: Assessment MSC: NCLEX: Physiological Integrity

1. A 68-year-old patient has been in the intensive care unit for 4 days and has a nursing diagnosis of disturbed sensory perception related to sleep deprivation. Which action should the nurse include in the plan of care? a. Administer prescribed sedatives or opioids at bedtime to promote sleep. b. Cluster nursing activities so that the patient has uninterrupted rest periods. c. Silence the alarms on the cardiac monitors to allow 30- to 40-minute naps. d. Eliminate assessments between 0100 and 0600 to allow uninterrupted sleep.

ANS: B Clustering nursing activities and providing uninterrupted rest periods will minimize sleep-cycle disruption. Sedative and opioid medications tend to decrease the amount of rapid eye movement (REM) sleep and can contribute to sleep disturbance and disturbed sensory perception. Silencing the alarms on the cardiac monitors would be unsafe in a critically ill patient, as would discontinuing assessments during the night. DIF: Cognitive Level: Apply (application) REF: 1601 TOP: Nursing Process: Planning MSC: NCLEX: Psychosocial Integrity

The nurse is assessing a patient with myocarditis before administering the scheduled dose of digoxin (Lanoxin). Which finding is most important for the nurse to communicate to the health care provider? a. Leukocytosis b. Irregular pulse c. Generalized myalgia d. Complaint of fatigue

ANS: B Myocarditis predisposes the heart to digoxin-associated dysrhythmias and toxicity. The other findings are common symptoms of myocarditis and there is no urgent need to report these.

29. When caring for a patient who has an arterial catheter in the left radial artery for arterial pressure-based cardiac output (APCO) monitoring, which information obtained by the nurse is most important to report to the health care provider? a. The patient has a positive Allen test. b. There is redness at the catheter insertion site. c. The mean arterial pressure (MAP) is 86 mm Hg. d. The dicrotic notch is visible in the arterial waveform.

ANS: B Redness at the catheter insertion site indicates possible infection. The Allen test is performed before arterial line insertion, and a positive test indicates normal ulnar artery perfusion. A MAP of 86 is normal and the dicrotic notch is normally present on the arterial waveform.

After receiving the following information about four patients during change-of-shift report, which patient should the nurse assess first? a. Patient with acute pericarditis who has a pericardial friction rub b. Patient who has just returned to the unit after balloon valvuloplasty c. Patient who has hypertrophic cardiomyopathy and a heart rate of 116 d. Patient with a mitral valve replacement who has an anticoagulant scheduled

ANS: B The patient who has just arrived after balloon valvuloplasty will need assessment for complications such as bleeding and hypotension. The information about the other patients is consistent with their diagnoses and does not indicate any complications or need for urgent assessment or intervention.

When the nurse is assessing a patient who is receiving a nitroprusside (Nipride) infusion to treat cardiogenic shock, which finding indicates that the medication is effective? a. No heart murmur is audible. b. Skin is warm, pink, and dry. c. Troponin level is decreased. d. Blood pressure is 90/40 mm Hg.

ANS: B Warm, pink, and dry skin indicates that perfusion to tissues is improved. Since nitroprusside is a vasodilator, the blood pressure may be low even if the medication is effective. Absence of a heart murmur and a decrease in troponin level are not indicators of improvement in shock. DIF: Cognitive Level: Application REF: 1721 | 1723 | 1733-1735 TOP: Nursing Process: Evaluation MSC: NCLEX: Physiological Integrity

When caring for a patient with infective endocarditis of the tricuspid valve, the nurse should monitor the patient for the development of a. flank pain. b. splenomegaly. c. shortness of breath. d. mental status changes.

ANS: C Embolization from the tricuspid valve would cause symptoms of pulmonary embolus. Flank pain, changes in mental status, and splenomegaly would be associated with embolization from the left-sided valves.

When the charge nurse is evaluating the skills of a new RN, which action by the new RN indicates a need for more education in the care of patients with shock? a. Placing the pulse oximeter on the ear for a patient with septic shock b. Keeping the head of the bed flat for a patient with hypovolemic shock c. Decreasing the room temperature to 68° F for a patient with neurogenic shock d. Increasing the nitroprusside (Nipride) infusion rate for a patient with a high SVR

ANS: C Patients with neurogenic shock may have poikilothermia. The room temperature should be kept warm to avoid hypothermia. The other actions by the new RN are appropriate. DIF: Cognitive Level: Application REF: 1721-1722 | 1724 OBJ: Special Questions: Delegation TOP: Nursing Process: Evaluation MSC: NCLEX: Safe and Effective Care Environment

The nurse establishes the nursing diagnosis of ineffective health maintenance related to lack of knowledge regarding long-term management of rheumatic fever when a 30-year-old recovering from rheumatic fever without carditis says which of the following? a. "I will need prophylactic antibiotic therapy for 5 years." b. "I will need to take aspirin or ibuprofen (Motrin) to relieve my joint pain." c. "I will call the doctor if I develop excessive fatigue or difficulty breathing." d. "I will be immune to further episodes of rheumatic fever after this infection."

ANS: D Patients with a history of rheumatic fever are more susceptible to a second episode. Patients with rheumatic fever without carditis require prophylaxis until age 20 and for a minimum of 5 years. The other patient statements are correct and would not support the nursing diagnosis of ineffective health maintenance.

Conductivity

Ability to transmit an impulse along a membrane in an orderly manner

ECG manifestation of junctional dysrhythmia

Abnormal P wave Normal QRS complex

Cheyne Strokes

Abnormal pattern of breathing characterized by progressively deeper & sometimes faster breathing, followed by a gradual decrease that results in a temporary stop in breathing called an apnea. This pattern repeats with each cycle usually taking 30 seconds to 2 minutes

What is the priority assessment by the nurse caring for a patient receiving IV nesiritide (Natrecor) to treat heart failure? c. Blood pressure

Although all identified assessments are appropriate for a patient receiving IV nesiritide, the priority assessment would be monitoring for hypotension, the main adverse effect of nesiritide.

Homocysteine

Amino acid produced during protein catabolism that has been identified as a risk factor for CV disease

atrial tachydysrhythmia

Atrial flutter identified by recurring, regular, sawtooth-shaped flutter waves that originate from a single ectopic focus in the right atrium. Patients with atrial flutter are at increased risk of stroke.

Second-Degree AV block Type 1 (Mobitz I, Wanckebach) treatment

Atropine Pacemaker

Junctional dysrhythmia treatment (3)

Atropine for escape rhythm Correct cause Drugs to reduce rate if tachycardia

The home care nurse visits a 73-year-old Hispanic woman with chronic heart failure. Which clinical manifestations, if assessed by the nurse, would indicate acute decompensated heart failure (pulmonary edema)? A Fatigue, orthopnea, and dependent edema B Severe dyspnea and blood-streaked, frothy sputum C Temperature is 100.4o F and pulse is 102 beats/minute D Respirations 26 breaths/minute despite oxygen by nasal cannula

B Severe dyspnea and blood-streaked, frothy sputum Clinical manifestations of pulmonary edema include anxiety, pallor, cyanosis, clammy and cold skin, severe dyspnea, use of accessory muscles of respiration, a respiratory rate > 30 breaths per minute, orthopnea, wheezing, and coughing with the production of frothy, blood-tinged sputum. Auscultation of the lungs may reveal crackles, wheezes, and rhonchi throughout the lungs. The heart rate is rapid, and blood pressure may be elevated or decreased.

A patient's localized infection has progressed to the point where septic shock is now suspected. Which of the following is an appropriate treatment modality for this patient? A. Insulin infusion B. Aggressive fluid resuscitation C. Intravenous administration of epinephrine D. Administration of nitrates and β-adrenergic blockers

B. Aggressive fluid resuscitation Rationale: Patients in septic shock require large amounts of fluid replacement. Nitrates and β-adrenergic blockers are most often used in the treatment of patients in cardiogenic shock. Epinephrine is indicated in anaphylactic shock, and insulin infusion is not normally necessary in the treatment of septic shock (but can be).

When caring for a patient in acute septic shock, the nurse would anticipate... A. Administering osmotic and/or loop diuretics. B. Infusing large amounts of intravenous fluids. C. Administering intravenous diphenhydramine (Benadryl). D. Assisting with insertion of a ventricular assist device (VAD).

B. Infusing large amounts of intravenous fluids. Rational: Septic shock is characterized by a decreased circulating blood volume. Volume expansion with the administration of intravenous fluids is the cornerstone of therapy. The administration of diuretics is inappropriate. VADs are useful for cardiogenic shock, not septic shock. Diphenhydramine (Benadryl) may be used for anaphylactic shock, but would not be helpful with septic shock.

A 25-year-old patient with a group A streptococcal pharyngitis does not want to take the antibiotics prescribed. What should the nurse tell the patient to encourage the patient to take the medications and avoid complications of the infection? A. "The complications of this infection will affect the skin, hair, and balance." B. "You will not feel well if you do not take the medicine and get over this infection." C. "Without treatment, you could get rheumatic fever, which can lead to rheumatic heart disease." D. "You may not want to take the antibiotics for this infection, but you will be sorry if you do not."

C. "Without treatment, you could get rheumatic fever, which can lead to rheumatic heart disease." Rheumatic fever (RF) is not common because of effective use of antibiotics to treat streptococcal infections. Without treatment, RF can occur and lead to rheumatic heart disease, especially in young adults. The complications do not include hair or balance. Saying that the patient will not feel well or that the patient will be sorry if the antibiotics are not taken is threatening to the patient and inappropriate for the nurse to say.

OBSTRUCTIVE SHOCK

Caused by problems that impair the ability of the normal heart muscle to pump effectively. The heart itself remains normal, but conditions outside the heart prevent either adequate filling of the heart or adequate contraction of the healthy heart muscle. -Most common causes: pericarditis & cardiac tamponade

Continuous ECG monitoring is done using a combination of leads II, V1, and MCL1.

A 55-year-old female patient develops acute pericarditis after a myocardial infarction. It is most important for the nurse to assess for which clinical manifestation of a possible complication? A. Presence of a pericardial friction rub B. Distant and muffled apical heart sounds C. Increased chest pain with deep breathing D. Decreased blood pressure with tachycardia

D. Decreased blood pressure with tachycardia Cardiac tamponade is a serious complication of acute pericarditis. Signs and symptoms indicating cardiac tamponade include narrowed pulse pressure, tachypnea, tachycardia, a decreased cardiac output, and decreased blood pressure. The other symptoms are consistent with acute pericarditis.

An 80-year-old patient with uncontrolled type 1 diabetes mellitus is diagnosed with aortic stenosis. When conservative therapy is no longer effective, the nurse knows that the patient will need to do or have what done?

D. Percutaneous transluminal balloon valvuloplasty (PTBV) procedure

An 80-year-old patient with uncontrolled type 1 diabetes mellitus is diagnosed with aortic stenosis. When conservative therapy is no longer effective, the nurse knows that the patient will need to do or have what done? A. Aortic valve replacement B. Take nitroglycerin for chest pain. C. Open commissurotomy (valvulotomy) procedure D. Percutaneous transluminal balloon valvuloplasty (PTBV) procedure

D. Percutaneous transluminal balloon valvuloplasty (PTBV) procedure

Which clinical finding would most likely indicate decreased cardiac output in a patient with aortic valve regurgitation?

D. shortnedd of breath on minimal exertion and a diastolic murmur

Chronic stable angina treatment

Decrease O2 demand and/or increase O2 supply: Short acting nitrates and Long-acting nitrates (nitroglycerin)

Third Degree (complete heart block) results in (increased or decreased) CO

Decreased

Decreased BP => ______ => Fluid movement into intravascular space.

Decreased capillary hydrostatic pressure

New technology using smart phones can obtain and save

ECG recordings and even detect atrial fibrillation.

All patients with septic shock develop "MODS". (T/F)

False SIRS

Cardiogenic shock is characterized by increased systemic vascular resistance, decreased CO, and decreased pulmonary artery wedge pressure. (T /F)

False hypovolemic shock

Distributive

Fluid shifted from central vascular space

The client has just returned from a cardiac catheterization. Which assessment data would warrant immediate intervention from the nurse? 3. The client refuses to keep the leg straight.

If the client bends the leg, it could cause the insertion site to bleed. This is arterial blood and the client could bleed to death very quickly, so this requires immediate intervention.

Beta Blockers

Inhibits sympathetic nervous stimulation of the heart. Decreases BP, HR, and contractility. Reduce ischemia. Decrease afterload.

QT Interval (ventricles)

Measured from beginning of QRS complex to end of T wave. Represents time taken for entire electrical depolarization and repolarization of the ventricles. Time: 0.34-0.43 Disturbances usually affecting repolarization more than depolarization and caused by drugs, electrolyte imbalances, and changes in heart rate

When caring for a patient with infective endocarditis, the nurse will assess the patient for which vascular manifestations (select all that apply)? A. Osler's nodes B. Janeway's lesions C. Splinter hemorrhages

Osler's nodes, Janeway's lesions, and splinter hemorrhages are all vascular manifestations of infective endocarditis.

What are Paroxysmal Supraventricular Tachycardias (PSVTs) associated with? (6)

Overexertion Stress Deep inspiration Stimulants Disease Digitalis toxicity

Review of oxygenation & tissue perfusion

Oxygenation & perfusion depend on how much oxygen from arterial blood perfuses the tissue. Tissue & organ perfusion is related to mean arterial pressure (MAP)

To assess the patient with pericarditis for evidence of a pericardial friction rub, the nurse should b. auscultate by placing the diaphragm of the stethoscope on the lower left sternal border.

Pericardial friction rubs are heard best with the diaphragm at the lower left sternal border.

Right Heart Failure

Represents failure of the right heart to pump blood forward into the pulmonary circulation. Blood backs up in the systemic circulation. ~ Causes peripheral edema and congestion of the abdominal organs, liver congestion, because blood is backing up to systemic circulation

The 66-year-old male client has his blood pressure (BP) checked at a health fair. The B/P is 168/98. Which action should the nurse implement first? Instruct the client to see his health-care provider as soon as possible.

The diastolic blood pressure should be less than 85 according to the American Heart Association; therefore, this client should see the health-care provider.

Increased urinary output

The nurse is evaluating the care & treatment for a patient in shock. Which finding indicates that the patient is having an appropriate response to the treatment?

Collaborative Care: Septic shock_• Vasopressor drug therapy may be added _

_once CVP is ≥8 mm Hg; vasopressin may be given to patients refractory to vasopressor therapy.

A patient with hypovolemic shock is receiving lactated Ringer's for fluid replacement therapy. During this therapy, it is most important for the nurse to monitor the patient's a. serum pH b. serum sodium c. serum potassium d. H/H

a, serum pH LR solution may increase lactate levels, which a damaged liver cannot convert to bicarbonate, and may intensify the metabolic lactic acidosis that occurs in progressive shock, necessitating careful attention to the patients acid-base balance. Sodium and potassium levels as well as hemoglobin levels should be monitored in all patients receiving fluid replacement therapy.

Decreased BP => Activation of sympathetic nervous system => ______ => decreased blood flow to kidneys, skin, and GI tract.

a-Adrenergic stimulation

Match each assessment area with the frequency with which the nurse should monitor its status when caring for an unstable patient with the acute phases of shock (some answers may not be used). 1. continuously, 2. q15min, 3. q1hr, 4. q2hr, 5. q4hr, 6. q8hr a. renal function b. HR, BP, CVP, PAP c. Neurologic function d. GI function e. Respiratory rate f. O2 saturation g. Breath sounds h. ECG i. Response to medication and fluid administration j. Normal body temperature

a. renal function - q1hr b. HR, BP, CVP, PAP - continuously c. Neurologic function - q1hr d. GI function - q4hr e. Respiratory rate - q15min f. O2 saturation - continuously g. Breath sounds - q1hr h. ECG - continuously i. Response to medication and fluid administration - q15min j. Normal body temperature - q4hr

If conduction is depressed and some areas of the heart are blocked (e.g., by infarction), the unblocked areas are

activated earlier than the blocked areas.

If performing cardioversion is it important to monitor patients ________________

airway

(secondary firing mechansims) Triggered beats (early or late) may come from

an ectopic focus or accessory pathway (area outside the normal conduction pathway) in the atria, AV node, or ventricles.

infarction

blockage of heart (embolism, thrombosis, etc.)

SYNCOPE

brief lapse in consciousness accompanied by a loss in postural tone (fainting).The causes of syncope are categorized as cardiovascular or noncardiovascular:

What is the priority assessment by the nurse caring for a patient receiving IV nesiritide (Natrecor) to treat heart failure? a. Urine output b. Lung sounds c. Blood pressure d. Respiratory rate

c. Blood pressure Although all identified assessments are appropriate for a patient receiving IV nesiritide, the priority assessment would be monitoring for hypotension, the main adverse effect of nesiritide.

A compensatory mechanism involved in HF that leads to inappropriate fluid retention and additional workload of the heart is: a. ventricular dilation b. ventricular hypertrophy c. neurohormonal response d. sympathetic nervous system activation

c. neurohormonal response Rationale: The following mechanisms in heart failure lead to inappropriate fluid retention and additional workload of the heart: activation of the renin-angiotensin-aldosterone system (RAAS) cascade and release of antidiuretic hormone from the posterior pituitary gland in response to low cerebral perfusion pressure that results from low cardiac output.

An electrophysiologic study (EPS) can identify

causes of heart blocks, tachydysrhythmias-dysrhythmias with rates>100 beats/minute, bradydysrhythmias-dysrhythmias with rates< 60 beats/minute and syncope.

In states of full excitability, the heart is

completely recovered.

A stable patient with acute decompensated heart failure (ADHF) suddenly becomes dyspneic. Before positioning the patient on the bedside, what should the nurse assess first? a. Urine output b. Heart rhythm c. Breath sounds d. Blood pressure

d. Blood pressure The nurse should evaluate the blood pressure before dangling the patient on the bedside because the blood pressure can decrease as blood pools in the periphery and preload decreases. If the patient's blood pressure is low or marginal, the nurse should put the patient in the semi-Fowler's position and use other measures to improve gas exchange.

The key factor in describing any type of shock is what? a. hypoxemia b. hypotension c. vascular collapse d. inadequate tissue perfusion

d. inadequate tissue perfusion Although all of the factors may be present, regardless of the cause, the end result in inadequate supply of oxygen and nutrients to body cells from inadequate tissue perfusion.

A patient has a spinal cord injury at T4. Vital signs include falling BP with bradycardia. The nurse recognizes that the patient is experiencing? a. a relative hypovolemia b. an absolute hypovolemia c. neurogenic shock from low blood flow d. neurogenic shock from massive vasodilation

d. neurogenic shock from massive vasodilation

EKG

electrodes are placed on chest and extremities to record electrical activity from 12 different views. Can identify many different heart conditions ( dysrhythmias, MI, pericarditis)

two most important CM of neurogenic shock

hypotension and bradycardia

can occur within 30 minutes of spinal cord injury

neurogenic shock

massive vasodilation without compensation because of loss of SNS tone

neurogenic shock

T wave represents

repolarization of the ventricles.

P wave

represents the depolarization of the atria (passage of an electrical impulse through the atria), causing atrial contraction.

non centralized telemetry monitoring does not

require constant surveillance.

The remaining six unipolar leads (V1 through V6) measure ......

the electrical forces in the horizontal plane (precordial leads)

When caring for a critically ill patient who is being mechanically ventilated, the nurse will astutely monitor for which clinical manifestation of multiple organ dysfunction syndrome (MODS)?

-Decreased respiratory compliance

What common drug is beneficial for anaphylactic shock?

-Diphenhydramine (Benadryl)

Diagnostic studies, Pericarditis

-ECG diagnose acute pericarditis, changes in 90%, diffuse ST elevation -Echocardiogram, effusion or tamponade -leukocytosis -elevated CRP, ESR -elevated troponin in pt with ST segment elevation

mitral valve stenosis, etiology

-Rheumatic heart disease leading cause in adults -congenital mitral stenosis -rheumatoid arthritis -systemic lupus erythematosus

When caring for a patient with mitral valve stenosis, it is most important that the nurse asses for c. shortness of breath on exertion.

The pressure gradient changes in mitral stenosis lead to fluid backup into the lungs, resulting in hypoxemia and dyspnea.

decreased renal blood flow during cardiogenic shock results in

sodium and water retention and decreased urine output

Severe Sepsis

The progression of sepsis with an amplified inflammatory response. -All tissues are involved & are hypoxic to some degree. -Some organs are experiencing cell death & dysfunction -Clinical manifestation: Lower oxygen saturation, rapid respiratory rate, decreased to absent urine output, & a change in the patient's cognition & affect

Bundles for Resuscitation & Manage. Severe Sepsis

-Sepsis Resuscitation Bundle- 1. Measure serum lactate levels. 2. Obtain blood cultures BEFORE admin. antibiotics. 3. Administer broad-spectrum antibiotic therapy within 1 hour of establishing diagnosis. 4. If either hypotension or a serum lactate level greater than 4 mmol/L institue: a. Infusion of 1000 mL of crystalloids or 300-500 mL of colloids over 30 minutes. b. If hypo doesn't respond by increasing MAP, start IV vasopressor therapy.

When the block is unidirectional, this uneven conduction may

*allow the initial impulse to reenter areas that were previously not excitable but have recovered. *The reentering impulse may be able to depolarize the atria and ventricles, causing a premature beat. * If the reentrant excitation continues, tachycardia occurs.

•In second-degree AV block

, type I (Mobitz I or Wenckebach heart block), there is a gradual lengthening of the PR interval until an atrial impulse is nonconducted and a QRS complex is blocked (missing).

Carditis, RF clinical manifestation

1) heart murmur: mitral or aortic regurgitation, mitral stenosis 2) cardaic enlargement and HF 3) pericarditis, muffled heart sounds, CP, pericardial rub, symptoms of effusion

The nurse identifies the nursing diagnosis of decreased cardiac output related to valvular insufficiency for the patient with infective endocarditis (IE) based on which assessment finding(s)? b. Urine output less than 30 mL/hr

Decreased renal perfusion caused by inadequate cardiac output will lead to decreased urine output.

V. fib treatment

Defib Drug therapy (epinephrine, vasopressin)

o

For patients with drug-refractory atrial fibrillation or who do not respond to electrical conversion, radiofrequency catheter ablation and the Maze procedure are options.

Anoxic

No oxygen

PVCs

are usually a benign finding in the patient with a healthy heart. In heart disease, depending on frequency, PVCs may reduce the CO and precipitate angina and HF.

Pericardial effusion, Pericarditis

-build-up of fluid in pericardium -compress nearby structures: pulmonary=cough, dyspnea, tachypnea -phrenic nerve compression= hiccups -laryngeal nerve compression=hoarseness

Dobutamine is an inotropic drug given specifically to improve-

-cardiac output (CO).

Secondary Cardiomyopathy

-cause is known -secondary to another disease process

Rheumatic heart disease

-chronic condition resulting form RF -scarring and deformity of heart valves

Stenosis

-constricting or narrowing -valve or artery

Rheumatic pancarditis

-40% of RF -all layers of heart are involved

Patho infective endocarditis

-blood turbulence within heart allows oranges to infect damaged valves or endothelial surfaces

The client with coronary artery disease asks the nurse, "Why do I get chest pain?" Which statement would be the most appropriate response by the nurse?

"Chest pain is caused by decreased oxygen to the heart muscle." - this stated in layman's terms - do not use medical terms - explain the correct procedure!

Which statement by the client diagnosed with coronary artery disease indicates that the client understands the discharge teaching concerning diet?

"I should bake or grill any meats I eat." - grill or bake do not fry - no more than 3 eggs a week, especially yolk - low fat milk, not whole milk - avoid pork products

The male client is diagnosed with coronary artery disease (CAD) and is prescribed sublingual nitroglycerin. Which statement indicates the client needs more teaching?

"If my chest pain is not gone with one tablet, I will go to the ER." - keep in the dark - the tabs should burn on sting when put under the tongue - client should take 1 pill every 5 minutes and after the third pill still chest pain then call 911

ECG CHANGES ASSOCIATED WITH ACUTE CORONARY SYNDROME

(1)The 12-lead ECG is the primary diagnostic tool used to evaluate patients presenting with ACS. (2)Definitive ECG changes that occur in response to ischemia, injury, or infarction of myocardial cells can be seen in the leads that face the area of involvement.

diagnostic tests for syncope

(1)echocardiography,(2) stress testing, (3)EPS,(4) head-up tilt-test,(5) Holter monitors, (6) event/loop recorders.

Treatment is often based on the cause of the PVCs

(e.g., oxygen therapy for hypoxia, electrolyte replacement). Drugs that can be considered include -adrenergic blockers, procainamide, amiodarone, or lidocaine (Xylocaine).

Dysrhythmias: Initial interventions

* Ensure ABCs. • Administer O2 via nasal cannula or non-rebreather mask. • Obtain baseline vital signs, including O2 saturation. • Obtain 12-lead ECG. • Initiate continuous ECG monitoring. • Identify underlying rate and rhythm. • Identify dysrhythmia. • Establish IV access. • Obtain baseline laboratory studies (e.g., CBC, electrolytes).

When assessing the cardiac rhythm

*make an accurate interpretation and immediately evaluate the patient's clinical status. Assess the patient's hemodynamic response to change in rhythm. Use information to guide the selection of your interventions. *+*Determination of the cause of dysrhythmias is a priority. At all times, the patient, not the "monitor," must be assessed and treated.

For symptomatic patients

, a transcutaneous pacemaker is used until a temporary transvenous pacemaker can be inserted followed by a permanent pacemaker.

• In first-degree AV block

, every impulse is conducted to the ventricles, but the duration of AV conduction is prolonged. First-degree AV block is usually not serious but can be a precursor of higher degrees of AV block. Patients with first-degree AV block are asymptomatic. There is no treatment.

Causes & types of shock by functional impairment

-Hypovolemic shock- Total body fluid decreased (in all fluid compartments) Specific Cause or risk factors- trauma, GI ulcer, surgery, inadequate clotting- hemophilia, liver disease, malnutrition, bone marrow suppression, cancer,anticoagulation therapy, diabetes insipidus, -Dehydration' hyperglycemia, vomiting, diarrhea, heavy diaphoresis, diuretic therapy, Nasogatric suction -Cardiogenic shock Direct pump failure (fluid volume not affected), specific cause or risk factor- myocardial Infarction, cardiac arrest, ventricular dysrhythmias (v fib, v tachy), cardiac amylodosis, cardiomyopathies- viral, toxic, myocardial degeneration. -Distributive shock- Fluid shifted from central vascular space (total body fluid volume normal or increased)- neural-induced, pain, anesthesia, stress, head trauma, chemical-induce- anaphylaxis, capillary leak, burns extensive- trauma, liver impairment Obstructive shock- Cardiac function decreased by non cardiac factor (indirect pump failure) total body fluid is not affected although central volume is decreased . Specific Cause or risk factors- Cardiac tamponade, arterial stenosis, pulmonary embolism, pulmonary hypertension, constructive pericarditis, thoracic tumors, tension pneumothorax.

What is the primary precipitating factor in septic shock?

-Infection leading to an inflammatory response

When caring for a patient in acute septic shock, the nurse would anticipate-

-Infusing large amounts of intravenous fluids.

STAGES OF SHOCK 4

-Initial stage of shock (early stage): When the patient's baseline MAP is decreased by less than 10. A heart & respiratory rate increased from the patient's baseline level or a slight increase in diastolic BP may be the only objective manifestion of this early stage of shock -Nonprogressive Stage of Shock (Compensatory stage): occurs when MAP decreases by 10-15 from baseline. Kidney & hormonal compensatory mechanisms are activated b/c cardiovascular responses alone are not enough to maintain MAP & supply oxygen to vital organs. -Kidneys & baroreceptors sense an ongoing decrease in MAP & trigger the release of renin, antidiuretic hormone (ADH), aldosterone, epinephrine, & norepinephrine to start kidney compensation. -ADH increases water reabsorption in the kidney, further reducing urine output, & also causes blood vessel constriction in the skin & other less vital tissue areas. -Tissue hypoxia occurs in nonvital organs (skin, GI tract) & in the kidney but it's not great enough to cause permanent damage. -Changes include acidosis (low blood pH) & hyperkalemia (increased blood potassium level). Subjective changes include : thirst & anxiety, objective changes include restlessness, tachycardia, increased respiratory rate, decreased urine output, falling systolic BP, rising diastolic BP, narrowing pulse pressure, cool extremities, & a 2%-5% decrease in oxygen saturation measured by pulse ox. Can remain in stage for hours. -PROGRESSIVE STAGE OF SHOCK (Intermediate Stage): occurs when there's a sustained decrease in MAP of more than 20 from baseline. -Compensatory mechanisms are functioning but can no longer deliver sufficient oxygen, even to vital organs. -Vital organs develop hypoxia & less vital organs become anoxie (no oxygen) & ischemic (call dysfunction or death from lack of oxygen). -Some tissues have severe call damage & die. -Subjective changes include severe thirst sensation & deeper anxiety. The patient may express a sense of "something bad" (impending doom) about to happen. They may seem confused; objective changes:a rapid weak pulse; low BP; pallor to cyanosis of oral mucosa & nail beds, cool & moist skin. REFRACTORY STAGE OF SHOCK (IRREVERSIBLE STAGE)- occurs when too much cell death & tissue damage result from too little oxygen reaching the tissues. Vital organs have overwhelming damage. Body can no longer respond effectively to interventions, & shock continues. Manifestations:rapid loss of consciousness; non palpable pulse; cold dusky extremities; slow shallow respirations; & unmeasureable oxygen saturation.

catheters

-Insertion of a CVP catheter allows pressure to be monitored in the patient's right atrium or superior vena cava while providing venous access. -Intra-arterial catheters allow continuous BP monitoring & are an access for arterial blood sampling.

What is the precipitating factor for hypovolemic shock?

-Loss of blood or flood from vasculature.

Nonsurgical Management

-Oxygen therapy -IV therapy: for fluid resuscitation **primary intervention** crystalloids- nonprotein- help maintain adequate fluid & electrolyte balance- NS or Ringer's lactate Colloids- large molecules (usually protein or starches)- protein containing-helps restore osmotic pressure & fluid volume Plasma- a cellular blood product containing clotting factors, is given to restore osmotic pressure when hematocrit & hemoglobin levels are within normal ranges.

Sepsis & Systemic Inflammatory Response Syndrome

-Sepsis is a condition in which infectious micro-organisms have entered the bloodstream. -Inflammatory responses become an enemy. -Patient has mild hypotension, a low urine output, & an increased respiratory rate. -Responses result in a hypodynamic state with decreased cardiac output -Temp of more than 100.4 or less than 96.8 -HR more than 90 -Respiratory more than 20 breaths -Abnormal WBC

Septic Shock

-Stage of sepsis & SIRS when multiple organ failure is evident & uncontrolled bleeding occurs. Etiology- Major cause of sepsis is a bacterial infection that escapes local control, fungal infections also cause sepsis -HALLMARK OF SEPSIS- an increasing serum lactate level, a normal or low total WBC count & a decreasing segmented neutrophil level with a rising band neutrophil level called left shift

What happens to BUN and creatinine in the case of cardiogenic shock?

-They are increased as a result of renal hypoperfusion.

Drug Therapy

-Used when the volume lost is severe & the patient doesn't respond sufficiently to fluid replacement & blood products. -Actions of drugs for shock increase venous return, improve cardiac contractility, or improve cardiac perfusion by dilating the coronary vessels. -Vasoconstricting drugs stimulate venous return by constricting blood vessels & decreasing venous pooling. Increase cardiac output, MAP, which help improve perfusion & oxygenation. dopamine (Intropin, Revimine); norepinephrine (Levophed); Nursing intervention: (dopamine) assess the patient for chest pain; monitor urine output hourly; assess BP every 15 minutes. -Inotropic drugs stimulate adrenergic receptors in the heart (beta1 receptors) & improve heart muscle cell contraction. Thus greater recoil occurs & more blood leaves the left ventricle during contraction. Dobutamine (Dobutrex)- Assess for chest pain, drug increases myocardial oxygen consumption & can cause angina or infarction. -Milrinone (Primacor)- Assess BP every 15 minutes; Hypertension is a sign of overdose. -Drugs enhancing myocardial perfusion help ensure that the heart is well perfused, esp when giving drugs to improve cardiac contraction so that aerobic metabolism is maintained in the heart cells & maximum contractility can occur. -Sodium nitroprusside (Nitropress, Nipride)- Assess BP @ least every 15 minutes. The vasodilating effect can cause systemic vasodilation & hypotension

Rheumatic Fever

-acute inflammatory disease of heart involving all layers -endocardium, myocardium and pericardium

Pericarditis Patho

-acute pericarditis -inflammatory response -influx of neutrophils -increased pericardial vascularity -fibrin deposition on epicardium

Pericarditis MI

-acute pericarditis= 48-72 hrs after MI -Dressler syndrome (late pericarditis)= 4-6 weeks after MI

Subacute infective endocarditis

-affects pt with preexisting valve disease

Pericardium function

-anchoring -lubrication decrease friction during systolic and diastolic heart movement -prevents excessive dilation of heart during diastole

Hypertrophic cardiomyopathy

-asymmetric left ventricular hypertrophy without ventricular dilation -septum becomes enlarged obstructs blood flow from left ventricle

Mitral valve regurgitation, physiology

-backward flow of blood from left ventricle into left atrium -incomplete valve closure during systole -increased cardiac workload to maintain CO

Mitral valve regurgitation, etiology

-defect in mitral leaflets, annulus, chordae tendineae, papillary muscles, left atrium, or left ventricle -mostly caused by MI, chronic rheumatic heard disease -mitral valve prolapse, ischemic papillary muscle dysfunction, infective endocarditis

mitral valve stenosis, clinical manifestation

-exertional dyspnea from reduced lung compliance -loud S1, low pitched diastolic murmur -hoarseness (atrial enlargment compressing laryngeal nerve) -hemoptysis (pulmonary hypertension) -CP from decreased CO and coronary perfusion -seizure or stroke (emboli) -AFib, fatigue and palpitations

Rheumatic Fever patho

-found mainly in valves, swelling and erosion of valve leaflets, calcification, stenosis -deposition of fibrin and blood cells forms vegetation -regurgitation of blood from dysfunctional valves -mitral and aortic most commonly affected

Pressure on either side of valuve

-in normal valve, opens unidirectionally -when open, pressure on both sides is equal -in stenotic valve, opening is smaller causing difference in pressure on both sides -level of stenosis determines difference in pressure, more stenosis=more pressure difference

The many deleterious effects of shock are all related to-

-inadequate perfusion and oxygenation of every body system.

regurgitation (incompetence, insufficiency)

-incomplete closure of valve -backward flow of blood

Cardiac tamponade

-increase in pericardial effusion volume -compression of heart -acute or subacute -chest pain, confusion, anxiety, restlessness -decreased CO, muffled heart sounds, narrowed pulse pressure -tachypnea, tachycardia -juglar venous distension

Infective endocarditis

-infection of endocardial (innermost) layer of heart and valves

Restrictive cardiomyopathy

-least common -impairs diastolic filling and stretch -systolic function unchanged

Erythema marginatum, RF clinical manifestation

-less common feature of RF -bright pink, nonpuritic, maplike macular lesions -trunk and proximal extremities -exacerbated by heat, warm bath

What is the precipitating factor for cardiogenic shock?

-loss or inadequate ventricular function leading to poor cardiac output.

Myocarditis diagnostic studies

-nonspecific EGC changes reflect pericardial involvement -dysrhythmias -mild leukocytosis -atypical lymphocytes -increased ESR, CRP -elevated troponin -endo-myocardial biopsy= histologic confirmation within first 6 weeks -neuclear scan, ECHO, MRI

TYPE OF SHOCKS- HYPOVOLEMIC

-occurs when too little circulating blood volume causes a MAP decrease, resulting in inadequate total body oxygenation- Reduced MAP slows blood flow resulting in decreased tissue perfusion. -Common problems leading: hemorrhage & dehydration

Hyperglycemia in the absence of diabetes can be an indicator of?

-possible onset of sepsis. People with no prior diabetic history who present with hyperglycemia may be going into septic shock.

Pericarditis Clinical manifestations

-severe, sharp chest pain -worse with deep inspiration, lying supine -relieved by sitting up and leaning forward -radating patin to neck, arm or shoulder -differential: angina -referred pain to trapezius makes it distinct -dyspnea

aortic valve regurgitation

...

The nurse is caring for a patient who is experiencing cardiogenic shock as a result of myocardial infarction. Which nursing assessment finding is most concerning? 1 PaO2 60 mm Hg 2 Blood pressure 100/56 mm Hg 3 Urine output 260 mL in eight hours 4 Heart rate 96 beats/minute

1 A PaO2 of 60 is below the normal 80 to 100 mm Hg. The patient experiencing cardiogenic shock will exhibit hypotension and tachycardia, and therefore a blood pressure of 100/56 mm Hg and heart rate of 96 would not apply. A urine output of 260 cc/8 hrs is borderline but not reportable without a continued trending pattern. Text Reference - p. 1633

What therapy is provided to a patient with acute respiratory distress syndrome (ARDS)? 1 Mechanical ventilation 2 Oxygen via a Venturi mask 3 Oxygen via a non-rebreather mask 4 Small volume nebulizer treatments

1 A patient with acute respiratory distress syndrome (ARDS) would be intubated and receive mechanical ventilation. Small volume nebulizer treatments would open airways; however, this intervention will not sufficiently treat ARDS. Oxygen via a Venturi mask or a non-rebreather mask would be insufficient to promote oxygenation and perfusion. Text Reference - p. 1651

What occurs when the inflammatory response is activated in a patient with systemic inflammatory response syndrome (SIRS)? 1 Release of mediators 2 Decrease in metabolism 3 Damage of the mesothelium 4 Decrease in vascular permeability

1 A release of mediators occurs when the inflammatory response is activated. Other changes that occur include an increase in metabolism or hypermetabolism, direct damage to the endothelium, and an increase in vascular permeability. Text Reference - p. 1649

How is systemic inflammatory response syndrome (SIRS) different from multiple organ dysfunction syndrome (MODS)? Correct1 Shock leads to SIRS, and SIRS causes MODS. 2 MODS is reversible, and SIRS has irreversible changes. Incorrect3 SIRS is caused by shock, and MODS is caused by perfusion deficits. 4 Homeostasis fails before SIRS, and homeostasis is maintained in MODS.

1 Any type of shock triggers the systemic inflammatory response. Generalized inflammation in organs remote from the initial shock is systemic inflammatory response syndrome (SIRS). Multiple organ dysfunction syndrome (MODS) results from SIRS. Both SIRS and MODS are reversible in the early stages. SIRS is caused by some kind of injury to the body such as sepsis, ischemia, infarction, and injury; SIRS can lead to MODS if not treated. In SIRS, a type of shock triggers a systemic inflammatory response, after which the body's homeostasis fails. In MODS, homeostasis fails and medical intervention is needed. Test-Taking Tip: Identify option components as correct or incorrect. This may help you identify a wrong answer. Text Reference - p. 1649

The nurse reviews the medical record of a patient with pneumonia and notes that the patient has hypotension, hypothermia, leukocytosis, and hypoxemia. What should the nurse infer from these findings? 1 The patient has septic shock. 2 The patient has neurogenic shock. 3 The patient has cardiogenic shock. 4 The patient has hypovolemic shock.

1 Septic shock is most commonly found in the patient having gram-negative bacterial infections, such as pneumonia. Because it is characterized by hypertension, hypothermia, leukocytosis, and hypoxemia in patients with infections, the nurse concludes that the patient has septic shock. Neurogenic shock is most commonly seen in the patient who has an injury. Cardiogenic shock is caused by cardiovascular disorders, such as a myocardial infarction and cardiomyopathy. Hypovolemic shock is caused by hemorrhage or trauma. Test-Taking Tip: Multiple-choice questions can be challenging, because students think that they will recognize the right answer when they see it or that the right answer will somehow stand out from the other choices. This is a dangerous misconception. The more carefully the question is constructed, the more each of the choices will seem like the correct response. Text Reference - p. 1637

The primary health care provider prescribes antibiotics and vasopressors for a patient. Which type of shock does the nurse expect to be treating? 1 Septic shock 2 Cardiogenic shock 3 Neurogenic shock 4 Anaphylactic shock

1 Septic shock occurs in response to infection. Therefore, antibiotics are prescribed for a patient with septic shock. Cardiogenic shock occurs when systolic or diastolic function of the heart is impaired. Sympathomimetic drugs are used for the treatment of cardiogenic shock. Injury to the spinal cord at the fifth thoracic vertebra or above causes neurogenic shock. Vasconstricting medications are prescribed to prevent vasodilation for a patient in septic shock. Anaphylactic shock is a life-threatening allergic reaction to a sensitizing substance. Antihistamines, bronchodilators, and corticosteroids are used in the treatment of anaphylactic shock. Test-Taking Tip: Do not worry if you select the same numbered answer repeatedly, because there usually is no pattern to the answers. Text Reference - p. 1645

nursing goals, infective endocarditis

1) normal baseline heart function 2) ADLs without fatigue 3) therapeutic regimen to prevent recurrence 4) health promotion, minimize risk of disease

When examining a patient with cardiogenic shock, which signs of peripheral hypoperfusion does the nurse expect? Select all that apply. 1 Cyanosis 2 Cold skin 3 Weak pulse 4 Bradycardia 5 Hypertension

1, 2, 3 Cyanosis, cold skin, and a weak pulse are the signs of peripheral hypoperfusion in cardiogenic shock. Bradycardia and hypertension are not seen in cardiogenic shock; instead, tachycardia and low blood pressure are noted. Text Reference - p. 1633

Which types of shock may cause reduced urinary output in a patient? Select all that apply. 1 Septic shock 2 Hypovolemic shock 3 Obstructive shock 4 Neurogenic shock 5 Anaphylactic shock 6 Cardiogenic shock.

1, 2, 3, 6 Decreased urine output is a clinical manifestation of septic, hypovolemic, obstructive and cardiogenic shock. Neurogenic shock is associated with bladder dysfunction. Anaphylactic shock is associated with urinary incontinence. Test-Taking Tip: Make certain that the answer you select is reasonable and obtainable under ordinary circumstances and that the action can be carried out in the given situation. Text Reference - p. 1635

The health care provider prescribes a dose of dobutamine for a patient in cardiogenic shock due to myocardial infarction. What appropriate actions should the nurse perform for safely administering the medication? Select all that apply. 1 Monitor heart rate and blood pressure. 2 Stop infusion if tachydysrhythmias develop. 3 Always administer with sodium bicarbonate. 4 Administer through a central line. 5 Use a glass bottle for infusion.

1, 2, 4 Doubutamine is a sympathomimetic medication. When used in therapy with dobutamine, the patient's heart rate and blood pressure should be continuously monitored, as they may worsen hypotension, requiring the addition of a vasopressor. The infusion should be stopped if tachydysrhythmias develop. The administration through a central line is recommended, because infiltration leads to tissue sloughing. The drug should not be administered with sodium bicarbonate, because it can get deactivated. Because dobutamine is not adsorbed in plastic containers, it is not necessary to administer the drug in glass bottles. Test-Taking Tip: Key words or phrases in the stem of the question such as first, primary, early, or best are important. Similarly, words such as only, always, never, and all in the alternatives are frequently evidence of a wrong response. As in life, no real absolutes exist in nursing; however, every rule has its exceptions, so answer with care. Text Reference - p. 1643

When examining a patient with septic shock, what symptoms would the nurse expect to find? Select all that apply. 1 Paralytic ileus 2 Gastrointestinal (GI) bleeding 3 Pulsus paradoxus 4 Distended jugular vein 5 Decreased urinary output

1, 2, 5 Patients suffering from septic shock may experience decreased tissue perfusion, which may result in a paralytic ileus, GI bleeding, and decreased urinary output. Pulsus paradoxus and jugular vein distension are found in obstructive shock and are mainly the result of compromised hemodynamics. Text Reference - p. 1637

A nurse caring for a patient with multiple organ dysfunction syndrome understands that the patient may be at increased risk of bleeding. What nursing interventions should the nurse perform to manage this patient? Select all that apply. 1 Observe bleeding sites. 2 Decrease fluid intake. 3 Provide enteral feedings. 4 Administer platelets and clotting factors. 5 Minimize traumatic interventions.

1, 4, 5 The patient with multiple organ dysfunction syndrome is at a risk of bleeding due to increased bleeding time, thrombocytopenia, and dysfunctional clotting process. The nursing interventions should be aimed at preventing potential bleeding and replacing factors being lost. The patient should be observed for frank or occult bleeding from potential sites. The factors like platelets and clotting factors should be replaced if deficient. Traumatic interventions such as intramuscular injections or multiple venipunctures should be avoided. Decreasing the fluid intake and providing enteral feedings will not help in minimizing hematologic complications. Text Reference - p. 1650

Cardiogenic shock care involves_

_hemodynamic monitoring, drug therapy (e.g., diuretics to reduce preload), and use of circulatory assist devices (e.g., intraaortic balloon pump, ventricular assist device).

Which client teaching should the nurse implement for the client diagnosed with coronary artery disease? Select all that apply.

1. Encourage a low-fat, low-cholesterol diet. 2. Instruct client to walk 30 minutes a day. 3. Decrease the salt intake to two (2) g a day. WRONG! restricted in hypertension 4. Refer to counselor for stress reduction techniques. 5. Teach the client to increase fiber in the diet.

The client has chronic atrial fibrillation. Which discharge teaching should the nurse discuss with the client?

1. Instruct the client to use a soft-bristle toothbrush. - they will be on anticoagulants, risk for bleeding - scheduled INR test for therapeutic levels of warfarin, PTT for heparin - synchronized cardioversion for new-onset atrial fib not chronic

The nurse is completing a neurovascular assessment on the client with chronic venous insufficiency. What should be included in this assessment? Select all that apply. 1. Assess for paresthesia. 2. Assess for pedal pulses. 3. Assess for paralysis. 4. Assess for pallor. 5. Assess for paresthesia.

1. The nurse should determine if the client has any numbness or tingling. 2. The nurse should determine if the client has pulses, the presence of which indicates there is no circulatory compromise. 3. The nurse should determine if the client can move the feet and legs. 4. The nurse should determine if the client's feet are pink or pale. 5. The nurse should assess the feet to determine if they are cold or warm.

1. A 20-year-old patient has acute infective endocarditis. While obtaining a nursing history, what should the nurse ask the patient about (select all that apply)? a. Renal dialysis b. IV drug abuse c. Recent dental work d. Cardiac catheterization e. Recent urinary tract infection

1. a, b, c, d, e. Recent dental, urologic, surgical, or gynecologic procedures and history of IV drug abuse, heart disease, cardiac catheterization or surgery, renal dialysis, and infections all increase the risk of infective endocarditis.

The critical care nurse is caring for a 55-year-old man who has a catheter in the right radial artery that is being used for continuous arterial blood pressure monitoring following his abdominal aortic aneurysm surgery. Which observation by the nurse would require an emergency intervention? a. Calculated mean arterial pressure is 74 mm Hg. b. Patient's head of bed elevation is at 30 degrees. c. Capillary refill time in the right hand is 5 seconds. d. Pressure bag attached to the arterial line is inflated to 270 mm Hg.

10. C Neurovascular status distal to the arterial insertion site is monitored hourly. If arterial flow is compromised, the limb will be cool and pale, with capillary refill time longer than 3 seconds. Symptoms of neurologic impairment include paresthesia, pain, or paralysis. Neurovascular impairment can result in loss of a limb and is an emergency. The pressure bag should be inflated to 300 mm Hg. Normal range for mean arterial pressure is 70 to 105 mm Hg. The backrest elevation may be up to 45 degrees unless the patient has orthostatic changes.

10. A patient with acute pericarditis has markedly distended jugular veins, decreased BP, tachycardia, tachypnea, and muffled heart sounds. The nurse recognizes that these symptoms occur when what happens? a. The pericardial space is obliterated with scar tissue and thickened pericardium b. Excess pericardial fluid compresses the heart and prevents adequate diastolic filling c. The parietal and visceral pericardial membranes adhere to each other, preventing normal myocardial contraction d. Fibrin accumulation on the visceral pericardium infiltrates into the myocardium, creating generalized myocardial dysfunction

10. b. The patient is experiencing a cardiac tamponade that consists of excess fluid in the pericardial sac, which compresses the heart and the adjoining structures, preventing normal filling and cardiac output. Fibrin accumulation, a scarred and thickened pericardium, and adherent pericardial membranes occur in chronic constrictive pericarditis.

The nurse is caring for a 34-year-old woman with acute decompensated heart failure who has a pulmonary artery catheter. Which assessment best indicates that the patient's condition is improving? a. Cardiac output (CO) is 3.5 L/minute. b. Central venous pressure (CVP) is 10 mm Hg. c. Pulmonary artery wedge pressure (PAWP) is 10 mm Hg. d. Systemic vascular resistance (SVR) is 1500 dynes/sec/cm-5

11. C PAWP is the most sensitive indicator of cardiac function and fluid volume status. Normal range for PAWP is 6 to 12 mm Hg. PAWP is increased in heart failure. Normal range for CVP is 2 to 8 mm Hg. An elevated CVP indicates right-sided heart failure or volume overload. Normal cardiac output is 4 to 8 L/minute. CO is decreased in heart failure. SVR is increased in left-sided heart failure. Normal SVR is 800 to 1200 dynes/sec/cm-5.

11. What explains the measurement of pulsus paradoxus with cardiac tamponade (select all that apply)? a. A difference of less than 10 mm Hg occurs. b. A difference of greater than 10 mm Hg occurs. c. It is measured with an automatic sphygmomanometer. d. Rapidly inflate the cuff until you hear sounds throughout the respiratory cycle. e. Subtract the number when sounds are heard in the respiratory cycle from the number when the first Korotkoff sound during expiration is heard.

11. b, e. Pulsus paradoxus is measured with a manually operated sphygmomanometer. The cuff is deflated slowly until the first Korotkoff sound during expiration is heard and the number is noted. The slow deflation of the cuff is continued until sounds are heard throughout the respiratory cycle and that number is subtracted from the first number. When the difference is >10 mm Hg, cardiac tamponade may be present. The difference is normally <10 mm Hg.

12. The patient with acute pericarditis is having a pericardiocentesis. Postoperatively what complication should the nurse monitor the patient for? a. Pneumonia b. Pneumothorax c. Myocardial infarction (MI) d. Cerebrovascular accident (CVA)

12. b. Pneumothorax may occur as a needle is inserted into the pericardial space to remove fluid for analysis and relieve cardiac pressure with pericardiocentesis. Other complications could include dysrhythmias, further cardiac tamponade, myocardial laceration, and coronary artery laceration.

13. Priority Decision: A patient with acute pericarditis has a nursing diagnosis of pain related to pericardial inflammation. What is the best nursing intervention for the patient? a. Administer opioids as prescribed on an around-the-clock schedule. b. Promote progressive relaxation exercises with the use of deep, slow breathing. c. Position the patient on the right side with the head of the bed elevated 15 degrees. d. Position the patient in Fowler's position with a padded over-the-bed table for the patient to lean on.

13. d. Relief from pericardial pain is often obtained by sitting up and leaning forward. Pain is increased by lying flat. The pain has a sharp, pleuritic quality that changes with respiration and patients take shallow breaths. Antiinflammatory medications may also be used to help control pain but opioids are not usually indicated.

A 68-year-old male patient diagnosed with sepsis is orally intubated on mechanical ventilation. Which action is most important for the nurse to take? a. Use the open-suctioning technique. b. Administer morphine for discomfort. c. Limit noise and cluster care activities. d. Elevate the head of the bed 30 degrees.

12. D The two major complications of endotracheal intubation are unplanned extubation and aspiration. To prevent aspiration all intubated patients and patients receiving enteral feedings must have the head of the bed (HOB) elevated a minimum of 30 to 45 degrees unless medically contraindicated. Closed-suction technique is preferred over the open-suction technique because oxygenation and ventilation are maintained during suctioning, and exposure to secretions is reduced. The nurse should provide comfort measures such as morphine to relieve anxiety and pain associated with intubation. To promote rest and sleep the nurse should limit noise and cluster activities.

16. What manifestations most strongly support a diagnosis of acute rheumatic fever? a. Carditis, polyarthritis, and erythema marginatum b. Polyarthritis, chorea, and decreased antistreptolysin O titer c. Organic heart murmurs, fever, and elevated erythrocyte sedimentation rate (ESR) d. Positive C-reactive protein, elevated white blood cells (WBCs), and subcutaneous nodules

16. a. Major criteria for the diagnosis of rheumatic fever include evidence of carditis, polyarthritis, chorea (often very late), erythema marginatum, and subcutaneous nodules. Minor criteria include all laboratory findings as well as fever, arthralgia, and a history of previous rheumatic fever. There also must be evidence of a previous group A streptococci infection (e.g., positive antistreptolysin O titer).

• Patients with septic shock often have _

_hypotension, respiratory failure, alteration in neurologic status, decreased urine output, and GI dysfunction.

The nurse is caring for a patient who has been on a mechanical ventilator for several days. Which weaning parameter would tell the nurse whether or not the patient has enough muscle strength to breathe without assistance? a. Tidal volume b. Minute ventilation c. Forced vital capacity d. Negative inspiratory force

14. D The negative inspiratory force (NIF) measures inspiratory muscle strength. Tidal volume and minute ventilation assess the patient's respiratory endurance. Forced vital capacity is not used as a measure to determine weaning from a ventilator.

14. When obtaining a nursing history for a patient with myocarditis, what should the nurse specifically question the patient about? a. Prior use of digoxin for treatment of cardiac problems b. Recent symptoms of a viral illness, such as fever and malaise c. A history of coronary artery disease (CAD) with or without an MI d. A recent streptococcal infection requiring treatment with penicillin

14. b. Viruses are the most common cause of myocarditis in the United States and early manifestations of myocarditis are often those of systemic viral infections. Myocarditis may also be associated with systemic inflammatory and metabolic disorders as well as with other microorganisms, drugs, or toxins. The patient with myocarditis is predisposed to drug-related dysrhythmias and toxicity with digoxin, so it is used very cautiously, if at all, in treatment of the condition.

18. A patient with rheumatic heart disease with carditis asks the nurse how long his activity will be restricted. What is the best answer by the nurse? a. "Full activity will be allowed as soon as acute symptoms have subsided." b. "Bed rest will be continued until symptoms of heart failure are controlled." c. "Nonstrenuous activities can be performed as soon as antibiotics are started." d. "Bed rest must be maintained until antiinflammatory therapy has been discontinued."

18. b. When carditis is present in the patient with rheumatic fever, ambulation is postponed until any symptoms of heart failure are controlled with treatment and full activity cannot be resumed until antiinflammatory therapy has been discontinued. In the patient without cardiac involvement, ambulation may be permitted as soon as acute symptoms have subsided and normal activity can be resumed when antiinflammatory therapy is discontinued.

When planning care for a patient on a mechanical ventilator, the nurse understands that the application of positive end-expiratory pressure (PEEP) to the ventilator settings has which therapeutic effect? a. Increased inflation of the lungs b. Prevention of barotrauma to the lung tissue c. Prevention of alveolar collapse during expiration d. Increased fraction of inspired oxygen concentration (FIO2) administration

15. C PEEP is positive pressure that is applied to the airway during exhalation. This positive pressure prevents the alveoli from collapsing, improving oxygenation and enabling a reduced FIO2 requirement. PEEP does not cause increased inflation of the lungs or prevent barotrauma. Actually auto-PEEP resulting from inadequate exhalation time may contribute to barotrauma.

15. Priority Decision: What is the most important role of the nurse in preventing rheumatic fever? a. Teach patients with infective endocarditis to adhere to antibiotic prophylaxis. b. Identify patients with valvular heart disease who are at risk for rheumatic fever. c. Encourage the use of antibiotics for treatment of all infections involving a sore throat. d. Promote the early diagnosis and immediate treatment of group A streptococcal pharyngitis.

15. d. Initial attacks of rheumatic fever and the development of rheumatic heart disease can be prevented by adequate treatment of group A streptococcal pharyngitis. Because streptococcal infection accounts for only about 20% of acute pharyngitis, cultures should be done to identify the organism and direct antibiotic therapy. Viral infections should not be treated with antibiotics. Prophylactic therapy is indicated in those who have valvular heart disease or have had rheumatic heart disease.

A patient's daughter asks the nurse what SIMV means on the settings of the mechanical ventilator attached to her father. Which statement best describes this mode of ventilation? a. "SIMV provides additional inspiratory pressure so that your father does not have to work as hard to breathe, thus enabling better oxygenation and a quicker recovery with fewer complications." b. "SIMV is a mode that allows the ventilator to totally control your father's breathing. It will prevent him from hyperventilating or hypoventilating, thus ensuring the best oxygenation." c. "SIMV is a mode that allows your father to breathe on his own, but the ventilator will control how deep a breath he will receive. The ventilator can sense when he wants a breath, and it will deliver it." d. "SIMV is a mode that allows your father to breathe on his own while receiving a preset number of breaths from the ventilator. He can breathe as much or as little as he wants beyond what the ventilator will breathe for him."

16. D SIMV stands for synchronized intermittent mandatory ventilation, a mode of ventilation in which the ventilator delivers a preset tidal volume at a preset frequency in synchrony with the patient's spontaneous breathing. Between ventilator-delivered breaths the patient is able to breathe spontaneously, receiving the preset FIO2 but self-regulates the rate and depth of those breaths. Pressure support ventilation (PSV) applies positive pressure only during inspiration. PSV is not used as a sole ventilator support during acute respiratory failure because of the risk of hypoventilation, but it does decrease the work of breathing. Pressure-control inverse ratio ventilation (PC-IRV) sets the ventilation pressure and the ratio of inspiration to expiration to control the patient's breathing. Assist-control ventilation (ACV) or assisted mandatory ventilation (AMV) delivers a preset rate of breaths but allows the patient to breathe spontaneously, with a preset tidal volume.

• Causes of cardiogenic shock include _

_include acute myocardial infarction (AMI), cardiomyopathy, blunt cardiac injury, severe systemic or pulmonary hypertension, and myocardial depression from metabolic problems.

• The compensatory stage _

_is clinically apparent and involves neural, hormonal, and biochemical compensatory mechanisms in an attempt to overcome the increasing consequences of anaerobic metabolism and to maintain homeostasis.

Which hematologic problem most significantly increases the risks associated with pulmonary artery (PA) catheter insertion? a. Leukocytosis b. Hypovolemia c. Hemolytic anemia d. Thrombocytopenia

18. D PA catheter insertion carries a significant risk of bleeding, a fact that is exacerbated when the patient has low levels of platelets. Leukocytosis, hypovolemia, and anemia are less likely to directly increase the risks associated with PA insertion.

• The initial stage of shock that occurs at a cellular level _

_is usually not clinically apparent.

Which interventions should the nurse perform prior to suctioning a patient who has an endotracheal (ET) tube using open-suction technique (select all that apply)? a. Put on clean gloves. b. Administer a bronchodilator. c. Perform a cardiopulmonary assessment. d. Hyperoxygenate the patient for 30 seconds. e. Insert a few drops of normal saline into the ET to break up secretions.

17. C,D Suctioning is preceded by a thorough assessment and hyperoxygenation for 30 seconds. Sterile, not clean, gloves are necessary, and it is not necessary to administer a bronchodilator. Instillation of normal saline into the ET tube is not an accepted standard practice.

Natriuretic Peptides - ANP (Atrial Natriuretic Peptide)

Has opposite effect of renin-angiotensin-aldosterone. It causes relaxation, inhibits sympathetic nervous system, and lowers blood pressure, lower cardiac output, oppose sympathetic nervous system effect

Which factor indicates that tracheotomy would be preferable to endotracheal intubation? a. The patient is unable to clear secretions. b. The patient is at high risk for aspiration. c. A long-term airway is probably necessary. d. An upper airway obstruction is impairing the patient's ventilation.

19. C A tracheotomy is indicated when the need for an artificial airway is expected to be long term. Aspiration risk, an inability to clear secretions, and upper airway obstruction are indications for an artificial airway, but these are not specific indications for tracheotomy.

19. What is an effect of valvular regurgitation? a. It causes a pressure gradient difference across an open valve. b. A pericardial friction rub is heard on the right sternal border of the chest. c. It leads to decreased flow of blood and hypertrophy of the preceding chamber. d. There is a backward flow of blood and volume overload in the preceding chamber.

19. d. Valvular regurgitation causes a backward flow of blood and volume overload in the preceding chamber. Without treatment, eventually hypertrophy of that chamber occurs. Stenosis causes a pressure gradient difference and decreased blood flow and hypertrophy of the preceding chamber. A pericardial friction rub is not related to valvular regurgitation but would be heard at the lower left sternal border of the chest.

A patient presents to the emergency department (ED) in a state of shock. On assessment, the nurse finds that the patient is cyanotic and has crackles on auscultation of the lungs. As which type of shock will the nurse classify this? 1 Neurogenic shock 2 Cardiogenic shock 3 Hypovolemic shock 4 Anaphylactic shock

2 A patient with cardiogenic shock shows peripheral hypoperfusion presenting as cyanosis and has crackles on auscultation of the lungs due to pulmonary congestion. In neurogenic shock, the patient demonstrates symptoms related to the injury such as hypotension and bradycardia. The patient in hypovolemic shock may experience tachycardia as a late sign. In anaphylactic shock, the patient may experience wheezing and stridor. Text Reference - p. 1633

A patient in shock is receiving 0.9 % NaCl (normal saline solution-NSS). Which nursing intervention is appropriate for this patient? 1 Monitor the patient's vital signs 2 Monitor for the signs of circulatory overload 3 Monitor for signs of hypernatremia in the patient 4 Monitor for allergic reactions and acute renal failure

2 Circulatory overload occurs due to fluid overload. Continuous infusion of 0.9% NaCl increases the fluid volume in the body and may cause circulatory overload. The patient's vital signs must be checked during transfusion of blood or blood products because they could cause an infection or an allergic reaction. Hypernatremia occurs when the patient is on 1.8%, 3%, and 5% NaCl infusions. Infusion of dextran-40 has a tendency to precipitate allergic reactions and acute renal failure. Test-Taking Tip: Do not worry if you select the same numbered answer repeatedly, because there usually is no pattern to the answers. Text Reference - p. 1642

When caring for a critically ill patient who is being mechanically ventilated, the nurse will astutely monitor for which clinical manifestation of multiple organ dysfunction syndrome (MODS)? 1 Increased serum albumin 2 Decreased respiratory compliance 3 Increased gastrointestinal (GI) motility 4 Decreased blood urea nitrogen (BUN)/creatinine ratio

2 Clinical manifestations of MODS include symptoms of respiratory distress, signs and symptoms of decreased renal perfusion, decreased serum albumin and prealbumin, decreased GI motility, acute neurologic changes, myocardial dysfunction, disseminated intravascular coagulation (DIC), and changes in glucose metabolism. Serum albumin is not increased, GI motility decreases in MODS, and the BUN/Creatinine ratio likely will increase. Text Reference - p. 1649

Which drug helps manage renal manifestations in a patient with systemic inflammatory response syndrome (SIRS) and multiple organ dysfunction syndrome (MODS)? 1 Sucralfate 2 Furosemide 3 Omeprazole 4 Acetaminophen

2 Furosemide is a loop diuretic that helps to manage renal manifestations in a patient with systemic inflammatory response syndrome (SIRS) and multiple organ dysfunction syndrome (MODS). Sucrafate is administered for prophylaxis against stress ulcers, which are gastrointestinal manifestations of SIRS and MODS. Omeprazole is a proton pump inhibitor that has the same action. Acetaminophen is an antipyretic drug given as an acute intervention to manage fevers in patients who are in shock. Text Reference - p. 1651

Which type of shock is associated with hyperglycemia, presence of pulmonary infiltrates in chest x-ray and increased levels of blood urea nitrogen (BUN)? 1 Septic 2 Cardiogenic 3 Obstructive 4 Hypovolemic

2 Increased blood levels of glucose, nitrogen, cardiac markers and presence of pulmonary infiltrates are seen in cardiogenic shock. Increased blood levels of lactate, glucose, and positive blood cultures are signs of septic shock. Manifestations of obstructive shock are specific to the area or organ of obstruction. Electrolyte imbalances and decreased hemoglobin and hematocrit are seen in hypovolemic shock. Test-Taking Tip: Identify option components as correct or incorrect. This may help you identify a wrong answer. Text Reference - p. 1635

Which type of shock is associated with bradycardia? 1 Septic shock 2 Neurogenic shock 3 Hypovolemic shock 4 Anaphylactic shock

2 Neurogenic shock is associated with bradycardia. Myocardial dysfunction and changes in body temperature are signs of septic shock. Reduction in preload, capillary refill and stroke volume are clinical manifestations of hypovolemic shock. Chest pain is seen in anaphylactic shock. Test-Taking Tip: Make certain that the answer you select is reasonable and obtainable under ordinary circumstances and that the action can be carried out in the given situation. Text Reference - p. 1635

Cardiogenic Shock

Caused by an alteration in cardiac function, cardiac output decrease, hypotension, tissue hypoxis due to myocardial infarction, arrhythmias, CAD (coronary artery disease), cardiomyopathy

Which type of shock can be treated by minimizing spinal cord trauma with stabilization? 1 Septic shock 2 Neurogenic shock 3 Anaphylactic shock 4 Hypovolemic shock

2 Neurogenic shock is caused by severe injury to the spinal cord and results in loss of sympathetic stimulation of blood vessels. Apart from administering vasoconstrictor agents, minimizing the spinal cord trauma with stabilization is a supporting therapy for neurogenic shock. Septic shock occurs in response to a systemic infection. Obtaining the cultures before starting antibiotics is appropriate care for septic shock. A life-threatening allergic reaction to a sensitizing substance causes anaphylactic shock. Avoiding exposure to allergens is supportive therapy for anaphylactic shock. Excessive loss of intravascular fluid causes hypovolemic shock. Besides restoring fluid volume, correcting the cause of fluid loss is supportive therapy. Test-Taking Tip: If the question asks for an immediate action or response, all of the answers may be correct, so base your selection on identified priorities for action. Text Reference - p. 1645

Which cardiovascular change is commonly found in patients with systemic inflammatory response syndrome (SIRS)? 1 Decrease in heart rate 2 Decrease in capillary refill 3 Decrease in central venous pressure 4 Decrease in pulmonary artery wedge pressure

2 Patients with systemic inflammatory response syndrome (SIRS) have decreased capillary refill. Other cardiovascular changes include increases rather than decreases in heart rate, central venous pressure, and pulmonary artery wedge pressure. Test-Taking Tip: Multiple-choice questions can be challenging, because students think that they will recognize the right answer when they see it or that the right answer will somehow stand out from the other choices. This is a dangerous misconception. The more carefully the question is constructed, the more each of the choices will seem like the correct response. Text Reference - p. 1649

Which system of the body is often the first to show signs of dysfunction in systemic inflammatory response syndrome (SIRS) and multiple organ dysfunction syndrome (MODS)? 1 Neurologic system 2 Respiratory system 3 Cardiovascular system 4 Gastrointestinal system

2 Systemic inflammatory response syndrome (SIRS) and multiple organ dysfunction syndrome (MODS) occur due to a systemic inflammatory response. Inflammatory mediators have a direct effect on the pulmonary vasculature. Thus, the respiratory system is often the first system to show signs of dysfunction. Signs of nervous system dysfunction, such as mental changes, can be early signs of SIRS and MODS. However, the nervous system is not the first system to show signs of SIRS and MODS. When the respiratory system is affected, ventilation-perfusion mismatch becomes worse. Tissue oxygen demands increase, leading to cardiovascular changes. Hence, cardiovascular changes occur after changes in the respiratory system. In the early stages of SIRS and MODS, there is shunting away of blood from the gastrointestinal tract, making it vulnerable to ischemic injury. However, such changes show signs of dysfunction later than does the respiratory system. Text Reference - p. 1649

A patient admitted to the hospital after a motor vehicle accident (MVA) is in hypovolemic shock. On examination, the nurse finds that the patient is becoming anxious, and the urine output is decreasing. What appropriate action should the nurse perform? 1 Prepare for administering blood products. 2 Begin crystalloid fluid replacement. 3 Start fluids only if deterioration occurs. 4 Wait for the patient to compensate naturally.

2 When the volume of blood loss is less than 30 percent, crystalloid fluid replacements are performed to reverse tissue dysfunction. Blood products are administered when the blood volume loss is more than 30 percent. The nurse should not wait for deterioration to occur to start the fluid replacement therapy. This is because the body can typically compensate naturally for a blood volume loss up to 15 percent. Test-Taking Tip: Be alert for grammatical inconsistencies. If the response is intended to complete the stem (an incomplete sentence) but makes no grammatical sense to you, it might be a distractor rather than the correct response. Question writers typically try to eliminate these inconsistencies. Text Reference - p. 1633

A nurse is examining a patient with anaphylactic shock due to an insect bite. What types of skin manifestations would the nurse expect to find? Select all that apply. 1 Pallor 2 Pruritus 3 Flushing 4 Urticaria 5 Cold, clammy skin

2, 3, 4 Insect bites may cause allergic reactions and anaphylactic shock. The skin manifestations may include pruritus, flushing, and urticaria caused by massive vasodilation, release of vasoactive mediators, and an increase in capillary permeability. Pallor and cold, clammy skin changes are usually seen in cardiogenic, hypovolemic, and obstructive shock caused by decreased circulatory volume and tissue perfusion. Text Reference - p. 1636

While planning the management of oxygen delivery in a patient with shock, what appropriate measures should the nurse undertake? Select all that apply. 1 Encourage the patient to move around to increase lung expansion. 2 Space activities that increase oxygen consumption. 3 Monitor continuously by using a central venous catheter. 4 Space activities that decrease oxygen consumption. 5 Administer supplemental oxygen as prescribed.

2, 3, 5 To optimize oxygen supply and ventilation in a patient suffering from shock, the activities that increase oxygen consumption should be evenly spaced. Mixed venous oxygen saturation should be monitored through a central venous catheter. The patient should not exert energy by excessive moving around because it increases oxygen demand. Text Reference - p. 1641

When examining a patient in the progressive stage of shock, which factors related to the gastrointestinal (GI) system should the nurse consider? Select all that apply. 1 Increased motility and peristalsis 2 Increased likelihood of GI ulcers 3 Increased ability to absorb nutrients 4 Increased risk of GI bleeding 5 Increased risk of bacterial migration from the GI tract to the bloodstream.

2, 4, 5 In the progressive stage of shock, the GI system gets affected by prolonged decreased tissue perfusion. As the blood supply to the GI tract is decreased, the normally protective mucosal barrier becomes ischemic. This ischemia predisposes the patient to ulcers and GI bleeding. It also increases the risk of bacterial migration from the GI tract to the blood. The decreased perfusion to the GI tract also results in a decreased ability to absorb nutrients, decreased motility, and slowed peristalsis. Text Reference - p. 1639

The client diagnosed with a DVT is placed on a medical unit. Which nursing interventions should be implemented? Select all that apply. 2. Instruct the client to stay in bed and not ambulate. 3. Encourage fluids and a diet high in roughage. 4. Monitor IV site every shift and PRN.

2. Clients should be on bed rest for five (5) to seven (7) days after diagnosis to allow time for the clot to adhere to the vein wall, thereby preventing embolization. 3. Bed rest and limited activity predispose the client to constipation. Fluids and diets high in fiber will help prevent constipation. Fluids will also help provide adequate fluid volume in the vasculature. 4. The client will be administered a heparin IV drip, which should be monitored.

What are the appropriate nursing interventions for the patient with delirium in the ICU (SATA)? a. Use clocke\s and calendars to maintain orientation b. Encourage round the clock presence of CG at the bedside c. Sedate the patient with appropriate drugs to protect the patient form harmful behaviors d. Silence all alarms, reduce overhead paging and avoid conversations around the patient e. Identify physiologic factors that may be contributing to the patient's confusion and irritability

2. Correct answers: a, c, e Rationale: The use of clocks and calendars can help orient the patient with delirium in the intensive care unit (ICU). If the patient demonstrates hyperactivity, insomnia, or delusions, management with neuroleptic drugs (e.g., haloperidol [Haldol]) can be considered. Physical conditions such as hemodynamic instability, hypoxemia, hypercarbia, electrolyte disturbances, and severe infections can precipitate delirium.

The nurse is developing a discharge-teaching plan for the client diagnosed with congestive heart failure. Which interventions should be included in the plan? Select all that apply.

2. Teach client how to count the radial pulse when taking digoxin, a cardiac glycoside. 3. Instruct client to remove the saltshaker from the dinner table.

Collaborative Care: Septic shock_• Patients in septic shock require _

_large amounts of fluid replacement; the goal is to achieve a target central venous pressure (CVP) of 8 to 12 mm Hg.

The nurse is caring for clients on a telemetry floor. Which nursing task would be most appropriate to delegate to an unlicensed nursing assistant? 2. Assist feeding the client diagnosed with congestive heart failure.

2. The nursing assistant can feed a client.

The nurse is caring for a client receiving heparin sodium via constant infusion. The heparin protocol reads to increase the IV rate by 100 units/hour if the PTT is less than 50 seconds. The current PTT level is 46 seconds. The heparin comes in 500 mL of D5W with 25,000 units of heparin added. The current rate on the IV pump is 18 mL per hour. At what rate should the nurse set the pump?

20 mL per hour

The patient has developed cardiogenic shock after a left anterior descending myocardial infection. Which circulatory-assist device should the nurse expect to use for this patient? a. Cardiopulmonary bypass b. Impedance cardiography (ICG) c. Intraaortic balloon pump (IABP) d. Central venous pressure (CVP) measurement

20. C The most commonly used mechanical circulatory-assist device is the intraaortic balloon pump (IABP), and it is used to decrease ventricular workload, increase myocardial perfusion, and augment circulation. Cardiopulmonary bypass provides circulation during open heart surgery. It is not used as an assist device after surgery. ICG is a noninvasive method to obtain cardiac output and assess thoracic fluid status. CVP measurement is an invasive measurement of right ventricular preload and reflects fluid volume problems.

20. Delegation Decision: An RN is working with a licensed practical nurse (LPN) in caring for a group of patients on a cardiac telemetry unit. A patient with aortic stenosis has the nursing diagnosis of activity intolerance related to fatigue and exertional dyspnea. Which nursing activity could be delegated to the LPN? a. Explain the reason for planning frequent periods of rest. b. Evaluate the patient's understanding of his disease process. c. Monitor BP, HR, RR, and SpO2 before, during, and after ambulation. d. Teach the patient which activities to choose that will gradually increase endurance.

20. c. Monitoring vital signs before and after ambulation is the collection of data. Instructions should be provided to the licensed practical nurse (LPN) regarding what changes in these vital signs should be reported to the RN. The other actions listed are RN responsibilities.

A 70-year-old patient in the ICU has become agitated and inattentive since his heart surgery. The nurse knows that this ICU psychosis frequently occurs in individuals with pre-existing dementia, history of alcohol abuse, and severe disease. What interventions should the nurse provide this patient to improve the patient's cognition (select all that apply)? a. Improve oxygenation. b. Provide a small amount of beer. c. Have the family stay with the patient. d. Enable the patient to sleep on a schedule with dim lights. e. Decrease sensory overload by conversing away from patient's room.

21. A,D,E ICU psychosis is from delirium in most ICU patients. Improving oxygenation, enabling the patient to sleep, and decreasing sensory overload along with orientation is all helpful in improving the patient's cognition. The beer may or may not be allowed for this patient, and the nurse should not assume that it will help. Having a family member stay with the patient to reorient the patient is helpful, but the family group may increase sensory overload with conversations not involving the patient.

The post-anesthesia care unit (PACU) has several patients with endotracheal tubes. Which patient should receive the least amount of endotracheal suctioning? a. Transplantation of a kidney b. Replacement of aortic valve c. Cerebral aneurysm resection d. Formation of an ileal conduit

22. C The nurse should avoid suctioning the patient after a craniotomy until it is necessary because suctioning will increase this patient's intracranial pressure. The patients with a kidney transplantation, aortic valve replacement, or formation of an ileal conduit will not be negatively affected by suctioning, although it should only be done when needed, not routinely.

• Immediate reaction in anaphylaxis causes _

_massive vasodilation, release of vasoactive mediators, and an increase in capillary permeability resulting in fluid leaks from the vascular space into the interstitial space.

21. What accurately describes mitral valve prolapse? a. Rapid onset prevents left chamber dilation b. May be caused by pulmonary hypertension c. Ballooning of valve into left atrium during ventricular systole d. Rapid development of pulmonary edema and cardiogenic shock

21. c. Mitral valve prolapse is the ballooning of the valve leaflets into the left atrium during ventricular systole. The rapid onset that prevents left chamber dilation and the rapid development of pulmonary edema and cardiogenic shock occur with acute mitral regurgitation. Pulmonary hypertension may contribute to tricuspid valve disease.

Chronic stable angina charateristics

Episodic pain lasting a few minutes, provoked by exertion or stress, Relieved by rest or nitroglycerin

22. What causes a sudden onset of cardiovascular collapse? a. Mitral stenosis b. Tricuspid valve disease c. Pulmonic valve stenosis d. Acute aortic regurgitation

22. d. Acute aortic regurgitation causes a sudden cardiovascular collapse. With mitral valve stenosis dyspnea is a prominent symptom and embolization may result from chronic atrial fibrillation. With tricuspid and pulmonic valve diseases, stenosis occurs more often than regurgitation. Tricuspid valve stenosis results in right atrial enlargement and elevated systemic venous pressures. Pulmonic valve stenosis results in right ventricular hypertension and hypertrophy.

• Protein-calorie malnutrition is one of the main manifestations _

_of hypermetabolism in shock; nutrition is vital to decreasing morbidity from shock.

23. The patient is admitted with angina, syncope, and dyspnea on exertion. In the assessment, the nurse notes a systolic murmur with a prominent S4. What will the nurse suspect is occurring with this patient? a. Mitral valve stenosis b. Aortic valve stenosis c. Acute mitral valve regurgitation d. Chronic mitral valve regurgitation

23. b. Aortic valve stenosis is identified with the triad of angina, syncope, and dyspnea on exertion, as well as the systolic murmur and prominent S4 heart sound. Mitral valve stenosis manifests as exertional dyspnea, hemoptysis, fatigue, atrial fibrillation, and a diastolic murmur. Acute mitral valve regurgitation has a new systolic murmur with pulmonary edema and cardiogenic shock rapidly developing. Chronic mitral valve regurgitation is identified with weakness, fatigue, exertional dyspnea, palpitations, an S3 gallop, and holosystolic murmur.

o The respiratory system is often the first system to show signs of dysfunction in SIRS and MODS,_

_often culminating in acute respiratory distress syndrome (ARDS).

Collaborative care: Anaphylactic shock_• Aggressive fluid replacement, _

_predominantly with colloids, is necessary.

24. Which drugs would the nurse expect to be included in those prescribed for patients with a mechanical valve replacement? a. Oral nitrates b. Anticoagulants c. Atrial antidysrhythmics d. β-adrenergic blocking agents

24. b. Patients with mechanical valves have an increased risk for thrombus formation. Therefore prophylactic anticoagulation therapy is used to prevent thrombus formation and systemic or pulmonary embolization. Nitrates are contraindicated for the patient with aortic stenosis because an adequate preload is necessary to open the stiffened aortic valve. Antidysrhythmics are used only if dysrhythmias occur and β-adrenergic blocking drugs may be used to control the heart rate if needed.

25. Priority Decision: A patient with symptomatic mitral valve prolapse has atrial and ventricular dysrhythmias. In addition to monitoring for decreased cardiac output related to the dysrhythmias, what is an important nursing intervention related to the dysrhythmias identified by the nurse? a. Monitor breathing pattern related to hypervolemia. b. Encourage calling for assistance when getting out of bed. c. Give sleeping pills to decrease paroxysmal nocturnal dyspnea. d. Teach the patient exercises to prevent recurrence of dysrhythmias.

25. b. Dysrhythmias frequently cause palpitations, lightheadedness, and dizziness and the patient should be carefully attended to to prevent falls. Hypervolemia and paroxysmal nocturnal dyspnea (PND) would be apparent in the patient with heart failure. Exercises will not prevent dysrhythmias.

During the irreversible stage, the patient will demonstrate _

_profound hypotension and hypoxemia, as well as organ failure; at this stage, recovery is unlikely.

26. A patient is scheduled for a percutaneous transluminal balloon valvuloplasty. The nurse understands that this procedure is indicated for which patient? a. Any patient with aortic regurgitation b. Older patients with aortic regurgitation c. Older patients with stenosis of any valve d. Young adult patients with mild mitral valve stenosis

26. c. This procedure has been used for repair of mitral, tricuspid, and pulmonic stenosis and less often for aortic stenosis. It is usually used for older patients and for those patients who are poor surgical risks because it is relatively easy and has good results and few complications.

With Cardiogenic shock,the overall goal is to _

_restore blood flow to the myocardium by restoring the balance between oxygen supply and demand.

27. A patient is scheduled for an open surgical valvuloplasty of the mitral valve. In preparing the patient for surgery, what should the nurse know about this surgery? a. Cardiopulmonary bypass is not required with this procedure. b. Valve repair is a palliative measure, whereas valve replacement is curative. c. The operative mortality rate is lower in valve repair than in valve replacement. d. Patients with valve repair do not require postoperative anticoagulation as do those who have valve replacement.

27. c. Repair of mitral or tricuspid valves has a lower operative mortality rate than does replacement and is becoming the surgical procedure of choice for these valvular diseases. Open repair is more precise than closed repair and requires cardiopulmonary bypass during surgery. All types of valve surgery are palliative, not curative, and patients require lifelong health care. Anticoagulation therapy is used for all valve surgery for at least some time postoperatively.

A patient's localized infection has progressed to the point where septic shock now is suspected. What medication is an appropriate treatment modality for this patient? 1 Insulin infusion 2 Intravenous (IV) administration of epinephrine 3 Aggressive IV crystalloid fluid resuscitation 4 Administration of nitrates and β-adrenergic blockers

3 Patients in septic shock require large amounts of crystalloid fluid replacement. Epinephrine is indicated in anaphylactic shock, and insulin infusion is not normally necessary in the treatment of septic shock (but can be). Nitrates and β-adrenergic blockers are used most often in the treatment of patients in cardiogenic shock. Text Reference - p. 1636

28. In which patient would a mechanical prosthetic valve be preferred over a biologic valve for valve replacement? a. 41-year-old man with peptic ulcer disease b. 22-year-old woman who desires to have children c. 35-year-old man with a history of seasonal asthma d. 62-year-old woman with early Alzheimer's disease

28. c. Mechanical prosthetic valves require long-term anticoagulation and this is a factor in making a decision about the type of valve to use for replacement. Patients who cannot take anticoagulant therapy, such as women of childbearing age, patients at risk for hemorrhage, patients who may not be compliant with anticoagulation therapy, and patients over age 65 may be candidates for the less durable biologic valves.

• Septic shock is the presence of _

_sepsis with hypotension despite fluid resuscitation along with the presence of inadequate tissue perfusion.

29. When performing discharge teaching for the patient following a mechanical valve replacement, the nurse determines that further instruction is needed when the patient says which statement? a. "I may begin an exercise program to gradually increase my cardiac tolerance." b. "I will always need to have my blood checked once a month for its clotting function." c. "I should take prophylactic antibiotics before I have dental or invasive medical procedures." d. "The biggest risk I have during invasive health procedures is bleeding because of my anticoagulants."

29. d. The greatest risk to a patient who has an artificial valve is the development of endocarditis with invasive medical or dental procedures. Before any of these procedures, antibiotic prophylaxis is necessary to prevent infection. Planning of an exercise program and monitoring anticoagulant therapy will be done.

Collaborative Care: Hypovolemic shock_• The underlying principles of managing patients with hypovolemic shock focus on _

_stopping the loss of fluid and restoring the circulating volume.

The nurse assesses a patient with multisystem organ dysfunction syndrome. What assessment finding is most indicative of deterioration? 1 Arterial PO2 of 95% 2 Pulse rate of 108 beats/minute 3 Total urine output of 120 mL over the past eight hours 4 Auscultation of fine bilateral crackles and a moist cough

3 A decrease in urine output to less than 30 mL/hr in an adult is an early indication of hypoperfusion to the kidneys, as well as other vital organs. This may or may not be accompanied by changes in vital signs. An arterial PO2 of 95%, a pulse rate of 108 beats/minute, and auscultation of crackles and a moist cough may also be indications of early deterioration but are not as specific as a low urinary output in identifying deterioration. Text Reference - p. 1649

A patient is brought to the emergency department (ED) after multiple bee stings. On assessment, the nurse finds that the patient has edema on the lips and tongue as well as chest pain, dizziness, wheezing, and stridor. What type of shock should the nurse document this as? 1 Septic shock 2 Neurogenic shock 3 Anaphylactic shock 4 Obstructive shock

3 Anaphylactic shock is an acute, life-threatening hypersensitivity reaction to a sensitizing substance that, in this case, is insect venom. The reaction quickly causes massive vasodilation, release of vasoactive mediators, and an increase in capillary permeability. As capillary permeability increases, fluid leaks from the vascular space into the interstitial space. The consequences of these pathophysiologic processes include edema on the lips and tongue, chest pain, wheezing, and stridor. Sepsis is a systemic inflammatory response to a documented or suspected infection. Neurogenic shock is a hemodynamic phenomenon that can occur within 30 minutes of a spinal cord injury at the fifth thoracic (T5) vertebra or above. Obstructive shock develops when a physical obstruction to blood flow occurs with decreased cardiac output. Text Reference - p. 1636

Why is there a loss of lean body mass in patients with systemic inflammatory response syndrome (SIRS) and multiorgan dysfunction syndrome (MODS)? 1 Hypoglycemia occurs. 2 Glucose is converted to glycogen. 3 Fatty acids are mobilized for fuel. 4 Glucose is converted to amino acids.

3 Both SIRS and MODS trigger a hypermetabolism response leading to mobilization of fatty acids for fuel. Such a catabolic state leads to loss of lean body mass. Because catecholamines and glucocorticoids are released, hyperglycemia occurs, not hypoglycemia. Hyperglycemia also occurs, because glycogen stores are converted into glucose. Once glycogen is depleted, amino acids are converted into glucose and there is a reduction in protein stores. Text Reference - p. 1649

Which medical emergency is caused by the failure of two or more organ systems? 1 Crush syndrome 2 Toxic shock syndrome 3 Multiple organ dysfunction syndrome (MODS) 4 Systemic inflammatory response syndrome (SIRS)

3 Multiple organ dysfunction syndrome is caused by the failure of two or more organ systems in an acutely ill patient. A crushing injury to the skeletal muscle causes crush syndrome, which is characterized by shock and renal failure. Bacterial toxins cause toxic shock syndrome, which is characterized by high fever, hypotension and malaise. Systemic inflammatory response syndrome (SIRS) is a systemic inflammatory response caused by infection, ischemia, infarction and injury. Test-Taking Tip: Identifying content and what is being asked about that content is critical to your choosing the correct response. Be alert for words in the stem of the item that are the same or similar in nature to those in one or two of the options. Text Reference - p. 1649

The nurse is caring for a patient who developed cardiogenic shock. Which medical diagnosis does the nurse suspect? 1 Urosepsis 2 Hemorrhage 3 Myocardial infarction 4 Tension pneumothorax

3 Myocardial infarction may produce necrotic areas of cardiac tissue that lead to impaired contractility and decreased cardiac output. This may lead to a cardiogenic shock state. Hemorrhage may lead to a hypovolemic shock state, tension pneumothorax may lead to an obstructive shock state, and urosepsis may lead to a septic shock state. Text Reference - p. 1633

A patient in neurogenic shock is receiving phenylephrine. Which nursing actions are appropriate when caring for this patient? 1 Monitoring for signs of dyspnea and pulmonary edema 2 Monitoring for signs of hypokalemia and hyperglycemia 3 Monitoring for signs of reflex bradycardia and restlessness 4 Monitoring for signs of hypothyroidism and Addison's disease

3 Phenylephrine is α-adrenergic agonist and may cause bradycardia and restlessness due to central nervous system stimulation. β-adrenergic agonists such as epinephrine cause dyspnea and pulmonary edema. Phenylephrine does not increase the elimination of potassium levels or blood glucose levels. Therefore, the patient does not have risk of hypokalemia and hyperglycemia. Phenylephrine does not impair thyroid and adrenal gland functioning. Therefore, the nurse will not monitor for the signs of hypothyroidism and Addison's disease. Test-Taking Tip: Identify option components as correct or incorrect. This may help you identify a wrong answer. Text Reference - p. 1643

CHEMICAL INDUCED DISTRIBUTIVE SHOCK

3 common origins: -anaphylaxis- one result of type I allergic reactions. Result is widespread loss of blood vessel tone & decreased cardiac output. -Sepsis- widespread infection that triggers a whole body inflammatory response. Leads to distributive shock when infectious microorganisms are present in blood. -Capillary leak syndrome- The response of capillaries to the presence of biologic mediators that change blood vessel integrity & allow fluid to shift from the blood vessels into the interstitial tissues. One in tissue, fluids are stagnant & can't deliver oxygen or remove tissue waste products. -Occurs when certain body chemicals or foreign substances in the blood & vessels start widespread changes in blood vessel walls. -Problems causing fluid shift: severe burns, liver disorders, ascites, peritonitis, paralytic ileus, severe malnutrition, large wounds .

A patient is admitted to the hospital with a suspected diagnosis of obstructive shock. What could be the possible causes of this type of shock? Select all that apply. 1 Hypersensitivity to antibiotics 2 Spinal cord injury 3 Cardiac tamponade 4 Tension pneumothorax 5 Superior vena cava syndrome

3, 4, 5 Obstructive shock develops when a physical obstruction to blood flow occurs resulting in decreased cardiac output. This can be caused by restricted diastolic filling of the right ventricle from compression caused by cardiac tamponade, tension pneumothorax, or superior vena cava syndrome. Hypersensitivity to antibiotics may cause anaphylactic shock. Spinal cord injury may lead to neurogenic shock. Text Reference - p. 1637

The nurse is administering oxygen therapy to a patient in septic shock. What are the possible factors that directly affect oxygen delivery in the patient and should be monitored? Select all that apply. 1 Urine output 2 White blood cells 3 Cardiac output 4 Available hemoglobin 5 Arterial oxygen saturation

3, 4, 5 Oxygen delivery depends on cardiac output, available hemoglobin, and arterial oxygen saturation (SaO2). The amount of blood that the heart pumps to the body may decide the amount of oxygen delivered to the tissues. Hemoglobin, the protein content in red blood cells, is responsible for carrying oxygen molecules. Low hemoglobin means a low oxygen supply to the tissues. Arterial oxygen saturation indicates the total oxygen carried by the blood in the arteries and implies the level of tissue oxygenation. The urine output and white blood cells are also important parameters to be monitored in a patient who suffers from a shock. However, these do not impact the oxygen delivery directly. Text Reference - p. 1641

The telemetry nurse is unable to read the telemetry monitor at the nurse's station. Which intervention should the telemetry nurse implement first?

3. Contact the client on the client call system.

• In severe sepsis and septic shock,

_the body's response to infection is exaggerated, resulting in an increase in inflammation and coagulation, and a decrease in fibrinolysis.

30. The patient is admitted post-radiation therapy with symptoms of cardiomyopathy (CMP). Which type of CMP should the nurse suspect that the patient is experiencing? a. Dilated b. Restrictive c. Takotsubo d. Hypertrophic

30. b. A secondary cause of restrictive cardiomyopathy (CMP) is radiation treatment to the thorax with stiffness of the ventricular wall occurring. Dilated CMP may have a genetic link, follow infectious myocarditis, or be related to an autoimmune process or excess alcohol ingestion. Takotsubo CMP is an acute stress-related syndrome that mimics acute coronary syndrome. It is most common in postmenopausal women. Hypertrophic CMP has a genetic link in about one half of all cases and is frequently seen in young athletic individuals.

31. What accurately describes dilated CMP (select all that apply)? a. Characterized by ventricular stiffness b. The least common type of cardiomyopathy c. The hyperdynamic systolic function creates a diastolic failure d. Echocardiogram reveals cardiomegaly with thin ventricular walls e. Often follows an infective myocarditis or exposure to toxins or drugs f. Differs from chronic heart failure in that there is no ventricular hypertrophy

31. d, e, f. Dilated CMP, the most common type of CMP, reveals cardiomegaly with thin ventricular walls on echocardiogram, as there is no ventricular hypertrophy, and may follow an infective myocarditis. As well, stasis of blood in the ventricles may contribute to systemic embolization. Restrictive CMP is the least common type and is characterized by ventricular stiffness. Hypertrophic CMP has hyperdynamic systolic function creating a diastolic failure, is characterized by massive thickening of intraventricular septum and ventricular wall, and may result in syncope during increased activity resulting from an obstructed aortic valve outflow.

33. When performing discharge teaching for a patient with any type of CMP, what should the nurse instruct the patient to do (select all that apply)? a. Eat a low-sodium diet. b. Go to the gym every day. c. Engage in stress reduction activities. d. Abstain from alcohol and caffeine intake. e. Avoid strenuous activity and allow for periods of rest. f. Suggest that caregivers learn cardiopulmonary resuscitation (CPR).

33. a, c, d, e, f. These topics can apply to any patient with CMP.

• The primary goal of drug therapy for shock is _

_the correction of decreased tissue perfusion. Vasopressor or vasodilator therapy is used according to patient needs to maintain the mean arterial pressure at the appropriate level.

• There is no specific diagnostic study to determine shock, _

_the diagnosis depends on the history and physical.

Collaborative care: Anaphylactic shock_• Epinephrine is _

_the drug of choice to treat anaphylactic shock

• MODS results from SIRS and is _

_the failure of two or more organ systems such that homeostasis cannot be maintained without intervention.

What type of medication does the nurse anticipate being prescribed by the health care provider to manage confusion, disorientation, and delirium in a patient with systemic inflammatory response syndrome (SIRS) and multiple organ dysfunction syndrome (MODS)? 1 Vasopressors 2 Loop diuretics 3 Proton pump inhibitors 4 Calcium channel blockers

4 Impaired perfusion of the brain may cause confusion, disorientation, and delirium in the patient. The health care provider is likely to prescribe calcium channel blockers to a patient exhibiting confusion, disorientation, and delirium to reduce cerebral vasospasm and improve perfusion of the brain. Vasopressors may be prescribed to combat cardiovascular dysfunction. Loop diuretics are prescribed if there is renal dysfunction. Proton pump inhibitors are prescribed to manage gastrointestinal symptoms. Text Reference - p. 1651

Collaborative Care: Septic shock_• Antibiotics are an important component of therapy and should be started within _

_the first hour of septic shock.

A patient is diagnosed with multiple organ dysfunction syndrome. While aggressive treatment is continued, the nurse suspects infection. What is the most appropriate action that the nurse should perform? 1 Discontinue the aggressive treatment. 2 Reduce oxygen delivery to the patient. 3 Wait for laboratory reports to confirm the suspicion. 4 Obtain a prescription for broad-spectrum antibiotic therapy.

4 If an infection is suspected, broad-spectrum antibiotics should be started immediately to limit the infection. Aggressive treatment for infection control should be carried out in parallel. These patients are usually hypoxemic. Therefore, oxygen should be administered strictly as prescribed. Cultures can be sent, and based on the reports, specific antibiotics can be added. Test-Taking Tip: Identify option components as correct or incorrect. This may help you identify a wrong answer. Example: If you are being asked to identify a diet that is specific to a certain condition, your knowledge about that condition would help you choose the correct response (e.g., cholecystectomy = low-fat, high-protein, low-calorie diet). Text Reference - p. 1650

Following coronary artery bypass graft surgery a patient has postoperative bleeding that requires returning to surgery to repair the leak. During surgery, the patient has a myocardial infarction (MI). After restoring the patient's body temperature to normal, which patient assessment is the most important for planning nursing care? 1 Cardiac index (CI) 5 L/min/m2 2 Central venous pressure (CVP) 8 mm Hg 3 Mean arterial pressure (MAP) 86 mm Hg 4 Pulmonary artery pressure (PAP) 28/14 mm Hg

4 Pulmonary hypertension as indicated by an elevated PAP indicates impaired forward flow of blood because of left ventricular dysfunction or hypoxemia. Both can be caused by the MI. The CI, CVP, and MAP readings are normal. Text Reference - p. 1639

What is the clinical manifestation of systemic inflammatory response syndrome (SIRS) and multiple organ dysfunction syndrome (MODS) on the respiratory system? 1 Pulmonary edema 2 Pulmonary fibrosis 3 Pulmonary embolism 4 Pulmonary hypertension

4 Systemic inflammatory response syndrome (SIRS) and multiple organ dysfunction syndrome (MODS) affect the respiratory system and lead to pulmonary hypertension. Pulmonary edema is caused by fluid accumulation in the air spaces. Pulmonary fibrosis is a respiratory disorder caused by scars in the lung tissues or inhalation of airborne toxins. A pulmonary embolism is caused by blood clots or blockage in the pulmonary arteries. Text Reference - p. 1651

What is the goal in the care of a systemic inflammatory response syndrome (SIRS) patient whose bilirubin level is 3 mg/dL? 1 Patient will be free of stress ulcers. 2 Patient will not feel abdominal distension. 3 Patient will maintain intraabdominal pressures. 4 Patient will maintain adequate tissue perfusion.

4 The nurse will plan to maintain adequate tissue perfusion for a systemic inflammatory response syndrome (SIRS) patient with bilirubin level of 3 mg/dL. An elevation of the bilirubin indicates impaired liver function. Stress ulcer prophylaxis is routine and would have been initiated before SIRS occurred. Abdominal distention and increased intraabdominal pressures are associated with impaired liver function. Monitoring these conditions is essential but is of a lower priority than maintaining tissue perfusion. Text Reference - p. 1651

What laboratory finding correlates with a medical diagnosis of cardiogenic shock? 1 Decreased liver enzymes 2 Increased white blood cells 3 Decreased red blood cells, hemoglobin, and hematocrit 4 Increased blood urea nitrogen (BUN) and serum creatinine levels

4 The renal hypoperfusion that accompanies cardiogenic shock results in increased BUN and creatinine levels. Impaired perfusion of the liver results in increased liver enzymes, whereas white blood cell levels typically do not increase in cardiogenic shock. Red blood cell indices are typically normal because of relative hypovolemia. Test-Taking Tip: Bring to your test prep with a positive attitude about yourself, your nursing knowledge, and your test-taking abilities. A positive attitude is achieved through self-confidence gained by effective study. This means (a) answering questions (assessment), (b) organizing study time (planning), (c) reading and further study (implementation), and (d) answering questions (evaluation). Text Reference - p. 1633

Which sign of neurologic dysfunction is commonly seen in both systemic inflammatory response syndrome (SIRS) and multiple organ dysfunction syndrome (MODS)? 1 Increased heart rate 2 Increased liver enzymes 3 Difficulty breathing 4 Confusion, agitation, and lethargy

4 The sign of neurologic dysfunction commonly seen in systemic inflammatory response syndrome (SIRS) and multiple organ dysfunction syndrome (MODS) is change in mental status, which may cause the patient to become confused, agitated, and lethargic. The patient's heart rate increases due to changes in the cardiovascular system. The patient's liver enzymes increase due to dysfunction in the hepatic system and finally the dysfunction results in hepatic encephalopathy. The patient's dyspnea is caused by changes in the respiratory system by inflammatory mediators. Test-Taking Tip: Identify option components as correct or incorrect. This may help you identify a wrong answer. Text Reference - p. 1649

The nurse reviews the plan of care for a patient with multisystem organ dysfunction syndrome. What is the most desirable outcome for the patient? 1 The patient will be free of signs and symptoms of sepsis 2 The patient will maintain a balanced fluid intake and output 3 The patient will experience enhanced overall well-being and mental rest 4 The patient will demonstrate improved perfusion and oxygenation of organs

4 The underlying pathophysiology of multisystem organ dysfunction syndrome (MODS) is a lack of perfusion to organs, resulting in tissue and/or organ hypoxia. Interventions to improve perfusion with fluids or medications improve patient outcomes. The outcomes listed in the other answer options are appropriate and desirable for the patient with MODS, but they are secondary to improved perfusion and oxygenation. Text Reference - p. 1647

Collaborative care: Neurogenic shock_• Treatment of hypotension and bradycardia involves _

_the use of vasopressors and atropine, respectively. Fluids are administered cautiously; the patient is monitored for hypothermia.

• Sepsis is a systemic inflammatory response _

_to a documented or suspected infection.

A patient is admitted to the emergency department (ED) for shock of unknown etiology. The first action by the nurse should be to a. administer oxygen. b. obtain a 12-lead electrocardiogram (ECG). c. obtain the blood pressure. d. check the level of consciousness.

ANS: A The initial actions of the nurse are focused on the ABCs—airway, breathing, and circulation—and administration of oxygen should be done first. The other actions should be accomplished as rapidly as possible after oxygen administration.

Which intervention should the nurse implement when administering a loop diuretic to a client diagnosed with coronary artery disease? 2. Assess the client's serum potassium level.

Loop diuretics cause potassium to be lost in the urine output. Therefore, the nurse should assess the client's potassium level, and if the client is hypokalemic, the nurse should question administering this medication.

Which assessment data would support that the client has a venous stasis ulcer? 1. Superficial pink open area on the medial part of the ankle.

The medial part of the ankle usually ulcerates because of edema that leads to stasis, which, in turn, causes the skin to break down.

Human serum albumin (5%), plasma protein fraction (5% albumin in 500 mL NSS) - Nursing Implications

Monitor for circulatory overload Mild S/E chills/fever/urticaria may develop More expensive than other colloids

The hemodynamic changes the nurse expects to find after successful initiation of IABP in a patient with cardiogenic shock include (SATA)? a. decreased SV b. decreased SVR c. decrased PAWP d. increased DBP e. decreased myocardial O2 consumption

5. Correct answers: b, c, d, e Rationale: The hemodynamic effects of intraaortic balloon inflation during diastole include increased diastolic blood pressure (BP), increased pressure in the aortic root, increased coronary artery perfusion pressure, and improved oxygen delivery to the myocardium. The hemodynamic effects of intraaortic balloon inflation during systole include decreased afterload (i.e., systemic vascular resistance [SVR]), decreased peak systolic pressure, decreased myocardial oxygen consumption, increased stroke volume (SV), and decreased preload (i.e., decreased pulmonary artery [PA] pressures), including decreased pulmonary artery wedge pressure (PAWP).

5. A patient with infective endocarditis of a prosthetic mitral valve develops a left hemiparesis and visual changes. What should the nurse expect to be included in collaborative management of the patient? a. Embolectomy b. Surgical valve replacement c. Administration of anticoagulants d. Higher than usual antibiotic dosages

5. b. Early valve replacement followed by prolonged antibiotic and anticoagulant therapy is recommended for these patients. Drug therapy for patients who develop endocarditis of prosthetic valves is often unsuccessful in eliminating the infection and preventing embolization.

If the patient in shock is to receive 1000 mL of normal saline in two hours, at what rate should the infusion pump be set? Fill in the blank. ___mL/hour

500 For the 1000 mL of normal saline to be infused in two hours, the infusion pump should be set at 500 mL per hour (1000 mL divided by two hours). Test-Taking Tip: When taking the NCLEX exam, an on-screen calculator will be available for you to determine your response, which you will then type in the provided space. Text Reference - p. 1641

The client with pericarditis is prescribed a nonsteroidal anti-inflammatory drug (NSAID). Which teaching instruction should the nurse discuss with the client? 3. Instruct the client to take the medication with food.

NSAIDs must be taken with food, milk, or antacids to help decrease gastric distress. NSAIDs reduce fever, inflammation, and pericardial pain.

Assess lips, nailbeds, conjunctiva, and oral mucous membranes For persons of darker skin, assessment of the effects of shock on skin can be detected by assessing the client's lips, nailbeds, conjunctiva, and oral mucosa. Assessing only generalized skin color may not reveal the signs until the late stages of shock.

A new nurse to the unit is in preceptorship. The preceptor knows the new nurse understands how to determine the effects of shock on a dark skinned client in the initial stage of shock, when the nurse states she will assess which of the following?

Which assessment data would warrant immediate intervention by the nurse? 1. The client diagnosed with DVT who complains of pain on inspiration.

A potentially life-threatening complication of DVT is a pulmonary embolus, which causes chest pain. The nurse should determine if the client has "thrown" a pulmonary embolus.

The purpose of adding PEEP to PPV is to a. increase FRC and improve oxygenation b. increase FIO2 in an attempt to wean the patient and avoid O2 toxicity c. determine if the patient is in synchrony with the ventilator or needs to be paralyzed d. determine if the patient is able to be weaned and avoid the risk of pneumomediastinum

6. Correct answer: a Rationale: Positive end-expiratory pressure (PEEP) is a ventilatory maneuver in which positive pressure is applied to the airway during exhalation. This increases functional residual capacity (FRC) and often improves oxygenation with restoration of lung volume that normally remains at the end of passive exhalation.

6. A patient with aortic valve endocarditis develops dyspnea, crackles in the lungs, and restlessness. What should the nurse suspect that the patient is experiencing? a. Pulmonary embolization from valve vegetations b. Vegetative embolization to the coronary arteries c. Valvular incompetence with resulting heart failure d. Nonspecific manifestations that accompany infectious diseases

6. c. The dyspnea, crackles, and restlessness that the patient is manifesting are symptoms of heart failure and decreased cardiac output (CO) that occurs in up to 80% of patients with aortic valve endocarditis as a result of aortic valve incompetence. Vegetative embolization from the aortic valve occurs throughout the arterial system and may affect any body organ. Pulmonary emboli occur in right-sided endocarditis.

The nursing management of a patient with an artificial airway includes a. maintaining ET tube cuff pressure at 30 cm H2O b. routine suctioning of the tube at least q2h c. observing for cardiac dysrhythmias during suctioning d. preventing tube dislodgement by limiting mouth care to lubrication of the lips

7. Correct answer: c Rationale: Potential complications associated with suctioning include hypoxemia, bronchospasm, increased intracranial pressure, dysrhythmias, hypertension, hypotension, mucosal damage, pulmonary bleeding, pain, and infection. Closely assess the patient before, during, and after the suctioning procedure. If the patient does not tolerate suctioning (e.g., decreased arterial oxygenation, increased or decreased blood pressure, sustained coughing, development of dysrhythmias), stop the procedure, and manually hyperventilate the patient with a bag valve mask and 100% oxygen.

7. Priority Decision: A patient hospitalized for 1 week with subacute infective endocarditis is afebrile and has no signs of heart damage. Discharge with outpatient antibiotic therapy is planned. During discharge planning with the patient, what is it most important for the nurse to do? a. Plan how his needs will be met while he continues on bed rest. b. Encourage the use of diversional activities to relieve boredom and restlessness. c. Teach the patient to avoid crowds and exposure to upper respiratory infections. d. Assess the patient's home environment in terms of family assistance and hospital access.

7. d. The patient with outpatient antibiotic therapy requires vigilant home nursing care and it is most important to determine the adequacy of the home environment for successful management of the patient. The patient is at risk for life-threatening complications, such as embolization and pulmonary edema, and must be able to access a hospital if needed. Bed rest will not be necessary for the patient without heart damage. Avoiding infections and planning diversional activities are indicated for the patient but are not the most important factors while he is on outpatient antibiotic therapy.

The client has just had a pericardiocentesis. Which interventions should the nurse implement? Select all that apply. 1. Monitor vital signs every 15 minutes for the first hour. 2. Assess the client's heart and lung sounds. 3. Record the amount of fluid removed as output. 4. Evaluate the client's cardiac rhythm.

71. $$1. The nurse should monitor the vital signs for any client who has just undergone surgery. $$2. A pericardiocentesis involves entering the pericardial sac. Assessing heart and lung sounds involves entering the pericardial sac and allows assessment for cardiac failure. $$3. The pericardial fluid is documented as output. $$4. Evaluating the client's cardiac rhythm allows the nurse to assess for cardiac failure, which is a complication of pericardial centesis.

The nurse monitors the patient with positive pressure mechanical vent for a. paralytic ileus because pressure on the abdominal contents affects bowel motility b. diuresis and sodium depletion because of increased release of ANP c. signs of CV insufficiency because pressure in the chest impedes venous return d. respiratory acidosis in a patient with COPD because of alveolar hyperventilation and increased PaO2 levels

8. Correct answer: c Rationale: Positive pressure ventilation affects circulation by transmission of increased mean airway pressure to the thoracic cavity. With increased intrathoracic pressure, thoracic vessels are compressed. Such compression results in decreased venous return to the heart, decreased left ventricular end-diastolic volume (preload), decreased cardiac output, and hypotension.

8. When instructing a patient with endocarditis how to prevent recurrence of the infection, what should the nurse teach the patient? a. Start on antibiotic therapy when exposed to persons with infections. b. Take one aspirin a day to prevent vegetative lesions from forming around the valves. c. Always maintain continuous antibiotic therapy to prevent the development of any systemic infection. d. Obtain prophylactic antibiotic therapy before certain invasive medical or dental procedures (e.g., dental cleaning).

8. d. Prophylactic antibiotic therapy should be initiated before invasive dental, medical, or surgical procedures to prevent recurrence of endocarditis. Continuous antibiotic therapy is indicated only in patients with implanted devices or ongoing invasive procedures. Symptoms of infection should be treated promptly but antibiotics are not used for exposure to infection.

A 64-year-old male patient admitted to the critical care unit for gastrointestinal hemorrhage complains of feeling tense and nervous. He appears restless with an increase in blood pressure and pulse. If the physical assessment shows no other changes, it is most important for the critical care nurse to take which action? a. Administer prescribed IV dose of lorazepam (Ativan). b. Stay with the patient and encourage expression of concerns. c. Ask a family member to remain at the bedside with the patient. d. Teach the patient how to use guided imagery to reduce anxiety.

9. B Anxiety is a common problem for critically ill patients. The nurse should first stay with the patient and encourage the patient to express concerns and needs. After expression of feelings, the nurse should determine the appropriate intervention if needed (e.g., lorazepam, guided imagery, family presence) and closely monitor the patient's hemodynamic parameters.

9. A patient is admitted to the hospital with a suspected acute pericarditis. To establish the presence of a pericardial friction rub, how should the nurse listen to the patient's chest? a. While timing the sound with the respiratory pattern b. With the bell of the stethoscope at the apex of the heart c. With the diaphragm of the stethoscope at the lower left sternal border of the chest d. With the diaphragm of the stethoscope to auscultate a high-pitched continuous rumbling sound

9. c. The stethoscope diaphragm at the left sternal border with the patient leaning forward is the best method to use to hear the high-pitched, grating sound of a pericardial friction rub. The sound does not radiate widely and occurs with the heartbeat. To differentiate a pericardial friction rub from a pleural friction rub, have the patient hold his or her breath. The rub will still be heard if it is cardiac in nature.

Physiological Integrity 25. Which action will the nurse include in the plan of care for a patient who was admitted with syncopal episodes of unknown origin? a. Instruct the patient to call for assistance before getting out of bed. b. Explain the association between various dysrhythmias and syncope. c. Educate the patient about the need to avoid caffeine and other stimulants. d. Tell the patient about the benefits of implantable cardioverter-defibrillators.

A A patient with fainting episodes is at risk for falls. The nurse will plan to minimize the risk by having assistance whenever the patient up. The other actions may be needed if dysrhythmias are found to be the cause of the patient's syncope, but are not appropriate for syncope of unknown origin. DIF: Cognitive Level: Apply (application) REF: 807 TOP: Nursing Process: Planning MSC:

Safe and Effective Care Environment 27. Which action by a new registered nurse (RN) who is orienting to the progressive care unit indicates a good understanding of the treatment of cardiac dysrhythmias? a. Injects IV adenosine (Adenocard) over 2 seconds to a patient with supraventricular tachycardia b. Obtains the defibrillator and quickly brings it to the bedside of a patient whose monitor shows asystole c. Turns the synchronizer switch to the "on" position before defibrillating a patient with ventricular fibrillation d. Gives the prescribed dose of diltiazem (Cardizem) to a patient with new-onset type II second degree AV block

A Adenosine must be given over 1 to 2 seconds to be effective. The other actions indicate a need for more education about treatment of cardiac dysrhythmias. The RN should hold the diltiazem until talking to the health care provider. The treatment for asystole is immediate CPR. The synchronizer switch should be "off" when defibrillating. DIF: Cognitive Level: Analyze (analysis) REF: 795 OBJ: Special Questions: Multiple Patients TOP: Nursing Process: Evaluation MSC:

Physiological Integrity 8. After the nurse gives IV atropine to a patient with symptomatic type 1, second-degree atrioventricular (AV) block, which finding indicates that the medication has been effective? a. Increase in the patient's heart rate b. Increase in strength of peripheral pulses c. Decrease in premature atrial contractions d. Decrease in premature ventricular contractions

A Atropine will increase the heart rate and conduction through the AV node. Because the medication increases electrical conduction, not cardiac contractility, the quality of the peripheral pulses is not used to evaluate the drug effectiveness. The patient does not have premature atrial or ventricular contractions. DIF: Cognitive Level: Apply (application) REF: 798 TOP: Nursing Process: Evaluation MSC:

Physiological Integrity 14. A 20-year-old has a mandatory electrocardiogram (ECG) before participating on a college soccer team and is found to have sinus bradycardia, rate 52. Blood pressure (BP) is 114/54, and the student denies any health problems. What action by the nurse is most appropriate? a. Allow the student to participate on the soccer team. b. Refer the student to a cardiologist for further diagnostic testing. c. Tell the student to stop playing immediately if any dyspnea occurs. d. Obtain more detailed information about the student's family health history.

A In an aerobically trained individual, sinus bradycardia is normal. The student's normal BP and negative health history indicate that there is no need for a cardiology referral or for more detailed information about the family's health history. Dyspnea during an aerobic activity such as soccer is normal. DIF: Cognitive Level: Apply (application) REF: 793 TOP: Nursing Process: Implementation MSC:

Physiological Integrity 12. Which intervention by a new nurse who is caring for a patient who has just had an implantable cardioverter-defibrillator (ICD) inserted indicates a need for more education about care of patients with ICDs? a. The nurse assists the patient to do active range of motion exercises for all extremities. b. The nurse assists the patient to fill out the application for obtaining a Medic Alert ID. c. The nurse gives amiodarone (Cordarone) to the patient without first consulting with the health care provider. d. The nurse teaches the patient that sexual activity usually can be resumed once the surgical incision is healed.

A The patient should avoid moving the arm on the ICD insertion site until healing has occurred in order to prevent displacement of the ICD leads. The other actions by the new nurse are appropriate for this patient. DIF: Cognitive Level: Apply (application) REF: 803 TOP: Nursing Process: Evaluation MSC:

At a clinic visit, the nurse provides dietary teaching for a 56-year-old woman who was recently hospitalized with an exacerbation of chronic heart failure. The nurse determines that teaching is successful if the patient makes which statement? A "I will limit the amount of milk and cheese in my diet." B "I can add salt when cooking foods but not at the table." C "I will take an extra diuretic pill when I eat a lot of salt." D "I can have unlimited amounts of foods labeled as reduced sodium ."

A "I will limit the amount of milk and cheese in my diet." Milk products should be limited to 2 cups per day for a 2500-mg sodium-restricted diet. Salt should not be added during food preparation or at the table. Diuretics should be taken as prescribed (usually daily) and not based on sodium intake. Foods labeled as reduced sodium contain at least 25% less sodium than regular.

A70-year-old woman with chronic heart failure and atrial fibrillation asks the nurse why warfarin (Coumadin) has been prescribed for her to continue at home. Which response by the nurse is accurate? A "The medication prevents blood clots from forming in your heart." B "The medication dissolves clots that develop in your coronary arteries." C "The medication reduces clotting by decreasing serum potassium levels." D "The medication increases your heart rate so that clots do not form in your heart."

A "The medication prevents blood clots from forming in your heart." Chronic heart failure causes enlargement of the chambers of the heart and an altered electrical pathway, especially in the atria. When numerous sites in the atria fire spontaneously and rapidly, atrial fibrillation occurs. Atrial fibrillation promotes thrombus formation within the atria with an increased risk of stroke and requires treatment with cardioversion, antidysrhythmics, and/or anticoagulants. Warfarin is an anticoagulant that interferes with hepatic synthesis of vitamin K-dependent clotting factors.

Intermediate or progressive stage The early stage of shock is demonstrated by a slightly elevated heart rate, and a normal to slightly decreased blood pressure. Refractory (also called irreversible) shock is characterized by profound tissue anoxia and cellular death. The refractory stage is manifested by severe hypotension; rapid, weak, or irregular pulse; disorientation; lethargy, or coma; pallor and cyanosis; cardiac arrest and death. The client's symptoms demonstrate intermediate shock.

A 42-year-old male presents with massive hemorrhage related to an industrial accident. His vital signs are heart rate 140, blood pressure 88/60, respirations 26 and shallow. The client's skin is cool and pale with generalized edema noted. The cardiac monitor reveals dysrhythmias. The client is awake and responds appropriately to questions. Based on this assessment data, the nurse expects the client is in which of the following stages of shock?

The patient with pericarditis is complaining of chest pain. After assessment, which intervention should the nurse expect to implement to provide pain relief? D. Nonsteroidal antiinflammatory drugs

Nonsteroidal antiinflammatory drugs (NSAIDs) will control pain and inflammation.

First-Degree AV block treatment?

Nope, just keep monitoring them

A patient admitted with heart failure appears very anxious and complains of shortness of breath. Which nursing actions would be appropriate to alleviate this patient's anxiety (select all that apply)? A Administer ordered morphine sulfate. B Position patient in a semi-Fowler's position. C Position patient on left side with head of bed flat. D Instruct patient on the use of relaxation techniques. E Use a calm, reassuring approach while talking to patient.

A Administer ordered morphine sulfate. B Position patient in a semi-Fowler's position. D Instruct patient on the use of relaxation techniques. E Use a calm, reassuring approach while talking to patient. Morphine sulfate reduces anxiety and may assist in reducing dyspnea. The patient should be positioned in semi-Fowler's position to improve ventilation that will reduce anxiety. Relaxation techniques and a calm reassuring approach will also serve to reduce anxiety.

The patient has heart failure (HF) with an ejection fraction of less than 40%. What core measures should the nurse expect to include in the plan of care for this patient (select all that apply)? A Left ventricular function is documented. B Controlling dysrhythmias will eliminate HF. C Prescription for digoxin (Lanoxin) at discharge D Prescription for angiotensin-converting enzyme (ACE) inhibitor at discharge E Education materials about activity, medications, weight monitoring, and what to do if symptoms worsen

A Left ventricular function is documented. D Prescription for angiotensin-converting enzyme (ACE) inhibitor at discharge E Education materials about activity, medications, weight monitoring, and what to do if symptoms worsen The Joint Commission has identified these three core measures for heart failure patients. Although controlling dysrhythmias will improve CO and workload, it will not eliminate HF. Prescribing digoxin for all HF patients is no longer done because there are newer effective drugs and digoxin toxicity occurs easily related to electrolyte levels and the therapeutic range must be maintained.

A patient with a recent diagnosis of heart failure has been prescribed furosemide (Lasix) in an effort to physiologically do what for the patient? A Reduce preload. B Decrease afterload. C Increase contractility. D Promote vasodilation.

A Reduce preload. Diuretics such as furosemide are used in the treatment of HF to mobilize edematous fluid, reduce pulmonary venous pressure, and reduce preload. They do not directly influence afterload, contractility, or vessel tone.

The patient with chronic heart failure is being discharged from the hospital. What information should the nurse emphasize in the patient's discharge teaching to prevent progression of the disease to ADHF? A Take medications as prescribed. B Use oxygen when feeling short of breath. C Only ask the physician's office questions. D Encourage most activity in the morning when rested.

A Take medications as prescribed. The goal for the patient with chronic HF is to avoid exacerbations and hospitalization. Taking the medications as prescribed along with nondrug therapies such as alternating activity with rest will help the patient meet this goal. If the patient needs to use oxygen at home, it will probably be used all the time or with activity to prevent respiratory acidosis. Many HF patients are monitored by a care manager or in a transitional program to assess the patient for medication effectiveness and monitor for patient deterioration and encourage the patient. This nurse manager can be asked questions or can contact the health care provider if there is evidence of worsening HF.

ANS: C The first manifestations of hypovolemic shock result from compensatory mechanisms. Signs of shock are first evident as changes in cardiovascular function. As shock progresses, changes in skin, respiration, and kidney function progress. The other questions would not identify early stages of shock.

A client brought to the emergency department after a motor vehicle accident is suspected of having internal bleeding. Which question does the nurse ask to determine whether the client is in the early stages of hypovolemic shock? a. "Are you more thirsty than normal?" b. "When was the last time you urinated?" c. "What is your normal heart rate?" d. "Is your skin usually cool and pale?"

ANS: A, C, E Septic shock manifests with decreased cardiac output, increased blood glucose, and increased serum lactate. The other parameters do not correlate with septic shock. page 823

A client has septic shock. Which hemodynamic parameters does the nurse correlate with this type of shock? (Select all that apply.) a. Decreased cardiac output b. Increased cardiac output c. Increased blood glucose d. Decreased blood glucose e. Increased serum lactate f. Decreased serum lactate

Notify the emergency room physician and prepare for an immediate needle thoracostomy and insertion of chest tube

A client involved in a motor vehicle accident is admitted to the emergency room with vital signs of heart rate 116, respirations 29, temperature 99.4 °F, and blood pressure 98/62. Upon assessment, the nurse notes obvious rib fractures to the right anterior chest, and does not detect breath sounds on the right upper lung field. The client's trachea is deviated to the side of the neck. Based on this assessment data, which of the following should be the nurse's immediate action?

ANS: A The client at risk for septic shock should be instructed to clean his or her toothbrush daily, either by running it through the dishwasher or by rinsing it in laundry bleach. Clients should be instructed to bathe daily and wash the armpits, the groin, and the rectal area. The client should refrain from cleaning pet litter boxes. Clients recovering from septic shock are not at higher risk for bleeding disorders.

A client recovering from septic shock is preparing for discharge home. What priority information does the nurse include in the teaching plan for this client? a. "Clean your toothbrush with laundry bleach daily." b. "Bathe every other day with antimicrobial soap." c. "Wash your hands after changing pet litter boxes." d. "Use an electric razor when you shave your face."

ANS: C The hypodynamic phase of septic shock is characterized by a rapid decrease in cardiac output, systolic blood pressure, and pulse pressure. The nurse must initiate drug therapy to maintain blood pressure and cardiac output. Accurate urinary output and blood cultures are important to the treatment but are not the priority when a client's pulse pressure is decreasing rapidly. The family should be updated appropriately.

A client was admitted 2 days ago with early stages of septic shock. Today the nurse notes that the client's systolic blood pressure, pulse pressure, and cardiac output are decreasing rapidly. Which intervention does the nurse do first? a. Insert a Foley catheter to monitor urine output closely. b. Ask the client's family to come to the hospital because death is near. c. Initiate the prescribed dobutamine (Dobutrex) intravenous drip. d. Obtain blood cultures before administering the next dose of antibiotics.

ANS: A When a local infection becomes systemic, the client develops a high-grade temperature, decreased urine output, and increased respiratory rate. Because of tachycardia and low blood pressure, the client may exhibit orthostatic hypotension. This is a subtle sign of systemic infection that requires further evaluation by the health care provider. The other signs are not manifestations of complications. Warmth and redness are expected with local infection.

A client who has a local infection of the right forearm is being discharged. The nurse teaches the client to seek immediate medical attention if which complication occurs? a. Dizziness on changing position b. Increased urine output c. Warmth and redness at site d. Low-grade temperature

ANS: B Ringer's lactate is an isotonic solution that acts as a volume expander. Also, the lactate acts as a buffer in the presence of acidosis. The other solutions do not contain any substance that would buffer or correct the client's acidosis.

A client who has acidosis resulting from hypovolemic shock has been prescribed intravenous fluid replacement. Which fluid does the nurse prepare to administer? a. Normal saline b. Ringer's lactate c. 5% dextrose in water d. 5% dextrose in 0.45% normal saline

ANS: A Blood cultures should be obtained before IV antibiotics are started. If hypotension occurs, fluid resuscitation is used first. CVP monitoring and vasopressor therapy are started if hypotension persists.

A client who has septic shock is admitted to the hospital. What priority intervention does the nurse implement first? a. Obtain two sets of blood cultures. b. Administer the prescribed IV vancomycin (Vancocin). c. Obtain central venous pressure (CVP) measurements. d. Administer the prescribed IV norepinephrine (Levophed).

The client has chronic atrial fibrillation. Which discharge teaching should the nurse discuss with the client? 1. Instruct the client to use a soft-bristle toothbrush.

A client with chronic atrial fibrillation will be taking an anticoagulant to help prevent clot formation. Therefore, the client is at risk for bleeding and should be instructed to use a soft-bristle toothbrush.

Sepsis & Septic Shock

A complex type of distributive shock that usually begins as a bacterial or fungal infection & progresses to a dangerous condition over a period of days.

The nurse is caring for clients on a surgical floor. Which client should be assessed first? 1. The client who is four (4) day post-operative abdominal surgery and is complaining of left calf pain when ambulating.

A complication of immobility after surgery is developing a DVT. This client with left calf pain should be assessed for a DVT.

NEURAL-INDUCED DISTRIBUTIVE SHOCK

A loss of MAP that occurs when sympathetic nerve impulses controlling blood vessel smooth muscle muscle are decreased & the smooth muscles relax, causing vasodilation.

Which client teaching should the nurse implement for the client diagnosed with coronary artery disease? Select all that apply. 1. Encourage a low-fat, low-cholesterol diet. 2. Instruct client to walk 30 minutes a day. 4. Refer to counselor for stress reduction techniques. 5. Increase fiber in the diet.

A low-fat, low-cholesterol diet will help decrease the buildup of atherosclerosis in the arteries. Walking will help increase collateral circulation. Stress reduction is encouraged for clients with CAD because this helps prevent excess stress on the heart muscle. Increasing fiber in the diet will help remove cholesterol via the gastrointestinal system.

The charge nurse is making assignments for clients on a cardiac unit. Which client should the charge nurse assign to a new graduate nurse? 3. The 75-year-old client scheduled for a cardiac catheterization.

A new graduate should be able to complete A pre-procedure checklist and get this client to the catheterization lab.

1 Prone positioning refers to the repositioning of a patient from a supine or lateral position to a prone position. This repositioning improves lung reexpansion through various mechanisms. Firstly, the gravity reverses the effects of fluid in the dependent parts of the lungs as the patient is moved from supine to prone. Secondly, in the prone position, the heart rests on the sternum, away from the lungs, contributing to an overall uniformity of pleural pressures. These two mechanisms help in better ventilation in the patient with respiratory failure. The prone position is a relatively safe supportive therapy used for critically ill patients with acute lung injury or ARDS and is used for improved oxygenation. Resting in other positions such as sitting, supine, or lateral may not help in oxygenation. Text Reference - p. 1622

A nurse is attending a patient with acute respiratory distress syndrome (ARDS). Which position is best for this patient? 1 Prone 2 Sitting 3 Supine 4 Lateral

1, 2, 5 It is extremely essential to monitor all ICU patients and prevent delirium. Sensory overload can lead to patient distress and anxiety. The nurse should limit noise in the ICU and help the patient to understand that some noises in the ICU cannot be prevented, for example, beeping of a cardiac monitor. The nurse can also limit noise levels by muting phones, setting alarms based on the patient's condition, and reducing unnecessary alarms. The use of clocks and calendars can help orient the patient to time and date. Regular assessment should be carried out using tools like the Confusion Assessment Method for the ICU and the Intensive Care Delirium Screening Checklist. Seeing a familiar face may make the patient comfortable; therefore, the presence of a caregiver is important. Giving regular sponge baths helps to maintain hygiene but doesn't affect delirium directly. Text Reference - p. 1601

A nurse is caring for a patient in ICU who is taking sedatives. What are the steps that a nurse should take in order to prevent delirium in this patient? Select all that apply. 1 Keep the noise in the ICU to a minimum. 2 Use clocks and a calendar to keep the patient oriented. 3 Ensure that there is minimal communication with the patient. 4 Give regular sponge baths to the patient, and monitor the urinary output. 5 Carry out frequent assessment for delirium by using the Confusion Assessment Method.

Immobilize of the victim's spine The primary intervention in trauma is immobilization of the client's cervical spine to avoid further trauma or paralysis. Placing the client on a spine board, and applying a cervical collar and head immobilizer is the priority, followed by airway management and treatment of hemorrhage or shock.

A nurse is part of an emergency response team arriving at the scene of a motor vehicle accident. One victim was thrown approximately 20 feet from the vehicle. The victim is unconscious, has labored respirations, and is bleeding from an open fracture of the left femur. Which of the following interventions is the nurse's first priority?

1, 2, 3 The primary goal of nutritional support is to prevent or correct nutritional deficiencies. This is usually done by the early provision of enteral nutrition or parenteral nutrition. Enteral nutrition preserves the structure and function of the gut mucosa and stops the movement of gut bacteria across the intestinal wall and into the bloodstream. In addition to this, early enteral nutrition is associated with fewer complications. Enteral feedings cannot be administered to all patients; in patients with paralytic ileus, intestinal obstruction, and GI ischemia, enteral feeding is contraindicated. In these patients, parenteral feeding is the best option. Text Reference - p. 1600

A nurse is starting enteral feeding through a nasogastric tube for a patient in ICU. What advantages of enteral feeding over parenteral feeding does the nurse identify in the patient? Select all that apply. 1 Preserves the structure and function of gut mucosa 2 Stops the movement of gut bacteria across the intestinal wall 3 Results in fewer complications 4 Prevents and corrects nutritional deficiencies 5 Can be administered to all patients

1, 3, 4 It is extremely important for a nurse to closely assess the patient before, during, and after the suctioning procedure. If the patient is unable to tolerate suctioning, stop the procedure and hyperoxygenate until equilibration occurs before attempting next suction pass. Decreased SpO2, increased or decreased BP, and development of dysrhythmias are indicators that the patient is not tolerating suction. Sustained coughing rather than absence of coughing also indicates that the patient is not tolerating suctioning. Presence of shivering and convulsions is not related to suctioning. Text Reference - p. 1616

A nurse is suctioning a patient. Which signs indicate that the patient is not tolerating suctioning? Select all that apply. 1 Decreased SpO2 2 Absence of coughing 3 Increased blood pressure (BP) 4 Development of dysrhythmias 5 Shivering and convulsions of the entire body

4 Central venous pressure (CVP) is a measure of the filling pressure of the right ventricle and is indicative of how the right side of the heart accommodates fluid load. A series of CVP measurements of 12 mm Hg or higher indicates failure of the right ventricle to handle venous return. A normal CVP measurement is 2 to 8 mm Hg. Cardiogenic shock and circulatory failure are late manifestations of heart failure in general and would likely show a decreased CVP and cardiac output. CVP may be increased with left ventricular failure; however, this is a late sign. It is possible to have both right and left failure at the same time. Text Reference - p. 1608

A nurse measures a patient's central venous pressure and recognizes a series of increased readings as directly indicative of: 1 Cardiogenic shock 2 Circulatory failure 3 Left ventricular failure 4 Right ventricular failure

1, 4, 5 It is important to remove the dentures of the patient during the process of oral intubation because the dentures can obstruct the airway. Inform the patient that brief restraint will be necessary for safety purposes. It is also necessary to brief the patient about the procedure to avoid any type of resistance during intubation. Metallic objects on the body do not interfere with the procedure of intubation, and, therefore, need not be removed. Before intubating, it is extremely important to preoxygenate the patient with 100% oxygen. This is because the patient will not get any oxygen supply during intubation for a short period. STUDY TIP: Avoid planning other activities that will add stress to your life between now and the time you take the licensure examination. Enough will happen spontaneously; do not plan to add to it. Text Reference - p. 1614

A patient admitted to the ICU is being intubated. What are the steps that a nurse should ensure for a safe intubation? Select all that apply. 1 Remove the dentures of the patient. 2 Ensure that the patient is not wearing any metallic objects. 3 Oxygenate the patient using a bag-valve-mask (BVM) and 95% oxygen before the procedure. 4 Inform the patient that brief restraint will be necessary. 5 Explain the procedure to the patient and also the patient's role in the procedure.

2, 3, 5 It is extremely important to follow aseptic measures to avoid infection following an IABP. Covering all the insertion sites with occlusive dressings avoids infections. Aseptic techniques should be followed during insertion and dressing changes to prevent infection. Prophylactic antibiotics prevent infections. Infection doesn't breed in the dressings if aseptic precautions are followed and the dressings are cleaned and replaced regularly. Replacing the lines every two hours is not necessary, but following aseptic conditions is. A culture swab helps to identify the presence of infection but d

A patient has received intraaortic balloon pump (IABP) therapy. In this case, what precautions should a nurse take to prevent any infection at the site? Select all that apply. 1 Replace the lines every two to three hours. 2 Cover all insertion sites with occlusive dressings. 3 Use strict aseptic technique line insertion and dressing changes. 4 Send culture swabs from the insertion site regularly. 5 Administer prophylactic antibiotics for the entire course of therapy

1, 4, 5 Following an intubation, it is important to confirm the placement of the endotracheal (ET) tube. This confirmation is obtained by x-ray after visualizing the ET tube correctly placed in the trachea. Auscultating lungs for breath sounds confirms that air is going into the lungs and not in the stomach. If the sounds are heard over the epigastrium, it indicates that the ET tube has gone in the stomach. Presence of carbon dioxide in exhaled air also confirms that the tube has gone into the lungs, and the breathing effort is normal. In this case, a CT scan is redundant. However, an x-ray is sufficient to confirm the placement of the ET tube. The patient may require a urinary catheter, but it is not an immediate intervention and can be done after intubation. Text Reference - p. 1614

A patient in ICU has been intubated for the relief of airway obstruction. What nursing actions should be performed to prevent complications after intubation? Select all that apply. 1 Obtain a chest x-ray to confirm the placement. 2 Obtain a computed tomography (CT) scan to note the placement. 3 Immediately catheterize the patient and check for urine output. 4 Auscultate lungs bilaterally and also epigastrium for breath sounds. 5 Use an end-tidal carbo

3 PaCO2 is the best indicator of alveolar hyperventilation or hypoventilation. Continuous PETCO2 monitoring can assess the patency of the airway and the presence of breathing. Continuous oxygen saturation (SpO2) provides objective data regarding tissue oxygenation. Central venous pressure (CVP) or pulmonary artery (PA) catheters with ScvO2 or SvO2 capability provide an indirect indication of the patient's tissue oxygenation status. Text Reference - p. 1615

A patient is admitted to the ICU and is on assisted ventilation. Which is the best indicator of inadequate alveolar oxygenation? 1 PETCO2 2 SpO2 3 PaCO2 4 ScvO2 or SvO2

1, 3, 4 Before PA catheter insertion, the patient is positioned supine and flat. The procedure is explained to the patient, and informed consent is obtained. The patient's electrolyte, acid-base, oxygenation, and coagulation status are noted. Imbalances such as hypokalemia, hypomagnesemia, hypoxemia, or acidosis can make the heart more irritable and increase the risk of ventricular dysrhythmia during catheter insertion. Coagulopathy increases the risk of hemorrhage. The procedure is never performed in a sitting position. The PA catheter is inserted through a sheath percutaneously into the internal jugular, subclavian, antecubital, or femoral vein using surgical asepsis. The insertion sites have to be dressed with occlusive dressings. Text Reference - p. 1608

A patient is advised to have a pulmonary artery (PA) catheter inserted for pulmonary artery pressure monitoring. What precautions are necessary prior to insertion of the catheter? Select all that apply. 1 Place the patient in the supine and flat position. 2 Position the patient sitting and with head turned laterally. 3 Explain the procedure to the patient and get consent. 4 Note the patient's electrolyte levels and oxygenation and coagulation status. 5 Do not cover the catheter insertion site with any dressings.

2, 4, 5 Appropriate patient selection for ventricular assist device (VAD) includes patients who are waiting for heart transplantation, who are diagnosed with Class IV heart disease, and have failed medical therapy, and who have failed to wean from cardiopulmonary bypass (CPB). Body surface area less than 1.3 m2 is a contraindication for ventricular assist device (VAD) therapy. Liver failure unrelated to a cardiac event is a contraindication for ventricular assist device (VAD) therapy. Text Reference - p. 1612

A patient is being considered for ventricular assist device (VAD) therapy. Which criteria indicate that this patient is an appropriate candidate for implantation of this device? Select all that apply. 1 Body surface area 1.1 m2 2 Placed on the heart transplantation list 3 Diagnosed with alcoholic liver failure 4 Diagnosed with Class IV heart disease 5 Unable to wean from the cardiopulmonary bypass (CPB) machine

3 Presence of condensate or water in tubing triggers a high-pressure ventilation alarm. Power failure triggers ventilator inoperative or low battery alarm. Insufficient gas flow and tracheotomy cuff leak triggers low tidal volume or minute ventilation alarm. Text Reference - p. 1620

A patient is being mechanically ventilated. A high-pressure ventilation alarm sounds. The nurse should assess for what cause of this type of alarm? 1 Power failure 2 Insufficient gas flow 3 Condensate in tubing 4 Tracheotomy cuff leak

3 Before inserting a line into the radial artery, an Allen test should be performed to confirm that ulnar circulation to the hand is adequate. In this test, pressure is applied to the radial and ulnar arteries simultaneously. The patient opens and closes the hand repeatedly until the hand blanches. When the pressure on the ulnar artery is released, the hand should return to a pink color within six seconds. If pinkness does not return within six seconds the ulnar artery is inadequate to maintain blood flow to the extremity and the radial artery should not be used for arterial line insertion. The phlebostatic axis is used to zero the arterial line, which would be done much later. Because of the risk of heparin-induced thrombocytopenia (HIT), heparinized saline should not be routinely used for the flush solution. The flush bag should be set to deliver 3 to 6 mL/hr. Text Reference - p. 1606

A patient is being prepared for insertion of an arterial measuring device. What should be done before the catheter is inserted into the patient's radial artery? 1 Locate the phlebostatic axis 2 Prepare a heparinized flush bag 3 Occlude the radial and ulnar arteries 4 Set an intravenous pump to deliver 15 mL/h

1, 3, 5 Nasal intubation is a blind procedure. There are chances that the tube may be misdirected. It may cause complications if there was a recent cranial surgery and fracture of the facial bones or the base of the skull. Suspected spinal fracture is a contraindication for oral intubation because it requires some movement of the neck and head. A deviated nasal septum can cause some difficulty in nasal intubation, although it is not a contraindication. STUDY TIP: Begin studying by setting goals. Make sure they are realistic. A goal of scoring 100% on all exams is not realistic, but scoring an 85% may be a better goal. Text Reference - p. 1614

A patient is being prepared for intubation using a nasal intubation technique. What absolute contraindications for nasal intubation should the nurse be aware of? Select all that apply. 1 Recent cranial surgery 2 Suspected spine injury 3 Fracture of facial bones 4 Deviated nasal septum 5 Fracture of the base of the skull

4 Indications for intraaortic balloon pump (IABP) therapy include acute myocardial infarction and cardiogenic shock. The use of the pump with this health problem allows time for emergent angiography. The pump is not used to reduce pressure in the pulmonary artery, improve coronary artery vessel perfusion, or to enhance the effectiveness of cardiac medications. Text Reference - p. 1610

A patient is experiencing cardiogenic shock after an acute myocardial infarction. Why would an intraaortic balloon pump (IABP) be beneficial for this patient? 1 Reduces pressure in the pulmonary artery 2 Improves coronary artery vessel perfusion 3 Enhances effectiveness of cardiac medications 4 Provides time for an emergency angiogram to be performed

2 Benzodiazepines, such as lorazepam, have anxioloytic activity and help to alleviate symptoms of anxiety in patients. Propofol is an anesthetic, and fentanyl is an analgesic agent. These two medications do not help to treat anxiety but are used to relax intubated, ventilated patients. Esomeprazole is a proton pump inhibitor that helps to reduce symptoms of peptic ulcer. Text Reference - p. 1600

A patient is experiencing symptoms of anxiety. The nurse anticipates that which medication will be prescribed? 1 Propofol 2 Lorazepam 3 Fentanyl 4 Esomeprazole

3 An SBT is recommended in patients who demonstrate weaning readiness. An SBT should be at least 30 minutes but no more than 120 minutes. At least 15 minutes but no more than 30 minutes, at least 30 minutes but no more than 60 minutes, and at least 60 minutes but no more than 120 minutes are not recommended time frames to determine weaning readiness. Text Reference - p. 1626

A patient is intubated. The nurse has to perform a spontaneous breathing trial (SBT) on this patient. For how long should this trial be done? 1 At least 15 minutes but not more than 30 minutes 2 At least 30 minutes but not more than 60 minutes 3 At least 30 minutes but not more than 120 minutes 4 At least 60 minutes but not more than 120 minutes

1, 2, 4 Patients have a higher risk for hospital-acquired pneumonia when they require mechanical ventilation. This is because the ET or tracheostomy tube bypasses the normal upper airway defenses. Additionally, poor nutritional state, immobility, and the underlying disease process make the patient more prone to infection. VAP is pneumonia that occurs 48 hours or more post-ET intubation. To prevent VAP, the health care team should strictly wash their hands before and after suctioning. An ET tube with a dorsal lumen above the cuff should be used to allow continuous suctioning of secretions in the subglottic area. Gloves should be worn whenever the nurse is in contact with the patient, and the nurse should change them frequently between activities to avoid cross-infection. If the ventilator circuit tubing is changed frequently, there is more risk of exposing the patient to various infections. Therefore, there should be no routine changes in the patient's ventilator circuit tubing. In addition to this, the head-of-bed should be elevated at a minimum of 30 to 45 degrees, unless medically contraindicated, to prevent pooling of secretions and facilitate suctioning. Text Reference - p. 1623

A patient is placed on a ventilator for assisted ventilation. What precautions should a nurse take to prevent the patient from ventilator-assisted pneumonia (VAP)? Select all that apply. 1 Wash hands before and after suctioning. 2 Use an endotracheal (ET) tube with a dorsal lumen above the cuff. 3 Change the patient's ventilator circuit tubing every two to three hours. 4 Wear gloves when in contact with the patient, and change gloves between activities. 5 Maintain the head-of-bed elevation at a minimum of 90 degrees unless medically contraindicated.

2, 4, 5, 6 The possible causes for a high-pressure limit alarm to go off include secretions, coughing, or gagging. It may also be set off in case of ventilator asynchrony if the patient is fighting the ventilator. Decreased compliance due to conditions like pulmonary edema may also cause the setting off of the high-pressure alarm. It can also be due to kinked or compressed tubing, which usually happens when the patient is biting on the endotracheal tube. Oversedation and loss of airway cause an apnea alarm and can also set off the low-pressure limit alarm. Text Reference - p. 1620

A patient is placed on mechanical ventilation. A nurse notices that the alarm for the high-pressure limit has been set off. What are the possible conditions that could give rise to this alarm? Select all that apply. 1 Oversedation 2 Secretions, coughing, or gagging 3 Loss of airway through total or partial extubation 4 Patient fighting the ventilator 5 Decreased compliance due to pulmonary edema 6 Kinked or compressed tubing

1, 3 Milrinone is a vasodilator. Vasodilation decreases preload and afterload. This medication does not directly affect the heart rate. Vasodilators cause the blood pressure to decrease. Vasodilators will improve cardiac output. Test-Taking Tip: Be alert for details about what you are being asked to do. In this question type, you are asked to select all options that apply to a given situation or patient. All options likely relate to the situation, but only some of the options may relate directly to the situation. Text Reference - p. 1604

A patient is prescribed milrinone. What effects on the patient's hemodynamic parameters should the nurse expect? Select all that apply. 1 Decreased preload 2 Increased heart rate 3 Decreased afterload 4 Increased blood pressure 5 Decreased cardiac output

6 The patient's airway pressure during expiration is 6 cm H2O. Normally during exhalation, the airway pressure drops to zero and exhalation occurs passively. The pressure in CPAP is delivered continuously during spontaneous breathing to prevent the patient's airway pressure from falling to zero. Therefore, if CPAP is 6 cm H2O, airway pressure during expiration is 6 cm H2O. Text Reference - p. 1621

A patient on mechanical ventilation is receiving a continuous positive airway pressure (CPAP) of 6 cm H2O. What is the patient's airway pressure during expiration? Record your answer using a whole number. ___________________ cm H2O

4 Intrathoracic pressure changes associated with positive pressure ventilation cause a decrease in production of atrial natriuretic peptide. Positive pressure ventilation also decreases cardiac output, which further decreases renal perfusion. A decrease in renal perfusion increases the production of renin, angiotensin and aldosterone, which results in sodium retention. Text Reference - p. 1623

A patient on positive pressure ventilation has increased sodium retention in the body. A decrease in production of which biologic factor may have caused sodium retention in the patient? 1 Renin 2 Angiotensin 3 Aldosterone 4 Atrial natriuretic peptide

Chest pain & hypertension

A patient receives dopamine 20 mcg/kg/min IV for treatment of shock. What does the nurse assess for while administering this drug?

Distributive-neural

A patient with a head injury was treated for a cerebral hematoma. After surgery, this patient is at risk for what type of shock?

2, 4, 6 Negative pressure ventilation is similar to normal ventilation in that expiration is passive and decreased intrathoracic pressures produce inspiration. Negative pressure ventilation uses intermittent subatmospheric pressure around the chest wall. This pressure reduces the intrathoracic pressure during inspiration. Negative pressure ventilation is noninvasive because it does not require an artificial airway. During inspiration, the chest is pulled outward because of the intermittent negative pressure around the chest wall. Negative pressure ventilation uses chambers that encase the chest wall, but not the upper airway. Text Reference - p. 1618

A patient with a spinal cord injury requires negative pressure ventilation. What statements should the nurse say to the patient's caregiver about negative pressure ventilation? Select all that apply. 1 "It is a type of invasive ventilation." 2 "It is similar to the normal ventilation." 3 "It pulls the chest inward during inspiration." 4 "It uses intermittent subatmospheric pressure." 5 "It uses chambers that encase the upper airway." 6 "It reduces intrathoracic pressure during inspiration."

3 A β-adrenergic blocking medication decreases stroke volume. Contractility is reduced by negative inotropes. An example of a negative inotrope is a β-adrenergic blocker. Since an increase in contractility increases stroke volume (SV) and myocardial oxygen requirements, a negative inotrope such as a β-adrenergic blocker will decrease stroke volume (SV). A β-adrenergic blocking medication will decrease rather than increase oxygen use. This medication does not affect myocardial cellular metabolism. Text Reference - p. 1604

A patient with hypertension is prescribed a β-adrenergic blocking medication. What effect should the nurse expect this medication to have on the patient's heart? 1 Increased oxygen use 2 Increased stroke volume 3 Decreased stroke volume 4 Decreased cellular metabolism

3 A central venous oxygen saturation (ScvO2) mixed venous oxygen saturation (SvO2) measurement of 48% is low. Metabolic demand exceeds oxygen supply in conditions that increase muscle movement and metabolic rate, including physiologic states such as seizures. Central venous oxygen saturation (ScvO2) mixed venous oxygen saturation (SvO2) measurements greater than 80% are caused by increased oxygen supply. Even though a low central venous oxygen saturation (ScvO2) mixed venous oxygen saturation (SvO2) measurement is associated with a decreased cardiac output, the patient is not experiencing a health problem such as cardiogenic shock caused by left ventricular pump failure that supports decreased cardiac output as the reason for the low measurement. Central venous oxygen saturation (ScvO2) mixed venous oxygen saturation (SvO2) measurements greater than 80% are caused by decreased oxygen demand. Text Reference - p. 1609

A patient with meningitis and seizures has a pulmonary arterial catheter inserted. The most recent central venous oxygen saturation (ScvO2) mixed venous oxygen saturation (SvO2) measurement is 48%. What should the nurse realize is the reason for this patient's measurement? 1 Increased oxygen supply 2 Decreased cardiac output 3 Increased oxygen demand 4 Decreased oxygen demand

1, 2, 5 In RSI, a sedative and paralytic are administered to the patient. Usually a sedative-hypnotic-amnesic agent like midazolam is used to make the patient unconscious. A rapid-onset opioid like fentanyl is also given to blunt the pain of the procedure. A paralytic drug like succinylcholine is then given to produce skeletal muscle paralysis. Before this, the patient is 100% oxygenated. After intubation, the patient is again 100% oxygenated, and tube placement is confirmed. Auscultation of the chest bilaterally and x-ray are some of the methods of confirming the placement of the tube. During intubation, the endotracheal (ET) tube is inserted through the nose or mouth. While performing RSI, no opening is made in the throat. However, an opening in the throat is made in a procedure called tracheostomy. Text Reference - p. 1614

A patient with severe respiratory distress is brought to the medical facility. The health care provider prescribes rapid-sequence intubation (RSI) to be done. What information should the nurse include when explaining the procedure to the family members? Select all that apply. 1 A sedative and a paralytic medication is administered so that the patient sleeps and does not feel the pain. 2 After intubation, 100% oxygen is given to the patient, and placement of the tube is confirmed. 3 After giving paralytic, an opening is made in the throat through which a tube is introduced. 4 After establishing the opening in the throat, the tube is placed and dressed properly. 5 A tube will be introduced in the patient's throat through the mouth so that ventilation can be established.

Increased pulse rate with warm, pink skin.

A patient's at risk for sepsis. Which assessment finding is most indicative of the hyper-dynamic activity that occurs in septic shock?

120 MAP is calculated by adding the systolic blood pressure to two times the diastolic blood pressure and dividing by three. For this patient that calculation would be 172 + 2(94)/3 = 120 mm Hg. Normal MAP is between 70 and 105 mm Hg. Text Reference - p. 1603

A patient's blood pressure is 172/94 mm Hg. What would the nurse calculate as being this patient's mean arterial pressure (MAP)? Record your answer using a whole number. ____________ mm Hg

4 SIMV stands for synchronized intermittent mandatory ventilation, a mode of ventilation in which the ventilator delivers a preset tidal volume at a preset frequency in synchrony with the patient's spontaneous breathing. Between ventilator-delivered breaths the patient is able to breathe spontaneously, receiving the preset FIO2, but self-regulates the rate and depth of those breaths. Pressure support ventilation (PSV) applies positive pressure only during inspiration. PSV is not used as a sole ventilator support during acute respiratory failure because of the risk of hypoventilation, but it does decrease the work of breathing. Pressure-control inverse ratio ventilation (PC-IRV) sets the ventilation pressure and the ratio of inspiration to expiration to control the patient's breathing. Assist-control ventilation (ACV) or assisted mandatory ventilation (AMV) delivers a preset rate of breaths, but allows the patient to breathe spontaneously, with a preset tidal volume. Text Reference - p. 1620

A patient's family member asks the nurse what SIMV means on the settings of the mechanical ventilator attached to the patient. Which statement best describes this mode of ventilation? 1 "SIMV provides additional inspiratory pressure so that your father does not have to work as hard to breathe, thus enabling better oxygenation and a quicker recovery with fewer complications." 2 "SIMV is a mode that allows the ventilator to totally control your father's breathing. It will prevent him from hyperventilating or hypoventilating, thus ensuring the best oxygenation." 3 "SIMV is a mode that allows your father to breathe on his own, but the ventilator will control how deep a breath he will receive. The ventilator can sense when he wants a breath and it will deliver it." 4 "SIMV is a mode that allows your father to breathe on his own while receiving a preset number of breaths from the ventilator. He can breathe as much or as little as he wants beyond what the ventilator will breathe for him."

Decreased pH with decreased Pao2 & increased Paco2.

A patient's showing early clinical manifestations of hypovolemic shock. The physician orders an ABG. Which ABG values does the nurse expect to see in hypovolemic shock?

Progressive

A post op hospitalized patient has a decrease in MAP of greater than 20 mm Hg from baseline value; elevated, thready pulse; decreased BP; shallow respirations of 26/min; pale skin; moderate acidosis; & moderate hyperkalemia. This patient's in what phase of shock?

1 After insertion and before using the PA catheter, a chest x-ray must be taken to confirm the catheter's position. A hemoglobin level is not needed before using the catheter for fluid administration. Electrolyte levels do not need to be evaluated before using the catheter for fluid administration. A 12-lead electrocardiogram is not needed before using the catheter for fluid administration. Text Reference - p. 1608

A pulmonary artery catheter has just been inserted through a patient's internal jugular vein. What should be done before the catheter is used for fluid administration? 1 Obtain a chest x-ray 2 Draw a hemoglobin level 3 Evaluate electrolyte levels 4 Obtain a 12-lead electrocardiogram

Patients with a heart transplantation are at risk for which complications in the first year after transplantation (select all that apply)? b. infection c. rejection e. sudden cardiac death

A variety of complications can occur after heart transplantation. In the first year after transplantation, the major causes of death are acute rejection and infection. Heart transplant recipients also are at risk for sudden cardiac death. Later, malignancy (especially lymphoma) and cardiac vasculopathy (accelerated CAD) are major causes of death.

Apply direct pressure to the femoral artery pressure point Bright red blood indicates arterial hemorrhage and bleeding must be controlled immediately to avoid death. Applying pressure to the femoral artery pressure point helps to control upper leg bleeding. The popliteal artery pressure point controls lower leg bleeding. Elevating the legs will not prevent arterial bleeding.

A young male sustains injuries related to a motorcycle accident. A nurse driving to work stops and assists the victim at the site of the accident. The victim is bleeding bright red blood profusely from his right upper leg. An observer at the scene has already activated the EMS system and the anticipated arrival time is less than 5 minutes. Which of the following actions is the most appropriate for the nurse to perform at this time?

Pulse rate

A young trauma patient is at risk for hypovolemic shock related to occult hemorrhage. What baseline indicator allows the nurse to recognize the early signs of shock?

Physiological Integrity OTHER 1. When preparing to defibrillate a patient. In which order will the nurse perform the following steps? (Put a comma and a space between each answer choice [A, B, C, D, E].) a. Turn the defibrillator on. b. Deliver the electrical charge. c. Select the appropriate energy level. d. Place the paddles on the patient's chest. e. Check the location of other staff and call out "all clear."

A, C, D, E, B This order will result in rapid defibrillation without endangering hospital staff. DIF: Cognitive Level: Analyze (analysis) REF: 802 TOP: Nursing Process: Implementation MSC:

The patient had myocarditis and is now experiencing fatigue, weakness, palpitations, and dyspnea at rest. The nurse assesses pulmonary crackles, edema, and weak peripheral pulses. Sinoatrial tachycardia is evident on the cardiac monitor. The Doppler echocardiography shows dilated cardiomyopathy. What collaborative and nursing care of this patient should be done to improve cardiac output and the quality of life? (Select all that apply.) A. Decrease preload and afterload. B. Relieve left ventricular outflow obstruction. C. Control heart failure by enhancing myocardial contractility. D. Improve diastolic filling and the underlying disease process. E. Improve ventricular filling by reducing ventricular contractility

A. Decrease preload and afterload. C. Control heart failure by enhancing myocardial contractility. The patient is experiencing dilated cardiomyopathy. To improve cardiac output and quality of life, drug, nutrition, and cardiac rehabilitation will be focused on controlling heart failure by decreasing preload and afterload and improving cardiac output, which will improve the quality of life. Relief of left ventricular outflow obstruction and improving ventricular filling by reducing ventricular contractility is done for hypertrophic cardiomyopathy. There are no specific treatments for restrictive cardiomyopathy, but interventions are aimed at improving diastolic filling and the underlying disease process

While doing an admission assessment, the nurse notes clubbing of the patient's fingers. Based on this finding, the nurse will question the patient about which disease process?

A. Endocarditis

A patient with septic shock has a BP of 70/46 mm Hg, pulse 136, respirations 32, temperature 104° F, and blood glucose 246 mg/dL. Which intervention ordered by the health care provider should the nurse implement first? a. Give normal saline IV at 500 mL/hr. b. Give acetaminophen (Tylenol) 650 mg rectally. c. Start insulin drip to maintain blood glucose at 110 to 150 mg/dL. d. Start norepinephrine (Levophed) to keep systolic blood pressure >90 mm Hg.

ANS: A Because of the low systemic vascular resistance (SVR) associated with septic shock, fluid resuscitation is the initial therapy. The other actions also are appropriate, and should be initiated quickly as well

Which is a priority nursing intervention for a patient during the acute phase of rheumatic fever?

A. administration of antibiotics as ordered

A patient is admitted with myocarditis. While performing the initial assessment, which clinical signs and symptoms might the nurse find (select all that apply):

A. angina B. peripheral edema D. pericardial friction rub

Which of the following diagnostic study best differentiates the various types of cardiomyopathy?

A. echocardiography

What is the most common dysrhythmia?

A. fib

A patient is diagnosed with mitral stenosis and new onset atrial fibrillation. Which intervention could the nurse delegate to NAP (select all that apply):

A. obtain and record daily weight C. observe for overt signs of bleeding D. obtain and record vital signs, including pulse oximetry

Assessment of an IV cocaine user with infective endocarditis should focus on which signs and symptoms (select all that apply):

A. retinal hemorrhage B. splinter hemorrhages C. presence of Osler's nodes E. painless erythematous macules on the palms and soles

The health care provider prescribes these actions for a patient who has possible septic shock with a BP of 70/42 mm Hg and oxygen saturation of 90%. In which order will the nurse implement the actions? Put a comma and space between each answer choice (a, b, c, d, etc.) ____________________ a. Obtain blood and urine cultures. b. Give vancomycin (Vancocin) 1 g IV. c. Infuse vasopressin (Pitressin) 0.01 units/min. d. Administer normal saline 1000 mL over 30 minutes. e. Titrate oxygen administration to keep O2 saturation >95%.

ANS: E, D, C, A, B The initial action for this hypotensive and hypoxemic patient should be to improve the oxygen saturation, followed by infusion of IV fluids and vasopressors to improve perfusion. Cultures should be obtained before administration of antibiotics.

A 198-lb patient is to receive a dobutamine infusion at 5 mcg/kg/minute. The label on the infusion bag states: dobutamine 250 mg in 250 mL normal saline. When setting the infusion pump, the nurse will set the infusion rate at how many mL per hour?

ANS: 27 In order to administer the dobutamine at the prescribed rate of 5 mcg/kg/minute from a concentration of 250 mg in 250 mL, the nurse will need to infuse 27 mL/hour.

2. The nurse is caring for a patient who has an intraortic balloon pump (IABP) following a massive heart attack. When assessing the patient, the nurse notices blood backing up into the IABP catheter. In which order should the nurse take the following actions? (Put a comma and a space between each answer choice [A, B, C, D].) a. Ensure that the IABP console has turned off. b. Assess the patient's vital signs and orientation. c. Obtain supplies for insertion of a new IABP catheter. d. Notify the health care provider of the IABP malfunction.

ANS: A, B, D, C Blood in the IABP catheter indicates a possible tear in the balloon. The console will shut off automatically to prevent complications such as air embolism. Next, the nurse will assess the patient and communicate with the health care provider about the patient's assessment and the IABP problem. Finally, supplies for insertion of a new IABP catheter may be needed, based on the patient assessment and the decision of the health care provider.

The patient has a heart rate of 40 beats/minute. The P waves are regular, and the Q waves are regular, but there is no relationship between the P wave and QRS complex. What treatment do you anticipate? A. Pacemaker B. Continue to monitor C. Carotid massage D. Defibrillation

ANS: A In third-degree atrioventricular (AV) block, there is no correlation between the impulse from the atrium to the ventricles and the ventricular rhythm seen. A pacemaker eventually is required. Action must be taken because this usually results in reduced cardiac output with subsequent ischemia if untreated. Carotid massage is vagal stimulation, and it can cause bradycardia. There is a problem in conduction, not abnormal contraction, and defibrillation is not used. Reference: 830

2. The nurse is caring for a patient who has an intraortic balloon pump (IABP) following a massive heart attack. When assessing the patient, the nurse notices blood backing up into the IABP catheter. In which order should the nurse take the following actions? (Put a comma and a space between each answer choice [A, B, C, D].) a. Ensure that the IABP console has turned off. b. Assess the patient's vital signs and orientation. c. Obtain supplies for insertion of a new IABP catheter. d. Notify the health care provider of the IABP malfunction.

ANS: A, B, D, C Blood in the IABP catheter indicates a possible tear in the balloon. The console will shut off automatically to prevent complications such as air embolism. Next, the nurse will assess the patient and communicate with the health care provider about the patient's assessment and the IABP problem. Finally, supplies for insertion of a new IABP catheter may be needed, based on the patient assessment and the decision of the health care provider. DIF: Cognitive Level: Analyze (analysis) REF: 1612 OBJ: Special Questions: Prioritization TOP: Nursing Process: Planning MSC: NCLEX: Physiological Integrity

1. When assisting with oral intubation of a patient who is having respiratory distress, in which order will the nurse take these actions? (Put a comma and a space between each answer choice [A, B, C, D, E].) a. Obtain a portable chest-x-ray. b. Position the patient in the supine position. c. Inflate the cuff of the endotracheal tube after insertion. d. Attach an end-tidal CO2 detector to the endotracheal tube. e. Oxygenate the patient with a bag-valve-mask device for several minutes.

ANS: E, B, C, D, A The patient is pre-oxygenated with a bag-valve-mask system for 3 to 5 minutes before intubation and then placed in a supine position. Following the intubation, the cuff on the endotracheal tube is inflated to occlude and protect the airway. Tube placement is assessed first with an end-tidal CO2 sensor, then with a chest x-ray. DIF: Cognitive Level: Analyze (analysis) REF: 1614 OBJ: Special Questions: Prioritization TOP: Nursing Process: Implementation MSC: NCLEX: Physiological Integrity

A patient has sought care after an episode of syncope of unknown origin. Which nursing action should you prioritize in the patient's subsequent diagnostic workup? A. Preparing to assist with a head-up tilt test B. Assessing the patient's knowledge of pacemakers C. Preparing an intravenous dose of a β-adrenergic blocker D. Teaching the patient about the role of antiplatelet aggregators

ANS: A A head-up tilt test is a common component of the diagnostic workup after episodes of syncope. Intravenous β-blockers are not indicated, and addressing pacemakers is premature and inappropriate at this stage of diagnosis. Patient education about antiplatelet aggregators is not directly relevant to the patient's syncope. Reference: 839

An elderly patient presents to the emergency department after a fall. She states she does not remember the incident. What is most important to assess first? A. Heart rate and rhythm B. Hemoglobin C. Home environment D. Alcohol consumption

ANS: A Although all options will be assessed eventually, determining a cardiac cause for this brief lapse of consciousness is most important. Reference: 839

The charge nurse is explaining the concept of pacemaker failure to capture to the new graduate. What information should the charge nurse give? A. It occurs when the electrical charge is insufficient. B. It occurs when the pacemaker does not recognize spontaneous heart activity. C. A complication is ventricular tachycardia. D. First-line treatment when this occurs is to turn down the electrical charge.

ANS: A Failure to capture occurs when the electrical charge to the myocardium is insufficient to produce atrial or ventricular contraction. It can result in serious bradycardia, and treatment includes increasing the electrical charge. Failure to sense occurs when the pacemaker fails to recognize spontaneous atrial or ventricular activity and fires inappropriately. This can result in ventricular tachycardia. Reference: 836

A patient in asystole is likely to receive which drug treatments? A. Atropine and epinephrine B. Lidocaine and amiodarone C. Digoxin and procainamide D. β-Adrenergic blockers and dopamine

ANS: A Normally, the patient in asystole cannot be successfully resuscitated. However, administration of atropine and epinephrine may prompt the return of depolarization and ventricular contraction. Reference: 832

Norepinephrine (Levophed) has been prescribed for a patient who was admitted with dehydration and hypotension. Which patient information indicates that the nurse should consult with the health care provider before administration of the norepinephrine? a. The patient's central venous pressure is 3 mm Hg. b. The patient is receiving low dose dopamine (Intropin). c. The patient is in sinus tachycardia at 100 to 110 beats/min. d. The patient has had no urine output since being admitted.

ANS: A Adequate fluid administration is essential before administration of vasopressors to patients with hypovolemic shock. The patient's low central venous pressure indicates a need for more volume replacement. The other patient data are not contraindications to norepinephrine administration. DIF: Cognitive Level: Application REF: 1733-1735 | 1736 TOP: Nursing Process: Implementation MSC: NCLEX: Physiological Integrity

Norepinephrine (Levophed) has been prescribed for a patient who was admitted with dehydration and hypotension. Which patient data indicate that the nurse should consult with the health care provider before starting the norepinephrine? a. The patient's central venous pressure is 3 mm Hg. b. The patient is in sinus tachycardia at 120 beats/min. c. The patient is receiving low dose dopamine (Intropin). d. The patient has had no urine output since being admitted.

ANS: A Adequate fluid administration is essential before administration of vasopressors to patients with hypovolemic shock. The patient's low central venous pressure indicates a need for more volume replacement. The other patient data are not contraindications to norepinephrine administration

Norepinephrine (Levophed) has been prescribed for a patient who was admitted with dehydration and hypotension. Which patient data indicate that the nurse should consult with the health care provider before starting the norepinephrine? a. The patient's central venous pressure is 3 mm Hg. b. The patient is in sinus tachycardia at 120 beats/min. c. The patient is receiving low dose dopamine (Intropin). d. The patient has had no urine output since being admitted.

ANS: A Adequate fluid administration is essential before administration of vasopressors to patients with hypovolemic shock. The patient's low central venous pressure indicates a need for more volume replacement. The other patient data are not contraindications to norepinephrine administration.

Norepinephrine (Levophed) has been prescribed for a patient who was admitted with dehydration and hypotension. Which patient data indicate that the nurse should consult with the health care provider before starting the norepinephrine? a. The patient's central venous pressure is 3 mm Hg. b. The patient is in sinus tachycardia at 120 beats/min. c. The patient is receiving low dose dopamine (Intropin). d. The patient has had no urine output since being admitted.

ANS: A Adequate fluid administration is essential before administration of vasopressors to patients with hypovolemic shock. The patient's low central venous pressure indicates a need for more volume replacement. The other patient data are not contraindications to norepinephrine administration.

A patient with septic shock has a BP of 70/46 mm Hg, pulse 136, respirations 32, temperature 104° F, and blood glucose 246 mg/dL. Which of these prescribed interventions will the nurse implement first? a. Give normal saline IV at 500 mL/hr. b. Infuse drotrecogin- (Xigris) 24 mcg/kg. c. Start insulin drip to maintain blood glucose at 110 to 150 mg/dL. d. Titrate norepinephrine (Levophed) to keep mean arterial pressure (MAP) at 65 to 70 mm Hg.

ANS: A Because of the low systemic vascular resistance (SVR) associated with septic shock, fluid resuscitation is the initial therapy. The other actions also are appropriate and should be initiated quickly as well. DIF: Cognitive Level: Application REF: 1735-1737 OBJ: Special Questions: Prioritization TOP: Nursing Process: Implementation MSC: NCLEX: Physiological Integrity

A patient with septic shock has a BP of 70/46 mm Hg, pulse 136, respirations 32, temperature 104° F, and blood glucose 246 mg/dL. Which intervention ordered by the health care provider should the nurse implement first? a. Give normal saline IV at 500 mL/hr. b. Give acetaminophen (Tylenol) 650 mg rectally. c. Start insulin drip to maintain blood glucose at 110 to 150 mg/dL. d. Start norepinephrine (Levophed) to keep systolic blood pressure >90 mm Hg.

ANS: A Because of the low systemic vascular resistance (SVR) associated with septic shock, fluid resuscitation is the initial therapy. The other actions also are appropriate, and should be initiated quickly as well.

A patient with septic shock has a BP of 70/46 mm Hg, pulse 136, respirations 32, temperature 104° F, and blood glucose 246 mg/dL. Which intervention ordered by the health care provider should the nurse implement first? a. Give normal saline IV at 500 mL/hr. b. Give acetaminophen (Tylenol) 650 mg rectally. c. Start insulin drip to maintain blood glucose at 110 to 150 mg/dL. d. Start norepinephrine (Levophed) to keep systolic blood pressure >90 mm Hg.

ANS: A Because of the low systemic vascular resistance (SVR) associated with septic shock, fluid resuscitation is the initial therapy. The other actions also are appropriate, and should be initiated quickly as well.

A patient with septic shock has a BP of 70/46 mm Hg, pulse 136, respirations 32, temperature 104° F, and blood glucose 246 mg/dL. Which intervention ordered by the health care provider should the nurse implement first? a. Give normal saline IV at 500 mL/hr. b.Give acetaminophen (Tylenol) 650 mg rectally. c. Start insulin drip to maintain blood glucose at 110 to 150 mg/dL. d. Start norepinephrine (Levophed) to keep systolic blood pressure >90 mm Hg.

ANS: A Because of the low systemic vascular resistance (SVR) associated with septic shock, fluid resuscitation is the initial therapy. The other actions also are appropriate, and should be initiated quickly as well.

The nurse is caring for a 78-year-old patient with aortic stenosis. Which assessment data obtained by the nurse would be most important to report to the health care provider? a. The patient complains of chest pressure when ambulating. b. A loud systolic murmur is heard along the right sternal border. c. A thrill is palpated at the second intercostal space, right sternal border. d. The point of maximum impulse (PMI) is at the left midclavicular line.

ANS: A Chest pressure (or pain) occurring with aortic stenosis is caused by cardiac ischemia, and reporting this information would be a priority. A systolic murmur and thrill are expected in a patient with aortic stenosis. A PMI at the left midclavicular line is normal.

The nurse is caring for a 64-year-old patient admitted with mitral valve regurgitation. Which information obtained by the nurse when assessing the patient should be communicated to the health care provider immediately? a. The patient has bilateral crackles. b. The patient has bilateral, 4+ peripheral edema. c. The patient has a loud systolic murmur across the precordium. d. The patient has a palpable thrill felt over the left anterior chest.

ANS: A Crackles that are audible throughout the lungs indicate that the patient is experiencing severe left ventricular failure with pulmonary congestion and needs immediate interventions such as diuretics. A systolic murmur and palpable thrill would be expected in a patient with mitral regurgitation. Although 4+ peripheral edema indicates a need for a change in therapy, it does not need to be addressed urgently.

A patient with septic shock has a urine output of 20 mL/hr for the past 3 hours. The pulse rate is 120 and the central venous pressure and pulmonary artery wedge pressure are low. Which of these orders by the health care provider will the nurse question? a. Give furosemide (Lasix) 40 mg IV. b. Increase normal saline infusion to 150 mL/hr. c. Administer hydrocortisone (SoluCortef) 100 mg IV. d. Prepare to give drotrecogin alpha (Xigris) 24 mcg/kg/hr.

ANS: A Furosemide will lower the filling pressures and renal perfusion further for the patient with septic shock. The other orders are appropriate. DIF: Cognitive Level: Application REF: 1724-1726 | 1731 | 1733 TOP: Nursing Process: Implementation MSC: NCLEX: Physiological Integrity

A 78-kg patient with septic shock has a urine output of 30 mL/hr for the past 3 hours. The pulse rate is 120/minute and the central venous pressure and pulmonary artery wedge pressure are low. Which order by the health care provider will the nurse question? a. Give PRN furosemide (Lasix) 40 mg IV. b. Increase normal saline infusion to 250 mL/hr. c. Administer hydrocortisone (Solu-Cortef) 100 mg IV. d. Titrate norepinephrine (Levophed) to keep systolic BP >90 mm Hg.

ANS: A Furosemide will lower the filling pressures and renal perfusion further for the patient with septic shock. The other orders are appropriate

0.9% NaCl (NSS) & Lactated Ringer's (LR) - Mechanism of Action

Fluid primarily remains in the intravascular space Increases intravascular volume

Hypertonic 1.8%, 3%, 5% NaCl - Mechanism of Action

Fluid remains in the intravascular space. Rapid volume expander

Crystalloid

Fluid replacement Doesn't cause allergic reactions -Normal saline, Ringer's Lactate

1. A 78-kg patient with septic shock has a urine output of 30 mL/hr for the past 3 hours. The pulse rate is 120/minute and the central venous pressure and pulmonary artery wedge pressure are low. Which order by the health care provider will the nurse question? a. Give PRN furosemide (Lasix) 40 mg IV. b. Increase normal saline infusion to 250 mL/hr. c. Administer hydrocortisone (Solu-Cortef) 100 mg IV. d. Titrate norepinephrine (Levophed) to keep systolic BP >90 mm Hg.

ANS: A Furosemide will lower the filling pressures and renal perfusion further for the patient with septic shock. The other orders are appropriate.

A 78-kg patient with septic shock has a urine output of 30 mL/hr for the past 3 hours. The pulse rate is 120/minute and the central venous pressure and pulmonary artery wedge pressure are low. Which order by the health care provider will the nurse question? a. Give PRN furosemide (Lasix) 40 mg IV. b. Increase normal saline infusion to 250 mL/hr. c. Administer hydrocortisone (Solu-Cortef) 100 mg IV. d. Titrate norepinephrine (Levophed) to keep systolic BP >90 mm Hg.

ANS: A Furosemide will lower the filling pressures and renal perfusion further for the patient with septic shock. The other orders are appropriate.

A patient in the intensive care unit with acute decompensated heart failure (ADHF) complains of severe dyspnea and is anxious, tachypneic, and tachycardic. All of the following medications have been ordered for the patient. The nurse's priority action will be to a. give IV morphine sulfate 4 mg. b. give IV diazepam (Valium) 2.5 mg. c. increase nitroglycerin (Tridil) infusion by 5 mcg/min. d. increase dopamine (Intropin) infusion by 2 mcg/kg/min.

ANS: A Morphine improves alveolar gas exchange, improves cardiac output by reducing ventricular preload and afterload, decreases anxiety, and assists in reducing the subjective feeling of dyspnea. Diazepam may decrease patient anxiety, but it will not improve the cardiac output or gas exchange. Increasing the dopamine may improve cardiac output, but it will also increase the heart rate and myocardial oxygen consumption. Nitroglycerin will improve cardiac output and may be appropriate for this patient, but it will not directly reduce anxiety and will not act as quickly as morphine to decrease dyspnea.

36. A patient who is receiving positive pressure ventilation is scheduled for a spontaneous breathing trial (SBT). Which finding by the nurse is most important to discuss with the health care provider before starting the SBT? a. New ST segment elevation is noted on the cardiac monitor. b. Enteral feedings are being given through an orogastric tube. c. Scattered rhonchi are heard when auscultating breath sounds. d. HYDROmorphone (Dilaudid) is being used to treat postoperative pain.

ANS: A Myocardial ischemia is a contraindication for ventilator weaning. The ST segment elevation is an indication that weaning should be postponed until further investigation and/or treatment for myocardial ischemia can be done. The other information will also be shared with the health care provider, but ventilator weaning can proceed when opioids are used for pain management, abnormal lung sounds are present, or enteral feedings are being used. DIF: Cognitive Level: Apply (application) REF: 1626 OBJ: Special Questions: Prioritization TOP: Nursing Process: Assessment MSC: NCLEX: Physiological Integrity

While caring for a patient with aortic stenosis, the nurse identifies a nursing diagnosis of acute pain related to decreased coronary blood flow. A priority nursing intervention for this patient would be to a. promote rest to decrease myocardial oxygen demand. b. teach the patient about the need for anticoagulant therapy. c. teach the patient to use sublingual nitroglycerin for chest pain. d. raise the head of the bed 60 degrees to decrease venous return.

ANS: A Rest is recommended to balance myocardial oxygen supply and demand and to decrease chest pain. The patient with aortic stenosis requires higher preload to maintain cardiac output, so nitroglycerin and measures to decrease venous return are contraindicated. Anticoagulation is not recommended unless the patient has atrial fibrillation.

11. Which assessment finding obtained by the nurse when caring for a patient with a right radial arterial line indicates a need for the nurse to take immediate action? a. The right hand is cooler than the left hand. b. The mean arterial pressure (MAP) is 77 mm Hg. c. The system is delivering 3 mL of flush solution per hour. d. The flush bag and tubing were last changed 3 days previously.

ANS: A The change in temperature of the left hand suggests that blood flow to the left hand is impaired. The flush system needs to be changed every 96 hours. A mean arterial pressure (MAP) of 75 mm Hg is normal. Flush systems for hemodynamic monitoring are set up to deliver 3 to 6 mL/hour of flush solution.

11. Which assessment finding obtained by the nurse when caring for a patient with a right radial arterial line indicates a need for the nurse to take immediate action? a. The right hand is cooler than the left hand. b. The mean arterial pressure (MAP) is 77 mm Hg. c. The system is delivering 3 mL of flush solution per hour. d. The flush bag and tubing were last changed 3 days previously.

ANS: A The change in temperature of the left hand suggests that blood flow to the left hand is impaired. The flush system needs to be changed every 96 hours. A mean arterial pressure (MAP) of 75 mm Hg is normal. Flush systems for hemodynamic monitoring are set up to deliver 3 to 6 mL/hour of flush solution. DIF: Cognitive Level: Apply (application) REF: 1606 TOP: Nursing Process: Assessment MSC: NCLEX: Physiological Integrity

During change-of-shift report, the nurse is told that a patient has been admitted with dehydration and hypotension after having vomiting and diarrhea for 4 days. Which finding is most important for the nurse to report to the health care provider? a. New onset of confusion b. Heart rate 112 beats/minute c. Decreased bowel sounds d. Pale, cool, and dry extremities

ANS: A The changes in mental status are indicative that the patient is in the progressive stage of shock and that rapid intervention is needed to prevent further deterioration. The other information is consistent with compensatory shock

During change-of-shift report, the nurse is told that a patient has been admitted with dehydration and hypotension after having vomiting and diarrhea for 4 days. Which finding is most important for the nurse to report to the health care provider? a. New onset of confusion b. Heart rate 112 beats/minute c. Decreased bowel sounds d. Pale, cool, and dry extremities

ANS: A The changes in mental status are indicative that the patient is in the progressive stage of shock and that rapid intervention is needed to prevent further deterioration. The other information is consistent with compensatory shock.

The patient with neurogenic shock is receiving a phenylephrine (Neo-Synephrine) infusion through a left forearm IV. Which assessment information obtained by the nurse indicates a need for immediate action? a. The patient's IV infusion site is cool and pale. b. The patient has warm, dry skin on the extremities. c. The patient has an apical pulse rate of 58 beats/min. d. The patient's urine output has been 28 mL over the last hour.

ANS: A The coldness and pallor at the infusion site suggest extravasation of the phenylephrine. The nurse should discontinue the IV and, if possible, infuse the medication into a central line. An apical pulse of 58 is typical for neurogenic shock but does not indicate an immediate need for nursing intervention. A 28 mL urinary output over 1 hour would require the nurse to monitor the output over the next hour, but an immediate change in therapy is not indicated. Warm, dry skin is consistent with early neurogenic shock, but it does not indicate a need for a change in therapy or immediate action. DIF: Cognitive Level: Application REF: 1733-1734 OBJ: Special Questions: Prioritization TOP: Nursing Process: Assessment MSC: NCLEX: Physiological Integrity

Which information obtained by the nurse when caring for a patient who has cardiogenic shock indicates that the patient may be developing multiple organ dysfunction syndrome (MODS)? a. The patient's serum creatinine level is elevated. b. The patient complains of intermittent chest pressure. c. The patient has crackles throughout both lung fields. d. The patient's extremities are cool and pulses are weak.

ANS: A The elevated serum creatinine level indicates that the patient has renal failure as well as heart failure. The crackles, chest pressure, and cool extremities are all consistent with the patient's diagnosis of cardiogenic shock. DIF: Cognitive Level: Application REF: 1740-1741 TOP: Nursing Process: Assessment MSC: NCLEX: Physiological Integrity

Which data collected by the nurse caring for a patient who has cardiogenic shock indicate that the patient may be developing multiple organ dysfunction syndrome (MODS)? a. The patient's serum creatinine level is elevated. b. The patient complains of intermittent chest pressure. c. The patient's extremities are cool and pulses are weak. d. The patient has bilateral crackles throughout lung fields.

ANS: A The elevated serum creatinine level indicates that the patient has renal failure as well as heart failure. The crackles, chest pressure, and cool extremities are all consistent with the patient's diagnosis of cardiogenic shock.

22. A patient with respiratory failure has arterial pressure-based cardiac output (APCO) monitoring and is receiving mechanical ventilation with peak end-expiratory pressure (PEEP) of 12 cm H2O. Which information indicates that a change in the ventilator settings may be required? a. The arterial pressure is 90/46. b. The heart rate is 58 beats/minute. c. The stroke volume is increased. d. The stroke volume variation is 12%.

ANS: A The hypotension suggests that the high intrathoracic pressure caused by the PEEP may be decreasing venous return and (potentially) cardiac output. The other assessment data would not be a direct result of PEEP and mechanical ventilation. DIF: Cognitive Level: Apply (application) REF: 1622-1624 TOP: Nursing Process: Evaluation MSC: NCLEX: Physiological Integrity

A patient is treated in the emergency department (ED) for shock of unknown etiology. The first action by the nurse should be to a. administer oxygen. b. attach a cardiac monitor. c. obtain the blood pressure. d. check the level of consciousness.

ANS: A The initial actions of the nurse are focused on the ABCs—airway, breathing, circulation—and administration of oxygen should be done first. The other actions should be accomplished as rapidly as possible after oxygen administration. DIF: Cognitive Level: Application REF: 1729-1731 | 1732 | 1733 | 1736-1737 OBJ: Special Questions: Prioritization TOP: Nursing Process: Implementation MSC: NCLEX: Physiological Integrity

A patient is admitted to the emergency department (ED) for shock of unknown etiology. The first action by the nurse should be to a. administer oxygen. b. obtain a 12-lead electrocardiogram (ECG). c. obtain the blood pressure. d. check the level of consciousness.

ANS: A The initial actions of the nurse are focused on the ABCs—airway, breathing, and circulation—and administration of oxygen should be done first. The other actions should be accomplished as rapidly as possible after oxygen administration

Two days after an acute myocardial infarction (MI), a patient complains of stabbing chest pain that increases with a deep breath. Which action will the nurse take first? a. Auscultate the heart sounds. b. Check the patient's temperature. c. Notify the patient's health care provider. d. Give the PRN acetaminophen (Tylenol).

ANS: A The patient's clinical manifestations and history are consistent with pericarditis, and the first action by the nurse should be to listen for a pericardial friction rub. Checking the temperature and notifying the health care provider are also appropriate actions but would not be done before listening for a rub. It is not stated for what symptom (e.g., headache) or finding (e.g., increased temperature) the PRN acetaminophen (Tylenol) is ordered.

The health-care provider ordered a femoral angiogram for the client diagnosed with arterial occlusive disease. Which intervention should the nurse implement? 4. Inform the client that fluids will be increased after the procedure.

Fluids will help flush the contrast dye out of the body and help prevent kidney damage.

A patient with shock of unknown etiology whose hemodynamic monitoring indicates BP 92/54, pulse 64, and an elevated pulmonary artery wedge pressure has the following collaborative interventions prescribed. Which intervention will the nurse question? a. Infuse normal saline at 250 mL/hr. b. Keep head of bed elevated to 30 degrees. c. Give nitroprusside (Nipride) unless systolic BP <90 mm Hg. d. Administer dobutamine (Dobutrex) to keep systolic BP >90 mm Hg.

ANS: A The patient's elevated pulmonary artery wedge pressure indicates volume excess. A normal saline infusion at 250 mL/hr will exacerbate this. The other actions are appropriate for the patient. DIF: Cognitive Level: Application REF: 1719 | 1721-1722 | 1735 TOP: Nursing Process: Planning MSC: NCLEX: Physiological Integrity

A nurse is caring for a patient with shock of unknown etiology whose hemodynamic monitoring indicates BP 92/54, pulse 64, and an elevated pulmonary artery wedge pressure. Which collaborative intervention ordered by the health care provider should the nurse question? a. Infuse normal saline at 250 mL/hr. b. Keep head of bed elevated to 30 degrees. c. Hold nitroprusside (Nipride) if systolic BP <90 mm Hg. d. Titrate dobutamine (Dobutrex) to keep systolic BP >90 mm Hg.

ANS: A The patient's elevated pulmonary artery wedge pressure indicates volume excess. A saline infusion at 250 mL/hr will exacerbate the volume excess. The other actions are appropriate for the patient.

A nurse is caring for a patient with shock of unknown etiology whose hemodynamic monitoring indicates BP 92/54, pulse 64, and an elevated pulmonary artery wedge pressure. Which collaborative intervention ordered by the health care provider should the nurse question? a. Infuse normal saline at 250 mL/hr. b. Keep head of bed elevated to 30 degrees. c. Hold nitroprusside (Nipride) if systolic BP <90 mm Hg. d. Titrate dobutamine (Dobutrex) to keep systolic BP >90 mm Hg.

ANS: A The patient's elevated pulmonary artery wedge pressure indicates volume excess. A saline infusion at 250 mL/hr will exacerbate the volume excess. The other actions are appropriate for the patient.

During a visit to a 78-year-old with chronic heart failure, the home care nurse finds that the patient has ankle edema, a 2-kg weight gain over the past 2 days, and complains of "feeling too tired to get out of bed." Based on these data, the best nursing diagnosis for the patient is a. activity intolerance related to fatigue. b. disturbed body image related to weight gain. c. impaired skin integrity related to ankle edema. d. impaired gas exchange related to dyspnea on exertion.

ANS: A The patient's statement supports the diagnosis of activity intolerance. There are no data to support the other diagnoses, although the nurse will need to assess for other patient problems.

A patient with neurogenic shock has just arrived in the emergency department after a diving accident. He has a cervical collar in place. Which of the following actions should the nurse take (select all that apply)? a. Prepare to administer atropine IV. b. Obtain baseline body temperature. c. Prepare for intubation and mechanical ventilation. d. Administer large volumes of lactated Ringer's solution. e. Administer high-flow oxygen (100%) by non-rebreather mask.

ANS: A, B, C, E All of the actions are appropriate except to give large volumes of lactated Ringer's solution. The patient with neurogenic shock usually has a normal blood volume, and it is important not to volume overload the patient. In addition, lactated Ringer's solution is used cautiously in all shock situations because the failing liver cannot convert lactate to bicarbonate. DIF: Cognitive Level: Application REF: 1736-1737 OBJ: Special Questions: Alternate Item Format TOP: Nursing Process: Implementation MSC: NCLEX: Physiological Integrity

Which preventive actions by the nurse will help limit the development of systemic inflammatory response syndrome (SIRS) in patients admitted to the hospital (select all that apply)? a. Use aseptic technique when caring for invasive lines or devices. b. Ambulate postoperative patients as soon as possible after surgery. c. Remove indwelling urinary catheters as soon as possible after surgery. d. Advocate for parenteral nutrition for patients who cannot take oral feedings. e. Administer prescribed antibiotics within 1 hour for patients with possible sepsis.

ANS: A, B, C, E Because sepsis is the most frequent etiology for SIRS, measures to avoid infection such as removing indwelling urinary catheters as soon as possible, use of aseptic technique, and early ambulation should be included in the plan of care. Adequate nutrition is important in preventing SIRS. Enteral, rather than parenteral, nutrition is preferred when patients are unable to take oral feedings because enteral nutrition helps maintain the integrity of the intestine, thus decreasing infection risk. Antibiotics should be administered within 1 hour after being prescribed to decrease the risk of sepsis progressing to SIRS

Which preventive actions by the nurse will help limit the development of systemic inflammatory response syndrome (SIRS) in patients admitted to the hospital (select all that apply)? a. Use aseptic technique when caring for invasive lines or devices. b. Ambulate postoperative patients as soon as possible after surgery. c. Remove indwelling urinary catheters as soon as possible after surgery. d. Advocate for parenteral nutrition for patients who cannot take oral feedings. e. Administer prescribed antibiotics within 1 hour for patients with possible sepsis.

ANS: A, B, C, E Because sepsis is the most frequent etiology for SIRS, measures to avoid infection such as removing indwelling urinary catheters as soon as possible, use of aseptic technique, and early ambulation should be included in the plan of care. Adequate nutrition is important in preventing SIRS. Enteral, rather than parenteral, nutrition is preferred when patients are unable to take oral feedings because enteral nutrition helps maintain the integrity of the intestine, thus decreasing infection risk. Antibiotics should be administered within 1 hour after being prescribed to decrease the risk of sepsis progressing to SIRS.

Which preventive actions by the nurse will help limit the development of systemic inflammatory response syndrome (SIRS) in patients admitted to the hospital (select all that apply)? a. Use aseptic technique when caring for invasive lines or devices. b. Ambulate postoperative patients as soon as possible after surgery. c. Remove indwelling urinary catheters as soon as possible after surgery. d. Advocate for parenteral nutrition for patients who cannot take oral feedings. e. Administer prescribed antibiotics within 1 hour for patients with possible sepsis.

ANS: A, B, C, E Because sepsis is the most frequent etiology for SIRS, measures to avoid infection such as removing indwelling urinary catheters as soon as possible, use of aseptic technique, and early ambulation should be included in the plan of care. Adequate nutrition is important in preventing SIRS. Enteral, rather than parenteral, nutrition is preferred when patients are unable to take oral feedings because enteral nutrition helps maintain the integrity of the intestine, thus decreasing infection risk. Antibiotics should be administered within 1 hour after being prescribed to decrease the risk of sepsis progressing to SIRS.

A patient with suspected neurogenic shock after a diving accident has arrived in the emergency department. A cervical collar is in place. Which actions should the nurse take (select all that apply)? a. Prepare to administer atropine IV. b. Obtain baseline body temperature. c. Infuse large volumes of lactated Ringer's solution. d. Provide high-flow oxygen (100%) by non-rebreather mask. e. Prepare for emergent intubation and mechanical ventilation.

ANS: A, B, D, E All of the actions are appropriate except to give large volumes of lactated Ringer's solution. The patient with neurogenic shock usually has a normal blood volume, and it is important not to volume overload the patient. In addition, lactated Ringer's solution is used cautiously in all shock situations because the failing liver cannot convert lactate to bicarbonate

A patient with suspected neurogenic shock after a diving accident has arrived in the emergency department. A cervical collar is in place. Which actions should the nurse take (select all that apply)? a. Prepare to administer atropine IV. b. Obtain baseline body temperature. c. Infuse large volumes of lactated Ringer's solution. d. Provide high-flow oxygen (100%) by non-rebreather mask. e. Prepare for emergent intubation and mechanical ventilation.

ANS: A, B, D, E All of the actions are appropriate except to give large volumes of lactated Ringer's solution. The patient with neurogenic shock usually has a normal blood volume, and it is important not to volume overload the patient. In addition, lactated Ringer's solution is used cautiously in all shock situations because the failing liver cannot convert lactate to bicarbonate.

Based on the Joint Commission Core Measures for patients with heart failure, which topics should the nurse include in the discharge teaching plan for a patient who has been hospitalized with chronic heart failure (select all that apply)? a. How to take and record daily weight b. Importance of limiting aerobic exercise c. Date and time of follow-up appointment d. Symptoms indicating worsening heart failure e. Actions and side effects of prescribed medications

ANS: A, C, D, E The Joint Commission Core Measures state that patients should be taught about prescribed medications, follow-up appointments, weight monitoring, and actions to take for worsening symptoms. Patients with heart failure are encouraged to begin or continue aerobic exercises such as walking, while self-monitoring to avoid excessive fatigue.

You are watching the cardiac monitor, and a patient's rhythm suddenly changes. There are no P waves. Instead, there are fine, wavy lines between the QRS complexes. The QRS complexes each measure 0.08 second (narrow), but they occur irregularly with a rate of 120 beats/minute. You correctly interpret that this rhythm is A. sinus tachycardia. B. atrial fibrillation. C. ventricular fibrillation. D. ventricular tachycardia.

ANS: B Atrial fibrillation is represented on the cardiac monitor by irregular R-R intervals and small fibrillatory (F) waves. There are no normal P waves because the atria are not contracting, just fibrillating. Reference: 827

impaired tissue perfusion and cellular metabolism

shock

You obtain a 6-second rhythm strip, and document the following analysis: atrial rate of 70 beats/minute, regular; ventricular rate of 40 beats/minute, regular; QRS of 0.04 second; no relationship between P waves and QRS complexes; and atria and ventricles beating independently of each other. What is the correct interpretation of this rhythm strip? A. Sinus dysrhythmias B. Third-degree heart block C. Wenckebach phenomenon D. Premature ventricular contractions

ANS: B Third-degree heart block represents a loss of communication between the atrium and ventricles. This is depicted on the rhythm strip because there is no relationship between the P waves, representing atrial contraction, and QRS complexes, representing ventricular contraction. The atria are beating totally on their own at 70 beats/minute, whereas the ventricles are pacing themselves at 40 beats/minute. Reference: 830

A normal cardiac impulse begins in the

sinoatrial (SA) node in the upper right atrium.

4. Following surgery for an abdominal aortic aneurysm, a patient's central venous pressure (CVP) monitor indicates low pressures. Which action is a priority for the nurse to take? a. Administer IV diuretic medications. b. Increase the IV fluid infusion per protocol. c. Document the CVP and continue to monitor. d. Elevate the head of the patient's bed to 45 degrees.

ANS: B A low CVP indicates hypovolemia and a need for an increase in the infusion rate. Diuretic administration will contribute to hypovolemia and elevation of the head may decrease cerebral perfusion. Documentation and continued monitoring is an inadequate response to the low CVP.

4. Following surgery for an abdominal aortic aneurysm, a patient's central venous pressure (CVP) monitor indicates low pressures. Which action is a priority for the nurse to take? a. Administer IV diuretic medications. b. Increase the IV fluid infusion per protocol. c. Document the CVP and continue to monitor. d. Elevate the head of the patient's bed to 45 degrees.

ANS: B A low CVP indicates hypovolemia and a need for an increase in the infusion rate. Diuretic administration will contribute to hypovolemia and elevation of the head may decrease cerebral perfusion. Documentation and continued monitoring is an inadequate response to the low CVP. DIF: Cognitive Level: Apply (application) REF: 1609 TOP: Nursing Process: Planning MSC: NCLEX: Physiological Integrity

After change-of-shift report in the progressive care unit, who should the nurse care for first? a. Patient who had an inferior myocardial infarction 2 days ago and has crackles in the lung bases b. Patient with suspected urosepsis who has new orders for urine and blood cultures and antibiotics c. Patient who had a T5 spinal cord injury 1 week ago and currently has a heart rate of 54 beats/minute d. Patient admitted with anaphylaxis 3 hours ago who now has clear lung sounds and a blood pressure of 108/58 mm Hg

ANS: B Antibiotics should be administered within the first hour for patients who have sepsis or suspected sepsis in order to prevent progression to systemic inflammatory response syndrome (SIRS) and septic shock. The data on the other patients indicate that they are more stable. Crackles heard only at the lung bases do not require immediate intervention in a patient who has had a myocardial infarction. Mild bradycardia does not usually require atropine in patients who have a spinal cord injury. The findings for the patient admitted with anaphylaxis indicate resolution of bronchospasm and hypotension

After change-of-shift report in the progressive care unit, who should the nurse care for first? a. Patient who had an inferior myocardial infarction 2 days ago and has crackles in the lung bases b. Patient with suspected urosepsis who has new orders for urine and blood cultures and antibiotics c. Patient who had a T5 spinal cord injury 1 week ago and currently has a heart rate of 54 beats/minute d. Patient admitted with anaphylaxis 3 hours ago who now has clear lung sounds and a blood pressure of 108/58 mm Hg

ANS: B Antibiotics should be administered within the first hour for patients who have sepsis or suspected sepsis in order to prevent progression to systemic inflammatory response syndrome (SIRS) and septic shock. The data on the other patients indicate that they are more stable. Crackles heard only at the lung bases do not require immediate intervention in a patient who has had a myocardial infarction. Mild bradycardia does not usually require atropine in patients who have a spinal cord injury. The findings for the patient admitted with anaphylaxis indicate resolution of bronchospasm and hypotension.

After change-of-shift report in the progressive care unit, who should the nurse care for first? a. Patient who had an inferior myocardial infarction 2 days ago and has crackles in the lung bases b. Patient with suspected urosepsis who has new orders for urine and blood cultures and antibiotics c. Patient who had a T5 spinal cord injury 1 week ago and currently has a heart rate of 54 beats/minute d. Patient admitted with anaphylaxis 3 hours ago who now has clear lung sounds and a blood pressure of 108/58 mm Hg

ANS: B Antibiotics should be administered within the first hour for patients who have sepsis or suspected sepsis in order to prevent progression to systemic inflammatory response syndrome (SIRS) and septic shock. The data on the other patients indicate that they are more stable. Crackles heard only at the lung bases do not require immediate intervention in a patient who has had a myocardial infarction. Mild bradycardia does not usually require atropine in patients who have a spinal cord injury. The findings for the patient admitted with anaphylaxis indicate resolution of bronchospasm and hypotension.

The following interventions are ordered by the health care provider for a patient who has respiratory distress and syncope after eating strawberries. Which will the nurse complete first? a. Start a normal saline infusion. b. Give epinephrine (Adrenalin). c. Start continuous ECG monitoring. d. Give diphenhydramine (Benadryl).

ANS: B Epinephrine rapidly causes peripheral vasoconstriction, dilates the bronchi, and blocks the effects of histamine and reverses the vasodilation, bronchoconstriction, and histamine release that cause the symptoms of anaphylaxis. The other interventions are also appropriate but would not be the first ones completed

Which finding is the best indicator that the fluid resuscitation for a patient with hypovolemic shock has been effective? a. Hemoglobin is within normal limits. b. Urine output is 60 mL over the last hour. c. Central venous pressure (CVP) is normal. d. Mean arterial pressure (MAP) is 72 mm Hg.

ANS: B Assessment of end organ perfusion, such as an adequate urine output, is the best indicator that fluid resuscitation has been successful. The hemoglobin level, CVP, and MAP are useful in determining the effects of fluid administration, but they are not as useful as data indicating good organ perfusion

Which finding is the best indicator that the fluid resuscitation for a patient with hypovolemic shock has been effective? a. Hemoglobin is within normal limits. b. Urine output is 60 mL over the last hour. c. Central venous pressure (CVP) is normal. d. Mean arterial pressure (MAP) is 72 mm Hg.

ANS: B Assessment of end organ perfusion, such as an adequate urine output, is the best indicator that fluid resuscitation has been successful. The hemoglobin level, CVP, and MAP are useful in determining the effects of fluid administration, but they are not as useful as data indicating good organ perfusion.

Which finding is the best indicator that the fluid resuscitation for a patient with hypovolemic shock has been effective? a. Hemoglobin is within normal limits. b. Urine output is 60 mL over the last hour. c. Central venous pressure (CVP) is normal. d. Mean arterial pressure (MAP) is 72 mm Hg.

ANS: B Assessment of end organ perfusion, such as an adequate urine output, is the best indicator that fluid resuscitation has been successful. The hemoglobin level, CVP, and MAP are useful in determining the effects of fluid administration, but they are not as useful as data indicating good organ perfusion.

The nurse identifies the nursing diagnosis of decreased cardiac output related to valvular insufficiency for the patient with infective endocarditis (IE) based on which assessment finding(s)? a. Fever, chills, and diaphoresis b. Urine output less than 30 mL/hr c. Petechiae on the inside of the mouth and conjunctiva d. Increase in heart rate of 15 beats/minute with walking

ANS: B Decreased renal perfusion caused by inadequate cardiac output will lead to decreased urine output. Petechiae, fever, chills, and diaphoresis are symptoms of IE, but are not caused by decreased cardiac output. An increase in pulse rate of 15 beats/minute is normal with exercise.

12. The central venous oxygen saturation (ScvO2) is decreasing in a patient who has severe pancreatitis. To determine the possible cause of the decreased ScvO2, the nurse assesses the patient's a. lipase. b. temperature. c. urinary output. d. body mass index.

ANS: B Elevated temperature increases metabolic demands and oxygen use by tissues, resulting in a drop in oxygen saturation of central venous blood. Information about the patient's body mass index, urinary output, and lipase will not help in determining the cause of the patient's drop in ScvO2.

12. The central venous oxygen saturation (ScvO2) is decreasing in a patient who has severe pancreatitis. To determine the possible cause of the decreased ScvO2, the nurse assesses the patient's a. lipase. b. temperature. c. urinary output. d. body mass index.

ANS: B Elevated temperature increases metabolic demands and oxygen use by tissues, resulting in a drop in oxygen saturation of central venous blood. Information about the patient's body mass index, urinary output, and lipase will not help in determining the cause of the patient's drop in ScvO2. DIF: Cognitive Level: Apply (application) REF: 1609 TOP: Nursing Process: Assessment MSC: NCLEX: Physiological Integrity

The following interventions are ordered by the health care provider for a patient who has respiratory distress and syncope after eating strawberries. Which will the nurse complete first? a. Start a normal saline infusion. b. Give epinephrine (Adrenalin). c. Start continuous ECG monitoring. d. Give diphenhydramine (Benadryl).

ANS: B Epinephrine rapidly causes peripheral vasoconstriction, dilates the bronchi, and blocks the effects of histamine and reverses the vasodilation, bronchoconstriction, and histamine release that cause the symptoms of anaphylaxis. The other interventions are also appropriate but would not be the first ones completed.

6. The intensive care unit (ICU) nurse educator will determine that teaching about arterial pressure monitoring for a new staff nurse has been effective when the nurse a. balances and calibrates the monitoring equipment every 2 hours. b. positions the zero-reference stopcock line level with the phlebostatic axis. c. ensures that the patient is supine with the head of the bed flat for all readings. d. rechecks the location of the phlebostatic axis when changing the patient's position.

ANS: B For accurate measurement of pressures, the zero-reference level should be at the phlebostatic axis. There is no need to rebalance and recalibrate monitoring equipment hourly. Accurate hemodynamic readings are possible with the patient's head raised to 45 degrees or in the prone position. The anatomic position of the phlebostatic axis does not change when patients are repositioned.

6. The intensive care unit (ICU) nurse educator will determine that teaching about arterial pressure monitoring for a new staff nurse has been effective when the nurse a. balances and calibrates the monitoring equipment every 2 hours. b. positions the zero-reference stopcock line level with the phlebostatic axis. c. ensures that the patient is supine with the head of the bed flat for all readings. d. rechecks the location of the phlebostatic axis when changing the patient's position.

ANS: B For accurate measurement of pressures, the zero-reference level should be at the phlebostatic axis. There is no need to rebalance and recalibrate monitoring equipment hourly. Accurate hemodynamic readings are possible with the patient's head raised to 45 degrees or in the prone position. The anatomic position of the phlebostatic axis does not change when patients are repositioned. DIF: Cognitive Level: Apply (application) REF: 1605 TOP: Nursing Process: Evaluation MSC: NCLEX: Safe and Effective Care Environment

After receiving report on the following patients, which patient should the nurse assess first? a. Patient with rheumatic fever who has sharp chest pain with a deep breath b. Patient with acute aortic regurgitation whose blood pressure is 86/54 mm Hg c. Patient with infective endocarditis who has a murmur and splinter hemorrhages d. Patient with dilated cardiomyopathy who has bilateral crackles at the lung bases

ANS: B Hypotension in patients with acute aortic regurgitation may indicate cardiogenic shock. The nurse should immediately assess this patient for other findings such as dyspnea or chest pain. The findings in the other patients are typical of their diagnoses and do not indicate a need for urgent assessment and intervention.

A 53-year-old patient with Stage D heart failure and type 2 diabetes asks the nurse whether heart transplant is a possible therapy. Which response by the nurse is most appropriate? a. "Because you have diabetes, you would not be a candidate for a heart transplant." b. "The choice of a patient for a heart transplant depends on many different factors." c. "Your heart failure has not reached the stage in which heart transplants are needed." d. "People who have heart transplants are at risk for multiple complications after surgery."

ANS: B Indications for a heart transplant include end-stage heart failure (Stage D), but other factors such as coping skills, family support, and patient motivation to follow the rigorous posttransplant regimen are also considered. Diabetic patients who have well-controlled blood glucose levels may be candidates for heart transplant. Although heart transplants can be associated with many complications, this response does not address the patient's question.

During the assessment of a 25-year-old patient with infective endocarditis (IE), the nurse would expect to find a. substernal chest pressure. b. a new regurgitant murmur. c. a pruritic rash on the chest. d. involuntary muscle movement.

ANS: B New regurgitant murmurs occur in IE because vegetations on the valves prevent valve closure. Substernal chest discomfort, rashes, and involuntary muscle movement are clinical manifestations of other cardiac disorders such as angina and rheumatic fever.

re-polarization is the

slower movement of ions across the membrane restoring the cell to the polarized state

To assess the patient with pericarditis for evidence of a pericardial friction rub, the nurse should a. listen for a rumbling, low-pitched, systolic murmur over the left anterior chest. b. auscultate by placing the diaphragm of the stethoscope on the lower left sternal border. c. ask the patient to cough during auscultation to distinguish the sound from a pleural friction rub. d. feel the precordial area with the palm of the hand to detect vibrations with cardiac contraction.

ANS: B Pericardial friction rubs are heard best with the diaphragm at the lower left sternal border. The nurse should ask the patient to hold his or her breath during auscultation to distinguish the sounds from a pleural friction rub. Friction rubs are not typically low pitched or rumbling and are not confined to systole. Rubs are not assessed by palpation.

29. When caring for a patient who has an arterial catheter in the left radial artery for arterial pressure-based cardiac output (APCO) monitoring, which information obtained by the nurse is most important to report to the health care provider? a. The patient has a positive Allen test. b. There is redness at the catheter insertion site. c. The mean arterial pressure (MAP) is 86 mm Hg. d. The dicrotic notch is visible in the arterial waveform.

ANS: B Redness at the catheter insertion site indicates possible infection. The Allen test is performed before arterial line insertion, and a positive test indicates normal ulnar artery perfusion. A MAP of 86 is normal and the dicrotic notch is normally present on the arterial waveform. DIF: Cognitive Level: Apply (application) REF: 1606 OBJ: Special Questions: Prioritization TOP: Nursing Process: Assessment MSC: NCLEX: Physiological Integrity

3. While family members are visiting, a patient has a respiratory arrest and is being resuscitated. Which action by the nurse is best? a. Tell the family members that watching the resuscitation will be very stressful. b. Ask family members if they wish to remain in the room during the resuscitation. c. Take the family members quickly out of the patient room and remain with them. d. Assign a staff member to wait with family members just outside the patient room.

ANS: B Research indicates that family members want the option of remaining in the room during procedures such as cardiopulmonary resuscitation (CPR) and that this decreases anxiety and facilitates grieving. The other options may be appropriate if the family decides not to remain with the patient.

3. While family members are visiting, a patient has a respiratory arrest and is being resuscitated. Which action by the nurse is best? a. Tell the family members that watching the resuscitation will be very stressful. b. Ask family members if they wish to remain in the room during the resuscitation. c. Take the family members quickly out of the patient room and remain with them. d. Assign a staff member to wait with family members just outside the patient room.

ANS: B Research indicates that family members want the option of remaining in the room during procedures such as cardiopulmonary resuscitation (CPR) and that this decreases anxiety and facilitates grieving. The other options may be appropriate if the family decides not to remain with the patient. DIF: Cognitive Level: Apply (application) REF: 1602 TOP: Nursing Process: Implementation MSC: NCLEX: Psychosocial Integrity

The nurse is admitting a patient with possible rheumatic fever. Which question on the admission health history will be most pertinent to ask? a. "Do you use any illegal IV drugs?" b. "Have you had a recent sore throat?" c. "Have you injured your chest in the last few weeks?" d. "Do you have a family history of congenital heart disease?"

ANS: B Rheumatic fever occurs as a result of an abnormal immune response to a streptococcal infection. Although illicit IV drug use should be discussed with the patient before discharge, it is not a risk factor for rheumatic fever, and would not be as pertinent when admitting the patient. Family history is not a risk factor for rheumatic fever. Chest injury would cause musculoskeletal chest pain rather than rheumatic fever.

When caring for a patient who has septic shock, which assessment finding is most important for the nurse to report to the health care provider? a. BP 92/56 mm Hg b. Skin cool and clammy c. Apical pulse 118 beats/min d. Arterial oxygen saturation 91%

ANS: B Since patients in the early stage of septic shock have warm and dry skin, the patient's cool and clammy skin indicates that shock is progressing. The other information also will be reported, but does not indicate deterioration of the patient's status. DIF: Cognitive Level: Application REF: 1723 OBJ: Special Questions: Prioritization TOP: Nursing Process: Assessment MSC: NCLEX: Physiological Integrity

Which intervention will the nurse include in the plan of care for a patient who has cardiogenic shock? a. Check temperature every 2 hours. b. Monitor breath sounds frequently. c. Maintain patient in supine position. d. Assess skin for flushing and itching.

ANS: B Since pulmonary congestion and dyspnea are characteristics of cardiogenic shock, the nurse should assess the breath sounds frequently. The head of the bed is usually elevated to decrease dyspnea in patients with cardiogenic shock. Elevated temperature and flushing or itching of the skin are not typical of cardiogenic shock

Which intervention will the nurse include in the plan of care for a patient who has cardiogenic shock? a. Check temperature every 2 hours. b. Monitor breath sounds frequently. c. Maintain patient in supine position. d. Assess skin for flushing and itching.

ANS: B Since pulmonary congestion and dyspnea are characteristics of cardiogenic shock, the nurse should assess the breath sounds frequently. The head of the bed is usually elevated to decrease dyspnea in patients with cardiogenic shock. Elevated temperature and flushing or itching of the skin are not typical of cardiogenic shock.

Which intervention will the nurse include in the plan of care for a patient who has cardiogenic shock? a. Check temperature every 2 hours. b. Monitor breath sounds frequently. c. Maintain patient in supine position. d. Assess skin for flushing and itching.

ANS: B Since pulmonary congestion and dyspnea are characteristics of cardiogenic shock, the nurse should assess the breath sounds frequently. The head of the bed is usually elevated to decrease dyspnea in patients with cardiogenic shock. Elevated temperature and flushing or itching of the skin are not typical of cardiogenic shock.

Which assessment finding in a patient who is admitted with infective endocarditis (IE) is most important to communicate to the health care provider? a. Generalized muscle aching b. Sudden onset right flank pain c. Janeway's lesions on the palms d. Temperature 100.7° F (38.1° C)

ANS: B Sudden onset of flank pain indicates possible embolization to the kidney and may require diagnostic testing such as a renal arteriogram and interventions to improve renal perfusion. The other findings are typically found in IE, but do not require any new interventions.

path of normal cardiac impulse is...

SA node, mycordium (interatrial/internodal paths=atrial contractions), travels to the atrioventricular (AV) node, through the bundle of His, and down the left and right bundle branches. It ends in the Purkinje fibers, which transmit the impulse to the ventricles.

2. Which hemodynamic parameter is most appropriate for the nurse to monitor to determine the effectiveness of medications given to a patient to reduce left ventricular afterload? a. Mean arterial pressure (MAP) b. Systemic vascular resistance (SVR) c. Pulmonary vascular resistance (PVR) d. Pulmonary artery wedge pressure (PAWP)

ANS: B Systemic vascular resistance reflects the resistance to ventricular ejection, or afterload. The other parameters will be monitored, but do not reflect afterload as directly.

A patient with cardiogenic shock has the following vital signs: BP 86/50, pulse 126, respirations 30. The PAWP is increased and cardiac output is low. The nurse will anticipate a. infusion of 5% human albumin. b. administration of furosemide (Lasix) IV. c. titration of an epinephrine (Adrenalin) drip. d. administration of hydrocortisone (SoluCortef).

ANS: B The PAWP indicates that the patient's preload is elevated and furosemide is indicated to reduce the preload and improve cardiac output. Epinephrine would further increase heart rate and myocardial oxygen demand. Normal saline infusion would increase the PAWP further. Hydrocortisone might be used for septic or anaphylactic shock. DIF: Cognitive Level: Application REF: 1735 | 1736 TOP: Nursing Process: Planning MSC: NCLEX: Physiological Integrity

A patient with cardiogenic shock has the following vital signs: BP 102/50, pulse 128, respirations 28. The pulmonary artery wedge pressure (PAWP) is increased and cardiac output is low. The nurse will anticipate an order for which medication? a. 5% human albumin b. Furosemide (Lasix) IV c. Epinephrine (Adrenalin) drip d. Hydrocortisone (Solu-Cortef)

ANS: B The PAWP indicates that the patient's preload is elevated, and furosemide is indicated to reduce the preload and improve cardiac output. Epinephrine would further increase heart rate and myocardial oxygen demand. 5% human albumin would also increase the PAWP. Hydrocortisone might be considered for septic or anaphylactic shock.

A patient with cardiogenic shock has the following vital signs: BP 102/50, pulse 128, respirations 28. The pulmonary artery wedge pressure (PAWP) is increased and cardiac output is low. The nurse will anticipate an order for which medication? a. 5% human albumin b. Furosemide (Lasix) IV c. Epinephrine (Adrenalin) drip d. Hydrocortisone (Solu-Cortef)

ANS: B The PAWP indicates that the patient's preload is elevated, and furosemide is indicated to reduce the preload and improve cardiac output. Epinephrine would further increase heart rate and myocardial oxygen demand. 5% human albumin would also increase the PAWP. Hydrocortisone might be considered for septic or anaphylactic shock.

A changed celled permeability from polarized states allow....

sodium to move rapidly into the cell, making the inside of the cell positive compared with the outside (depolarization).

34. The nurse educator is evaluating the performance of a new registered nurse (RN) who is providing care to a patient who is receiving mechanical ventilation with 15 cm H2O of peak end-expiratory pressure (PEEP). Which action indicates that the new RN is safe? a. The RN plans to suction the patient every 1 to 2 hours. b. The RN uses a closed-suction technique to suction the patient. c. The RN tapes connection between the ventilator tubing and the ET. d. The RN changes the ventilator circuit tubing routinely every 48 hours.

ANS: B The closed-suction technique is used when patients require high levels of PEEP (>10 cm H2O) to prevent the loss of PEEP that occurs when disconnecting the patient from the ventilator. Suctioning should not be scheduled routinely, but it should be done only when patient assessment data indicate the need for suctioning. Taping connections between the ET and the ventilator tubing would restrict the ability of the tubing to swivel in response to patient repositioning. Ventilator tubing changes increase the risk for ventilator-associated pneumonia (VAP) and are not indicated routinely. DIF: Cognitive Level: Apply (application) REF: 1616 OBJ: Special Questions: Delegation TOP: Nursing Process: Implementation MSC: NCLEX: Safe and Effective Care Environment

When developing a community health program to decrease the incidence of rheumatic fever, which action would be most important for the community health nurse to include? a. Vaccinate high-risk groups in the community with streptococcal vaccine. b. Teach community members to seek treatment for streptococcal pharyngitis. c. Teach about the importance of monitoring temperature when sore throats occur. d. Teach about prophylactic antibiotics to those with a family history of rheumatic fever.

ANS: B The incidence of rheumatic fever is decreased by treatment of streptococcal infections with antibiotics. Family history is not a risk factor for rheumatic fever. There is no immunization that is effective in decreasing the incidence of rheumatic fever. Teaching about monitoring temperature will not decrease the incidence of rheumatic fever.

26. When evaluating a patient with a central venous catheter, the nurse observes that the insertion site is red and tender to touch and the patient's temperature is 101.8° F. What should the nurse plan to do next? a. Give analgesics and antibiotics as ordered. b. Discontinue the catheter and culture the tip. c. Change the flush system and monitor the site. d. Check the site more frequently for any swelling.

ANS: B The information indicates that the patient has a local and systemic infection caused by the catheter, and the catheter should be discontinued. Changing the flush system, giving analgesics, and continued monitoring will not help prevent or treat the infection. Administration of antibiotics is appropriate, but the line should still be discontinued to avoid further complications such as endocarditis.

26. When evaluating a patient with a central venous catheter, the nurse observes that the insertion site is red and tender to touch and the patient's temperature is 101.8° F. What should the nurse plan to do next? a. Give analgesics and antibiotics as ordered. b. Discontinue the catheter and culture the tip. c. Change the flush system and monitor the site. d. Check the site more frequently for any swelling.

ANS: B The information indicates that the patient has a local and systemic infection caused by the catheter, and the catheter should be discontinued. Changing the flush system, giving analgesics, and continued monitoring will not help prevent or treat the infection. Administration of antibiotics is appropriate, but the line should still be discontinued to avoid further complications such as endocarditis. DIF: Cognitive Level: Apply (application) REF: 1611 TOP: Nursing Process: Planning MSC: NCLEX: Physiological Integrity

27. An 81-year-old patient who has been in the intensive care unit (ICU) for a week is now stable and transfer to the progressive care unit is planned. On rounds, the nurse notices that the patient has new onset confusion. The nurse will plan to a. give PRN lorazepam (Ativan) and cancel the transfer. b. inform the receiving nurse and then transfer the patient. c. notify the health care provider and postpone the transfer. d. obtain an order for restraints as needed and transfer the patient.

ANS: B The patient's history and symptoms most likely indicate delirium associated with the sleep deprivation and sensory overload in the ICU environment. Informing the receiving nurse and transferring the patient is appropriate. Postponing the transfer is likely to prolong the delirium. Benzodiazepines and restraints contribute to delirium and agitation. DIF: Cognitive Level: Apply (application) REF: 1601 TOP: Nursing Process: Planning MSC: NCLEX: Psychosocial Integrity

A patient with rheumatic fever has subcutaneous nodules, erythema marginatum, and polyarthritis. Based on these findings, which nursing diagnosis would be most appropriate? a. Pain related to permanent joint fixation b. Activity intolerance related to arthralgia c. Risk for infection related to open skin lesions d. Risk for impaired skin integrity related to pruritus

ANS: B The patient's joint pain will lead to difficulty with activity. The skin lesions seen in rheumatic fever are not open or pruritic. Although acute joint pain will be a problem for this patient, joint inflammation is a temporary clinical manifestation of rheumatic fever and is not associated with permanent joint changes.

After receiving change-of-shift report on a heart failure unit, which patient should the nurse assess first? a. Patient who is taking carvedilol (Coreg) and has a heart rate of 58 b. Patient who is taking digoxin and has a potassium level of 3.1 mEq/L c. Patient who is taking isosorbide dinitrate/hydralazine (BiDil) and has a headache d. Patient who is taking captopril (Capoten) and has a frequent nonproductive cough

ANS: B The patient's low potassium level increases the risk for digoxin toxicity and potentially fatal dysrhythmias. The nurse should assess the patient for other signs of digoxin toxicity and then notify the health care provider about the potassium level. The other patients also have side effects of their medications, but their symptoms do not indicate potentially life-threatening complications.

When planning care for a patient hospitalized with a streptococcal infective endocarditis (IE), which intervention is a priority for the nurse to include? a. Monitor labs for streptococcal antibodies. b. Arrange for placement of a long-term IV catheter. c. Teach the importance of completing all oral antibiotics. d. Encourage the patient to begin regular aerobic exercise.

ANS: B Treatment for IE involves 4 to 6 weeks of IV antibiotic therapy in order to eradicate the bacteria, which will require a long-term IV catheter such as a peripherally inserted central catheter (PICC) line. Rest periods and limiting physical activity to a moderate level are recommended during the treatment for IE. Oral antibiotics are not effective in eradicating the infective bacteria that cause IE. Blood cultures, rather than antibody levels, are used to monitor the effectiveness of antibiotic therapy.

After receiving 2 L of normal saline, the central venous pressure for a patient who has septic shock is 10 mm Hg, but the blood pressure is still 82/40 mm Hg. The nurse will anticipate an order for a. nitroglycerine (Tridil). b. norepinephrine (Levophed). c. sodium nitroprusside (Nipride). d. methylprednisolone (Solu-Medrol).

ANS: B When fluid resuscitation is unsuccessful, vasopressor drugs are administered to increase the systemic vascular resistance (SVR) and blood pressure, and improve tissue perfusion. Nitroglycerin would decrease the preload and further drop cardiac output and BP. Methylprednisolone (Solu-Medrol) is considered if blood pressure does not respond first to fluids and vasopressors. Nitroprusside is an arterial vasodilator and would further decrease SVR

After receiving 2 L of normal saline, the central venous pressure for a patient who has septic shock is 10 mm Hg, but the blood pressure is still 82/40 mm Hg. The nurse will anticipate an order for a. nitroglycerine (Tridil). b. norepinephrine (Levophed). c. sodium nitroprusside (Nipride). d. methylprednisolone (Solu-Medrol).

ANS: B When fluid resuscitation is unsuccessful, vasopressor drugs are administered to increase the systemic vascular resistance (SVR) and blood pressure, and improve tissue perfusion. Nitroglycerin would decrease the preload and further drop cardiac output and BP. Methylprednisolone (Solu-Medrol) is considered if blood pressure does not respond first to fluids and vasopressors. Nitroprusside is an arterial vasodilator and would further decrease SVR.

After receiving 2 L of normal saline, the central venous pressure for a patient who has septic shock is 10 mm Hg, but the blood pressure is still 82/40 mm Hg. The nurse will anticipate an order for a. nitroglycerine (Tridil). b. norepinephrine (Levophed). c. sodium nitroprusside (Nipride). d. methylprednisolone (Solu-Medrol).

ANS: B When fluid resuscitation is unsuccessful, vasopressor drugs are administered to increase the systemic vascular resistance (SVR) and blood pressure, and improve tissue perfusion. Nitroglycerin would decrease the preload and further drop cardiac output and BP. Methylprednisolone (Solu-Medrol) is considered if blood pressure does not respond first to fluids and vasopressors. Nitroprusside is an arterial vasodilator and would further decrease SVR.

Which teaching points should you include when providing discharge instructions to a patient with a new permanent pacemaker and to the caregiver (select all that apply)? A. Avoid or limit air travel. B. Take and record a pulse rate daily. C. Obtain and wear a Medic Alert ID or bracelet at all times. D. Avoid lifting the arm on the side of the pacemaker above the shoulder. E. Avoid microwave ovens because they interfere with pacemaker function.

ANS: B,C,D Air travel is not restricted. Inform airport security about the pacemaker because it may set off the metal detector. If a hand-held screening wand is used, it should not be placed directly over the pacemaker. Manufacturer information may vary regarding the effect of metal detectors on the function of the pacemaker. The patient should monitor the pulse and inform the primary care provider if it drops below predetermined rate. A Medic Alert ID or bracelet should be worn at all times. The patient should avoid lifting the arm on the pacemaker side above the shoulder until it is approved by the primary care provider. Microwave ovens are safe to use and do not interfere with pacemaker function. Table 36-13 provides additional discharge teaching guidelines for a patient with a pacemaker. Reference: 837

You are caring for a patient 24 hours after pacemaker insertion. Which nursing intervention is most appropriate at this time? A. Reinforcing the pressure dressing as needed B. Encouraging range-of-motion exercises of the involved arm C. Assessing the incision for any redness, swelling, or discharge D. Applying wet-to-dry dressings every 4 hours to the insertion site

ANS: C After pacemaker insertion, it is important for the patient to limit activity of the involved arm to minimize pacemaker lead displacement. The nonpressure dressing is kept dry until removed, usually 24 hours postoperatively. It is important for you to observe signs of infection by assessing for redness, swelling, or discharge from the incision site. Reference: 836-837

When computing a heart rate from the ECG tracing, you count 15 of the small blocks between the R waves of a patient whose rhythm is regular. From these data, you calculate the patient's heart rate to be what? A. 60 beats/minute B. 75 beats/minute C. 100 beats/minute D. 150 beats/minute

ANS: C Because each small block on the ECG paper represents 0.04 second, 1500 of them represent 1 minute. By dividing the number of small blocks (15 in this case) into 1500, you can calculate the heart rate in a patient whose rhythm is regular (in this case, 100). Reference: 821

Important teaching for the patient scheduled for a radiofrequency catheter ablation procedure includes explaining that A. ventricular bradycardia may be induced and treated during the procedure. B. catheters will be placed in both femoral arteries to allow double-catheter use. C. the procedure will destroy areas of the conduction system that are causing rapid heart rhythms. D. a general anesthetic will be given to prevent the awareness of any "sudden cardiac death" experiences.

ANS: C Radiofrequency catheter ablation therapy uses electrical energy to "burn" or ablate areas of the conduction system as definitive treatment for tachydysrhythmias. Reference: 837

Which statement best describes the electrical activity of the heart represented by measuring the PR interval on the ECG? A. The length of time it takes to depolarize the atrium B. The length of time it takes for the atria to depolarize and repolarize C. The length of time for the electrical impulse to travel from the sinoatrial (SA) node to the Purkinje fibers D. The length of time it takes for the electrical impulse to travel from the SA node to the AV node

ANS: C The electrical impulse in the heart must travel from the SA node through the AV node and into the Purkinje fibers for synchronous atrial and ventricular contraction to occur. The P wave represents atrial contraction, and the R wave is part of the QRS complex that represents ventricular contraction. When measuring the time from the beginning of the P wave to the beginning of the QRS (PR interval), you are identifying the length of time it takes for the electrical impulse to travel from the SA node to the Purkinje fibers. Reference: 818-819

STEMI

ST-elevation myocardial infarction. It's caused by an occlusive thrombus. EMERGENCY SITUATION

The patient has an electrocardiographic (ECG) tracing that is 50 beats/minute, the rhythm is regular, and there is a P wave before every QRS complex. The QRS has a normal shape and duration, and the PR interval is normal. What is you response? A. Administer atropine by intravenous push (IVP). B. Administer epinephrine by IVP. C. Monitor the patient for syncope. D. Attach an external pacemaker.

ANS: C The rhythm described is sinus bradycardia. Treatment depends on the patient's response and whether adequate perfusion is occurring. If the patient tolerates the rhythm, no treatment is given. Reference: 824

A patient admitted with acute coronary syndrome (ACS) has continuous ECG monitoring. An examination of the rhythm strip reveals the following characteristics: atrial rate of 74 beats/minute and regular; ventricular rate of 62 beats/minute and irregular; P wave with a normal shape; PR interval that lengthens progressively until a P wave is not conducted; and QRS complex with a normal shape. Your priority nursing intervention involves A. performing synchronized cardioversion. B. administering 1 mg of epinephrine by IVP. C. observing for symptoms of hypotension or angina. D. preparing the patient for a transcutaneous pacemaker.

ANS: C The rhythm is a second-degree AV block, type I (Mobitz I or Wenckebach heart block). The rhythm is identified by a gradual lengthening of the PR interval. Type I AV block is usually a result of myocardial ischemia or infarction and typically is transient and well tolerated. You should assess for bradycardia, hypotension, and angina. If the patient becomes symptomatic, atropine or a temporary pacemaker may be needed. Reference: 830

You are monitoring the electrocardiogram of a patient admitted with ACS. Which ECG characteristics most suggest ischemia? A. Sinus rhythm with a pathologic Q wave B. Sinus rhythm with an elevated ST segment C. Sinus rhythm with a depressed ST segment D. Sinus rhythm with premature atrial contractions

ANS: C The typical ECG changes seen in myocardial ischemia include ST-segment depression or T-wave inversion, or both. Reference: 837

Which ECG characteristics are consistent with a diagnosis of ventricular tachycardia (VT)? A. Unmeasurable rate and rhythm B. Rate of 150 beats/minute; inverted P wave C. Rate of 200 beats/minute; P wave not visible D. Rate of 125 beats/minute; normal QRS complex

ANS: C VT is associated with a rate of 150 to 250 beats/minute, and the P wave is not normally visible. P-wave inversion and a normal QRS complex are not associated with VT. Rate and rhythm are not measurable in ventricular fibrillation. Reference: 831

13. An intraaortic balloon pump (IABP) is being used for a patient who is in cardiogenic shock. Which assessment data indicate to the nurse that the goals of treatment with the IABP are being met? a. Urine output of 25 mL/hr b. Heart rate of 110 beats/minute c. Cardiac output (CO) of 5 L/min d. Stroke volume (SV) of 40 mL/beat

ANS: C A CO of 5 L/min is normal and indicates that the IABP has been successful in treating the shock. The low SV signifies continued cardiogenic shock. The tachycardia and low urine output also suggest continued cardiogenic shock.

13. An intraaortic balloon pump (IABP) is being used for a patient who is in cardiogenic shock. Which assessment data indicate to the nurse that the goals of treatment with the IABP are being met? a. Urine output of 25 mL/hr b. Heart rate of 110 beats/minute c. Cardiac output (CO) of 5 L/min d. Stroke volume (SV) of 40 mL/beat

ANS: C A CO of 5 L/min is normal and indicates that the IABP has been successful in treating the shock. The low SV signifies continued cardiogenic shock. The tachycardia and low urine output also suggest continued cardiogenic shock. DIF: Cognitive Level: Apply (application) REF: 1603 TOP: Nursing Process: Evaluation MSC: NCLEX: Physiological Integrity

13. An intraaortic balloon pump (IABP) is being used for a patient who is in cardiogenic shock. Which assessment data indicate to the nurse that the goals of treatment with the IABP are being met? a. Urine output of 25 mL/hr b. Heart rate of 110 beats/minute c. Cardiac output (CO) of 5 L/min d. Stroke volume (SV) of 40 mL/beat

ANS: C A CO of 5 L/min is normal and indicates that the IABP has been successful in treating the shock. The low SV signifies continued cardiogenic shock. The tachycardia and low urine output also suggest continued cardiogenic shock. DIF: Cognitive Level: Apply (application) REF: 1603 TOP: Nursing Process: Evaluation MSC: NCLEX: Physiological Integrity.

The emergency department (ED) receives notification that a patient who has just been in an automobile accident is being transported to your facility with anticipated arrival in 1 minute. In preparation for the patient's arrival, the nurse will obtain a. 500 mL of 5% albumin. b. lactated Ringer's solution. c. two 14-gauge IV catheters. d. dopamine (Intropin) infusion.

ANS: C A patient with multiple trauma may require fluid resuscitation to prevent or treat hypovolemic shock, so the nurse will anticipate the need for 2 large bore IV lines to administer normal saline. Lactated Ringer's solution should be used cautiously and will not be ordered until the patient has been assessed for possible liver abnormalities. Although colloids may sometimes be used for volume expansion, crystalloids should be used as the initial therapy for fluid resuscitation. Vasopressor infusion is not used as the initial therapy for hypovolemic shock. DIF: Cognitive Level: Application REF: 1731 | 1732 | 1733 TOP: Nursing Process: Planning MSC: NCLEX: Physiological Integrity

The emergency department (ED) nurse receives report that a patient involved in a motor vehicle crash is being transported to the facility with an estimated arrival in 1 minute. In preparation for the patient's arrival, the nurse will obtain a. hypothermia blanket. b. lactated Ringer's solution. c. two 14-gauge IV catheters. d. dopamine (Intropin) infusion.

ANS: C A patient with multiple trauma may require fluid resuscitation to prevent or treat hypovolemic shock, so the nurse will anticipate the need for 2 large bore IV lines to administer normal saline. Lactated Ringer's solution should be used cautiously and will not be ordered until the patient has been assessed for possible liver abnormalities. Vasopressor infusion is not used as the initial therapy for hypovolemic shock. Patients in shock need to be kept warm not cool.

The emergency department (ED) nurse receives report that a patient involved in a motor vehicle crash is being transported to the facility with an estimated arrival in 1 minute. In preparation for the patient's arrival, the nurse will obtain a. hypothermia blanket. b. lactated Ringer's solution. c. two 14-gauge IV catheters. d. dopamine (Intropin) infusion.

ANS: C A patient with multiple trauma may require fluid resuscitation to prevent or treat hypovolemic shock, so the nurse will anticipate the need for 2 large bore IV lines to administer normal saline. Lactated Ringer's solution should be used cautiously and will not be ordered until the patient has been assessed for possible liver abnormalities. Vasopressor infusion is not used as the initial therapy for hypovolemic shock. Patients in shock need to be kept warm not cool.

Which client problem has priority for the client with a cardiac dysrhythmia? 2. Decreased cardiac output.

Any abnormal electrical activity of the heart causes decreased cardiac output.

The emergency department (ED) nurse receives report that a patient involved in a motor vehicle crash is being transported to the facility with an estimated arrival in 1 minute. In preparation for the patient's arrival, the nurse will obtain a. hypothermia blanket. b. lactated Ringer's solution. c. two 14-gauge IV catheters. d. dopamine (Intropin) infusion.

ANS: C A patient with multiple trauma may require fluid resuscitation to prevent or treat hypovolemic shock, so the nurse will anticipate the need for 2 large bore IV lines to administer normal saline. Lactated Ringer's solution should be used cautiously and will not be ordered until the patient has been assessed for possible liver abnormalities. Vasopressor infusion is not used as the initial therapy for hypovolemic shock. Patients in shock need to be kept warm not cool.

A patient who is receiving dobutamine (Dobutrex) for the treatment of acute decompensated heart failure (ADHF) has the following nursing interventions included in the plan of care. Which action will be most appropriate for the registered nurse (RN) to delegate to an experienced licensed practical/vocational nurse (LPN/LVN)? a. Assess the IV insertion site for signs of extravasation. b. Teach the patient the reasons for remaining on bed rest. c. Monitor the patient's blood pressure and heart rate every hour. d. Titrate the rate to keep the systolic blood pressure >90 mm Hg.

ANS: C An experienced LPN/LVN would be able to monitor BP and heart rate and would know to report significant changes to the RN. Teaching patients, making adjustments to the drip rate for vasoactive medications, and monitoring for serious complications such as extravasation require RN level education and scope of practice.

During discharge teaching with a 68-year-old patient who had a mitral valve replacement with a mechanical valve, the nurse instructs the patient on the a. use of daily aspirin for anticoagulation. b. correct method for taking the radial pulse. c. need for frequent laboratory blood testing. d. need to avoid any physical activity for 1 month.

ANS: C Anticoagulation with warfarin (Coumadin) is needed for a patient with mechanical valves to prevent clotting on the valve. This will require frequent international normalized ratio (INR) testing. Daily aspirin use will not be effective in reducing the risk for clots on the valve. Monitoring of the radial pulse is not necessary after valve replacement. Patients should resume activities of daily living as tolerated.

Which diagnostic test will be most useful to the nurse in determining whether a patient admitted with acute shortness of breath has heart failure? a. Serum troponin b. Arterial blood gases c. B-type natriuretic peptide d. 12-lead electrocardiogram

ANS: C B-type natriuretic peptide (BNP) is secreted when ventricular pressures increase, as they do with heart failure. Elevated BNP indicates a probable or very probable diagnosis of heart failure. A twelve-lead electrocardiogram, arterial blood gases, and troponin may also be used in determining the causes or effects of heart failure but are not as clearly diagnostic of heart failure as BNP

Which action by the nurse will determine if the therapies ordered for a patient with chronic constrictive pericarditis are effective? a. Assess for the presence of a paradoxical pulse. b. Monitor for changes in the patient's sedimentation rate. c. Assess for the presence of jugular venous distention (JVD). d. Check the electrocardiogram (ECG) for ST segment changes.

ANS: C Because the most common finding on physical examination for a patient with chronic constrictive pericarditis is jugular venous distention, a decrease in JVD indicates improvement. Paradoxical pulse, ST-segment ECG changes, and changes in sedimentation rates occur with acute pericarditis but are not expected in chronic constrictive pericarditis.

20. The nurse notes thick, white secretions in the endotracheal tube (ET) of a patient who is receiving mechanical ventilation. Which intervention will be most effective in addressing this problem? a. Increase suctioning to every hour. b. Reposition the patient every 1 to 2 hours. c. Add additional water to the patient's enteral feedings. d. Instill 5 mL of sterile saline into the ET before suctioning.

ANS: C Because the patient's secretions are thick, better hydration is indicated. Suctioning every hour without any specific evidence for the need will increase the incidence of mucosal trauma and would not address the etiology of the ineffective airway clearance. Instillation of saline does not liquefy secretions and may decrease the SpO2. Repositioning the patient is appropriate but will not decrease the thickness of secretions. DIF: Cognitive Level: Apply (application) REF: 1617 TOP: Nursing Process: Implementation MSC: NCLEX: Physiological Integrity

Which assessment data would require immediate intervention by the nurse for the client who is six (6) hours post-operative abdominal aortic aneurysm repair? 1. Absent bilateral pedal pulses.

Any neurovascular abnormality in the client's lower extremities indicates the graft is occluded or possibly bleeding and requires immediate intervention by the nurse.

Which finding about a patient who is receiving vasopressin (Pitressin) to treat septic shock is most important for the nurse to communicate to the health care provider? a. The patient's urine output is 18 mL/hr. b. The patient's heart rate is 110 beats/minute. c. The patient is complaining of chest pain. d. The patient's peripheral pulses are weak.

ANS: C Because vasopressin is a potent vasoconstrictor, it may decrease coronary artery perfusion. The other information is consistent with the patient's diagnosis and should be reported to the health care provider but does not indicate a need for a change in therapy

Which finding about a patient who is receiving vasopressin (Pitressin) to treat septic shock is most important for the nurse to communicate to the health care provider? a. The patient's urine output is 18 mL/hr. b. The patient's heart rate is 110 beats/minute. c. The patient is complaining of chest pain. d. The patient's peripheral pulses are weak.

ANS: C Because vasopressin is a potent vasoconstrictor, it may decrease coronary artery perfusion. The other information is consistent with the patient's diagnosis and should be reported to the health care provider but does not indicate a need for a change in therapy.

A patient with heart failure has a new order for captopril (Capoten) 12.5 mg PO. After administering the first dose and teaching the patient about the drug, which statement by the patient indicates that teaching has been effective? a. "I will be sure to take the medication with food." b. "I will need to eat more potassium-rich foods in my diet." c. "I will call for help when I need to get up to use the bathroom." d. "I will expect to feel more short of breath for the next few days."

ANS: C Captopril can cause hypotension, especially after the initial dose, so it is important that the patient not get up out of bed without assistance until the nurse has had a chance to evaluate the effect of the first dose. The angiotensin-converting enzyme (ACE) inhibitors are potassium sparing, and the nurse should not teach the patient to purposely increase sources of dietary potassium. Increased shortness of breath is expected with the initiation of -adrenergic blocker therapy for heart failure, not for ACE inhibitor therapy. ACE inhibitors are best absorbed when taken an hour before eating.

The nurse will plan discharge teaching about the need for prophylactic antibiotics when having dental procedures for which patient? a. Patient admitted with a large acute myocardial infarction. b. Patient being discharged after an exacerbation of heart failure. c. Patient who had a mitral valve replacement with a mechanical valve. d. Patient being treated for rheumatic fever after a streptococcal infection.

ANS: C Current American Heart Association guidelines recommend the use of prophylactic antibiotics before dental procedures for patients with prosthetic valves to prevent infective endocarditis (IE). The other patients are not at risk for IE.

18. The nurse notes premature ventricular contractions (PVCs) while suctioning a patient's endotracheal tube. Which action by the nurse is a priority? a. Decrease the suction pressure to 80 mm Hg. b. Document the dysrhythmia in the patient's chart. c. Stop and ventilate the patient with 100% oxygen. d. Give antidysrhythmic medications per protocol.

ANS: C Dysrhythmias during suctioning may indicate hypoxemia or sympathetic nervous system stimulation. The nurse should stop suctioning and ventilate the patient with 100% oxygen. Lowering the suction pressure will decrease the effectiveness of suctioning without improving the hypoxemia. Because the PVCs occurred during suctioning, there is no need for antidysrhythmic medications (which may have adverse effects) unless they recur when the suctioning is stopped and patient is well oxygenated. DIF: Cognitive Level: Apply (application) REF: 1616 OBJ: Special Questions: Prioritization TOP: Nursing Process: Implementation MSC: NCLEX: Physiological Integrity

A patient has recently started on digoxin (Lanoxin) in addition to furosemide (Lasix) and captopril (Capoten) for the management of heart failure. Which assessment finding by the home health nurse is a priority to communicate to the health care provider? a. Presence of 1 to 2+ edema in the feet and ankles b. Palpable liver edge 2 cm below the ribs on the right side c. Serum potassium level 3.0 mEq/L after 1 week of therapy d. Weight increase from 120 pounds to 122 pounds over 3 days

ANS: C Hypokalemia can predispose the patient to life-threatening dysrhythmias (e.g., premature ventricular contractions), and potentiate the actions of digoxin and increase the risk for digoxin toxicity, which can also cause life-threatening dysrhythmias. The other data indicate that the patient's heart failure requires more effective therapies, but they do not require nursing action as rapidly as the low serum potassium level.

28. The family members of a patient who has just been admitted to the intensive care unit (ICU) with multiple traumatic injuries have just arrived in the ICU waiting room. Which action should the nurse take next? a. Explain ICU visitation policies and encourage family visits. b. Immediately take the family members to the patient's bedside. c. Describe the patient's injuries and the care that is being provided. d. Invite the family to participate in a multidisciplinary care conference.

ANS: C Lack of information is a major source of anxiety for family members and should be addressed first. Family members should be prepared for the patient's appearance and the ICU environment before visiting the patient for the first time. ICU visiting should be individualized to each patient and family rather than being dictated by rigid visitation policies. Inviting the family to participate in a multidisciplinary conference is appropriate but should not be the initial action by the nurse.

28. The family members of a patient who has just been admitted to the intensive care unit (ICU) with multiple traumatic injuries have just arrived in the ICU waiting room. Which action should the nurse take next? a. Explain ICU visitation policies and encourage family visits. b. Immediately take the family members to the patient's bedside. c. Describe the patient's injuries and the care that is being provided. d. Invite the family to participate in a multidisciplinary care conference.

ANS: C Lack of information is a major source of anxiety for family members and should be addressed first. Family members should be prepared for the patient's appearance and the ICU environment before visiting the patient for the first time. ICU visiting should be individualized to each patient and family rather than being dictated by rigid visitation policies. Inviting the family to participate in a multidisciplinary conference is appropriate but should not be the initial action by the nurse. DIF: Cognitive Level: Apply (application) REF: 1602 OBJ: Special Questions: Prioritization TOP: Nursing Process: Implementation MSC: NCLEX: Psychosocial Integrity

A nurse is assessing a patient who is receiving a nitroprusside (Nipride) infusion to treat cardiogenic shock. Which finding indicates that the medication is effective? a. No new heart murmurs b. Decreased troponin level c. Warm, pink, and dry skin d. Blood pressure 92/40 mm Hg

ANS: C Warm, pink, and dry skin indicates that perfusion to tissues is improved. Since nitroprusside is a vasodilator, the blood pressure may be low even if the medication is effective. Absence of a heart murmur and a decrease in troponin level are not indicators of improvement in shock.

A 21-year-old woman is scheduled for percutaneous transluminal balloon valvuloplasty to treat mitral stenosis. Which information should the nurse include when explaining the advantages of valvuloplasty over valve replacement to the patient? a. Biologic valves will require immunosuppressive drugs after surgery. b. Mechanical mitral valves need to be replaced sooner than biologic valves. c. Lifelong anticoagulant therapy will be needed after mechanical valve replacement. d. Ongoing cardiac care by a health care provider is not necessary after valvuloplasty.

ANS: C Long-term anticoagulation therapy is needed after mechanical valve replacement, and this would restrict decisions about career and childbearing in this patient. Mechanical valves are durable and last longer than biologic valves. All valve repair procedures are palliative, not curative, and require lifelong health care. Biologic valves do not activate the immune system, and immunosuppressive therapy is not needed.

14. The nurse is caring for a patient who has an intraaortic balloon pump in place. Which action should be included in the plan of care? a. Position the patient supine at all times. b. Avoid the use of anticoagulant medications. c. Measure the patient's urinary output every hour. d. Provide passive range of motion for all extremities

ANS: C Monitoring urine output will help determine whether the patient's cardiac output has improved and also help monitor for balloon displacement. The head of the bed can be elevated up to 30 degrees. Heparin is used to prevent thrombus formation. Limited movement is allowed for the extremity with the balloon insertion site to prevent displacement of the balloon. DIF: Cognitive Level: Apply (application) REF: 1613 TOP: Nursing Process: Planning MSC: NCLEX: Physiological Integrity

14. The nurse is caring for a patient who has an intraaortic balloon pump in place. Which action should be included in the plan of care? a. Position the patient supine at all times. b. Avoid the use of anticoagulant medications. c. Measure the patient's urinary output every hour. d. Provide passive range of motion for all extremities.

ANS: C Monitoring urine output will help determine whether the patient's cardiac output has improved and also help monitor for balloon displacement. The head of the bed can be elevated up to 30 degrees. Heparin is used to prevent thrombus formation. Limited movement is allowed for the extremity with the balloon insertion site to prevent displacement of the balloon. DIF: Cognitive Level: Apply (application) REF: 1613 TOP: Nursing Process: Planning MSC: NCLEX: Physiological Integrity

The nurse plans discharge teaching for a patient with chronic heart failure who has prescriptions for digoxin (Lanoxin) and hydrochlorothiazide (HydroDIURIL). Appropriate instructions for the patient include a. limit dietary sources of potassium. b. take the hydrochlorothiazide before bedtime. c. notify the health care provider if nausea develops. d. skip the digoxin if the pulse is below 60 beats/minute.

ANS: C Nausea is an indication of digoxin toxicity and should be reported so that the provider can assess the patient for toxicity and adjust the digoxin dose, if necessary. The patient will need to include potassium-containing foods in the diet to avoid hypokalemia. Patients should be taught to check their pulse daily before taking the digoxin and if the pulse is less than 60, to call their provider before taking the digoxin. Diuretics should be taken early in the day to avoid sleep disruption.

A patient with massive trauma and possible spinal cord injury is admitted to the emergency department (ED). Which finding by the nurse will help confirm a diagnosis of neurogenic shock? a. Cool, clammy skin b. Inspiratory crackles c. Apical heart rate 48 beats/min d. Temperature 101.2° F (38.4° C)

ANS: C Neurogenic shock is characterized by hypotension and bradycardia. The other findings would be more consistent with other types of shock. DIF: Cognitive Level: Comprehension REF: 1721-1722 | 1723 TOP: Nursing Process: Assessment MSC: NCLEX: Physiological Integrity

A 19-year-old patient with massive trauma and possible spinal cord injury is admitted to the emergency department (ED). Which assessment finding by the nurse will help confirm a diagnosis of neurogenic shock? a. Inspiratory crackles. b. Cool, clammy extremities. c. Apical heart rate 45 beats/min. d. Temperature 101.2° F (38.4° C).

ANS: C Neurogenic shock is characterized by hypotension and bradycardia. The other findings would be more consistent with other types of shock

A 19-year-old patient with massive trauma and possible spinal cord injury is admitted to the emergency department (ED). Which assessment finding by the nurse will help confirm a diagnosis of neurogenic shock? a. Inspiratory crackles. b. Cool, clammy extremities. c. Apical heart rate 45 beats/min. d. Temperature 101.2° F (38.4° C).

ANS: C Neurogenic shock is characterized by hypotension and bradycardia. The other findings would be more consistent with other types of shock.

A 19-year-old patient with massive trauma and possible spinal cord injury is admitted to the emergency department (ED). Which assessment finding by the nurse will help confirm a diagnosis of neurogenic shock? a. Inspiratory crackles. b. Cool, clammy extremities. c. Apical heart rate 45 beats/min. d. Temperature 101.2° F (38.4° C).

ANS: C Neurogenic shock is characterized by hypotension and bradycardia. The other findings would be more consistent with other types of shock.

An older patient with cardiogenic shock is cool and clammy and hemodynamic monitoring indicates a high systemic vascular resistance (SVR). Which intervention should the nurse anticipate doing next? a. Increase the rate for the dopamine (Intropin) infusion. b. Decrease the rate for the nitroglycerin (Tridil) infusion. c. Increase the rate for the sodium nitroprusside (Nipride) infusion. d. Decrease the rate for the 5% dextrose in normal saline (D5/.9 NS) infusion.

ANS: C Nitroprusside is an arterial vasodilator and will decrease the SVR and afterload, which will improve cardiac output. Changes in the D5/.9 NS and nitroglycerin infusions will not directly decrease SVR. Increasing the dopamine will tend to increase SVR.

An older patient with cardiogenic shock is cool and clammy and hemodynamic monitoring indicates a high systemic vascular resistance (SVR). Which intervention should the nurse anticipate doing next? a. Increase the rate for the dopamine (Intropin) infusion. b. Decrease the rate for the nitroglycerin (Tridil) infusion. c. Increase the rate for the sodium nitroprusside (Nipride) infusion. d. Decrease the rate for the 5% dextrose in normal saline (D5/.9 NS) infusion.

ANS: C Nitroprusside is an arterial vasodilator and will decrease the SVR and afterload, which will improve cardiac output. Changes in the D5/.9 NS and nitroglycerin infusions will not directly decrease SVR. Increasing the dopamine will tend to increase SVR.

An older patient with cardiogenic shock is cool and clammy and hemodynamic monitoring indicates a high systemic vascular resistance (SVR). Which intervention should the nurse anticipate doing next? a. Increase the rate for the dopamine (Intropin) infusion. b. Decrease the rate for the nitroglycerin (Tridil) infusion. c. Increase the rate for the sodium nitroprusside (Nipride) infusion. d. Decrease the rate for the 5% dextrose in normal saline (D5/.9 NS) infusion.

ANS: C Nitroprusside is an arterial vasodilator and will decrease the SVR and afterload, which will improve cardiac output. Changes in the D5/.9 NS and nitroglycerin infusions will not directly decrease SVR. Increasing the dopamine will tend to increase SVR.

5. When caring for a patient with pulmonary hypertension, which parameter is most appropriate for the nurse to monitor to evaluate the effectiveness of the treatment? a. Central venous pressure (CVP) b. Systemic vascular resistance (SVR) c. Pulmonary vascular resistance (PVR) d. Pulmonary artery wedge pressure (PAWP)

ANS: C PVR is a major contributor to pulmonary hypertension, and a decrease would indicate that pulmonary hypertension was improving. The other parameters also may be monitored but do not directly assess for pulmonary hypertension. DIF: Cognitive Level: Apply (application) REF: 1603-1604 TOP: Nursing Process: Evaluation MSC: NCLEX: Physiological Integrity

Which statement by a patient with restrictive cardiomyopathy indicates that the nurse's discharge teaching about self-management has been most effective? a. "I will avoid taking aspirin or other antiinflammatory drugs." b. "I will need to limit my intake of salt and fluids even in hot weather." c. "I will take antibiotics when my teeth are cleaned at the dental office." d. "I should begin an exercise program that includes things like biking or swimming."

ANS: C Patients with restrictive cardiomyopathy are at risk for infective endocarditis and should use prophylactic antibiotics for any procedure that may cause bacteremia. The other statements indicate a need for more teaching by the nurse. Dehydration and vigorous exercise impair ventricular filling in patients with restrictive cardiomyopathy. There is no need to avoid salt (unless ordered), aspirin, or NSAIDs.

To evaluate the effectiveness of the omeprazole (Prilosec) being administered to a patient with systemic inflammatory response syndrome (SIRS), which assessment will the nurse make? a. Auscultate bowel sounds. b. Ask the patient about nausea. c. Monitor stools for occult blood. d. Check for abdominal distention.

ANS: C Proton pump inhibitors are given to decrease the risk for stress ulcers in critically ill patients. The other assessments also will be done, but these will not help in determining the effectiveness of the omeprazole administration. DIF: Cognitive Level: Application REF: 1735-1737 | 1742-1743 TOP: Nursing Process: Evaluation MSC: NCLEX: Physiological Integrity

IV sodium nitroprusside (Nipride) is ordered for a patient with acute pulmonary edema. During the first hours of administration, the nurse will need to titrate the nitroprusside rate if the patient develops a. ventricular ectopy. b. a dry, hacking cough. c. a systolic BP <90 mm Hg. d. a heart rate <50 beats/minute.

ANS: C Sodium nitroprusside is a potent vasodilator, and the major adverse effect is severe hypotension. Coughing and bradycardia are not adverse effects of this medication. Nitroprusside does not cause increased ventricular ectopy.

23. A nurse is weaning a 68-kg male patient who has chronic obstructive pulmonary disease (COPD) from mechanical ventilation. Which patient assessment finding indicates that the weaning protocol should be stopped? a. The patient's heart rate is 97 beats/min. b. The patient's oxygen saturation is 93%. c. The patient respiratory rate is 32 breaths/min. d. The patient's spontaneous tidal volume is 450 mL.

ANS: C Tachypnea is a sign that the patient's work of breathing is too high to allow weaning to proceed. The patient's heart rate is within normal limits, although the nurse should continue to monitor it. An oxygen saturation of 93% is acceptable for a patient with COPD. A spontaneous tidal volume of 450 mL is within the acceptable range. DIF: Cognitive Level: Apply (application) REF: 1627 TOP: Nursing Process: Evaluation MSC: NCLEX: Physiological Integrity

The nurse working on the heart failure unit knows that teaching an older female patient with newly diagnosed heart failure is effective when the patient states that a. she will take furosemide (Lasix) every day at bedtime. b. the nitroglycerin patch is applied when any chest pain develops. c. she will call the clinic if her weight goes from 124 to 128 pounds in a week. d. an additional pillow can help her sleep if she is feeling short of breath at night.

ANS: C Teaching for a patient with heart failure includes information about the need to weigh daily and notify the health care provider about an increase of 3 pounds in 2 days or 3 to 5 pounds in a week. Nitroglycerin patches are used primarily to reduce preload (not to prevent chest pain) in patients with heart failure and should be used daily, not on an "as needed" basis. Diuretics should be taken earlier in the day to avoid nocturia and sleep disturbance. The patient should call the clinic if increased orthopnea develops, rather than just compensating by further elevating the head of the bed.

A patient with chronic heart failure who is taking a diuretic and an angiotensin-converting enzyme (ACE) inhibitor and who is on a low-sodium diet tells the home health nurse about a 5-pound weight gain in the last 3 days. The nurse's priority action will be to a. have the patient recall the dietary intake for the last 3 days. b. ask the patient about the use of the prescribed medications. c. assess the patient for clinical manifestations of acute heart failure. d. teach the patient about the importance of restricting dietary sodium.

ANS: C The 5-pound weight gain over 3 days indicates that the patient's chronic heart failure may be worsening. It is important that the patient be assessed immediately for other clinical manifestations of decompensation, such as lung crackles. A dietary recall to detect hidden sodium in the diet, reinforcement of sodium restrictions, and assessment of medication compliance may be appropriate interventions but are not the first nursing actions indicated.

Which assessment finding obtained by the nurse when assessing a patient with acute pericarditis should be reported immediately to the health care provider? a. Pulsus paradoxus 8 mm Hg b. Blood pressure (BP) of 168/94 c. Jugular venous distention (JVD) to jaw level d. Level 6 (0 to 10 scale) chest pain with a deep breath

ANS: C The JVD indicates that the patient may have developed cardiac tamponade and may need rapid intervention to maintain adequate cardiac output. Hypertension would not be associated with complications of pericarditis, and the BP is not high enough to indicate that there is any immediate need to call the health care provider. A pulsus paradoxus of 8 mm Hg is normal. Level 6/10 chest pain should be treated but is not unusual with pericarditis.

The patient with neurogenic shock is receiving a phenylephrine (Neo-Synephrine) infusion through a right forearm IV. Which assessment finding obtained by the nurse indicates a need for immediate action? a. The patient's heart rate is 58 beats/minute. b. The patient's extremities are warm and dry. c. The patient's IV infusion site is cool and pale. d. The patient's urine output is 28 mL over the last hour.

ANS: C The coldness and pallor at the infusion site suggest extravasation of the phenylephrine. The nurse should discontinue the IV and, if possible, infuse the medication into a central line. An apical pulse of 58 is typical for neurogenic shock but does not indicate an immediate need for nursing intervention. A 28-mL urinary output over 1 hour would require the nurse to monitor the output over the next hour, but an immediate change in therapy is not indicated. Warm, dry skin is consistent with early neurogenic shock, but it does not indicate a need for a change in therapy or immediate action

The patient with neurogenic shock is receiving a phenylephrine (Neo-Synephrine) infusion through a right forearm IV. Which assessment finding obtained by the nurse indicates a need for immediate action? a. The patient's heart rate is 58 beats/minute. b. The patient's extremities are warm and dry. c. The patient's IV infusion site is cool and pale. d. The patient's urine output is 28 mL over the last hour.

ANS: C The coldness and pallor at the infusion site suggest extravasation of the phenylephrine. The nurse should discontinue the IV and, if possible, infuse the medication into a central line. An apical pulse of 58 is typical for neurogenic shock but does not indicate an immediate need for nursing intervention. A 28-mL urinary output over 1 hour would require the nurse to monitor the output over the next hour, but an immediate change in therapy is not indicated. Warm, dry skin is consistent with early neurogenic shock, but it does not indicate a need for a change in therapy or immediate action.

The patient with neurogenic shock is receiving a phenylephrine (Neo-Synephrine) infusion through a right forearm IV. Which assessment finding obtained by the nurse indicates a need for immediate action? a. The patient's heart rate is 58 beats/minute. b. The patient's extremities are warm and dry. c. The patient's IV infusion site is cool and pale. d. The patient's urine output is 28 mL over the last hour.

ANS: C The coldness and pallor at the infusion site suggest extravasation of the phenylephrine. The nurse should discontinue the IV and, if possible, infuse the medication into a central line. An apical pulse of 58 is typical for neurogenic shock but does not indicate an immediate need for nursing intervention. A 28-mL urinary output over 1 hour would require the nurse to monitor the output over the next hour, but an immediate change in therapy is not indicated. Warm, dry skin is consistent with early neurogenic shock, but it does not indicate a need for a change in therapy or immediate action.

Which topic will the nurse plan to include in discharge teaching for a patient with systolic heart failure and an ejection fraction of 33%? a. Need to begin an aerobic exercise program several times weekly b. Use of salt substitutes to replace table salt when cooking and at the table c. Benefits and side effects of angiotensin-converting enzyme (ACE) inhibitors d. Importance of making an annual appointment with the primary care provider

ANS: C The core measures for the treatment of heart failure established by The Joint Commission indicate that patients with an ejection fraction (EF) <40% receive an ACE inhibitor to decrease the progression of heart failure. Aerobic exercise may not be appropriate for a patient with this level of heart failure, salt substitutes are not usually recommended because of the risk of hyperkalemia, and the patient will need to see the primary care provider more frequently than annually

While admitting an 82-year-old with acute decompensated heart failure to the hospital, the nurse learns that the patient lives alone and sometimes confuses the "water pill" with the "heart pill." When planning for the patient's discharge the nurse will facilitate a a. consult with a psychologist. b. transfer to a long-term care facility. c. referral to a home health care agency. d. arrangements for around-the-clock care.

ANS: C The data about the patient suggest that assistance in developing a system for taking medications correctly at home is needed. A home health nurse will assess the patient's home situation and help the patient develop a method for taking the two medications as directed. There is no evidence that the patient requires services such as a psychologist consult, long-term care, or around-the-clock home care.

A patient who has been involved in a motor vehicle crash arrives in the emergency department (ED) with cool, clammy skin; tachycardia; and hypotension. Which intervention ordered by the health care provider should the nurse implement first? a. Insert two large-bore IV catheters. b. Initiate continuous electrocardiogram (ECG) monitoring. c. Provide oxygen at 100% per non-rebreather mask. d. Draw blood to type and crossmatch for transfusions.

ANS: C The first priority in the initial management of shock is maintenance of the airway and ventilation. ECG monitoring, insertion of IV catheters, and obtaining blood for transfusions should also be rapidly accomplished but only after actions to maximize oxygen delivery have been implemented.

A patient who has been involved in a motor vehicle crash arrives in the emergency department (ED) with cool, clammy skin; tachycardia; and hypotension. Which intervention ordered by the health care provider should the nurse implement first? a. Insert two large-bore IV catheters. b. Initiate continuous electrocardiogram (ECG) monitoring. c. Provide oxygen at 100% per non-rebreather mask. d. Draw blood to type and crossmatch for transfusions.

ANS: C The first priority in the initial management of shock is maintenance of the airway and ventilation. ECG monitoring, insertion of IV catheters, and obtaining blood for transfusions should also be rapidly accomplished but only after actions to maximize oxygen delivery have been implemented.

A patient who has been involved in a motor vehicle crash arrives in the emergency department (ED) with cool, clammy skin; tachycardia; and hypotension. Which intervention ordered by the health care provider should the nurse implement first? a. Insert two large-bore IV catheters. b. Initiate continuous electrocardiogram (ECG) monitoring. c. Provide oxygen at 100% per non-rebreather mask. d. Draw blood to type and crossmatch for transfusions.

ANS: C The first priority in the initial management of shock is maintenance of the airway and ventilation. ECG monitoring, insertion of IV catheters, and obtaining blood for transfusions should also be rapidly accomplished but only after actions to maximize oxygen delivery have been implemented.

19. Which assessment finding obtained by the nurse when caring for a patient receiving mechanical ventilation indicates the need for suctioning? a. The patient's oxygen saturation is 93%. b. The patient was last suctioned 6 hours ago. c. The patient's respiratory rate is 32 breaths/minute. d. The patient has occasional audible expiratory wheezes.

ANS: C The increase in respiratory rate indicates that the patient may have decreased airway clearance and requires suctioning. Suctioning is done when patient assessment data indicate that it is needed, not on a scheduled basis. Occasional expiratory wheezes do not indicate poor airway clearance, and suctioning the patient may induce bronchospasm and increase wheezing. An oxygen saturation of 93% is acceptable and does not suggest that immediate suctioning is needed. DIF: Cognitive Level: Apply (application) REF: 1616 TOP: Nursing Process: Assessment MSC: NCLEX: Physiological Integrity

While assessing a 68-year-old with ascites, the nurse also notes jugular venous distention (JVD) with the head of the patient's bed elevated 45 degrees. The nurse knows this finding indicates a. decreased fluid volume. b. jugular vein atherosclerosis. c. increased right atrial pressure. d. incompetent jugular vein valves.

ANS: C The jugular veins empty into the superior vena cava and then into the right atrium, so JVD with the patient sitting at a 45-degree angle reflects increased right atrial pressure. JVD is an indicator of excessive fluid volume (increased preload), not decreased fluid volume. JVD is not caused by incompetent jugular vein valves or atherosclerosis.

8. Which action is a priority for the nurse to take when the low pressure alarm sounds for a patient who has an arterial line in the left radial artery? a. Fast flush the arterial line. b. Check the left hand for pallor. c. Assess for cardiac dysrhythmias. d. Rezero the monitoring equipment.

ANS: C The low pressure alarm indicates a drop in the patient's blood pressure, which may be caused by cardiac dysrhythmias. There is no indication to rezero the equipment. Pallor of the left hand would be caused by occlusion of the radial artery by the arterial catheter, not by low pressure. There is no indication of a need for flushing the line. DIF: Cognitive Level: Apply (application) REF: 1606 OBJ: Special Questions: Prioritization TOP: Nursing Process: Implementation MSC: NCLEX: Physiological Integrity

A patient who has just been admitted with pulmonary edema is scheduled to receive the following medications. Which medication should the nurse question before giving? d. Carvedilol (Coreg) 3.125 mg

Although carvedilol is appropriate for the treatment of chronic heart failure, it is not used for patients with acute decompensated heart failure (ADHF) because of the risk of worsening the heart failure.

31. The nurse notes that a patient's endotracheal tube (ET), which was at the 22-cm mark, is now at the 25-cm mark and the patient is anxious and restless. Which action should the nurse take next? a. Offer reassurance to the patient. b. Bag the patient at an FIO2 of 100%. c. Listen to the patient's breath sounds. d. Notify the patient's health care provider.

ANS: C The nurse should first determine whether the ET tube has been displaced into the right mainstem bronchus by listening for unilateral breath sounds. If so, assistance will be needed to reposition the tube immediately. The other actions are also appropriate, but detection and correction of tube malposition are the most critical actions. DIF: Cognitive Level: Apply (application) REF: 1614 OBJ: Special Questions: Prioritization TOP: Nursing Process: Implementation MSC: NCLEX: Physiological Integrity

A patient recovering from heart surgery develops pericarditis and complains of level 6 (0 to 10 scale) chest pain with deep breathing. Which ordered PRN medication will be the most appropriate for the nurse to give? a. Fentanyl 1 mg IV b. IV morphine sulfate 4 mg c. Oral ibuprofen (Motrin) 600 mg d. Oral acetaminophen (Tylenol) 650 mg

ANS: C The pain associated with pericarditis is caused by inflammation, so nonsteroidal antiinflammatory drugs (NSAIDs) (e.g., ibuprofen) are most effective. Opioid analgesics are usually not used for the pain associated with pericarditis.

An outpatient who has chronic heart failure returns to the clinic after 2 weeks of therapy with metoprolol (Toprol XL). Which assessment finding is most important for the nurse to report to the health care provider? a. 2+ pedal edema b. Heart rate of 56 beats/minute c. Blood pressure (BP) of 88/42 mm Hg d. Complaints of fatigue

ANS: C The patient's BP indicates that the dose of metoprolol may need to be decreased because of hypotension. Bradycardia is a frequent adverse effect of -adrenergic blockade, but the rate of 56 is not unusual with â-adrenergic blocker therapy. -Adrenergic blockade initially will worsen symptoms of heart failure in many patients, and patients should be taught that some increase in symptoms, such as fatigue and edema, is expected during the initiation of therapy with this class of drugs

The client diagnosed with a myocardial infarction is six (6) hours post-right femoral percutaneous transluminal angioplasty (PTCA), also known as balloon surgery. Which assessment data would require immediate intervention by the nurse? 3. The client is complaining of numbness in the right foot.

Any neurovascular assessment data that are abnormal require intervention by the nurse; numbness may indicate decreased blood supply to the right foot.

When caring for a patient with mitral valve stenosis, it is most important that the nurse assess for a. diastolic murmur. b. peripheral edema. c. shortness of breath on exertion. d. right upper quadrant tenderness.

ANS: C The pressure gradient changes in mitral stenosis lead to fluid backup into the lungs, resulting in hypoxemia and dyspnea. The other findings also may be associated with mitral valve disease but are not indicators of possible hypoxemia.

A patient with a history of chronic heart failure is admitted to the emergency department (ED) with severe dyspnea and a dry, hacking cough. Which action should the nurse do first? a. Auscultate the abdomen. b. Check the capillary refill. c. Auscultate the breath sounds. d. Assess the level of orientation.

ANS: C This patient's severe dyspnea and cough indicate that acute decompensated heart failure (ADHF) is occurring. ADHF usually manifests as pulmonary edema, which should be detected and treated immediately to prevent ongoing hypoxemia and cardiac/respiratory arrest. The other assessments will provide useful data about the patient's volume status and also should be accomplished rapidly, but detection (and treatment) of pulmonary complications is the priority.

24. The nurse is caring for a patient receiving a continuous norepinephrine (Levophed) IV infusion. Which patient assessment finding indicates that the infusion rate may need to be adjusted? a. Heart rate is 58 beats/minute. b. Mean arterial pressure (MAP) is 56 mm Hg. c. Systemic vascular resistance (SVR) is elevated. d. Pulmonary artery wedge pressure (PAWP) is low.

ANS: C Vasoconstrictors such as norepinephrine (Levophed) will increase SVR, and this will increase the work of the heart and decrease peripheral perfusion. The infusion rate may need to be decreased. Bradycardia, hypotension (MAP of 56 mm Hg), and low PAWP are not associated with norepinephrine infusion.

24. The nurse is caring for a patient receiving a continuous norepinephrine (Levophed) IV infusion. Which patient assessment finding indicates that the infusion rate may need to be adjusted? a. Heart rate is 58 beats/minute. b. Mean arterial pressure (MAP) is 56 mm Hg. c. Systemic vascular resistance (SVR) is elevated. d. Pulmonary artery wedge pressure (PAWP) is low.

ANS: C Vasoconstrictors such as norepinephrine (Levophed) will increase SVR, and this will increase the work of the heart and decrease peripheral perfusion. The infusion rate may need to be decreased. Bradycardia, hypotension (MAP of 56 mm Hg), and low PAWP are not associated with norepinephrine infusion. DIF: Cognitive Level: Apply (application) REF: 1604 TOP: Nursing Process: Evaluation MSC: NCLEX: Physiological Integrity

A nurse is assessing a patient who is receiving a nitroprusside (Nipride) infusion to treat cardiogenic shock. Which finding indicates that the medication is effective? a. No new heart murmurs b. Decreased troponin level c. Warm, pink, and dry skin d. Blood pressure 92/40 mm Hg

ANS: C Warm, pink, and dry skin indicates that perfusion to tissues is improved. Since nitroprusside is a vasodilator, the blood pressure may be low even if the medication is effective. Absence of a heart murmur and a decrease in troponin level are not indicators of improvement in shock.

A nurse is assessing a patient who is receiving a nitroprusside (Nipride) infusion to treat cardiogenic shock. Which finding indicates that the medication is effective? a. No new heart murmurs b. Decreased troponin level c. Warm, pink, and dry skin d. Blood pressure 92/40 mm Hg

ANS: C Warm, pink, and dry skin indicates that perfusion to tissues is improved. Since nitroprusside is a vasodilator, the blood pressure may be low even if the medication is effective. Absence of a heart murmur and a decrease in troponin level are not indicators of improvement in shock.

After receiving 1000 mL of normal saline, the central venous pressure for a patient who has septic shock is 10 mm Hg, but the blood pressure is still 82/40 mm Hg. The nurse will anticipate the administration of a. nitroglycerine (Tridil). b. drotrecogin alpha (Xigris). c. norepinephrine (Levophed). d. sodium nitroprusside (Nipride).

ANS: C When fluid resuscitation is unsuccessful, vasopressor drugs are administered to increase the systemic vascular resistance (SVR) and improve tissue perfusion. Nitroglycerin would decrease the preload and further drop cardiac output and BP. Drotrecogin alpha may decrease inappropriate inflammation and help prevent systemic inflammatory response syndrome, but it will not directly improve blood pressure. Nitroprusside is an arterial vasodilator and would further decrease SVR. DIF: Cognitive Level: Application REF: 1731 | 1733-1735 TOP: Nursing Process: Planning MSC: NCLEX: Physiological Integrity

The ECG monitor of a patient in the cardiac care unit after a myocardial infarction (MI) indicates ventricular bigeminy with a rate of 50 beats/minute. You anticipate A. performing defibrillation. B. treatment with IV lidocaine. C. insertion of a temporary, transvenous pacemaker. D. assessing the patient's response to the dysrhythmia.

ANS: D A premature ventricular contraction (PVC) is a contraction originating in an ectopic focus in the ventricles. When every other beat is a PVC, the rhythm is called ventricular bigeminy. PVCs are usually a benign finding in the patient with a normal heart. In heart disease, PVCs may reduce the cardiac output and precipitate angina and heart failure, depending on the frequency. Because PVCs in coronary artery disease or acute MI indicate ventricular irritability, the patient's physiologic response to PVCs must be monitored. Assessment of the patient's hemodynamic status is important to determine whether treatment with drug therapy is needed. Reference: 830

The client is admitted to the telemetry unit diagnosed with acute exacerbation of congestive heart failure (CHF). Which signs/symptoms would the nurse expect to find when assessing this client?

Apical pulse rate of 110 and 4+ pitting edema of feet.

The patient has chronic atrial fibrillation (AF). What action do you anticipate? A. Monitoring the PR interval B. Defibrillation with 360 joule C. Teaching the patient to monitor the pulse deficit D. Teaching the patient to take an anticoagulant daily

ANS: D The chaotic atrial activity results in blood stasis that can lead to embolic events. Patients with chronic AF are given an anticoagulant, most often warfarin (Coumadin), to prevent the formation of emboli. There is no PR interval in AF because the P wave is absent, replaced by chaotic fibrillatory waves. Defibrillation is an elective procedure in chronic AF and is performed at lower levels of electricity. Pulse deficit is a higher-level skill and is not taught to the patient. Reference: 827

Following an acute myocardial infarction, a previously healthy 63-year-old develops clinical manifestations of heart failure. The nurse anticipates discharge teaching will include information about a. digitalis preparations. b. -adrenergic blockers. c. calcium channel blockers. d. angiotensin-converting enzyme (ACE) inhibitors.

ANS: D ACE inhibitor therapy is currently recommended to prevent the development of heart failure in patients who have had a myocardial infarction and as a first-line therapy for patients with chronic heart failure. Digoxin therapy for heart failure is no longer considered a first-line measure, and digoxin is added to the treatment protocol when therapy with other medications such as ACE-inhibitors, diuretics, and -adrenergic blockers is insufficient. Calcium channel blockers are not generally used in the treatment of heart failure. The -adrenergic blockers are not used as initial therapy for new onset heart failure.

The nurse is obtaining a health history from a 24-year-old patient with hypertrophic cardiomyopathy (HC). Which information obtained by the nurse is most important? a. The patient has a history of a recent upper respiratory infection. b. The patient has a family history of coronary artery disease (CAD). c. The patient reports using cocaine a "couple of times" as a teenager. d. The patient's 29-year-old brother died from a sudden cardiac arrest.

ANS: D About half of all cases of HC have a genetic basis, and it is the most common cause of sudden cardiac death in otherwise healthy young people. The information about the patient's brother will be helpful in planning care (such as an automatic implantable cardioverter-defibrillator [AICD

A patient who has just been admitted with pulmonary edema is scheduled to receive the following medications. Which medication should the nurse question before giving? a. Furosemide (Lasix) 60 mg b. Captopril (Capoten) 25 mg c. Digoxin (Lanoxin) 0.125 mg d. Carvedilol (Coreg) 3.125 mg

ANS: D Although carvedilol is appropriate for the treatment of chronic heart failure, it is not used for patients with acute decompensated heart failure (ADHF) because of the risk of worsening the heart failure. The other medications are appropriate for the patient with ADHF.

Which assessment is most important for the nurse to make in order to evaluate whether treatment of a patient with anaphylactic shock has been effective? a. Pulse rate b. Orientation c. Blood pressure d. Oxygen saturation

ANS: D Because the airway edema that is associated with anaphylaxis can affect airway and breathing, the oxygen saturation is the most critical assessment. Improvements in the other assessments also will be expected with effective treatment of anaphylactic shock. DIF: Cognitive Level: Application REF: 1724-1725 | 1732 TOP: Nursing Process: Evaluation MSC: NCLEX: Physiological Integrity

Which assessment information is most important for the nurse to obtain to evaluate whether treatment of a patient with anaphylactic shock has been effective? a. Heart rate b. Orientation c. Blood pressure d. Oxygen saturation

ANS: D Because the airway edema that is associated with anaphylaxis can affect airway and breathing, the oxygen saturation is the most critical assessment. Improvements in the other assessments will also be expected with effective treatment of anaphylactic shock

Which assessment information is most important for the nurse to obtain to evaluate whether treatment of a patient with anaphylactic shock has been effective? a. Heart rate b. Orientation c. Blood pressure d. Oxygen saturation

ANS: D Because the airway edema that is associated with anaphylaxis can affect airway and breathing, the oxygen saturation is the most critical assessment. Improvements in the other assessments will also be expected with effective treatment of anaphylactic shock.

Which assessment information is most important for the nurse to obtain to evaluate whether treatment of a patient with anaphylactic shock has been effective? a. Heart rate b. Orientation c. Blood pressure d. Oxygen saturation

ANS: D Because the airway edema that is associated with anaphylaxis can affect airway and breathing, the oxygen saturation is the most critical assessment. Improvements in the other assessments will also be expected with effective treatment of anaphylactic shock.

The nurse is caring for a patient who is receiving IV furosemide (Lasix) and morphine for the treatment of acute decompensated heart failure (ADHF) with severe orthopnea. Which clinical finding is the best indicator that the treatment has been effective? a. Weight loss of 2 pounds in 24 hours b. Hourly urine output greater than 60 mL c. Reduction in patient complaints of chest pain d. Reduced dyspnea with the head of bed at 30 degrees

ANS: D Because the patient's major clinical manifestation of ADHF is orthopnea (caused by the presence of fluid in the alveoli), the best indicator that the medications are effective is a decrease in dyspnea with the head of the bed at 30 degrees. The other assessment data also may indicate that diuresis or improvement in cardiac output has occurred, but are not as specific to evaluating this patient's response.

The nurse obtains a health history from a 65-year-old patient with a prosthetic mitral valve who has symptoms of infective endocarditis (IE). Which question by the nurse is most appropriate? a. "Do you have a history of a heart attack?" b. "Is there a family history of endocarditis?" c. "Have you had any recent immunizations?" d. "Have you had dental work done recently?"

ANS: D Dental procedures place the patient with a prosthetic mitral valve at risk for infective endocarditis (IE). Myocardial infarction (MI), immunizations, and a family history of endocarditis are not risk factors for IE.

9. Which action will the nurse need to do when preparing to assist with the insertion of a pulmonary artery catheter? a. Determine if the cardiac troponin level is elevated. b. Auscultate heart and breath sounds during insertion. c. Place the patient on NPO status before the procedure. d. Attach cardiac monitoring leads before the procedure.

ANS: D Dysrhythmias can occur as the catheter is floated through the right atrium and ventricle, and it is important for the nurse to monitor for these during insertion. Pulmonary artery catheter insertion does not require anesthesia, and the patient will not need to be NPO. Changes in cardiac troponin or heart and breath sounds are not expected during pulmonary artery catheter insertion.

9. Which action will the nurse need to do when preparing to assist with the insertion of a pulmonary artery catheter? a. Determine if the cardiac troponin level is elevated. b. Auscultate heart and breath sounds during insertion. c. Place the patient on NPO status before the procedure. d. Attach cardiac monitoring leads before the procedure.

ANS: D Dysrhythmias can occur as the catheter is floated through the right atrium and ventricle, and it is important for the nurse to monitor for these during insertion. Pulmonary artery catheter insertion does not require anesthesia, and the patient will not need to be NPO. Changes in cardiac troponin or heart and breath sounds are not expected during pulmonary artery catheter insertion. DIF: Cognitive Level: Apply (application) REF: 1608 TOP: Nursing Process: Planning MSC: NCLEX: Physiological Integrity

16. To verify the correct placement of an oral endotracheal tube (ET) after insertion, the best initial action by the nurse is to a. auscultate for the presence of bilateral breath sounds. b. obtain a portable chest x-ray to check tube placement. c. observe the chest for symmetric chest movement with ventilation. d. use an end-tidal CO2 monitor to check for placement in the trachea.

ANS: D End-tidal CO2 monitors are currently recommended for rapid verification of ET placement. Auscultation for bilateral breath sounds and checking chest expansion are also used, but they are not as accurate as end-tidal CO2 monitoring. A chest x-ray confirms the placement but is done after the tube is secured. DIF: Cognitive Level: Apply (application) REF: 1614-1615 TOP: Nursing Process: Evaluation MSC: NCLEX: Physiological Integrity

35. The nurse is caring for a patient with a subarachnoid hemorrhage who is intubated and placed on a mechanical ventilator with 10 cm H2O of peak end-expiratory pressure (PEEP). When monitoring the patient, the nurse will need to notify the health care provider immediately if the patient develops a. oxygen saturation of 93%. b. respirations of 20 breaths/minute. c. green nasogastric tube drainage. d. increased jugular venous distention.

ANS: D Increases in jugular venous distention in a patient with a subarachnoid hemorrhage may indicate an increase in intracranial pressure (ICP) and that the PEEP setting is too high for this patient. A respiratory rate of 20, O2 saturation of 93%, and green nasogastric tube drainage are within normal limits. DIF: Cognitive Level: Apply (application) REF: 1623-1624 TOP: Nursing Process: Assessment MSC: NCLEX: Physiological Integrity

When teaching the patient with newly diagnosed heart failure about a 2000-mg sodium diet, the nurse explains that foods to be restricted include a. canned and frozen fruits. b. fresh or frozen vegetables. c. eggs and other high-protein foods. d. milk, yogurt, and other milk products.

ANS: D Milk and yogurt naturally contain a significant amount of sodium, and intake of these should be limited for patients on a diet that limits sodium to 2000 mg daily. Other milk products, such as processed cheeses, have very high levels of sodium and are not appropriate for a 2000-mg sodium diet. The other foods listed have minimal levels of sodium and can be eaten without restriction.

A patient with cardiogenic shock is cool and clammy and hemodynamic monitoring indicates a high systemic vascular resistance (SVR). Which action will the nurse anticipate taking? a. Increase the rate for the prescribed dopamine (Intropin) infusion. b. Decrease the rate for the prescribed nitroglycerin (Tridil) infusion. c. Decrease the rate for the prescribed 5% dextrose in water (D5W) infusion. d. Increase the rate for the prescribed sodium nitroprusside (Nipride) infusion.

ANS: D Nitroprusside is an arterial vasodilator and will decrease the SVR and afterload, which will improve cardiac output. Changes in the D5W and nitroglycerin infusions will not directly increase SVR. Increasing the dopamine will tend to increase SVR. DIF: Cognitive Level: Application REF: 1733-1734 TOP: Nursing Process: Planning MSC: NCLEX: Physiological Integrity

7. When monitoring for the effectiveness of treatment for a patient with a large anterior wall myocardial infarction, the most important information for the nurse to obtain is a. central venous pressure (CVP). b. systemic vascular resistance (SVR). c. pulmonary vascular resistance (PVR). d. pulmonary artery wedge pressure (PAWP).

ANS: D PAWP reflects left ventricular end diastolic pressure (or left ventricular preload) and is a sensitive indicator of cardiac function. Because the patient is high risk for left ventricular failure, the PAWP must be monitored. An increase will indicate left ventricular failure. The other values would also provide useful information, but the most definitive measurement of changes in cardiac function is the PAWP.

7. When monitoring for the effectiveness of treatment for a patient with a large anterior wall myocardial infarction, the most important information for the nurse to obtain is a. central venous pressure (CVP). b. systemic vascular resistance (SVR). c. pulmonary vascular resistance (PVR). d. pulmonary artery wedge pressure (PAWP).

ANS: D PAWP reflects left ventricular end diastolic pressure (or left ventricular preload) and is a sensitive indicator of cardiac function. Because the patient is high risk for left ventricular failure, the PAWP must be monitored. An increase will indicate left ventricular failure. The other values would also provide useful information, but the most definitive measurement of changes in cardiac function is the PAWP. DIF: Cognitive Level: Apply (application) REF: 1607 TOP: Nursing Process: Evaluation MSC: NCLEX: Physiological Integrity

When the nurse educator is evaluating the skills of a new registered nurse (RN) caring for patients experiencing shock, which action by the new RN indicates a need for more education? a. Placing the pulse oximeter on the ear for a patient with septic shock b. Keeping the head of the bed flat for a patient with hypovolemic shock c. Increasing the nitroprusside (Nipride) infusion rate for a patient with a high SVR d. Maintaining the room temperature at 66° to 68° F for a patient with neurogenic shock

ANS: D Patients with neurogenic shock may have poikilothermia. The room temperature should be kept warm to avoid hypothermia. The other actions by the new RN are appropriate

When the nurse educator is evaluating the skills of a new registered nurse (RN) caring for patients experiencing shock, which action by the new RN indicates a need for more education? a. Placing the pulse oximeter on the ear for a patient with septic shock b. Keeping the head of the bed flat for a patient with hypovolemic shock c. Increasing the nitroprusside (Nipride) infusion rate for a patient with a high SVR d. Maintaining the room temperature at 66° to 68° F for a patient with neurogenic shock

ANS: D Patients with neurogenic shock may have poikilothermia. The room temperature should be kept warm to avoid hypothermia. The other actions by the new RN are appropriate.

When the nurse educator is evaluating the skills of a new registered nurse (RN) caring for patients experiencing shock, which action by the new RN indicates a need for more education? a. Placing the pulse oximeter on the ear for a patient with septic shock b. Keeping the head of the bed flat for a patient with hypovolemic shock c. Increasing the nitroprusside (Nipride) infusion rate for a patient with a high SVR d. Maintaining the room temperature at 66° to 68° F for a patient with neurogenic shock

ANS: D Patients with neurogenic shock may have poikilothermia. The room temperature should be kept warm to avoid hypothermia. The other actions by the new RN are appropriate.

To evaluate the effectiveness of the pantoprazole (Protonix) ordered for a patient with systemic inflammatory response syndrome (SIRS), which assessment will the nurse perform? a. Auscultate bowel sounds. b. Palpate for abdominal pain. c. Ask the patient about nausea. d. Check stools for occult blood.

ANS: D Proton pump inhibitors are given to decrease the risk for stress ulcers in critically ill patients. The other assessments also will be done, but these will not help in determining the effectiveness of the pantoprazole administration

To evaluate the effectiveness of the pantoprazole (Protonix) ordered for a patient with systemic inflammatory response syndrome (SIRS), which assessment will the nurse perform? a. Auscultate bowel sounds. b. Palpate for abdominal pain. c. Ask the patient about nausea. d. Check stools for occult blood.

ANS: D Proton pump inhibitors are given to decrease the risk for stress ulcers in critically ill patients. The other assessments also will be done, but these will not help in determining the effectiveness of the pantoprazole administration.

To evaluate the effectiveness of the pantoprazole (Protonix) ordered for a patient with systemic inflammatory response syndrome (SIRS), which assessment will the nurse perform? a. Auscultate bowel sounds. b. Palpate for abdominal pain. c. Ask the patient about nausea. d. Check stools for occult blood.

ANS: D Proton pump inhibitors are given to decrease the risk for stress ulcers in critically ill patients. The other assessments also will be done, but these will not help in determining the effectiveness of the pantoprazole administration.

The nurse has identified a nursing diagnosis of acute pain related to inflammatory process for a patient with acute pericarditis. The priority intervention by the nurse for this problem is to a. teach the patient to take deep, slow breaths to control the pain. b. force fluids to 3000 mL/day to decrease fever and inflammation. c. remind the patient to request opioid pain medication every 4 hours. d. place the patient in Fowler's position, leaning forward on the overbed table.

ANS: D Sitting upright and leaning forward frequently will decrease the pain associated with pericarditis. Forcing fluids will not decrease the inflammation or pain. Taking deep breaths will tend to increase pericardial pain. Opioids are not very effective at controlling pain caused by acute inflammatory conditions and are usually ordered PRN. The patient would receive scheduled doses of a nonsteroidal antiinflammatory drug (NSAID).

The nurse is caring for a patient with mitral regurgitation. Referring to the figure below, where should the nurse listen to best hear any murmur that the patient has? a. 1 b. 2 c. 3 d. 4

ANS: D Sounds from the mitral valve are best heard at the apex of the heart, fifth intercostal space, midclavicular line.

During change-of-shift report, the nurse learns that a patient has been admitted with dehydration and hypotension after having vomiting and diarrhea for 3 days. Which finding is most important for the nurse to report to the health care provider? a. Decreased bowel sounds b. Apical pulse 110 beats/min c. Pale, cool, and dry extremities d. New onset of confusion and agitation

ANS: D The changes in mental status are indicative that the patient is in the progressive stage of shock and that rapid intervention is needed to prevent further deterioration. The other information is consistent with compensatory shock. DIF: Cognitive Level: Application REF: 1728-1729 OBJ: Special Questions: Prioritization TOP: Nursing Process: Assessment MSC: NCLEX: Physiological Integrity

37. After change-of-shift report on a ventilator weaning unit, which patient should the nurse assess first? a. Patient who failed a spontaneous breathing trial and has been placed in a rest mode on the ventilator b. Patient who is intubated and has continuous partial pressure end-tidal CO2 (PETCO2) monitoring c. Patient with a central venous oxygen saturation (ScvO2) of 69% while on bilevel positive airway pressure (BiPAP) d. Patient who was successfully weaned and extubated 4 hours ago and now has no urine output for the last 6 hours

ANS: D The decreased urine output may indicate acute kidney injury or that the patient's cardiac output and perfusion of vital organs have decreased. Any of these causes would require rapid action. The data about the other patients indicate that their conditions are stable and do not require immediate assessment or changes in their care. Continuous PETCO2 monitoring is frequently used when patients are intubated. The rest mode should be used to allow patient recovery after a failed SBT, and an ScvO2 of 69% is within normal limits. DIF: Cognitive Level: Analyze (analysis) REF: 1625 | 1627 OBJ: Special Questions: Prioritization; Multiple Patients TOP: Nursing Process: Planning MSC: NCLEX: Safe and Effective Care Environment

38. After change-of-shift report, which patient should the progressive care nurse assess first? a. Patient who was extubated in the morning and has a temperature of 101.4° F (38.6° C) b. Patient with bilevel positive airway pressure (BiPAP) for sleep apnea whose respiratory rate is 16 c. Patient with arterial pressure monitoring who is 2 hours post-percutaneous coronary intervention who needs to void d. Patient who is receiving IV heparin for a venous thromboembolism and has a partial thromboplastin time (PTT) of 98 sec

ANS: D The findings for this patient indicate high risk for bleeding from an elevated (nontherapeutic) PTT. The nurse needs to adjust the rate of the infusion (dose) per the health care provider's parameters. The patient with BiPAP for sleep apnea has a normal respiratory rate. The patient recovering from the percutaneous coronary intervention will need to be assisted with voiding and this task could be delegated to unlicensed assistive personnel. The patient with a fever may be developing ventilator-associated pneumonia, but addressing the bleeding risk is a higher priority. DIF: Cognitive Level: Analyze (analysis) REF: 1600 OBJ: Special Questions: Prioritization; Multiple Patients TOP: Nursing Process: Planning MSC: NCLEX: Safe and Effective Care Environment

A patient who has been involved in a motor vehicle crash is admitted to the emergency department (ED) with cool, clammy skin; tachycardia; and hypotension. Which of these prescribed interventions should the nurse implement first? a. Place the patient on continuous cardiac monitor. b. Draw blood to type and crossmatch for transfusions. c. Insert two 14-gauge IV catheters in antecubital space. d. Administer oxygen at 100% per non-rebreather mask

ANS: D The first priority in the initial management of shock is maintenance of the airway and ventilation. Cardiac monitoring, insertion of IV catheters, and obtaining blood for transfusions also should be rapidly accomplished, but only after actions to maximize oxygen delivery have been implemented. DIF: Cognitive Level: Application REF: 1732 OBJ: Special Questions: Prioritization TOP: Nursing Process: Implementation

32. The nurse educator is evaluating the care that a new registered nurse (RN) provides to a patient receiving mechanical ventilation. Which action by the new RN indicates the need for more education? a. The RN increases the FIO2 to 100% before suctioning. b. The RN secures a bite block in place using adhesive tape. c. The RN asks for assistance to reposition the endotracheal tube. d. The RN positions the patient with the head of bed at 10 degrees.

ANS: D The head of the patient's bed should be positioned at 30 to 45 degrees to prevent ventilator-associated pneumonia. The other actions by the new RN are appropriate. DIF: Cognitive Level: Apply (application) REF: 1623 OBJ: Special Questions: Delegation TOP: Nursing Process: Evaluation MSC: NCLEX: Safe and Effective Care Environment

15. While waiting for cardiac transplantation, a patient with severe cardiomyopathy has a ventricular assist device (VAD) implanted. When planning care for this patient, the nurse should anticipate a. giving immunosuppressive medications. b. preparing the patient for a permanent VAD. c. teaching the patient the reason for complete bed rest. d. monitoring the surgical incision for signs of infection.

ANS: D The insertion site for the VAD provides a source for transmission of infection to the circulatory system and requires frequent monitoring. Patient's with VADs are able to have some mobility and may not be on bed rest. The VAD is a bridge to transplantation, not a permanent device. Immunosuppression is not necessary for nonbiologic devices like the VAD. DIF: Cognitive Level: Apply (application) REF: 1613 TOP: Nursing Process: Planning MSC: NCLEX: Physiological Integrity

17. To maintain proper cuff pressure of an endotracheal tube (ET) when the patient is on mechanical ventilation, the nurse should a. inflate the cuff with a minimum of 10 mL of air. b. inflate the cuff until the pilot balloon is firm on palpation. c. inject air into the cuff until a manometer shows 15 mm Hg pressure. d. inject air into the cuff until a slight leak is heard only at peak inflation.

ANS: D The minimal occluding volume technique involves injecting air into the cuff until an air leak is present only at peak inflation. The volume to inflate the cuff varies with the ET and the patient's size. Cuff pressure should be maintained at 20 to 25 mm Hg. An accurate assessment of cuff pressure cannot be obtained by palpating the pilot balloon. DIF: Cognitive Level: Understand (comprehension) REF: 1615 TOP: Nursing Process: Implementation MSC: NCLEX: Physiological Integrity

30. The nurse responds to a ventilator alarm and finds the patient lying in bed holding the endotracheal tube (ET). Which action should the nurse take next? a. Activate the rapid response team. b. Provide reassurance to the patient. c. Call the health care provider to reinsert the tube. d. Manually ventilate the patient with 100% oxygen.

ANS: D The nurse should ensure maximal patient oxygenation by manually ventilating with a bag-valve-mask system. Offering reassurance to the patient, notifying the health care provider about the need to reinsert the tube, and activating the rapid response team are also appropriate after the nurse has stabilized the patient's oxygenation. DIF: Cognitive Level: Apply (application) REF: 1617 OBJ: Special Questions: Prioritization TOP: Nursing Process: Implementation MSC: NCLEX: Physiological Integrity

A common initial mediator that causes endothelial damage leading to SIRS and MODS is "endotoxin". (T/F)

True

A patient admitted with acute dyspnea is newly diagnosed with dilated cardiomyopathy. Which information will the nurse plan to teach the patient about managing this disorder? a. A heart transplant should be scheduled as soon as possible. b. Elevating the legs above the heart will help relieve dyspnea. c. Careful compliance with diet and medications will prevent heart failure. d. Notify the doctor about any symptoms of heart failure such as shortness of breath.

ANS: D The patient should be instructed to notify the health care provider about any worsening of heart failure symptoms. Because dilated cardiomyopathy does not respond well to therapy, even patients with good compliance with therapy may have recurrent episodes of heart failure. Elevation of the legs above the heart will worsen symptoms (although this approach is appropriate for a patient with hypertrophic cardiomyopathy). The patient with terminal or end-stage cardiomyopathy may consider heart transplantation.

21. Four hours after mechanical ventilation is initiated for a patient with chronic obstructive pulmonary disease (COPD), the patient's arterial blood gas (ABG) results include a pH of 7.51, PaO2 of 82 mm Hg, PaCO2 of 26 mm Hg, and HCO3- of 23 mEq/L (23 mmol/L). The nurse will anticipate the need to a. increase the FIO2. b. increase the tidal volume. c. increase the respiratory rate. d. decrease the respiratory rate.

ANS: D The patient's PaCO2 and pH indicate respiratory alkalosis caused by too high a respiratory rate. The PaO2 is appropriate for a patient with COPD and increasing the respiratory rate and tidal volume would further lower the PaCO2. DIF: Cognitive Level: Analyze (analysis) REF: 1615-1616 TOP: Nursing Process: Planning MSC: NCLEX: Physiological Integrity

10. When assisting with the placement of a pulmonary artery (PA) catheter, the nurse notes that the catheter is correctly placed when the monitor shows a a. typical PA pressure waveform. b. tracing of the systemic arterial pressure. c. tracing of the systemic vascular resistance. d. typical PA wedge pressure (PAWP) tracing.

ANS: D The purpose of a PA line is to measure PAWP, so the catheter is floated through the pulmonary artery until the dilated balloon wedges in a distal branch of the pulmonary artery, and the PAWP readings are available. After insertion, the balloon is deflated and the PA waveform will be observed. Systemic arterial pressures are obtained using an arterial line and the systemic vascular resistance is a calculated value, not a waveform.

10. When assisting with the placement of a pulmonary artery (PA) catheter, the nurse notes that the catheter is correctly placed when the monitor shows a a. typical PA pressure waveform. b. tracing of the systemic arterial pressure. c. tracing of the systemic vascular resistance. d. typical PA wedge pressure (PAWP) tracing.

ANS: D The purpose of a PA line is to measure PAWP, so the catheter is floated through the pulmonary artery until the dilated balloon wedges in a distal branch of the pulmonary artery, and the PAWP readings are available. After insertion, the balloon is deflated and the PA waveform will be observed. Systemic arterial pressures are obtained using an arterial line and the systemic vascular resistance is a calculated value, not a waveform. DIF: Cognitive Level: Understand (comprehension) REF: 1608 TOP: Nursing Process: Assessment MSC: NCLEX: Physiological Integrity

Bradycardia with hypotension is characteristic of neurogenic shock. (T /F)

True

Which action could the nurse delegate to unlicensed assistive personnel (UAP) trained as electrocardiogram (ECG) technicians working on the cardiac unit? a. Select the best lead for monitoring a patient with an admission diagnosis of Dressler syndrome. b. Obtain a list of herbal medications used at home while admitting a new patient with pericarditis. c. Teach about the need to monitor the weight daily for a patient who has hypertrophic cardiomyopathy. d. Check the heart monitor for changes in rhythm while a patient who had a valve replacement ambulates.

ANS: D Under the supervision of registered nurses (RNs), UAP check the patient's cardiac monitor and obtain information about changes in heart rate and rhythm with exercise. Teaching and obtaining information about home medications (prescribed or complementary) and selecting the best leads for monitoring patients require more critical thinking and should be done by the RN.

While caring for a 23-year-old patient with mitral valve prolapse (MVP) without valvular regurgitation, the nurse determines that discharge teaching has been effective when the patient states that it will be necessary to a. take antibiotics before any dental appointments. b. limit physical activity to avoid stressing the heart. c. take an aspirin a day to prevent clots from forming on the valve. d. avoid use of over-the-counter (OTC) medications that contain stimulant drugs.

ANS: D Use of stimulant medications should be avoided by patients with MVP because these may exacerbate symptoms. Daily aspirin and restricted physical activity are not needed by patients with mild MVP. Antibiotic prophylaxis is needed for patients with MVP with regurgitation but will not be necessary for this patient.

25. When caring for the patient with a pulmonary artery (PA) pressure catheter, the nurse observes that the PA waveform indicates that the catheter is in the wedged position. Which action should the nurse take next? a. Zero balance the transducer. b. Activate the fast flush system. c. Notify the health care provider. d. Deflate and reinflate the PA balloon.

ANS: D When the catheter is in the wedge position, blood flow past the catheter is obstructed, placing the patient at risk for pulmonary infarction. A health care provider or advanced practice nurse should be called to reposition the catheter. The other actions will not correct the wedging of the PA catheter.

For which dysrhythmia is defibrillation primarily indicated? A. Ventricular fibrillation B. Third-degree AV block C. Uncontrolled atrial fibrillation D. Ventricular tachycardia with a pulse

ANS:A Defibrillation is always indicated in the treatment of ventricular fibrillation. Drug treatments are normally used in the treatment of uncontrolled atrial fibrillation and for ventricular tachycardia with a pulse (if the patient is stable). Otherwise, synchronized cardioversion is used (as long as the patient has a pulse). Pacemakers are the treatment of choice for third-degree heart block. Reference: 833

You prepare a patient for synchronized cardioversion knowing that cardioversion differs from defibrillation in that A. defibrillation requires lower dose of electrical energy. B. cardioversion is indicated for treatment of atrial bradydysrhythmias. C. defibrillation is synchronized to deliver a shock during the QRS complex. D. patients should be sedated if cardioversion is done on a nonemergent basis.

ANS:D Synchronized cardioversion is the therapy of choice for the patient with hemodynamically unstable ventricular or supraventricular tachydysrhythmias. A synchronized circuit in the defibrillator delivers a countershock that is programmed to occur on the R wave of the QRS complex seen on the ECG tracing. The synchronizer switch must be turned on when cardioversion is planned. The procedure for synchronized cardioversion is the same as for defibrillation, with the following exceptions. If synchronized cardioversion is done on a nonemergent basis, the patient is sedated before the procedure. The initial energy needed for synchronized cardioversion is less than the energy needed for defibrillation. Reference: 833

Excitability

Ability to be electrically stimulated

Automaticity

Ability to initiate an impulse spontaneously and continuously

Contractility

Ability to respond mechanically to an impulse

The nurse is obtaining a health history from a 24-year-old patient with hypertrophic cardiomyopathy (HC). Which information obtained by the nurse is most important? d. The patient's 29-year-old brother died from a sudden cardiac arrest.

About half of all cases of HC have a genetic basis, and it is the most common cause of sudden cardiac death in otherwise healthy young people.

Which type of shock causes an absence of bowel sounds? 1 Cardiogenic shock 2 Neurogenic shock 3 Hypovolemic shock 4 Anaphylactic shock

Absence of bowel sounds is associated with hypovolemic shock. Decreased bowel sounds are seen with cardiogenic shock. Bowel dysfunction is associated with neurogenic shock. Abdominal pain, nausea and vomiting are seen with anaphylactic shock. Test-Taking Tip: Identifying content and what is being asked about that content is critical to your choosing the correct response. Be alert for words in the stem of the item that are the same or similar in nature to those in one or two of the options. Text Reference - p. 1635

Which nursing diagnosis would be priority for the client diagnosed with myocarditis? Activity intolerance related to impaired cardiac muscle function.

Activity intolerance is priority for the client with myocarditis, an inflammation of the heart muscle. Nursing care is aimed at decreasing myocardial work and maintaining cardiac output.

Which interventions should the nurse discuss with the client diagnosed with atherosclerosis?Select all that apply. 1. Include significant other in the discussion. 2. Stop smoking or using any type of tobacco products

Adherence to lifestyle modifications is enhanced when the client receives support from significant others. 2. Tobacco use is the most significant modifi- able risk factor that contributes to the development of atherosclerosis.

Nursing management chronic stable angina

Administration of supplemental oxygen, Assess vital signs, pulse oximetry, 12-lead ECG, Prompt pain relief first with a nitrate followed by an opioid analgesic, if needed, Auscultation of heart sounds

. The nurse and an unlicensed nursing assistant are caring for a 64-year-old client who is four (4) hours post-operative bilateral femoral-popliteal bypass surgery. Which nursing task should be delegated to the unlicensed nursing assistant? Elevate the foot of the client's bed.

After the surgery, the client's legs will be elevated to help decrease edema. The surgery has corrected the decreased blood supply to the lower legs.

Non-modifiable risk factors for CAD

Age (after 65 years old incidence in men and women become =), gender (highest in middle-aged men), family Hx, ethnicity (caucasian at greater risk than African Americans), genetic predisposition

Assessment

Age is important- Hypovolemic shock from trauma is more common in young adults & other types of shock- more common in older adults. -Information about urine output is especially important because urine output is reduced during the first stages of shock even when fluid intake is normal.

dilation of the blood vessels.

All of the following can result in hypoperfusion EXCEPT:

Hemodynamic monitoring in the patient with cardiogenic shock will reveal an increased PAWP and a decreased cardiac output. (T /F)

True

In anaphylactic shock, death may occur as a result of respiratory failure. (T /F)

True

After having an MI, the nurse notes the patient has jugular venous distention, gained weight, developed peripheral edema, and has a heart rate of 108/minute. What should the nurse suspect is happening? D Right-sided HF

An MI is a primary cause of heart failure. The jugular venous distention, weight gain, peripheral edema, and increased heart rate are manifestations of right-sided heart failure.

After having an MI, the nurse notes the patient has jugular venous distention, gained weight, developed peripheral edema, and has a heart rate of 108/minute. What should the nurse suspect is happening? d. Right-sided HF

An MI is a primary cause of heart failure. The jugular venous distention, weight gain, peripheral edema, and increased heart rate are manifestations of right-sided heart failure.

The nurse on the telemetry unit has just received the A.M. shift report. Which client should the nurse assess first? 1. The client diagnosed with myocardial infarction who has an audible S3 heart sound.

An S3 heart sound indicates left ventricular failure, and the nurse must assess this client first because it is an emergency situation.

The intensive care department nurse is assessing the client who is 12 hours postmyocardial infarction. The nurse assesses a S3 heart sound. Which intervention should the nurse implement? 1. Notify the health-care provider immediately.

An S3 indicates left ventricular failure and should be reported to the health-care provider. It is a potential life-threatening complication of a myocardial infarction.

The nurse is unable to assess a pedal pulse in the client diagnosed with arterial occlusive disease. Which intervention should the nurse implement first? 1. Complete a neurovascular assessment.

An absent pulse is not uncommon in a client diagnosed with arterial occlusive disease, but the nurse must ensure that the feet can be moved and are warm, which indicates adequate blood supply to the feet.

2 To ensure adequate tracheal perfusion, the nurse should maintain cuff pressure at 20 to 25 cm H2O. Excess cuff pressure can damage the tracheal mucosa. Lesser cuff pressure may cause the ET tube to become destabilized and extubate. Text Reference - p. 1615

An endotracheal (ET) tube is inserted in a patient. The nurse inflates the cuff to stabilize the tube. How much cuff pressure should be maintained to keep it inflated and ensure adequate tracheal perfusion? 1 10-15 cm H2O 2 20-25 cm H2O 3 30-35 cm H2O 4 40-45 cm H2O

1, 3, 4 Whenever a patient is intubated, repositioning and retaping the tube after 24 hours are essential. For the orally intubated patient, remove the bite block and the old tape or ties. Provide oral hygiene, and then reposition the ET tube to the opposite side of the mouth. There is no need to give anesthesia for maintaining oral care. There is no point in physically restraining the patient, because it will cause discomfort and anxiety. Text Reference - p. 1617

An endotracheal (ET) tube is placed for a patient. What interventions should the nurse perform to maintain oral hygiene and care? Select all that apply. 1 Reposition and retape the ET tube at least every 24 hours. 2 Keep the buccal cavity and nasal cavity anesthetized. 3 Replace the bite block and reconfirm proper cuff inflation and tube placement regularly. 4 Provide oral hygiene with repositioning of the ET tube to the opposite side of the mouth after care. 5 Keep the patient restrained to avoid dislodging the tube.

The health-care provider prescribes an ACE inhibitor for the client diagnosed with essential hypertension. Which statement is the most appropriate rationale for administering this medication? 3. ACE inhibitors prevent vasoconstriction and sodium and water retention.

Angiotensin-converting enzyme (ACE) inhibitors prevent the conversion of angiotensin I to angiotensin II, and this, in turn, prevents vasoconstriction and sodium and water retention.

The nurse is administering a dose of digoxin (Lanoxin) to a patient with heart failure (HF). The nurse would become concerned with the possibility of digitalis toxicity if the patient reported which symptom(s)? D Anorexia and nausea

Anorexia, nausea, vomiting, blurred or yellow vision, and cardiac dysrhythmias are all signs of digitalis toxicity.

The nurse is administering a dose of digoxin (Lanoxin) to a patient with heart failure (HF). The nurse would become concerned with the possibility of digitalis toxicity if the patient reported which symptom(s)? d. Anorexia and nausea

Anorexia, nausea, vomiting, blurred or yellow vision, and cardiac dysrhythmias are all signs of digitalis toxicity.

The nurse is caring for a patient with acute respiratory distress syndrome (ARDS). Which intervention should the nurse initiate to prevent stress ulcers? A. Begin enteral feedings as soon as bowel sounds are present. B. Administer prescribed lorazepam (Ativan) to reduce anxiety. C. Observe stool for frank bleeding and occult blood. D. Maintain head of the bed elevation at 30 to 45 degrees.

Answer: A. Begin enteral feedings as soon as bowel sounds are present. Rationale: Stress ulcers prevention includes early initiation of enteral nutrition to protect gastrointestinal (GI) tract from mucosal damage. Antiulcer agents such as histamine (H2)-receptor antagonists, proton pump inhibitors, and mucosal protecting agents are also indicated to prevent stress ulcers. Stress ulcers are not caused by anxiety. Stress ulcers are related to GI ischemia from hypotension, shock, and acidosis. Monitoring for GI bleeding will not prevent stress ulcers. Ventilator-associated pneumonia related to aspiration is prevented by elevation of the head of bed to 30 to 45 degrees. Ch. 68

The nurse is caring for a patient with multiple fractured ribs. Which clinical manifestation, if experienced by the patient, is an early indication that the patient is developing respiratory failure? A. Kussmaul respirations and hypotension B. Frequent position changes and agitation C. Cyanosis and increased capillary refill time D. Tachycardia and pursed lip breathing

Answer: B. Frequent position changes and agitation Rationale: A change in mental status is an early indication of respiratory failure; the brain is sensitive to variations in oxygenation, arterial carbon dioxide levels, and acid-base balance. Restlessness, confusion, agitation, and combative behavior suggest inadequate oxygen delivery to the brain. Ch. 68

A patient with severe sepsis is receiving drotrecogin alfa (Xigris). What should the nurse monitor during the infusion of this medication? A. Absolute neutrophil count and C-reactive protein B. Hemoglobin, platelet count, and prothrombin time C. Serum creatinine and blood urea nitrogen levels D. D-dimer and glycosylated hemoglobin A1C

Answer: B. Hemoglobin, platelet count, and prothrombin time Rationale: Drotrecogin alfa (Xigris) is a recombinant form of activated protein C and is indicated for patients with severe sepsis and a high risk of death. Drotrecogin alfa interrupts the body's response to severe sepsis, including bleeding and clotting abnormalities. The most common side effect of drotrecogin alfa is bleeding. The nurse should monitor hemoglobin, platelets, prothrombin time, and partial thromboplastin time. Ch. 66/67

The nurse cares for a patient who is orally intubated on mechanical ventilation. Which action is most important for the nurse to take? A. Limit noise and cluster activities. B. Maintain head of bed elevation at 30 degrees. C. Administer morphine for discomfort. D. Use the open-suction technique.

Answer: B. Maintain head of bed elevation at 30 degrees. Rationale: The two major complications of endotracheal intubation are unplanned extubation and aspiration. To prevent aspiration all intubated patients and patients receiving enteral feedings must have the head of the bed (HOB) elevated a minimum of 30 to 45 degrees unless medically contraindicated. To promote rest and sleep the nurse should limit noise and cluster activities. Closed-suction technique is preferred over the open-suction technique because oxygenation and ventilation are maintained during suctioning, and exposure to secretions is reduced. The nurse should provide comfort measures such as morphine to relieve anxiety and pain associated with intubation. Ch. 66/67

The nurse is caring for a patient with acute decompensated heart failure who has a pulmonary artery catheter. Which assessment best indicates that the patient's condition is improving? A. Central venous pressure (CVP) is 8 mm Hg. B. Pulmonary artery wedge pressure (PAWP) is 10 mm Hg. C. Cardiac output (CO) is 3.8 L/minute. D. Systemic vascular resistance (SVR) is 1700 dynes/sec/cm-5.

Answer: B. Pulmonary artery wedge pressure (PAWP) is 10 mm Hg. Rationale: PAWP is the most sensitive indicator of cardiac function and fluid volume status. Normal range for PAWP is 6 to 12 mm Hg. PAWP is increased in heart failure. Normal range for CVP is 2 to 8 mm Hg; elevated CVP indicates right-sided heart failure or volume overload. CO is decreased in heart failure; normal cardiac output is 4 to 8 L/minute. SVR is increased in left-sided heart failure; normal SVR is 800 to 1200 dynes/sec/cm-5. Ch. 66/67

A patient with acute respiratory distress syndrome (ARDS) is on positive pressure ventilation (PPV). The patient's cardiac index is 1.4 L/minute and pulmonary artery wedge pressure is 8 mm Hg. What order by the physician is important for the nurse to question? A. Change the intravenous (IV) rate from 75 to 125 mL/hour. B. Initiate a dobutamine (Dobutrex) infusion at 3 mcg/kg/minute. C. Increase positive end-expiratory pressure (PEEP) from 10 to 15 cm H2O. D. Administer one unit packed red blood cells over the next 2 hours.

Answer: C. Increase positive end-expiratory pressure (PEEP) from 10 to 15 cm H2O. Rationale: Patients on PPV and PEEP frequently experience decreased cardiac output (CO) and cardiac index (CI). High levels of PEEP increase intrathoracic pressure and cause decreased venous return; this results in decreased CO. Interventions to improve CO include lowering the PEEP, administering crystalloid fluids or colloid solutions, use of inotropic drugs (e.g., dobutamine, dopamine). Packed red blood cells may also be administered to improve CO and oxygenation if the hemoglobin is less than 9 or 10 mg/dL. Ch. 68

A patient with aspiration pneumonia develops severe respiratory distress. Which diagnostic finding would indicate to the nurse that the patient has developed acute respiratory distress syndrome (ARDS)? A. PaCO2 of 31 mm Hg and pH 7.51 B. PaO2 of 64 mm Hg on 24% FIO2 C. PaO2 of 42 mm Hg on 80% FIO2 D. PaCO2 of 70 mm Hg and pH 7.29

Answer: C. PaO2 of 42 mm Hg on 80% FIO2 Rationale: Refractory hypoxemia indicates ARDS. PaO2 of 42 mm Hg on 80% FIO2; the PaO2/FIO2 ratio is 52.5 that indicates ARDS (PaO2/FIO2 ratio less than 200). PaCO2 of 31 mm Hg and pH 7.51 indicates respiratory alkalosis. PaO2 of 64 mm Hg on 24% FIO2; the PaO2/FIO2 ratio is 267that indicates an acute lung injury (PaO2/FIO2 ratio between 200 to 300). PaCO2 of 70 mm Hg and pH 7.29 indicates respiratory acidosis. (Fraction of inspired oxygen, FIO2.) Ch. 68

A patient is admitted to the emergency department with gastrointestinal bleeding. The patient's vital signs are blood pressure 78/58 mm Hg, pulse 124 beats/minute, respirations 28 breaths/minute, and temperature 97.2° F (36.2° C). Which physician order should the nurse complete first? A. Administer norepinephrine (Levophed) by continuous IV infusion. B. Insert a nasogastric tube and an indwelling bladder catheter. C. Rapidly administer 1000 mL normal saline solution intravenously. D. Obtain a 12-lead electrocardiogram, chest radiograph, and arterial blood gases.

Answer: C. Rapidly administer 1000 mL normal saline solution intravenously. Rationale: Isotonic crystalloids, such as normal saline solution, should be used in the initial resuscitation of hypovolemic shock. Vasopressor drugs (e.g., norepinephrine) may be considered if the patient does not respond to fluid resuscitation and blood products. Other orders (e.g., insertion of nasogastric tube and indwelling bladder catheter and obtaining the diagnostic studies) can be initiated after fluid resuscitation is initiated. Ch. 66/67

A patient's arterial blood gas result is pH 7.31, PaCO2 49 mm Hg, HCO3 26 mEq/L, and PaO2 52 mm Hg. What action is most important for the nurse to take? A. Instruct the patient to breathe into a paper bag. B. Increase fluid intake to 2500 mL per 24 hours. C. Perform chest physical therapy four times per day. D. Initiate oxygen at 2 L/minute by nasal cannula.

Answer: D. Initiate oxygen at 2 L/minute by nasal cannula. Rationale: The arterial blood gas results indicate the patient is in uncompensated respiratory acidosis with moderate hypoxemia. Oxygen therapy is indicated to correct hypoxemia secondary to V/Q mismatch. Supplemental oxygen should be initiated at 1 to 3 L/min by nasal cannula, or 24% to 32% by simple face mask or Venturi mask to improve the PaO2. Breathing into a paper bag is indicated for hyperventilation to correct respiratory alkalosis. Hydration is indicated for thick secretions, and chest physical therapy is indicated for patients with 30 mL or more of sputum production per day. Ch. 68

The nurse is caring for a patient in cardiogenic shock after an acute myocardial infarction. Which clinical manifestations would be of most concern to the nurse? A. Agitation, respiratory rate of 32 breaths/minute, and serum creatinine level of 2.6 mg/dL B. Mean arterial pressure of 54 mm Hg, jaundice, and cold, clammy skin C. Restlessness, heart rate of 124 beats/minute, and hypoactive bowel sounds D. PaO2 of 38 mm Hg, serum lactate level of 46.5 mcg/dL, and bleeding from puncture sites

Answer: D. PaO2 of 38 mm Hg, serum lactate level of 46.5 mcg/dL, and bleeding from puncture sites Rationale: Severe hypoxemia, lactic acidosis, and bleeding are clinical manifestations of the irreversible state of shock; recovery from this stage is not likely because of multiple organ system failure. Agitation, tachypnea, and increased serum creatinine are clinical manifestations of the progressive stage of shock. Decreased mean arterial pressure, jaundice, and cold, clammy skin are also clinical manifestations of the progressive stage of shock. Restlessness, tachycardia, and hypoactive bowel sounds are clinical manifestations that occur during the compensatory stage of shock. Ch. 66/67

In septic shock, bacterial endotoxins cause vascular changes that result in decreased systemic vascular resistance with increased cardiac output. (T /F)

True

The "respiratory" system is often the first to show evidence of dysfunction in SIRS and MODS. (T/F)

True

When teaching the patient with acute rheumatic fever, identify the rationale for the use of each of the following drugs in the patient's treatment plan. Antibiotics: Aspirin: Corticosteroids: Nonsteroidal antiinflammatory drugs (NSAIDs):

Antibiotics: To eliminate any residual group A β-hemolytic streptococci; prevent spread of infection; prevent recurrent infection Aspirin: Antiinflammatory effect to control fever and arthritic and joint manifestations Corticosteroids: Antiinflammatory effect to control fever and inflammation of severe carditis Nonsteroidal antiinflammatory drugs (NSAIDs): Antiinflammatory effect to control fever and joint manifestations

The client has just received a mechanical valve replacement. Which behavior by the client indicates the client needs more teaching? 3. The client takes an enteric-coated aspirin daily.

Aspirin and nonsteroidal anti-inflammatory drugs (NSAIDs) interfere with clotting and may potentiate the effects of the anticoagulant therapy, which the client with a mechanical valve will be prescribed. Therefore, the client should not take aspirin daily.

The nurse is caring for a client diagnosed with a myocardial infarction who is experiencing chest pain. Which interventions should the nurse implement? Select all that apply. 2. Administer an aspirin orally. 3. Apply oxygen via a nasal cannula.

Aspirin is an antiplatelet medication and should be administered orally. Oxygen will help decrease myocardial ischemia, thereby decreasing pain.

The client is scheduled for a right femoral cardiac catheterization. Which nursing intervention should the nurse implement after the procedure?

Assess the client's neurovascular status. - assess for pulses, paresthesia, paralysis, coldness and pallor - HOB elevated no more than 10 degrees, client kept in bedrest

The nurse enters the room of the client diagnosed with congestive heart failure. The client is lying in bed gasping for breath, is cool and clammy, and has buccal cyanosis. Which intervention would the nurse implement first?

Assist the client to a sitting position THEN vital and check pulse ox then sponge the client's forehead

The nurse is developing a nursing care plan for a client diagnosed with congestive heart failure. A nursing diagnosis of "decreased cardiac output related to inability of the heart to pump effectively" is written. Which short-term goal would be best for the client? The client will: 2. Have an audible S1 and S2 with no S3 heard by end of shift.

Audible S1 and S2 sounds are normal for a heart with adequate output. An audible S3 sound might indicate left ventricular failure that could be life threatening.

3 Auto-positive end-expiratory pressure (auto-PEEP) is caused by inadequate exhalation time. Barotrauma, hemodynamic instability and increased work of breathing are the results and not the causes of the auto-PEEP. Text Reference - p. 1621

Auto-positive end-expiratory pressure (PEEP) is the additional PEEP over what is set by the health care provider. What causes auto-PEEP during mechanical ventilation? 1 Barotrauma 2 Hemodynamic instability 3 Inadequate exhalation time 4 Increased work of breathing

AED

Automatic external defibrillators have rhythm detection capability and the ability to advise the operator to deliver a shock using hands-free defibrillator pads.

What should the nurse recognize as an indication for the use of dopamine (Intropin) in the care of a patient with heart failure? A Acute anxiety B Hypotension and tachycardia C Peripheral edema and weight gain D Paroxysmal nocturnal dyspnea (PND)

B Hypotension and tachycardia Dopamine is a β-adrenergic agonist whose inotropic action is used for treatment of severe heart failure accompanied by hemodynamic instability. Such a state may be indicated by tachycardia accompanied by hypotension. PND, anxiety, edema, and weight gain are common signs and symptoms of heart failure, but these do not necessarily warrant the use of dopamine.

Which diagnostic test will be most useful to the nurse in determining whether a patient admitted with acute shortness of breath has heart failure? c. B-type natriuretic peptide

B-type natriuretic peptide (BNP) is secreted when ventricular pressures increase, as they do with heart failure.

Physiological Integrity 21. A 19-year-old student comes to the student health center at the end of the semester complaining that, "My heart is skipping beats." An electrocardiogram (ECG) shows occasional premature ventricular contractions (PVCs). What action should the nurse take next? a. Start supplemental O2 at 2 to 3 L/min via nasal cannula. b. Ask the patient about current stress level and caffeine use. c. Ask the patient about any history of coronary artery disease. d. Have the patient taken to the hospital emergency department (ED).

B In a patient with a normal heart, occasional PVCs are a benign finding. The timing of the PVCs suggests stress or caffeine as possible etiologic factors. It is unlikely that the patient has coronary artery disease, and this should not be the first question the nurse asks. The patient is hemodynamically stable, so there is no indication that the patient needs to be seen in the ED or that oxygen needs to be administered. DIF: Cognitive Level: Apply (application) REF: 799 OBJ: Special Questions: Prioritization TOP: Nursing Process: Implementation MSC:

Physiological Integrity 24. A patient whose heart monitor shows sinus tachycardia, rate 132, is apneic and has no palpable pulses. What is the first action that the nurse should take? a. Perform synchronized cardioversion. b. Start cardiopulmonary resuscitation (CPR). c. Administer atropine per agency dysrhythmia protocol. d. Provide supplemental oxygen via non-rebreather mask.

B The patient's clinical manifestations indicate pulseless electrical activity and the nurse should immediately start CPR. The other actions would not be of benefit to this patient. DIF: Cognitive Level: Apply (application) REF: 800 OBJ: Special Questions: Prioritization TOP: Nursing Process: Implementation MSC:

Physiological Integrity 5. The nurse notes that a patient's cardiac monitor shows that every other beat is earlier than expected, has no visible P wave, and has a QRS complex that is wide and bizarre in shape. How will the nurse document the rhythm? a. Ventricular couplets b. Ventricular bigeminy c. Ventricular R-on-T phenomenon d. Multifocal premature ventricular contractions

B Ventricular bigeminy describes a rhythm in which every other QRS complex is wide and bizarre looking. Pairs of wide QRS complexes are described as ventricular couplets. There is no indication that the premature ventricular contractions (PVCs) are multifocal or that the R-on-T phenomenon is occurring. DIF: Cognitive Level: Apply (application) REF: 799 TOP: Nursing Process: Assessment MSC:

A patient with a diagnosis of heart failure has been started on a nitroglycerin patch by his primary care provider. What should this patient be taught to avoid? A High-potassium foods B Drugs to treat erectile dysfunction C Nonsteroidal antiinflammatory drugs D Over-the-counter H2 -receptor blockers

B Drugs to treat erectile dysfunction The use of erectile drugs concurrent with nitrates creates a risk of severe hypotension and possibly death. High-potassium foods, NSAIDs, and H2-receptor blockers do not pose a risk in combination with nitrates.

The nurse is preparing to administer digoxin to a patient with heart failure. In preparation, laboratory results are reviewed with the following findings: sodium 139 mEq/L, potassium 5.6 mEq/L, chloride 103 mEq/L, and glucose 106 mg/dL. What should the nurse do next? A Withhold the daily dose until the following day. B Withhold the dose and report the potassium level. C Give the digoxin with a salty snack, such as crackers. D Give the digoxin with extra fluids to dilute the sodium level.

B Withhold the dose and report the potassium level The normal potassium level is 3.5 to 5.0 mEq/L. The patient is hyperkalemic, which makes the patient more prone to digoxin toxicity. For this reason, the nurse should withhold the dose and report the potassium level. The physician may order the digoxin to be given once the potassium level has been treated and decreases to within normal range.

What do you do if the patients starts going into v. fib during cardioversion?

Turn off sync and defib!

The nurse provides discharge instructions for a 40-year-old woman who is newly diagnosed with cardiomyopathy. Which statement, if made by the patient, indicates that further teaching is necessary? A."I will avoid lifting heavy objects." B. "I can drink alcohol in moderation." C."My family will need to take a CPR course." D. "I will reduce stress by learning guided imagery."

B. "I can drink alcohol in moderation." Patients with cardiomyopathy should avoid alcohol consumption, especially in patients with alcohol-related dilated cardiomyopathy. Avoiding heavy lifting and stress, as well as family members learning CPR, are recommended teaching points.

The nurse performs discharge teaching for a 68-year-old man who is newly diagnosed with infective endocarditis with a history of IV substance abuse. Which statement by the patient indicates to the nurse that teaching was successful? A."I will need antibiotics before having any invasive procedure or surgery." B. "I will inform my dentist about my hospitalization for infective endocarditis." C."I should not be alarmed if I have difficulty breathing or pink-tinged sputum." D. "An elevated temperature is expected and can be managed by taking acetaminophen."

B. "I will inform my dentist about my hospitalization for infective endocarditis." Patients with infective endocarditis should inform their dental providers of their health history. Antibiotic prophylaxis is recommended for patients with a history of infective endocarditis who have certain dental procedures performed. Antibiotics are not indicated before genitourinary or gastrointestinal procedures unless an infection is present. Patients should immediately report the presence of fever or clinical manifestations indicating heart failure to their health care provider.

The patient had a history of rheumatic fever and has been diagnosed with mitral valve stenosis. The patient is planning to have a biologic valve replacement. What protective mechanisms should the nurse teach the patient about using after the valve replacement?

B. Antibiotic prophylaxis for dental care

The patient had a history of rheumatic fever and has been diagnosed with mitral valve stenosis. The patient is planning to have a biologic valve replacement. What protective mechanisms should the nurse teach the patient about using after the valve replacement? A. Long-term anticoagulation therapy B. Antibiotic prophylaxis for dental care C. Exercise plan to increase cardiac tolerance D. Take β-adrenergic blockers to control palpitations.

B. Antibiotic prophylaxis for dental care The patient will need to use antibiotic prophylaxis for dental care to prevent endocarditis. Long-term anticoagulation therapy is not used with biologic valve replacement unless the patient has atrial fibrillation. An exercise plan to increase cardiac tolerance is needed for a patient with heart failure. Taking β-adrenergic blockers to control palpitations is prescribed for mitral valve prolapse, not valve replacement.

The nurse would recognize which of the following clinical manifestations as suggestive of sepsis? A. Respiratory rate of seven breaths per minute B. Hyperglycemia in the absence of diabetes C. Sudden diuresis unrelated to drug therapy D. Bradycardia with sudden increase in blood pressure

B. Hyperglycemia in the absence of diabetes Rationale: Hyperglycemia in patients with no history of diabetes is a diagnostic criterion for sepsis. Oliguria, not diuresis, typically accompanies sepsis along with tachycardia and tachypnea.

The nurse conducts a complete physical assessment on a patient admitted with infective endocarditis. Which finding is significant?

B. Regurgitant murmur at the mitral valve area

The nurse conducts a complete physical assessment on a patient admitted with infective endocarditis. Which finding is significant? A. Respiratory rate of 18 and heart rate of 90 B. Regurgitant murmur at the mitral valve area C. Heart rate of 94 and capillary refill time of 2 seconds D. Point of maximal impulse palpable in fourth intercostal space

B. Regurgitant murmur at the mitral valve area A regurgitant murmur of the aortic or mitral valves would indicate valvular disease, which is a complication of endocarditis. All the other findings are within normal limits.

The nurse is caring for a patient who is receiving IV furosemide (Lasix) and morphine for the treatment of acute decompensated heart failure (ADHF) with severe orthopnea. Which clinical finding is the best indicator that the treatment has been effective? d. Reduced dyspnea with the head of bed at 30 degrees

Because the patient's major clinical manifestation of ADHF is orthopnea (caused by the presence of fluid in the alveoli), the best indicator that the medications are effective is a decrease in dyspnea with the head of the bed at 30 degrees

The nurse knows the client understands the teaching concerning a low-fat, lowcholesterol diet when the client selects which meal? 3. Baked chicken, baked potato, and skim milk.

Baked, broiled, or grilled meats are recommended; a plain baked potato is appropriate; and skim milk is low in fat—so this meal is appropriate for a low-fat, lowcholesterol diet.

Which finding about a patient who is receiving vasopressin (Pitressin) to treat septic shock is most important for the nurse to communicate to the health care provider? a. The patient's urine output is 18 mL/hr. b. The patient's heart rate is 110 beats/minute. c. The patient is complaining of chest pain. d. The patient's peripheral pulses are weak.

Because vasopressin is a potent vasoconstrictor, it may decrease coronary artery perfusion. The other information is consistent with the patient's diagnosis and should be reported to the health care provider but does not indicate a need for a change in therapy.

A 50-year-old man who develops third-degree heart block reports feeling chest pressure and shortness of breath. Which instructions should the nurse provide to the patient before initiating emergency transcutaneous pacing?

Before initiating transcutaneous pacing (TCP) therapy, it is important to tell the patient what to expect. The nurse should explain that the muscle contractions created by the pacemaker when the current passes through the chest wall are uncomfortable. Pacing for complete heart block will not convert the heart rhythm to normal. Overdrive pacing is used for very fast heart rates. Transcutaneous pacing is delivered through pacing pads adhered to the skin.

The nurse is teaching a class on valve replacements. Which statement identifies a disadvantage of having a biologic tissue valve replacement? 4. The valve has to be replaced frequently.

Biologic valves deteriorate and need to be replaced frequently; this is a disadvantage of them. Mechanical valves do not deteriorate and do not have to be replaced often.

Cyanosis

Blue skin & mucous membranes. Discoloration results from poor circulation or inadequate oxygenation of the blood

What does a wide QRS complex indicate?

Bundle branch block Ventricular conduction delay

Safe and Effective Care Environment 23. A patient who is on the progressive care unit develops atrial flutter, rate 150, with associated dyspnea and chest pain. Which action that is included in the hospital dysrhythmia protocol should the nurse do first? a. Obtain a 12-lead electrocardiogram (ECG). b. Notify the health care provider of the change in rhythm. c. Give supplemental O2 at 2 to 3 L/min via nasal cannula. d. Assess the patient's vital signs including oxygen saturation.

C Because this patient has dyspnea and chest pain in association with the new rhythm, the nurse's initial actions should be to address the patient's airway, breathing, and circulation (ABC) by starting with oxygen administration. The other actions also are important and should be implemented rapidly. DIF: Cognitive Level: Apply (application) REF: 793 OBJ: Special Questions: Prioritization TOP: Nursing Process: Implementation MSC:

Physiological Integrity 3. A patient has a junctional escape rhythm on the monitor. The nurse will expect the patient to have a heart rate of _____ beats/minute. a. 15 to 20 b. 20 to 40 c. 40 to 60 d. 60 to 100

C If the sinoatrial (SA) node fails to discharge, the atrioventricular (AV) node will automatically discharge at the normal rate of 40 to 60 beats/minute. The slower rates are typical of the bundle of His and the Purkinje system and may be seen with failure of both the SA and AV node to discharge. The normal SA node rate is 60 to 100 beats/minute. DIF: Cognitive Level: Understand (comprehension) REF: 797 TOP: Nursing Process: Assessment MSC:

Physiological Integrity 10. Which information will the nurse include when teaching a patient who is scheduled for a radiofrequency catheter ablation for treatment of atrial flutter? a. The procedure will prevent or minimize the risk for sudden cardiac death. b. The procedure will use cold therapy to stop the formation of the flutter waves. c. The procedure will use electrical energy to destroy areas of the conduction system. d. The procedure will stimulate the growth of new conduction pathways between the atria.

C Radiofrequency catheter ablation therapy uses electrical energy to "burn" or ablate areas of the conduction system as definitive treatment of atrial flutter (i.e., restore normal sinus rhythm) and tachydysrhythmias. All other statements regarding the procedure are incorrect. DIF: Cognitive Level: Apply (application) REF: 805 TOP: Nursing Process: Implementation MSC:

Safe and Effective Care Environment 28. A patient reports dizziness and shortness of breath for several days. During cardiac monitoring in the emergency department (ED), the nurse obtains the following electrocardiographic (ECG) tracing. The nurse interprets this heart rhythm as a. junctional escape rhythm. b. accelerated idioventricular rhythm. c. third-degree atrioventricular (AV) block. d. sinus rhythm with premature atrial contractions (PACs).

C The inconsistency between the atrial and ventricular rates and the variable P-R interval indicate that the rhythm is third-degree AV block. Sinus rhythm with PACs will have a normal rate and consistent P-R intervals with occasional PACs. An accelerated idioventricular rhythm will not have visible P waves. DIF: Cognitive Level: Apply (application) REF: 798 TOP: Nursing Process: Assessment MSC:

Physiological Integrity 29. A patient who is complaining of a "racing" heart and feeling "anxious" comes to the emergency department. The nurse places the patient on a heart monitor and obtains the following electrocardiographic (ECG) tracing. Which action should the nurse take next? a. Prepare to perform electrical cardioversion. b. Have the patient perform the Valsalva maneuver. c. Obtain the patient's vital signs including oxygen saturation. d. Prepare to give a -blocker medication to slow the heart rate.

C The patient has sinus tachycardia, which may have multiple etiologies such as pain, dehydration, anxiety, and myocardial ischemia. Further assessment is needed before determining the treatment. Vagal stimulation or -blockade may be used after further assessment of the patient. Electrical cardioversion is used for some tachydysrhythmias, but would not be used for sinus tachycardia. DIF: Cognitive Level: Analyze (analysis) REF: 793 OBJ: Special Questions: Prioritization TOP: Nursing Process: Implementation MSC:

Physiological Integrity 26. Which nursing action can the registered nurse (RN) delegate to experienced unlicensed assistive personnel (UAP) working as a telemetry technician on the cardiac care unit? a. Decide whether a patient's heart rate of 116 requires urgent treatment. b. Monitor a patient's level of consciousness during synchronized cardioversion. c. Observe cardiac rhythms for multiple patients who have telemetry monitoring. d. Select the best lead for monitoring a patient admitted with acute coronary syndrome.

C UAP serving as telemetry technicians can monitor cardiac rhythms for individuals or groups of patients. Nursing actions such as assessment and choice of the most appropriate lead based on ST segment elevation location require RN-level education and scope of practice. DIF: Cognitive Level: Analyze (analysis) REF: 15 OBJ: Special Questions: Delegation TOP: Nursing Process: Planning MSC:

A 54-year-old male patient who had bladder surgery 2 days ago develops acute decompensated heart failure (ADHF) with severe dyspnea. Which action by the nurse would be indicated first? A Perform a bladder scan to assess for urinary retention. B Restrict the patient's oral fluid intake to 500 mL per day. C Assist the patient to a sitting position with arms on the overbed table. D Instruct the patient to use pursed-lip breathing until the dyspnea subsides.

C Assist the patient to a sitting position with arms on the overbed table. The nurse should place the patient with ADHF in a high Fowler's position with the feet horizontal in the bed or dangling at the bedside. This position helps decrease venous return because of the pooling of blood in the extremities. This position also increases the thoracic capacity, allowing for improved ventilation. Pursed-lip breathing helps with obstructive air trapping but not with acute pulmonary edema. Restricting fluids takes considerable time to have an effect.

What is the priority assessment by the nurse caring for a patient receiving IV nesiritide (Natrecor) to treat heart failure? A Urine output B Lung sounds C Blood pressure D Respiratory rate

C Blood pressure Although all identified assessments are appropriate for a patient receiving IV nesiritide, the priority assessment would be monitoring for hypotension, the main adverse effect of nesiritide.

The nurse prepares to administer digoxin (Lanoxin) 0.125 mg to an 82-year-old man admitted with influenza and a history of chronic heart failure. What should the nurse assess before giving the medication? A Prothrombin time B Urine specific gravity C Serum potassium level D Hemoglobin and hematocrit

C Serum potassium level Serum potassium should be monitored because hypokalemia increases the risk for digoxin toxicity. Changes in prothrombin time, urine specific gravity, and hemoglobin or hematocrit would not require holding the digoxin dose.

Which of the following laboratory findings fits with a diagnosis of cardiogenic shock? A. Decreased liver enzymes B. Increased white blood cells C. Increased BUN & creatinine levels D. Decreased red blood cells, hemoglobin, and hematocrit

C. Increased BUN & creatinine levels Rationale: The renal hypoperfusion that accompanies cardiogenic shock results in increased BUN and creatinine levels. Impaired perfusion of the liver results in increased liver enzymes while red blood cell indices are typically normal because of relative hypovolemia. White blood cell levels do not typically rise in cardiogenic shock.

A 72-year-old man with a history of aortic stenosis is admitted to the emergency department. He reports severe left-sided chest pressure radiating to the jaw. Which medication, if ordered by the health care provider, should the nurse question?

C. Nitroglycerin

The nurse is teaching a class on arterial essential hypertension. Which modifiable risk factors would the nurse include when preparing this presentation? 2. Discuss sedentary lifestyle and smoking cessation.

Sedentary lifestyle is discouraged in clients with hypertension, and daily isotonic exercises are recommended. Smoking increases the atherosclerotic process in vessels; causes vasoconstriction of vessels; and adheres to hemoglobin, decreasing oxygen levels.

What nursing action should the nurse prioritize during the care of a patient who has recently recovered from rheumatic fever? A. Teach the patient how to manage his or her physical activity. B. Teach the patient about the need for ongoing anticoagulation. C. Teach the patient about the need for continuous antibiotic prophylaxis. D. Teach the patient about the need to maintain standard infection control procedures.

C. Teach the patient about the need for continuous antibiotic prophylaxis. Patients with a history of rheumatic fever frequently require ongoing antibiotic prophylaxis, an intervention that necessitates education. This consideration is more important than activity management in preventing recurrence. Anticoagulation is not indicated in this patient population. Standard precautions are indicated for all patients.

The nurse is caring for a patient with chronic constrictive pericarditis. Which assessment finding reflects a more serious complication of this condition?

C. jugular venous distention

The nurse is caring for a patient newly admitted with heart failure secondary to dilated cardiomyopathy. Which of the following interventions would be a priority?

C. monitor the patient's response to prescribed medications

PEA treatment

CPR Intubation IV epinephrine Treat underlying cause

Asystole treatment

CPR and ACLS Epinephrine and/or vasopressin Intubation

The client who has had a myocardial infarction is admitted to the telemetry unit from intensive care. Which referral would be most appropriate for the client?

CARDIAC REHABILITATION

Which interventions should be used for anaphylactic shock (select all that apply)? a. antibiotics b. vasodilatior c. antihistamine d. oxygen supplementation e. colloid volume expansion f. crystalloid volume expansion

CDE

The client shows ventricular fibrillation on the telemetry at the nurse's station. Which action should the telemetry nurse implement first?

Call a STAT code. - treatment choice defibrillator but not action

A patient with heart failure has a new order for captopril (Capoten) 12.5 mg PO. After administering the first dose and teaching the patient about the drug, which statement by the patient indicates that teaching has been effective? c. "I will call for help when I need to get up to use the bathroom."

Captopril can cause hypotension, especially after the initial dose, so it is important that the patient not get up out of bed without assistance until the nurse has had a chance to evaluate the effect of the first dose.

Diagnosis/ Treatment Chronic Stable Angina

Cardiac catheterization, coronary angiography, and Coronary Revascularization

CK-MB

Cardiospecific isoenzyme that is released from cells when myocardial tissue is injured

Key features:Shock

Cardiovascular manifestations: decreased cardiac output, increased pulse rate, thready pulse, decreased BP, narrowed pulse pressure, postural hypotension ¥, low central venous pressure, flat neck & hand veins in dependent positions, alow capillary refill nailbeds Respiratory - increased respiratory rate, shallow depth of respirations, increased paco2, increased paco2, cyanosis (lips & nail beds) Neuromuscular- anxiety, restlessness, increased thirst LATE- Decreased CNS activity (lethargy to coma); Generalized muscle weakness, diminished or absent deep tendon reflexes, sluggish pupillary response to light. KIDNEY- decreased urine output; increased specific gravity; sugar & acetone present in urine INTEGUMENTARY- Cool to cold; pale to mottled to cyanotic; moist, clammy; mouth dry; pastelike coating presenting GASTROINTESTINAL- Decreased motility; diminished or absent bowel sounds; nausea & vomiting; constipation.

Ischemic

Cell dysfunction or death from lack of oxygen

The client is diagnosed with pericarditis. Which are the most common signs/symptoms the nurse would expect to find when assessing the client? 4. Increased chest pain with inspiration.

Chest pain is the most common symptom of pericarditis, usually has an abrupt onset, and is aggravated by respiratory movements (deep inspiration, coughing), changes in body position, and swallowing.

A 70-year-old woman with chronic heart failure and atrial fibrillation asks the nurse why warfarin (Coumadin) has been prescribed for her to continue at home. Which response by the nurse is accurate? A "The medication prevents blood clots from forming in your heart."

Chronic heart failure causes enlargement of the chambers of the heart and an altered electrical pathway, especially in the atria.

Which assessment data would the nurse expect to find in the client diagnosed with chronic venous insufficiency? Brown discolored skin

Chronic venous insufficiency leads to chronic edema that, in turn, causes a brownish pigmentation to the skin.

The home care nurse visits a 73-year-old Hispanic woman with chronic heart failure. Which clinical manifestations, if assessed by the nurse, would indicate acute decompensated heart failure (pulmonary edema)? B Severe dyspnea and blood-streaked, frothy sputum

Clinical manifestations of pulmonary edema include anxiety, pallor, cyanosis, clammy and cold skin, severe dyspnea, use of accessory muscles of respiration, a respiratory rate > 30 breaths per minute, orthopnea, wheezing, and coughing with the production of frothy, blood-tinged sputum.

to prepare patient for placement of ECG/electrode pads...

Clip excessive hair on the chest wall with scissors. Gently rub the skin with dry gauze until slightly pink. If the skin is oily, wipe with alcohol first. If the patient is diaphoretic, apply a skin protectant before placing the electrode.

The nurse is teaching the client diagnosed with arterial occlusive disease. Which interventions should the nurse include in the teaching? Select all that apply. 1. Wash legs and feet daily in warm water. 2. Apply moisturizing cream to feet. 4. Do not wear any type of knee stocking. 5. Wear clean white cotton socks.

Cold water causes vasoconstriction and hot water may burn the client's feet; therefore, warm tepid water should be recommended. 2. Moisturizing prevents drying of the feet. 4. This will further decrease circulation to the legs. 5. Colored socks have dye and dirty socks may cause foot irritation that may lead to breaks in the skin.

The nurse is discussing the importance of exercising with a client who is diagnosed with CAD. Which statement best describes the scientific rationale for encouraging 30 minutes of walking daily to help prevent complications of atherosclerosis? 1. Exercise promotes the development of collateral circulation.

Collateral circulation is the development of blood supply around narrowed arteries; it helps prevent complications of atherosclerosis, including myocardial infarction, cerebrovascular accidents, and peripheral vascular disease. Exercise promotes the development of collateral circulation.

Troponin

Contractile proteins that are released after an MI >2.3 ng/mL

What is the correct order of the pathophysiologic steps involved in systemic inflammatory response syndrome (SIRS)? 1. Activation of coagulation cascade 2. Release of mediators 3. Increase in vascular permeability 4. Leakage of mediators and proteins into interstitial space 5. Digestion of foreign debris by white blood cells

Correct 1. Release of mediators Correct 2. Increase in vascular permeability Correct 3. Leakage of mediators and proteins into interstitial space Correct 4. Digestion of foreign debris by white blood cells Correct 5. Activation of coagulation cascade Release of mediators occurs in a patient with systemic inflammatory response syndrome (SIRS) when the inflammatory response is activated. This release increases the vascular permeability. As a result, mediators and proteins leak out of the blood vessels into the interstitial space. White blood cells digest the foreign debris and the coagulation cascade is activated. Text Reference - p. 1649

PVC treatment

Correct cause Antidysrhythmic

The nurse is caring for a 64-year-old patient admitted with mitral valve regurgitation. Which information obtained by the nurse when assessing the patient should be communicated to the health care provider immediately? a. The patient has bilateral crackles.

Crackles that are audible throughout the lungs indicate that the patient is experiencing severe left ventricular failure with pulmonary congestion and needs immediate interventions such as diuretics.

The nurse will plan discharge teaching about the need for prophylactic antibiotics when having dental procedures for which patient?. c. Patient who had a mitral valve replacement with a mechanical valve.

Current American Heart Association guidelines recommend the use of prophylactic antibiotics before dental procedures for patients with prosthetic valves to prevent infective endocarditis (IE).

A stable patient with acute decompensated heart failure (ADHF) suddenly becomes dyspneic. Before positioning the patient on the bedside, what should the nurse assess first? A Urine output B Heart rhythm C Breath sounds D Blood pressure

D Blood pressure The nurse should evaluate the blood pressure before dangling the patient on the bedside because the blood pressure can decrease as blood pools in the periphery and preload decreases. If the patient's blood pressure is low or marginal, the nurse should put the patient in the semi-Fowler's position and use other measures to improve gas exchange.

Beyond the first year after a heart transplant, the nurse knows that what is a major cause of death? A Infection B Acute rejection C Immunosuppression D Cardiac vasculopathy

D Cardiac vasculopathy Beyond the first year after a heart transplant, malignancy (especially lymphoma) and cardiac vasculopathy (accelerated CAD) are the major causes of death. During the first year after transplant, infection and acute rejection are the major causes of death. Immunosuppressive therapy will be used for posttransplant management to prevent rejection and increases the patient's risk of an infection.

Physiological Integrity 15. When analyzing the rhythm of a patient's electrocardiogram (ECG), the nurse will need to investigate further upon finding a(n) a. isoelectric ST segment. b. P-R interval of 0.18 second. c. Q-T interval of 0.38 second. d. QRS interval of 0.14 second.

D Because the normal QRS interval is 0.04 to 0.10 seconds, the patient's QRS interval of 0.14 seconds indicates that the conduction through the ventricular conduction system is prolonged. The P-R interval and Q-T interval are within normal range, and ST segment should be isoelectric (flat). DIF: Cognitive Level: Apply (application) REF: 791 TOP: Nursing Process: Assessment MSC:

Physiological Integrity 6. A patient has a normal cardiac rhythm and a heart rate of 72 beats/minute. The nurse determines that the P-R interval is 0.24 seconds. The most appropriate intervention by the nurse would be to a. notify the health care provider immediately. b. give atropine per agency dysrhythmia protocol. c. prepare the patient for temporary pacemaker insertion. d. document the finding and continue to monitor the patient.

D First-degree atrioventricular (AV) block is asymptomatic and requires ongoing monitoring because it may progress to more serious forms of heart block. The rate is normal, so there is no indication that atropine is needed. Immediate notification of the health care provider about an asymptomatic rhythm is not necessary. DIF: Cognitive Level: Apply (application) REF: 798 TOP: Nursing Process: Implementation MSC:

Physiological Integrity 17. Which laboratory result for a patient with multifocal premature ventricular contractions (PVCs) is most important for the nurse to communicate to the health care provider? a. Blood glucose 243 mg/dL b. Serum chloride 92 mEq/L c. Serum sodium 134 mEq/L d. Serum potassium 2.9 mEq/L

D Hypokalemia increases the risk for ventricular dysrhythmias such as PVCs, ventricular tachycardia, and ventricular fibrillation. The health care provider will need to prescribe a potassium infusion to correct this abnormality. Although the other laboratory values also are abnormal, they are not likely to be the etiology of the patient's PVCs and do not require immediate correction. DIF: Cognitive Level: Apply (application) REF: 799 OBJ: Special Questions: Prioritization TOP: Nursing Process: Assessment MSC:

Physiological Integrity 4. The nurse obtains a rhythm strip on a patient who has had a myocardial infarction and makes the following analysis: no visible P waves, P-R interval not measurable, ventricular rate 162, R-R interval regular, and QRS complex wide and distorted, QRS duration 0.18 second. The nurse interprets the patient's cardiac rhythm as a. atrial flutter. b. sinus tachycardia. c. ventricular fibrillation. d. ventricular tachycardia.

D The absence of P waves, wide QRS, rate >150 beats/minute, and the regularity of the rhythm indicate ventricular tachycardia. Atrial flutter is usually regular, has a narrow QRS configuration, and has flutter waves present representing atrial activity. Sinus tachycardia has P waves. Ventricular fibrillation is irregular and does not have a consistent QRS duration. DIF: Cognitive Level: Apply (application) REF: 794 TOP: Nursing Process: Assessment MSC:

Physiological Integrity 7. A patient who was admitted with a myocardial infarction experiences a 45-second episode of ventricular tachycardia, then converts to sinus rhythm with a heart rate of 98 beats/minute. Which of the following actions should the nurse take next? a. Immediately notify the health care provider. b. Document the rhythm and continue to monitor the patient. c. Perform synchronized cardioversion per agency dysrhythmia protocol. d. Prepare to give IV amiodarone (Cordarone) per agency dysrhythmia protocol.

D The burst of sustained ventricular tachycardia indicates that the patient has significant ventricular irritability, and antidysrhythmic medication administration is needed to prevent further episodes. The nurse should notify the health care provider after the medication is started. Defibrillation is not indicated given that the patient is currently in a sinus rhythm. Documentation and continued monitoring are not adequate responses to this situation. DIF: Cognitive Level: Apply (application) REF: 799 TOP: Nursing Process: Implementation MSC:

Physiological Integrity 22. The nurse has received change-of-shift report about the following patients on the progressive care unit. Which patient should the nurse see first? a. A patient who is in a sinus rhythm, rate 98, after having electrical cardioversion 2 hours ago b. A patient with new onset atrial fibrillation, rate 88, who has a first dose of warfarin (Coumadin) due c. A patient with second-degree atrioventricular (AV) block, type 1, rate 60, who is dizzy when ambulating d. A patient whose implantable cardioverter-defibrillator (ICD) fired two times today who has a dose of amiodarone (Cordarone) due

D The frequent firing of the ICD indicates that the patient's ventricles are very irritable, and the priority is to assess the patient and administer the amiodarone. The other patients may be seen after the amiodarone is administered. DIF: Cognitive Level: Analyze (analysis) REF: 803 OBJ: Special Questions: Prioritization; Multiple Patients TOP: Nursing Process: Implementation MSC:

Physiological Integrity 19. A patient's cardiac monitor shows sinus rhythm, rate 64. The P-R interval is 0.18 seconds at 1:00 AM, 0.22 seconds at 2:30 PM, and 0.28 seconds at 4:00 PM. Which action should the nurse take next? a. Place the transcutaneous pacemaker pads on the patient. b. Administer atropine sulfate 1 mg IV per agency dysrhythmia protocol. c. Document the patient's rhythm and assess the patient's response to the rhythm. d. Call the health care provider before giving the next dose of metoprolol (Lopressor).

D The patient has progressive first-degree atrioventricular (AV) block, and the -blocker should be held until discussing the medication with the health care provider. Documentation and assessment are appropriate but not fully adequate responses. The patient with first-degree AV block usually is asymptomatic, and a pacemaker is not indicated. Atropine is sometimes used for symptomatic bradycardia, but there is no indication that this patient is symptomatic. DIF: Cognitive Level: Apply (application) REF: 798 OBJ: Special Questions: Prioritization TOP: Nursing Process: Implementation MSC:

Physiological Integrity 20. A patient develops sinus bradycardia at a rate of 32 beats/minute, has a blood pressure (BP) of 80/42 mm Hg, and is complaining of feeling faint. Which actions should the nurse take next? a. Recheck the heart rhythm and BP in 5 minutes. b. Have the patient perform the Valsalva maneuver. c. Give the scheduled dose of diltiazem (Cardizem). d. Apply the transcutaneous pacemaker (TCP) pads.

D The patient is experiencing symptomatic bradycardia, and treatment with TCP is appropriate. Continued monitoring of the rhythm and BP is an inadequate response. Calcium channel blockers will further decrease the heart rate, and the diltiazem should be held. The Valsalva maneuver will further decrease the rate. DIF: Cognitive Level: Apply (application) REF: 804 OBJ: Special Questions: Prioritization TOP: Nursing Process: Implementation MSC:

Psychosocial Integrity 11. After providing a patient with discharge instructions on the management of a new permanent pacemaker, the nurse knows that teaching has been effective when the patient states a. "I will avoid cooking with a microwave oven or being near one in use." b. "It will be 1 month before I can take a bath or return to my usual activities." c. "I will notify the airlines when I make a reservation that I have a pacemaker." d. "I won't lift the arm on the pacemaker side up very high until I see the doctor."

D The patient is instructed to avoid lifting the arm on the pacemaker side above the shoulder to avoid displacing the pacemaker leads. The patient should notify airport security about the presence of a pacemaker before going through the metal detector, but there is no need to notify the airlines when making a reservation. Microwave oven use does not affect the pacemaker. The insertion procedure involves minor surgery that will have a short recovery period. DIF: Cognitive Level: Apply (application) REF: 805 TOP: Nursing Process: Evaluation MSC:

Physiological Integrity 2. The nurse needs to quickly estimate the heart rate for a patient with a regular heart rhythm. Which method will be best to use? a. Count the number of large squares in the R-R interval and divide by 300. b. Print a 1-minute electrocardiogram (ECG) strip and count the number of QRS complexes. c. Calculate the number of small squares between one QRS complex and the next and divide into 1500. d. Use the 3-second markers to count the number of QRS complexes in 6 seconds and multiply by 10.

D This is the quickest way to determine the ventricular rate for a patient with a regular rhythm. All the other methods are accurate, but take longer. DIF: Cognitive Level: Apply (application) REF: 789-790 TOP: Nursing Process: Assessment MSC:

The nurse is administering a dose of digoxin (Lanoxin) to a patient with heart failure (HF). The nurse would become concerned with the possibility of digitalis toxicity if the patient reported which symptom(s)? A Muscle aches B Constipation C Pounding headache D Anorexia and nausea

D Anorexia and nausea Anorexia, nausea, vomiting, blurred or yellow vision, and cardiac dysrhythmias are all signs of digitalis toxicity. The nurse would become concerned and notify the health care provider if the patient exhibited any of these symptoms.

A male patient with a long-standing history of heart failure has recently qualified for hospice care. What measure should the nurse now prioritize when providing care for this patient? A Taper the patient off his current medications. B Continue education for the patient and his family. C Pursue experimental therapies or surgical options. D Choose interventions to promote comfort and prevent suffering.

D Choose interventions to promote comfort and prevent suffering. The central focus of hospice care is the promotion of comfort and the prevention of suffering. Patient education should continue, but providing comfort is paramount. Medications should be continued unless they are not tolerated. Experimental therapies and surgeries are not commonly used in the care of hospice patients.

After having an MI, the nurse notes the patient has jugular venous distention, gained weight, developed peripheral edema, and has a heart rate of 108/minute. What should the nurse suspect is happening? A ADHF B Chronic HF C Left-sided HF D Right-sided HF

D Right-sided HF An MI is a primary cause of heart failure. The jugular venous distention, weight gain, peripheral edema, and increased heart rate are manifestations of right-sided heart failure.

The elderly client has coronary artery disease. Which question should the nurse ask the client during the client teaching? 3. "Are you sexually active?"

Sexual activity is a risk factor for angina resulting from coronary artery disease. The client's being elderly should not affect the nurse's assessment of the client's concerns about sexual activity.

When caring for a critically ill patient who is being mechanically ventilated, the nurse will astutely monitor for which of the following clinical manifestations of multiple organ dysfunction syndrome (MODS)? A. Increased gastrointestinal (GI) motility B. Increased serum albumin C. Decreased blood urea nitrogen (BUN)/creatinine ratio D. Decreased respiratory compliance

D. Decreased respiratory compliance Rationale: Clinical manifestations of MODS include symptoms of respiratory distress, signs and symptoms of decreased renal perfusion, decreased serum albumin and prealbumin, decreased GI motility, acute neurologic changes, myocardial dysfunction, disseminated intravascular coagulation (DIC), and changes in glucose metabolism.

Which assessment finding in a patient who is admitted with infective endocarditis (IE) is most important to communicate to the health care provider? b. Sudden onset right flank pain

Sudden onset of flank pain indicates possible embolization to the kidney and may require diagnostic testing such as a renal arteriogram and interventions to improve renal perfusion.

Priority nursing management for a patient with myocarditis includes interventions related to:

D. oxygenation and ventillation

TREATMENT OF DYSRHYTHMIAS

Defibrillation, the passage of a DC electrical shock through the heart to depolarize the cells of the myocardium, is the most effective method of terminating VF and pulseless VT.

Concentric Hypertrophy

Disproportionate increase in the thickness, seen in people suffering from hypertension

The nurse obtains a health history from a 65-year-old patient with a prosthetic mitral valve who has symptoms of infective endocarditis (IE). Which question by the nurse is most appropriate? "Have you had dental work done recently?

Dental procedures place the patient with a prosthetic mitral valve at risk for infective endocarditis (IE).

Eccentric Hypertrophy

Disproportionate ventricular dilation, decrease in the thickness of the ventricular wall

Along with persistent, crushing chest pain, which signs/symptoms would make the nurse suspect that the client is experiencing a myocardial infarction?

Diaphoresis and cool clammy skin. - these are systemic reaction

Cardiogenic

Direct pump failure

What can be a source of QRS Interval variation?

Disturbance in conduction in bundle branches or in ventricles

What can cause variation in the PR interval?

Disturbance in conduction usually in AV node, bundle of His, or bundle branches but can be in atria as well

What can cause variation in the QT interval?

Disturbances usually affecting repolarization more than depolarization and caused by drugs, electrolyte imbalances, and changes in heart rate

What can cause a variation in the T wave?

Disturbances usually caused by electrolyte imbalances, ischemia, or infarction

What can cause a variation in ST segment?

Disturbances usually caused by ischemia, injury, or infarction

A patient with a recent diagnosis of heart failure has been prescribed furosemide (Lasix) in an effort to physiologically do what for the patient? A Reduce preload.

Diuretics such as furosemide are used in the treatment of HF to mobilize edematous fluid, reduce pulmonary venous pressure, and reduce preload.

A patient with a recent diagnosis of heart failure has been prescribed furosemide (Lasix) in an effort to physiologically do what for the patient? a. Reduce preload.

Diuretics such as furosemide are used in the treatment of HF to mobilize edematous fluid, reduce pulmonary venous pressure, and reduce preload.

The nurse is discussing the importance of exercise with the client diagnosed with coronary artery disease. Which intervention should the nurse implement?

Do not walk outside if it is less than 40˚F. - when it is cold outside, it causes vasoconstriction and lower oxygen to heart muscles - perform isotonic exercises not isometric - walk at least 30 minutes not 15 minutes for 3-4 times - wear supportive shoes not sandals!

You are caring for a patient with ADHF who is receiving IV dobutamine (Dobutrex). You know that this drug is ordered because it (select all that apply): c. improves contractility e. works on the B1-receptors in the heart.

Dobutamine (Dobutrex) has a positive chronotropic effect and increases heart rate and improves contractility. It is a selective β-adrenergic agonist and works primarily on the β1-adrenergic receptors in the heart. It is frequently used in the short-term management of acute decompensated heart failure (ADHF).

What should the nurse recognize as an indication for the use of dopamine (Intropin) in the care of a patient with heart failure? B Hypotension and tachycardia

Dopamine is a β-adrenergic agonist whose inotropic action is used for treatment of severe heart failure accompanied by hemodynamic instability.

What should the nurse recognize as an indication for the use of dopamine (Intropin) in the care of a patient with heart failure? b. Hypotension and tachycardia

Dopamine is a β-adrenergic agonist whose inotropic action is used for treatment of severe heart failure accompanied by hemodynamic instability.

Evaluation of Dysrhythmias

Dysrhythmias result from various abnormalities and disease states, and the cause of a dysrhythmia influences the treatment.

A client is being seen in the clinic to R/O mitral valve stenosis. Which assessment data would be most significant? 1. The client complains of shortness of breath when walking.

Dyspnea on exertion (DOE) is typically the earliest manifestation of mitral valve stenosis.

A patient is admitted to the hospital with possible acute pericarditis. The nurse should plan to teach the patient about the purpose of a. echocardiography.

Echocardiograms are useful in detecting the presence of the pericardial effusions associated with pericarditis

Locally produced vasoactive substances

Endothelins (function as vasoconstrictors) & inflammatory mediators

Multiple organ dysfunction syndrome (MODS ) "may occur independently" from SIRS. (T/F)

False "results"

Hypovolemic shock from relative hypovolemia may occur with diabetes insipidus. (T /F)

False, absolute hypovolemia

S wave:

First negative (downward) deflection after the R wave. Not usually measured

R wave:

First positive (upward) deflection in the QRS complex. Not usually measured

Monounsaturated fat

Fish oil, nuts, olives May help lower your blood cholesterol level when used in place of saturated fat in your diet.

Which actions should the surgical scrub nurse take to prevent from personally developing a DVT? 2. Flex the leg muscles and change the leg positions frequently.

Flexing the leg muscles and changing positions assist the blood to return to the heart and move out of the peripheral vessels.

The nurse is monitoring the ECGs of several patients on a cardiac telemetry unit. The patients are directly visible to the nurse, and all of the patients are observed to be sitting up and talking with visitors. Which patient's rhythm would require the nurse to take immediate action?

Frequent premature ventricular contractions (PVCs) (greater than 1 every 10 beats) may reduce the cardiac output and precipitate angina and heart failure, depending on their frequency. Because PVCs in CAD or acute MI indicate ventricular irritability, the patient's physiologic response to PVCs must be monitored. Frequent PVCs most likely must be treated with oxygen therapy, electrolyte replacement, or antidysrhythmic agents.

The nurse is discharging a client diagnosed with DVT from the hospital. Which discharge instructions should be provided to the client? 3. Avoid green leafy vegetables and notify the HCP of red or brown urine.

Green leafy vegetables contain vitamin K, which is the antidote for warfarin. These foods will interfere with the action of warfarin. Red or brown urine may indicate bleeding.

How are Paroxysmal Supraventricular Tachycardia (PSVTs) manifested? (5)

HR is 150-220 beats/minute (add for clarification) HR > 180 leads to decreased cardiac output and stroke volume Hypotension Dyspnea Angina

Third Degree (complete heart block) cause

Heart disease, systemic diseases, drugs

The nurse and an unlicensed assistive personnel (UAP) are caring for four clients on a telemetry unit. Which nursing task would be best for the nurse to delegate to the UAP?

Help position the client who is having a portable x-ray done. - unstable patient from ICU is not delegated - teaching is not delegated

o

High ventricular rates (over 100/min) and the loss of the atrial "kick" (atrial contraction reflected by a sinus P wave) can decrease CO and cause serious consequences such as chest pain and HF.

A patient has recently started on digoxin (Lanoxin) in addition to furosemide (Lasix) and captopril (Capoten) for the management of heart failure. Which assessment finding by the home health nurse is a priority to communicate to the health care provider? c. Serum potassium level 3.0 mEq/L after 1 week of therapy

Hypokalemia can predispose the patient to life-threatening dysrhythmias (e.g., premature ventricular contractions), and potentiate the actions of digoxin and increase the risk for digoxin toxicity, which can also cause life-threatening dysrhythmias.

A patient with chronic HF and atrial fibrillation is treated with a digitalis glycoside and a loop diuretic. To prevent possible complications of this combination of drugs, what does the nurse need to do (select all that apply)? a. Monitor serum potassium levels b. teach the patient how to take a pulse rate.

Hypokalemia, which can be caused by the use of potassium-depleting diuretics (e.g., thiazides, loop diuretics), is one of the most common causes of digitalis toxicity. Low serum levels of potassium enhance the action of digitalis, causing a therapeutic dose to achieve toxic levels. Hypokalemia can also precipitate dysrhythmias. Monitoring the serum potassium levels of patients receiving digitalis preparations and potassium-depleting diuretics is essential. Patients taking digitalis preparations should be taught how to measure their pulse rate because bradycardia and atrioventricular blocks are late signs of digitalis toxicity. In addition, patients should know what pulse rate would necessitate a call to the health care provider.

After receiving report on the following patients, which patient should the nurse assess first? b. Patient with acute aortic regurgitation whose blood pressure is 86/54 mm Hg

Hypotension in patients with acute aortic regurgitation may indicate cardiogenic shock.

Risk Factors for Shock (older adult)

Hypovolemic- diuretic therapy, diminished thirst reflex, immobility, use of aspirin-containing products, use of therapies (Ginkgo biloba); anticoagulant therapy Cardiogenic- Diabetes mellitus; presence of cardiomyopathies Distributive: Diminished immune response; reduced skin integrity; presence of cancer; peripheral neuropathy; strokes; institutionalization (hospital or extended care); malnutrition; anemia Obstructive- Pulmonary hypertension; presence of cancer

o

ICD delivers a defibrillator shock or acts as a pacemaker as needed. Education of the patient and caregiver is of extreme importance.

Adaptive Responses & Events During Hypovolemic

INITIAL STAGE- Decrease in baseline MAP of 5-10. Increased sympathetic stimulation- mild vasoconstriction; increased HR. NONPROGRESSIVE STAGE- Decrease in MAP of 10-15 from patient's baseline value. Continued sympathetic stimulation- moderate vasoconstriction; increased HR; decreased pulse pressure; Chemical compensation- renin aldosterone & antidiuretic hormone secretion, increased vasoconstriction; decreased urine output; stimulation of third reflex; some anaerobic metabolism in nonvital organs- mild acidosis & mild hyperkalemia PROGRESSIVE STAGE-Decrease in MAP >20 from patients baseline value; anoxia of nonvital organs; overall metabolism is anaerobic-moderate acidosis, moderate hyperkalemia, tissue, ischemia REFRACTORY STAGE- severe tissue hypoxia, with ischemia & necrosis; release of myocardial depressant factor from pancreas; build up of toxic metabolites. Death.

A 38-year-old teacher who reported dizziness and shortness of breath while supervising recess is admitted with a dysrhythmia. Which medication, if ordered, requires the nurse to carefully monitor the patient for asystole?

IV adenosine (Adenocard) is the first drug of choice to convert supraventricular tachycardia to a normal sinus rhythm. Adenosine is administered IV rapidly (over 1 or 2 seconds) followed by a rapid, normal saline flush. The nurse should monitor the patient's ECG continuously because a brief period of asystole after adenosine administration is common and expected. Atropine sulfate increases heart rate, whereas lanoxin and metoprolol slow the heart rate.

Which intervention should the nurse implement when defibrillating a client who is in ventricular fibrillation? 4. Shout "all clear" prior to defibrillating the client.

If any member of the health-care team is touching the client or the bed during defibrillation, that person could possibly be shocked. Therefore, the nurse should shout "all clear."

The telemetry nurse is unable to read the telemetry monitor at the nurse's station. Which intervention should the telemetry nurse implement first?. 3. Contact the client on the client call system.

If the client answers the call light and is not experiencing chest pain, then there is probably a monitor artifact, which is not a life-threatening emergency. After talking with the client, send a nurse to the room to check the monitor.

Dextran - Nursing Implications

Increases risk of bleeding Important to monitor pt for allergic reactions and acute renal failure.

Activation of Neurohumoral

Influences such as the sympathetic nervous system, may lead to problems. Sympathetic nervous system activity will increase & can cause problems, can lead to tachycardia, oxygen demand will increase

What is NSS & LR used for? (What type of shock)

Initial volume replacement in most types of shock

MI CV clinical manifestations

Initially, increase in BP and HR then decrease in BP. crackles suggest left ventricular dysfunction, jugular vein distension and peripheral edema suggest right, abnormal heart sounds (s3 or s2 or new murmur)

The client with coronary artery disease is prescribed a Holter monitor. Which intervention should the nurse implement?

Instruct client to keep a diary of activity, especially when having chest pain. - important, so physician can compare ECG recordings with different level of activity - it should not be removed for any reason - does not affect meds regimen - perform all activities as usual while wearing Holter

Chronic Stable Angina

Intermittent chest pain occurring over a long period with the same pattern of onset, duration & intensity of symptoms

Which assessment data would cause the nurse to suspect the client has atherosclerosis? 3. Intermittent claudication.

Intermittent claudication is a sign of generalized atherosclerosis and is a marker of atherosclerosis.

Sinus tachycardia- etiology

It is associated with stressors such as exercise, fever, pain, hypotension, hypovolemia, anemia, hypoxia, hypoglycemia, myocardial ischemia, heart failure (HF), hyperthyroidism, anxiety, and fear. It can also be an effect of certain drugs.Treatment is based on the underlying cause.

The client is experiencing multifocal premature ventricular contractions. Which antidysrhythmic medication would the nurse expect the health-care provider to order for this client?

LIDOCAINE - suppresses ventricular ectopy and is the drug of choice for ventricular dysrhythmias ATROPINE - decreases vagal stimulation, for asystole DIGOXIN - slows heart rate, increases contractility, for atrial fibrillation ADENOSINE - choice drug for supraventricular tachycardia

A 21-year-old woman is scheduled for percutaneous transluminal balloon valvuloplasty to treat mitral stenosis. Which information should the nurse include when explaining the advantages of valvuloplasty over valve replacement to the patient? c. Lifelong anticoagulant therapy will be needed after mechanical valve replacement.

Long-term anticoagulation therapy is needed after mechanical valve replacement, and this would restrict decisions about career and childbearing in this patient

The client is admitted for surgical repair of an 8-cm abdominal aortic aneurysm. Which sign/symptom would make the nurse suspect the client has an expanding AAA? 1. Complaints of low back pain.

Low back pain is present because of the pressure of the aneurysm on the lumbar nerves; this is a serious symptom, usually indicating that the aneurysm is expanding rapidly and about to rupture.

C-reactive protein

Marker of inflammation. May have value in predicting risk of cardiac disease.

Hespan - Nursing Implications

May be 50% less costly than albumin Use cautiously in pts w/heart failure, renal failure, or bleeding disorders (due to antiplatelet effect).

pr interval

Measured from beginning of P wave to beginning of QRS complex. Represents time taken for impulse to spread through the atria, AV node and bundle of His, bundle branches, and Purkinje fibers, to a point immediately preceding ventricular contraction. Time: 0.12-0.20 Disturbance in conduction usually in AV node, bundle of His, or bundle branches but can be in atria as well

QRS Interval (ventricles)

Measured from beginning to end of QRS complex. Represents time taken for depolarization (contraction) of both ventricles (systole). <0.12 Disturbance in conduction in bundle branches or in ventricles

ST Segment (ventricles)

Measured from the S wave of the QRS complex to the beginning of the T wave. Represents the time between ventricular depolarization and repolarization (diastole). Should be isoelectric (flat). Time: 0.12 Disturbances (e.g., elevation, depression) usually caused by ischemia, injury, or infarction

Capillary Leak Syndrome

Mediators change blood vessel integrity & allow fluid to shift from the vascular space to the interstitial tissues

A patient in the intensive care unit with acute decompensated heart failure (ADHF) complains of severe dyspnea and is anxious, tachypneic, and tachycardic. All of the following medications have been ordered for the patient. The nurse's priority action will be to a. give IV morphine sulfate 4 mg.

Morphine improves alveolar gas exchange, improves cardiac output by reducing ventricular preload and afterload, decreases anxiety, and assists in reducing the subjective feeling of dyspnea. Diazepam may decrease patient anxiety, but it will not improve the cardiac output or gas exchange.

A patient admitted with heart failure appears very anxious and complains of shortness of breath. Which nursing actions would be appropriate to alleviate this patient's anxiety (select all that apply)? A Administer ordered morphine sulfate. B Position patient in a semi-Fowler's position. D Instruct patient on the use of relaxation techniques. E Use a calm, reassuring approach while talking to patient.

Morphine sulfate reduces anxiety and may assist in reducing dyspnea. The patient should be positioned in semi-Fowler's position to improve ventilation that will reduce anxiety. Relaxation techniques and a calm reassuring approach will also serve to reduce anxiety.

Physical Assessment/ Clinical Manifestation

Most manifestations of hypovolemic shock are caused by the changes resulting from compensatory efforts. -Compensatory mechanisms : are physiologic responses that try to keep an adequate blood flow to vital organs. -Cardiovascular changes- start with decreased MAP, leading to compensatory responses. Earliest sign!!! Pulse rate increases to keep cardiac output & MAP at normal levels, even though the actual stroke volume per beat is decreased. Increased HR- earliest manifestation of shock! Peripheral pulses difficult to palpate & are blocked with light pressure. -Systolic pressure decreases as shock progresses & cardiac output decreases. -Pulse ox- 90-95%- nonprogressive stage of shock; 75-80% - progressive stage of shock. -Respiratory rate increases during hypovolemic- to ensure that oxygen intake is increased so that it can be delivered to critical tissues. -Kidney & urinary changes occur with shock to compensate for decreased MAP by saving body water through decreased filtration & increased water reabsorption. Decreased urine output is a sensitive indicator of early shock! -Skin changes- reduced skin perfusion. This allows more blood to circulate to the vital organs which can't tolerate low oxygen levels. -Central Nervous System Changes- 1st manifestation as thirst. Thirst is caused by stimulation of the thirst centers in the brain in response to decreased blood volume. -Skeletal Muscle Changes- Muscle weakness & pain in response to tissue hypoxia & anaerobic metabolism.

The nurse plans discharge teaching for a patient with chronic heart failure who has prescriptions for digoxin (Lanoxin) and hydrochlorothiazide (HydroDIURIL). Appropriate instructions for the patient include c. notify the health care provider if nausea develops.

Nausea is an indication of digoxin toxicity and should be reported so that the provider can assess the patient for toxicity and adjust the digoxin dose, if necessary.

Unstable Angina Charateristics

New-onset angina, chronic stable angina that increases in frequency, duration, or severity, occurs at rest or with minimal exertion and lasts more than 10 minutes

The intensive care department nurse is assessing the client who is 12 hours post-myocardial infarction. The nurse assesses an S3 heart sound. Which intervention should the nurse implement?

Notify the health-care provider immediately. - means left ventricular failure, should be reported ASAP

Which assessment data would warrant immediate intervention in the client diagnosed with arterial occlusive disease? 3. The client has numbness and tingling.

Numbness and tingling are paresthesia, which is a sign of a severely decreased blood supply to the lower extremities.

The nurse performs discharge teaching for a 74-year-old woman with an implantable cardioverter-defibrillator. Which statement by the patient indicates to the nurse that further teaching is needed?

Patients should be taught to report any signs of infection at incision site (e.g., redness, swelling, drainage) or fever to their primary care provider immediately. Teach the patient to inform airport security of presence of ICD because it may set off the metal detector. If hand-held screening wand is used, it should not be placed directly over the ICD. Teach the patient to avoid standing near antitheft devices in doorways of stores and public buildings, and to walk through them at a normal pace. Caregivers should learn cardiopulmonary resuscitation (CPR).

CARDIOGENIC SHOCK

Occurs when the actual heart muscle is unhealthy & pumping & is directly impaired. Myocardial infarction- most common cause of direct pump failure.

Distributive-septic

Older adult with UTI Patient with pneumonia Older adult with sacral pressure ulcers Patient with MRSA infection

The male client is diagnosed with Guillain Barré syndrome (GB) and is in the intensive care unit on a ventilator. Which cardiovascular rationale explains implementing passive range of motion (ROM) exercises? 4. They help to prevent DVTs by movement of the blood through the veins.

One reason for performing range of motion exercises is to assist the blood vessels in the return of blood to the heart, preventing DVT.

The client is diagnosed with an abdominal aortic aneurysm. Which statement would the nurse expect the client to make during the admission assessment? 2. "I don't have any abdominal pain or any type of problems."

Only about two-fifths of clients with AAA have symptoms; the remainder are asymptomatic.

Where do junctional dysrhythmias originate?

Originate at AV node (SA node fails to fire or impulse blocked at AV node) --> AV node become pacer --> retrograde transmission to atria

The male client diagnosed with essential hypertension has been prescribed an alphaadrenergic blocker. Which intervention should the nurse discuss with the client? Change position slowly when going from lying to sitting position.

Orthostatic hypotension may occur when the blood pressure is decreasing and may lead to dizziness and light-headedness so the client should change position slowly.

The health-care provider has ordered an angiotensin-converting enzyme (ACE) inhibitor for the client diagnosed with congestive heart failure. Which discharge instructions should the nurse include? 2. Teach the client how to prevent orthostatic hypotension.

Orthostatic hypotension may occur with ACE inhibitors as a result of vasodilation. Therefore, the nurse should instruct the client to rise slowly and sit on the side of the bed until equilibrium is restored.

What triggers Paroxysmal Supraventricular Tachycardia (PSVT)?

PAC triggers a run of repeated premature beats

The client is being admitted with Coumadin (warfarin), an anticoagulant, toxicity. Which laboratory data should the nurse monitor?. 3. International Normalized Ratio (INR).

PT/INR is a test to monitor warfarin (Coumadin) action in the body.

The nurse suspects cardiac tamponade in a patient who has acute pericarditis. To assess for the presence of pulsus paradoxus, the nurse should a. note when Korotkoff sounds are auscultated during both inspiration and expiration.

Pulsus paradoxus exists when there is a gap of greater than 10 mm Hg between when Korotkoff sounds can be heard during only expiration and when they can be heard throughout the respiratory cycle.

Second-Degree AV block Type 2 (Mobitz II) treatment

Pacemaker

Third Degree (complete heart block) treatment

Pacemaker Drugs to increase heart rate if needed while awaiting pacing

PAC Manifestation

Palpitations Heart "skips a beat"

Which signs/symptoms should the nurse assess in any client who has a long-term valvular heart disease? Select all that apply. 1. Paroxysmal nocturnal dyspnea. 2. Orthopnea. 3. Cough.

Paroxysmal nocturnal dyspnea is a sudden attack of respiratory distress usually occurring at night because of the reclining position and occurs in valvular disorders. This is an abnormal condition in which a Client must sit or stand to breathe comfortably and occurs in valvular disorders. Coughing occurs when the client with long-term valvular disease has difficulty breathing when walking or performing any type of activity.

A patient who has chronic heart failure tells the nurse, "I was fine when I went to bed, but I woke up in the middle of the night feeling like I was suffocating!" The nurse will document this assessment finding as c. paroxysmal nocturnal dyspnea.

Paroxysmal nocturnal dyspnea is caused by the reabsorption of fluid from dependent body areas when the patient is sleeping and is characterized by waking up suddenly with the feeling of suffocation.

Patient in Hypovolemic Shock!

Patent Airway IV catheter, or maintain an established one Administer oxygen Elevate patient's feet, keeping head flat or elevated to no more than 30 degree angle Examine for overt bleeding If overt bleeding, apply direct pressure Drugs are prescribed Increase rate of IV fluid delivery DO NOT LEAVE PATIENT

Cardiogenic

Patient who had a myocardial infarction

Hypovolemic

Patient who had a ruptured aortic aneurysm. Patient with ruptured spleen resulting from trauma Patient with dehydration Patient with diabetes insipidus Patient receiving excessive diuretic therapy Patient who has cancer of the head & neck, with a nasogastric tube Patient receiving heparin therapy

Distributive-Chemical

Patient who had an allergic reaction to a medication Patient with insect bites

The nurse establishes the nursing diagnosis of ineffective health maintenance related to lack of knowledge regarding long-term management of rheumatic fever when a 30-year-old recovering from rheumatic fever without carditis says which of the following? d. "I will be immune to further episodes of rheumatic fever after this infection."

Patients with a history of rheumatic fever are more susceptible to a second episode.

What nursing action should the nurse prioritize during the care of a patient who has recently recovered from rheumatic fever? C. Teach the patient about the need for continuous antibiotic prophylaxis.

Patients with a history of rheumatic fever frequently require ongoing antibiotic prophylaxis, an intervention that necessitates education

The nurse performs discharge teaching for a 68-year-old man who is newly diagnosed with infective endocarditis with a history of IV substance abuse. Which statement by the patient indicates to the nurse that teaching was successful? B. "I will inform my dentist about my hospitalization for infective endocarditis."

Patients with infective endocarditis should inform their dental providers of their health history.

Which statement by a patient with restrictive cardiomyopathy indicates that the nurse's discharge teaching about self-management has been most effective? c. "I will take antibiotics when my teeth are cleaned at the dental office."

Patients with restrictive cardiomyopathy are at risk for infective endocarditis and should use prophylactic antibiotics for any procedure that may cause bacteremia.

Ejection Fraction

Percentage of blood pumped out of ventricles each contraction ~ Normal: 55%-70% of the blood within the ventricles are going to be ejected ~ Changes with systolic and diastolic ventricular dysfunction ~ Used for the assessment of systolic ventricular dysfunction

While admitting a patient with pericarditis, the nurse will assess for what manifestations of this disorder? A. Pulsus paradoxus

Pericarditis can lead to cardiac tamponade, an emergency situation. Pulsus paradoxus greater than 10 mm Hg is a sign of cardiac tamponade that should be assessed at least every 4 hours in a patient with pericarditis.

Atrial flutter treatment

Pharmacologic agent Electrical cardioversion Radiofrequency ablation

Manifestation of Heart Failure

Physiologic effects of the impaired pumping ability of the heart ~ Respiratory manifestation: acute pulmonary edema, Cheyne strokes ~ Fatigue, weakness & mental confusion ~ Fluid retention & edema ~ Cachexia ~ Cyanosis ~ Arrhythmias & sudden cardiac death

Short acting and long acting nitrites

Promote peripheral vasodialation, decreasing preload and afterload. Promote coronary artery vasodilation. May prevent or control coronary vasospasm

The client diagnosed with essential hypertension is taking a loop diuretic daily. Which assessment data would require immediate intervention by the nurse? 2. The client has a weight gain of 2 kg within 1-2 days.

Rapid weight gain—for example, 2 kg in 1-2 days—indicates that the loop diuretic is not working effectively; 2 kg equals 4.4 lbs; 1 L of fluid weighs l kg.

The nurse is admitting a patient with possible rheumatic fever. Which question on the admission health history will be most pertinent to ask? b. "Have you had a recent sore throat?"

Rheumatic fever occurs as a result of an abnormal immune response to a streptococcal infection.

The nurse is discussing discharge teaching with the client who is three (3) days postoperative abdominal aortic aneurysm repair. Which discharge instructions should the nurse include when teaching the client? 1. Notify HCP of any redness or irritation of incision.

Redness or irritation of the incision indicates infection and should be reported immediately to the HCP.

Reducing risk factors for CAD (elevated serum lipids)

Reduce total fat intake, reduce sat. fat intake, take prescribed drugs, engage in physical activity

The client is admitted to the medical unit to rule out carditis. Which question should the nurse ask the client during the admission interview to support this diagnosis? 2. "Did you have rheumatic fever as a child?"

Rheumatic fever, a systemic inflammatory disease caused by an abnormal immune response to pharyngeal infection by group A beta-hemolytic streptococci, causes carditis in about 50% of the people.

Blood/Blood Products Packed RBCs/Fresh frozen plasma/Platelets - Mechanism of Action

Replaces blood loss, increases oxygen-carrying capability. Replaces coagulation factors. Helps control bleeding caused by thrombocytopenia.

Left-Sided Heart Failure

Represents failure of the left heart to move from pulmonary circulation into the systemic circulation ~ Blood backs up in the pulmonary circulation

T Wave (ventricles)

Represents time for ventricular repolarization. Should be upright. Time: 0.16 Disturbances (e.g., tall, peaked; inverted) usually caused by electrolyte imbalances, ischemia, or infarction

The unlicensed nursing assistant is caring for the client diagnosed with chronic venous insufficiency. Which action would warrant immediate intervention from the nurse? 1. Applying compression stockings before going to bed.

Research shows that removing the compression stockings while the client is in bed promotes perfusion of the subcutaneous tissue. The foot of the bed should be elevated.

The nurse has written an outcome goal "demonstrates tolerance for increased activity" for a client diagnosed with congestive heart failure. Which intervention should the nurse implement to assist the client to achieve this outcome? 4. Plan for frequent rest periods.

Scheduling activities and rest periods allows the client to participate in his or her own care and addresses the desired outcome.

Multiple organ dysfunction Syndrome

Sequence of cell damage caused by the massive release of toxic metabolites & enzymes. Once the damage has stsrted, the sequence becomes a vicious cycle as more dead cells break open & release harmful metabolites. These trigger small clots (microthrombi) to form, which block tissue oxygenation & damage more cells, -Liver, heart, brain, & kidney function are lost first. -Most profound change is damage to the heart muscle. One cause of this damage is the release of myocardial depressant factor (MDF) from the ischemic pancreas.

myoglobin

Serum contractions rise 30-60 minutes after MI and raidly clear from circulation

The nurse prepares to administer digoxin (Lanoxin) 0.125 mg to an 82-year-old man admitted with influenza and a history of chronic heart failure. What should the nurse assess before giving the medication? C Serum potassium level

Serum potassium should be monitored because hypokalemia increases the risk for digoxin toxicity.

Which intervention should the nurse implement when defibrillating a client who is in ventricular fibrillation?

Shout "all clear" prior to defibrillating the client. - if anybody is touching the client or bed , that person could possibly be shocked - adults defibrillated at 360 joules - oxygen should be removed to prevent spark - use defib pads pr gels not petroleum gel

The client that is one (1)-day postoperative coronary artery bypass surgery is exhibiting sinus tachycardia. Which intervention should the nurse implement? 4. Determine if the client is having pain

Sinus tachycardia means the sino-atrial node is the pacemaker, but the rate is greater than 100 because of pain, anxiety, or fever. The nurse must determine the cause and treat appropriately. There is no specific medication for sinus tachycardia

The nurse has identified a nursing diagnosis of acute pain related to inflammatory process for a patient with acute pericarditis. The priority intervention by the nurse for this problem is to d. place the patient in Fowler's position, leaning forward on the overbed table.

Sitting upright and leaning forward frequently will decrease the pain associated with pericarditis.

IV sodium nitroprusside (Nipride) is ordered for a patient with acute pulmonary edema. During the first hours of administration, the nurse will need to titrate the nitroprusside rate if the patient develops c. a systolic BP <90 mm Hg.

Sodium nitroprusside is a potent vasodilator, and the major adverse effect is severe hypotension.

What is the difference between stable and unstable v. tach?

Stable = patient has pulse Unstable = pulseless

The HCP prescribes an HMG-COA reductase inhibitor (statin) medication to a client with CAD. Which should the nurse teach the client about this medication? 2. This medication should be taken in the evening.

Statin medications should be taken in the evening for best results because the enzyme that destroys cholesterol works best in the evening and the medication enhances this process.

The client diagnosed with rule-out myocardial infarction is experiencing chest pain while walking to the bathroom. Which action should the nurse implement first? 3. Have the client sit down immediately.

Stopping all activity will decrease the need of the myocardium for oxygen and may help decrease the chest pain.

The nurse is discussing angina with a client who is diagnosed with coronary artery disease. Which action should the client take first when experiencing angina? 2. Stop the activity immediately and rest.

Stopping the activity decreases the heart's need for oxygen and may help decrease the angina, chest pain.

Along with persistent, crushing chest pain, which signs/symptoms would make the nurse suspect that the client is experiencing a myocardial infarction? 2. Diaphoresis and cool clammy skin.

Sweating is a systemic reaction to the MI. The body vasoconstricts to shunt blood from the periphery to the trunk of the body; this, in turn, leads to cold, clammy skin.

Third Degree (complete heart block) can lead to...

Syncope, HF, shock

The nurse recognizes that primary manifestations of systolic failure include: a. decreased EF and increased PAWP

Systolic heart failure results in systolic failure in the left ventricle (LV). The LV loses its ability to generate enough pressure to eject blood forward through the aorta.

What are the 6Ts of pulseless electrical activity (PEA)

Tamponade Tension pneumothorax Thrombosis (pulmonary) Thrombosis (MI) Toxins Trauma

The nurse working on the heart failure unit knows that teaching an older female patient with newly diagnosed heart failure is effective when the patient states that c. all the clinic if her weight goes from 124 to 128 pounds in a week.

Teaching for a patient with heart failure includes information about the need to weigh daily and notify the health care provider about an increase of 3 pounds in 2 days or 3 to 5 pounds in a week.

A patient with chronic heart failure who is taking a diuretic and an angiotensin-converting enzyme (ACE) inhibitor and who is on a low-sodium diet tells the home health nurse about a 5-pound weight gain in the last 3 days. The nurse's priority action will be to. c. assess the patient for clinical manifestations of acute heart failure.

The 5-pound weight gain over 3 days indicates that the patient's chronic heart failure may be worsening.

The nurse is teaching the Dietary Approaches to Stop Hypertension (DASH) diet to a client diagnosed with essential hypertension. Which statement indicates that the client understands the client teaching concerning the DASH diet? 1. "I should eat at least four (4) to five (5) servings of vegetables a day."

The DASH diet has proved beneficial in lowering blood pressure. It recommends eating a diet high in vegetables and fruits.

The client with coronary artery disease is prescribed a Holter monitor. Which intervention should the nurse implement? 1. Instruct client to keep a diary of activity, especially when having chest pain.

The Holter monitor is a 24-hour electrocardiogram, and the client must keep an accurate record of activity so that the health-care provider can compare the ECG recordings with different levels of activity.

o

The ICD sensing system monitors the HR and rhythm and identifies VT and VF.

Disseminated intravascular coagulation (DIC)

The ICU nurse observes petechiae, ecchymoses, & blood oozing from gums & other mucous membranes of a patient with septic shock. How does the nurse interpret this finding?

Which assessment finding obtained by the nurse when assessing a patient with acute pericarditis should be reported immediately to the health care provider? c. Jugular venous distention (JVD) to jaw level

The JVD indicates that the patient may have developed cardiac tamponade and may need rapid intervention to maintain adequate cardiac output.

Based on the Joint Commission Core Measures for patients with heart failure, which topics should the nurse include in the discharge teaching plan for a patient who has been hospitalized with chronic heart failure (select all that apply)? a. How to take and record daily weight c. Date and time of follow-up appointment d. Symptoms indicating worsening heart failure e. Actions and side effects of prescribed medications

The Joint Commission Core Measures state that patients should be taught about prescribed medications, follow-up appointments, weight monitoring, and actions to take for worsening symptoms. Patients with heart failure are encouraged to begin or continue aerobic exercises such as walking, while self-monitoring to avoid excessive fatigue.

The patient has heart failure (HF) with an ejection fraction of less than 40%. What core measures should the nurse expect to include in the plan of care for this patient (select all that apply)? a. Left ventricular function is documented. d. Prescription for angiotensin-converting enzyme (ACE) inhibitor at discharge e. Education materials about activity, medications, weight monitoring, and what to do if symptoms worsen

The Joint Commission has identified these three core measures for heart failure patients. Although controlling dysrhythmias will improve CO and workload, it will not eliminate HF.

The patient has heart failure (HF) with an ejection fraction of less than 40%. What core measures should the nurse expect to include in the plan of care for this patient (select all that apply)? A Left ventricular function is documented. D Prescription for angiotensin-converting enzyme (ACE) inhibitor at discharge E Education materials about activity, medications, weight monitoring, and what to do if symptoms worsen

The Joint Commission has identified these three core measures for heart failure patients. Although controlling dysrhythmias will improve CO and workload, it will not eliminate HF. Prescribing digoxin for all HF patients is no longer done because there are newer effective drugs and digoxin toxicity occurs easily related to electrolyte levels and the therapeutic range must be maintained.

The nurse and an unlicensed nursing assistant are caring for four clients on a telemetry unit. Which nursing task would be best for the nurse delegate to the unlicensed nursing assistant? 4. Help position the client who is having a portable x-ray done

The UAP can assist the x-ray technician in positioning the client for the portable xray. This does not require judgment.

RHYTHM IDENTIFICATION AND TREATMENT

The ability to recognize normal and abnormal cardiac rhythms, called dysrhythmias, is an essential nursing skill.

Pacemakers

The artificial cardiac pacemaker is an electronic device used to pace the heart when the normal conduction pathway is damaged or diseased.provide antibradycardia, antitachycardia, and overdrive pacing.permanent pacemaker is implanted totally within the body.A specialized type of cardiac pacing is used for the management of HF.

A male patient with a long-standing history of heart failure has recently qualified for hospice care. What measure should the nurse now prioritize when providing care for this patient? D Choose interventions to promote comfort and prevent suffering.

The central focus of hospice care is the promotion of comfort and the prevention of suffering.

A male patient with a long-standing history of heart failure has recently qualified for hospice care. What measure should the nurse now prioritize when providing care for this patient? d. Choose interventions to promote comfort and prevent suffering.

The central focus of hospice care is the promotion of comfort and the prevention of suffering.

The nurse just received the A.M. shift report. Which client should the nurse assess first? The client diagnosed with deep vein thrombosis who is complaining of chest pain.

The chest pain could be a pulmonary embolus secondary to deep vein thrombosis and requires immediate intervention by the nurse.

The client is one (1) day post-operative abdominal aortic aneurysm repair. Which information from the unlicensed nursing assistant would require immediate intervention from the nurse? 2. The client's urinary output is 90 mL in six (6) hours.

The client must have 30 mL urinary output every hour. Clients who are post-operative AAA are at high risk for renal failure because of the anatomical location of the AAA near the renal arteries.

The client has just returned from a cardiac catheterization. Which assessment data would warrant immediate intervention from the nurse?

The client refuses to keep the leg straight.

The nurse is developing a discharge-teaching plan for the client diagnosed with congestive heart failure. Which intervention should be included in the plan? Select all that apply. 2. Teach client how to count the radial pulse when taking digoxin, a cardiac glycoside. 3. Instruct client to remove the saltshaker from the dinner table.

The client should not take digoxin if radial pulse is less than 60. The client should be on a low-sodium diet to prevent water retention.

The male client is diagnosed with coronary artery disease (CAD) and is prescribed sublingual nitroglycerin. Which statement indicates the client needs more teaching? 4. "If my chest pain is not gone with one tablet, I will go to the ER."

The client should take one tablet every five (5) minutes and, if no relief occurs after the third tablet, have someone drive him to the emergency department or call 911.

The client is admitted to the telemetry unit diagnosed with acute exacerbation of congestive heart failure (CHF). Which signs/symptoms would the nurse expect to find when assessing this client? 1. Apical pulse rate of 110 and 4 pitting edema of feet.

The client with CHF would exhibit tachycardia (apical pulse rate of 110), dependent edema, fatigue, third heart sounds, lung congestion, and change in mental status.

The nurse is administering a calcium channel blocker to the client diagnosed with a myocardial infarction. Which assessment data would cause the nurse to question administering this medication?

The client's blood pressure is 90/62. - apical pulse between 60-100 can give - serum calcium is not monitored for Ca channel blockers - if BP lower than 90 do not give

The nurse is administering a calcium channel blocker to the client diagnosed with a myocardial infarction. Which assessment data would cause the nurse to question administering this medication? 4. The client's blood pressure is 90/62.

The client's blood pressure is low, and a calcium channel blocker would cause the blood pressure to bottom out.

Which topic will the nurse plan to include in discharge teaching for a patient with systolic heart failure and an ejection fraction of 33%? c. Benefits and side effects of angiotensin-converting enzyme (ACE) inhibitors

The core measures for the treatment of heart failure established by The Joint Commission indicate that patients with an ejection fraction (EF) <40% receive an ACE inhibitor to decrease the progression of heart failure.

A massive gastrointestinal bleed has resulted in hypovolemic shock in an older patient. What is a priority nursing diagnosis? a Acute pain b Impaired tissue integrity c Decreased cardiac output d Ineffective tissue perfusion

d Ineffective tissue perfusion The many deleterious effects of shock are all related to inadequate perfusion and oxygenation of every body system. This nursing diagnosis supersedes the other diagnoses.

ANS: D During severe sepsis, interventions should focus on decreasing hypoxia, maintaining acid-base balance, keeping blood glucose levels as normal as possible, maintaining organ perfusion, minimizing adrenal insufficiency, and decreasing microemboli. Treatment should include administration of low-dose corticosteroids, insulin drip with blood glucose checks every 1 to 2 hours, hourly intake and output monitoring, and an increase in ventilator rate and tidal volume.

The intensive care nurse is caring for an intubated client who has severe sepsis that led to acute respiratory distress. Which nursing intervention is most appropriate during this stage of sepsis? a. Check blood glucose levels every 4 hours. b. Monitor intake and urinary output twice each shift. c. Decrease ventilator rate and tidal volume. d. Administer prescribed low-dose corticosteroids.

1, 3, 4 In order to achieve the designation as a pediatric critical care nurse (CCRN) the nurse will require registered nurse licensure, successful completion of a written test, and pediatric intensive care clinical experience. An advance practice degree and recommendation from a pediatric physician are not required to achieve CCRN certification. Text Reference - p. 1599

The critical care nurse is pursuing certification as a pediatric critical care nurse (CCRN). What will the nurse need to accomplish to achieve this certification? Select all that apply. 1 Registered nurse licensure 2 An advanced practice degree 3 Successful completion of a written test 4 Pediatric intensive care clinical experience 5 Recommendation from a pediatric physician

While admitting an 82-year-old with acute decompensated heart failure to the hospital, the nurse learns that the patient lives alone and sometimes confuses the "water pill" with the "heart pill." When planning for the patient's discharge the nurse will facilitate a c. referral to a home health care agency.

The data about the patient suggest that assistance in developing a system for taking medications correctly at home is needed.

During progressive shock, blood that normally perfuses the kidneys is shunted to the heart and brain, resulting in renal hypoperfusion.

The family of a 44-year-old male, suffering hemorrhagic shock as a result of a motor vehicle accident, states that they do not understand why the client is now on dialysis when he did not sustain injury to the kidneys in the accident. The nurse's most appropriate response is based on which of the following?

ANS: B Hypovolemic shock can be caused by dehydration. A client who has bulimia is at highest risk for dehydration owing to excessive vomiting. Basketball, smoking, and traveling do not put the client at risk for hypovolemic shock.

The emergency department nurse is triaging clients. Which client does the nurse assess most carefully for hypovolemic shock? a. 15-year-old adolescent who plays high school basketball b. 24-year-old computer specialist who has bulimia c. 48-year-old truck driver who has a 40-pack-year history of smoking d. 62-year-old business executive who travels frequently

A compensatory mechanism involved in HF that leads to inappropriate fluid retention and additional workload of the heart is: c. neurohormonal response

The following mechanisms in heart failure lead to inappropriate fluid retention and additional workload of the heart: activation of the renin-angiotensin-aldosterone system (RAAS) cascade and release of antidiuretic hormone from the posterior pituitary gland in response to low cerebral perfusion pressure that results from low cardiac output.

1 Patients requiring ventilation up to 3 days are said to have received short-term ventilation. More than 3 days (7, 14, and 20 days) denotes long-term ventilation. Text Reference - p. 1625

The health care provider advises short-term ventilation for a patient. How many days would be considered short-term ventilation? 1 Up to 3 days 2 Up to 7 days 3 Up to 14 days 4 Up to 20 days

The client diagnosed with a myocardial infarction asks the nurse, "Why do I have to rest and take it easy? My chest doesn't hurt anymore." Which statement would be the nurse's best response? 1. "Your heart is damaged and needs about four (4) to six (6) weeks to heal."

The heart tissue is dead, stress or activity may cause heart failure, and it does take about six (6) weeks for scar tissue to form.

The nurse has just received the A.M. shift report. Which client would the nurse assess first? 3. The client with arterial occlusive disease who cannot move the foot.

The inability to move the foot means that a severe neurovascular compromise has occurred, and the nurse should assess this client first.

When developing a community health program to decrease the incidence of rheumatic fever, which action would be most important for the community health nurse to include? b. Teach community members to seek treatment for streptococcal pharyngitis.

The incidence of rheumatic fever is decreased by treatment of streptococcal infections with antibiotics.

ANS: A Any problem that impairs oxygen delivery to tissues and organs can start the syndrome of shock and lead to a life-threatening emergency. Shock represents the "whole-body response," affecting all organs in a predictable sequence. Compensation mechanisms attempt to maintain homeostasis and deliver necessary oxygen to organs but eventually will fail without reversal of the cause of shock, resulting in death. page 809

The intensive care nurse is educating the spouse of a client who is being treated for shock. The spouse states, "The doctor said she has shock. What is that?" What is the nurse's best response? a. "Shock occurs when oxygen to the body's tissues and organs is impaired." b. "Shock is a serious condition, but it is not a life-threatening emergency." c. "Shock progresses slowly and can be stopped by the body's normal compensation." d. "Shock is a condition that affects only specific body organs like the kidneys."

4 Progressive care units (PCUs), also called intermediate care units, provide a transition between the intensive care unit (ICU) and the general care unit or discharge. Generally, PCU patients are at risk for serious complications, but their risk is lower than that of the ICU patient. Therefore, the nurse anticipates that the patient with a stable blood pressure on a stable dose of a vasoactive drug will be transferred to the PCU. A patient who required a new drip overnight for hypotension and a patient who crashed and required intubation and mechanical ventilation are unstable and the nurse does not anticipate their transfer to the PCU. A preoperative heart transplant patient is often admitted to the PCU. A patient who is 12 hours post transplantation is not expected to be transferred to the PCU at this time. Text Reference - p. 1599

The intensive care unit (ICU) charge nurse is reviewing patient medical records during the overnight shift. Which patient does the nurse anticipate will be transferred to the progressive care unit (PCU)? 1 The patient who required a new drip overnight for hypotension. 2 The patient who was intubated and ventilated after crashing overnight. 3 The patient who is 12 hours postop for a heart and lung transplantation. 4 The patient whose blood pressure is 100/70 mm Hg on a stable dose of a vasoactive drug.

implantable cardioverter defibrillator (ICD)

The is used for patients who have survived SCD, have spontaneous sustained VT, have syncope with inducible ventricular tachycardia/fibrillation during EPS, and are at high risk for future life-threatening dysrhythmias.

The client diagnosed with arterial occlusive disease is one (1) day post-operative right femoral popliteal bypass. Which intervention should the nurse implement? 4. Assess the client's leg dressing every four (4) hours.

The leg dressing needs to be assessed for hemorrhaging or signs of infection.

Which client would be most likely to develop an abdominal aortic aneurysm? 3. A 69-year-old male with peripheral vascular disease.

The most common cause of AAA is atherosclerosis (which is the cause of peripheral vascular disease); it occurs in men four (4) times more often than women and primarily in Caucasians.

The client is diagnosed with a small abdominal aortic aneurysm. Which interventions should be included in the discharge teaching? Select all that apply. 1. Tell the client to exercise three (3) times a week for 30 minutes. 2. Encourage the client to eat a low-fat, low-cholesterol diet. 4. Discuss with the client the importance of losing weight.

The most common cause of AAA is atherosclerosis, so teaching should address this area. 2. A low-fat, low-cholesterol diet will help decrease development of atherosclerosis. 4. Losing weight will help decrease the pressure on the AAA and will help address decreasing the cholesterol level.

The client diagnosed with congestive heart failure is complaining of leg cramps at night. Which nursing interventions should be implemented? 2. Monitor the client's potassium level and assess the client's intake of bananas and orange juice.

The most probable cause of the leg cramping is potassium excretion as a result of diuretic medication. Bananas and orange juice are foods that are high in potassium.

4 Negative inspiratory force (NIF) is the amount of negative pressure that a patient is able to generate to initiate spontaneous respirations. An NIF of less than -20 cm H2O is an indicator for weaning but the more negative the number, the better the indication for weaning. Therefore -60 cm H2O is the best indication for weaning. Text Reference - p. 1626

The negative inspiratory force (NIF) is measured in a patient who is on positive pressure ventilation. Among the values given, which NIF value is the best indication for weaning? 1 -30 cm H2O 2 -40 cm H2O 3 -50 cm H2O 4 -60 cm H2O

2 Frank-Starling's law explains the effects of preload and states that the more a myocardial fiber is stretched during filling, the more it shortens during systole and the greater the force of the contraction. Cardiac index (CI) is the measurement of the cardiac output adjusted for body surface area (BSA). It is a more precise measurement of the efficiency of the heart's pumping action. Systemic vascular resistance (SVR) is opposition encountered by the left ventricle to blood flow by the vessels. Pulmonary vascular resistance (PVR) is opposition encountered by the right ventricle to blood flow by the vessels. Text Reference - p. 1604

The nurse educator is preparing a lecture on hemodynamic monitoring. What should the educator use to explain the effects of preload? 1 Cardiac index 2 Frank-Starling's law 3 Systemic vascular resistance 4 Pulmonary vascular resistance

1 Critical care nursing is a specialty dealing with human responses to life-threatening problems. Medial surgical nursing is a specialty dealing with the care of adult patients in a variety of settings. Mental health nursing is a specialty dealing with people of all ages with mental illness or mental distress. Maternal newborn nursing is a specialty dealing with the care of women throughout their pregnancy and childbirth. Text Reference - p. 1598

The nurse educator is teaching a group of nursing students about critical care nursing. Which statement by a student indicates appropriate understanding of this nursing specialty? 1 "Critical care nursing is a specialty dealing with human responses to life-threatening problems." 2 "Critical care nursing is a specialty dealing with the care of adult patients in a variety of settings." 3 "Critical care nursing is a specialty dealing with people of all ages with mental illness or mental distress." 4 "Critical care nursing is a specialty dealing with the care of women throughout their pregnancy and childbirth."

1, 4, 5 Pulmonary artery catheterization helps to monitor and manage the care of patients who are at high risk for hemodynamic compromise. Pulmonary artery catheterization may cause trauma in the blood vessels and worsen symptoms of coagulopathy. Pulmonary artery catheterization increases the risk of trauma in patients with mechanical tricuspid valves and endocardial pacemakers; therefore, it is contraindicated in the patient with coagulopathy, mechanical tricuspid valve, and transvenous pacemaker. Pulmonary artery catheterization is performed in patients with cardiogenic shock and fulminant myocarditis to detect the risk of heart failure. Text Reference - p. 1607

The nurse identifies that pulmonary artery catheterization is contraindicated for patients with a history of what? Select all that apply. 1 Coagulopathy 2 Cardiogenic shock 3 Fulminant myocarditis 4 Endocardial pacemaker 5 Mechanical tricuspid valve

1 Critical care nurses will face ethical dilemmas related to the care of patients. This can occur over perceived issues of delivering futile or nonbeneficial care, such as attempting to resuscitate a neonate born at 21 weeks of gestation. This situation does not represent a societal, temporal, or sequential dilemma. Text Reference - p. 1599

The nurse in the newborn intensive care unit (NICU) is providing care to a neonate born at 21 weeks gestation who is being resuscitated in the delivery room. The standard gestation compatible with life is 23 to 24 weeks gestation. Which type of dilemma could the nurse be facing? 1 Ethical 2 Societal 3 Temporal 4 Sequential

ANS: B The client receiving sodium nitroprusside should have his or her blood pressure assessed every 15 minutes. Higher doses can cause systemic vasodilation and can increase shock. The nurse should monitor the client's pain, urinary output, and extremities, but these assessments do not directly relate to the nitroprusside infusion.

The nurse is administering prescribed sodium nitroprusside (Nipride) intravenously to a client who has shock. Which nursing intervention is a priority when administering this medication? a. Ask if the client has chest pain every 30 minutes. b. Assess the client's blood pressure every 15 minutes. c. Monitor the client's urinary output every hour. d. Observe the client's extremities every 4 hours.

ANS: B The syndrome of hypovolemic shock results in inadequate tissue perfusion and oxygenation; thus some cells are metabolizing anaerobically. Such metabolism increases the production of lactic acid, resulting in an increase in hydrogen ion production and acidosis. Other laboratory values associated with acidosis include increased creatinine (impaired renal function) and increased partial pressure of arterial carbon dioxide. Urine specific gravity is not associated with acidosis. page 812

The nurse is assessing a client who has hypovolemic shock. Which laboratory value indicates that the client is at risk for acidosis? a. Decreased serum creatinine b. Increased serum lactic acid c. Increased urine specific gravity d. Decreased partial pressure of arterial carbon dioxide

ANS: A An increase in heart and respiratory rates (heart rate first) from the client's baseline and a slight increase in diastolic blood pressure may be the only objective manifestations of early shock. These findings do not correlate with other stages of shock.

The nurse is assessing a client who has septic shock. The following assessment data were collected: Baseline Data Today's Data Heart rate 75 beats/min 98 beats/min Blood pressure 125/65 mm Hg 128/75 mm Hg Respiratory rate 12 breaths/min 18 breaths/min Urinary output 40 mL/hr 40 mL/hr The nurse correlates these findings with which stage of shock? a. Early b. Compensatory c. Intermediate d. Refractory

ANS: D The late phase of sepsis-induced distributive shock is characterized by most of the same cardiovascular manifestations as any other type of shock. The distinguishing feature is lack of ability to clot blood, causing the client to bleed from areas of minor trauma and to bleed spontaneously. The other manifestations are associated with all types of shock.

The nurse is assessing a client who was admitted for treatment of shock. Which manifestation indicates that the client's shock is caused by sepsis? a. Hypotension b. Pale clammy skin c. Anxiety and confusion d. Oozing of blood at the IV site

2 Inadequate pain control is often linked with agitation and anxiety and is known to add to the stress response. Pain is associated with infection, but the patient is at the highest risk for developing anxiety. Dementia and depression are not associated with unrelieved pain. Text Reference - p. 1600

The nurse is assessing a patient in the intensive care unit (ICU). The patient is intubated and exhibiting symptoms of pain. Based on this data what condition is the patient at the highest risk for developing? 1 Anxiety 2 Infection 3 Dementia 4 Depression

ANS: C Certain conditions or treatments that cause immune suppression, such as having cancer and being treated with chemotherapeutic agents, aspirin, and certain antibiotics, can predispose a person to septic shock. The other client situations do not increase the client's risk for septic shock.

The nurse is assessing clients in the emergency department. Which client is at highest risk for developing septic shock? a. 25-year-old man who has irritable bowel syndrome b. 37-year-old woman who is 20% above ideal body weight c. 68-year-old woman who is being treated with chemotherapy d. 82-year-old man taking beta blockers for hypertension

ANS: D Anaphylaxis damages cells and causes release of large amounts of histamine and other inflammatory chemicals. This results in massive blood vessel dilation and increased capillary leak, which manifests as swelling. The other clinical manifestations do not relate to anaphylaxis or distributive shock.

The nurse is caring for a client who has had an anaphylactic event. Which priority question does the nurse ask to determine whether the client is experiencing distributive shock? a. "Is your blood pressure higher than usual?" b. "Are you having pain in your throat?" c. "Have you been vomiting?" d. "Are you usually this swollen?"

ANS: C IV therapy for fluid resuscitation is the primary intervention for hypovolemic shock. A dopamine hydrochloride drip is a secondary treatment if the client does not respond to fluids. Aminoglycosides and heparin are given to clients with septic shock.

The nurse is caring for a client who has hypovolemic shock. After administering oxygen, what is the priority intervention for this client? a. Administer an aminoglycoside. b. Initiate a dopamine hydrochloride (Intropin) drip. c. Administer crystalloid fluids. d. Initiate an intravenous heparin drip.

Decreasing urine output Increasing HR

The nurse is caring for a patient at risk for hypovolemic shock. Which indicators of shock does the nurse monitor for?

Localized infected area

The nurse is caring for a patient at risk for septic shock from a wound infection. In order to prevent systemic inflammatory response syndrome, the nurse's priority is to monitor which factor?

Metabolic acidosis

The nurse is caring for a patient in septic shock. The nurse notes that the rate & depth of respirations is markedly increased. The nurse interprets this as a possible manifestation of the respiratory system compensating for which condition?

1, 2, 3 Intraaortic balloon therapy destroys platelets and may cause thrombocytopenia. Monitoring hemodynamic parameters helps estimate the loss of platelets and development of thrombus. Enlarged blood clots reduce blood flow to the tissues and result in hypoxemia. Therefore, measurement of oxygen deprivation levels assesses for thrombus formation in the patient. Peripheral neurovascular damage is generally observed as a result of intraaortic balloon therapy and should be monitored. Spinach is rich in vitamin K and increases the formation of clotting factors. Warfarin is an anticoagulant used to prevent future development of blood clots. Spinach and warfarin should be avoided for patients with thrombocytopenia. Text Reference - p. 1611

The nurse is caring for a patient receiving intraaortic balloon therapy. Which nursing interventions are appropriate for this patient? Select all that apply. 1 Monitoring oxygen deprivation levels 2 Monitoring hemodynamic parameters 3 Monitoring neurovascular complications 4 Including spinach in the patient's diet 5 Providing warfarin therapy

2 Tachypnea and disorientation are early and subtle signs of deterioration. The rapid response team (RRT) brings rapid and immediate care to unstable patients in non-critical care units. While assessing pain, documenting the data in the medical record, and administering prescribed medications such as antihypertensive medications are all appropriate actions, they are not the priority nursing actions in this situation. Text Reference - p. 1599

The nurse is providing care to a postoperative patient on a medical-surgical unit. The patient is experiencing tachypnea and becomes disoriented at times. Which is the priority action by the nurse in this situation? 1 Assessing the current level of pain 2 Activating the rapid response team 3 Documenting the data in the medical record 4 Administering the prescribed antihypertensive medication

4 Intraaortic balloon therapy has potential complications such as site infection, thromboembolism, arterial trauma, hematologic complications, and hemorrhage from the insertion site. Maintaining the head of the bed below 45 degrees helps to prevent breathlessness in the patient in the event of arterial trauma. Monitoring coagulation profiles, hematocrit, and platelet count is beneficial when the patient has a hematologic complication. An occlusive dressing prevents risk of surface infection but is not used to treat arterial trauma in the patient. Heparin helps to prevent thromboembolism but does not treat arterial trauma. Text Reference - p. 1612

The nurse is caring for a patient who has suffered arterial trauma during intraaortic balloon pump (IABP) therapy. What is the appropriate nursing action for this patient? 1 Monitor coagulation profiles 2 Apply an occlusive dressing 3 Administer prophylactic heparin 4 Maintain head of the bed below 45 degrees

1, 3, 4 Administering IV fluids helps thin respiratory secretions and facilitates suctioning. Postural drainage and percussion every two hours helps move secretions into larger airways and promotes the removal through suctioning. Supplemental humidification helps thin secretions and promotes suctioning. The patient has an endotracheal tube and will not be given an oral diet. Instilling normal saline into the endotracheal tube can cause asphyxia. Text Reference - p. 1617

The nurse is caring for a patient with an artificial airway. Which nursing interventions ensure endotracheal tube patency? Select all that apply. 1 Provide adequate hydration 2 Provide gluten rich food 3 Promote postural drainage 4 Provide supplemental humidification 5 Instilling normal saline into the endotracheal tube

Antibiotics

The nurse is caring for a patient with septic shock. Which therapy specific to the management of shock for this patient does the nurse anticipate will be used?

ANS: A Distributive shock is the type of shock that occurs when blood volume is not lost from the body but is distributed to the interstitial tissues, where it cannot circulate and deliver oxygen. Neurally-induced distributive shock may be caused by pain, anesthesia, stress, spinal cord injury, or head trauma. The other clients are at risk for hypovolemic and cardiogenic shock. pg 812

The nurse is caring for multiple clients in the emergency department. The client with which condition is at highest risk for distributive shock? a. Severe head injury from a motor vehicle accident b. Diabetes insipidus from polycystic kidney disease c. Ischemic cardiomyopathy from severe coronary artery disease d. Vomiting of blood from a gastrointestinal ulcer

1 Preload, afterload, and contractility determine stroke volume (SV). Cardiac output and heart rate are used to determine stroke volume; however, body surface area is used to determine cardiac index. Mean arterial pressure is used to determine afterload, not stroke volume. Cardiac index is a more precise measurement of the efficiency of the heart's pumping action; it is not used to determine stroke volume. Text Reference - p. 1603

The nurse is concerned about a patient's stroke volume. What determines stroke volume? 1 Preload, afterload, and contractility 2 Cardiac output, heart rate, and body surface area 3 Afterload, cardiac output, and mean arterial pressure 4 Cardiac index, mean arterial pressure, and blood pressure

1, 2, 3, 5 Hemodynamic effects of intraaortic balloon pump therapy (IABP) include increased stroke volume leading to warm skin and increased urine output. The decrease in afterload improves breath sounds. Improved stroke volume also improves mentation. The pump has no direct effect on blood pressure regulation. Text Reference - p. 1611

The nurse is evaluating a patient receiving intraaortic balloon pump (IABP) therapy. Which findings indicate that the pump is improving the patient's health status? Select all that apply. 1 Warm and dry skin 2 Urine output 50 mL/hr 3 Breath sounds clear bilaterally 4 Blood pressure 168/88 mm Hg 5 Oriented to person, place, and time

Hematocrit at 34% Maintaining a hematocrit at 30% to 35% and a hemoglobin level between 12.5 and 14.5 g/100 mL is the goal of blood administration. Frequent monitoring of hemoglobin and hematocrit is essential to evaluate therapy of blood transfusions.

The nurse is evaluating the effectiveness of blood transfusions to a client with hypovolemic shock, secondary to hemorrhage. Which of the following indicates to the nurse that a therapeutic effect of the administration of blood has been achieved?

1 Accurate oxygen saturation of hemoglobin (SpO2) measurements may be difficult to obtain on patients who are hypothermic, are receiving vasopressor therapy, or experiencing shock. A body temperature of 95.4° F is hypothermic and is most likely the reason this measurement is difficult to obtain. A cardiac glycoside is not a vasopressor and would not cause this difficulty. A blood pressure of 118/72 mm Hg and heart rate of 72 with occasional ectopy are not manifestations of shock and would not cause this difficulty. Text Reference - p. 1609

The nurse is having difficulty obtaining an accurate oxygen saturation of hemoglobin (SpO2) measurement on a patient. What should the nurse consider as the reason for this difficulty? 1 Body temperature 95.4° F 2 Receiving a cardiac glycoside 3 Blood pressure of 118/72 mm Hg 4 Heart rate of 72 beats/minute with occasional ectopy

ANS: C Dopamine hydrochloride causes vasoconstriction that in turn increases cardiac output and mean arterial pressure, thereby improving tissue perfusion and oxygenation. Tachycardia is not a desired response but often occurs as a side effect. pg 818

The nurse is monitoring a client in hypovolemic shock who has been placed on a dopamine hydrochloride (Intropin) drip. Which manifestation is a desired response to this medication? a. Decrease in blood pressure b. Increase in heart rate c. Increase in cardiac output d. Decrease in mean arterial pressure

ANS: A Therapy during the second (late) phase of septic shock is aimed at enhancing the blood's ability to clot. Enoxaparin would increase the client's risk of bleeding and therefore should not be administered during the last phase of septic shock. Administering clotting factors, plasma, platelets, and other blood products will assist the client's blood to clot. Intravenous insulin to control hyperglycemia and antibiotic therapy would continue in the late phases of septic shock.

The nurse is planning care for a client with late-phase septic shock. All of the following treatments have been prescribed. Which prescription does the nurse question? a. Enoxaparin (Lovenox) 40 mg subcutaneous twice daily b. Transfusion of 2 units of fresh frozen plasma c. Regular insulin intravenous drip per protocol d. Cefazolin (Ancef) 1 g IV every 6 hours

3, 4, 5 Interventions to reduce the risk of infection in a patient with an intraaortic balloon pump (IABP) include covering the site with an occlusive dressing, administering prophylactic antibiotics as prescribed, and using strict aseptic technique with dressing changes. Keeping the lower extremities extended would help prevent arterial trauma caused by insertion or displacement of the balloon. Turning and repositioning every two hours would help prevent the development of stasis pneumonia. Text Reference - p. 1612

The nurse is planning care for a patient receiving intraaortic balloon pump (IABP) therapy. What interventions should be included to reduce this patient's risk of developing an infection? Select all that apply. 1 Keep lower extremities extended 2 Turn and reposition every two hours 3 Cover the site with an occlusive dressing 4 Administer prophylactic antibiotics as prescribed 5 Use strict aseptic technique with dressing changes

3 Stroke volume variation (SVV) is the variation of the arterial pulsation caused by heart-lung interaction. It is a sensitive indicator of preload responsiveness when used on select patients. SVV is used only for patients on controlled mechanical ventilation with a fixed respiratory rate and a fixed tidal volume of 8 mL/kg. SVV is not used on patients who have spontaneous respirations even though an arterial line does need to be in place. The patient may have continuous cardiac monitoring but the patient needs to be intubated rather than provided with oxygen via a face mask. The patient needs to be intubated; however, nasal intubation is not identified as a requirement. It is not identified that the patient needs to be receiving positive end expiration pressure through the ventilator. Text Reference - p. 1607

The nurse is preparing a patient for arterial pressure-based cardiac output (APCO) measuring. What patient criteria must be met before this measuring device can be used to determine the patient's stroke volume variation (SVV)? 1 Spontaneous respirations and placement of an arterial line 2 Continuous cardiac monitoring and application of oxygen via face mask 3 Controlled mechanical ventilation and fixed respiratory rate and tidal volume 4 Nasal intubation and positive end expiration pressure setting on the ventilator

ANS: C Sodium nitroprusside (Nipride) must be protected from light to prevent degradation of the drug. It should be delivered via pump. This medication does not have any effect on respiratory rate. Hypertension is a sign of milrinone (Primacor) overdose.

The nurse is preparing to administer sodium nitroprusside (Nipride) to a client. Which important action related to the administration of this drug does the nurse implement? a. Assess the client's respiratory rate. b. Administer the medication with gravity tubing. c. Protect the medication from light with an opaque bag. d. Monitor for hypertensive crisis.

4 When an intensive care unit (ICU) patient exhibits manifestations of delirium it is the nurse's priority to address physiologic factors that could be contributing to the patient's symptoms. Correction of oxygenation by increasing the patient's oxygen is the priority intervention for this patient. Placing a clock in the room and updating the calendar in the room are important when providing care to a patient with delirium but these are not the priorities in the current situation. Administering an opioid pain medication is likely to enhance the clinical manifestations of delirium. Text Reference - p. 1601

The nurse is providing care for a patient who is receiving care in the intensive care unit (ICU). The patient is exhibiting symptoms of delirium. Which will the nurse address on priority when providing care to this patient? 1 Placing a clock in the room 2 Updating the calendar in the room 3 Administering opioid analgesics for pain 4 Administering increased oxygen, per order

1, 3, 4, 5 Allowing a patient's family to be present during resuscitative efforts during a cardiac arrest can help in the grieving process if the patient dies. It also reduces the fear and anxiety that the family is feeling, allows the patient's family to support their loved one, and helps the family overcome any doubts about the patient's condition. Allowing the family to be in the room during resuscitative efforts has not been shown to decrease the likelihood of malpractice lawsuits. Text Reference - p. 1602

The nurse is providing care to a patient in the intensive care unit (ICU) who is being coded due to cardiac arrest. Why would it be appropriate for the patient's family to be present in this situation? Select all that apply. 1 It aids in the grieving process if the patient dies. 2 It decreases the likelihood of malpractice lawsuits. 3 It may reduce the fear and anxiety the family is feeling. 4 It allows the patient's family to support their loved one. 5 It may help the family overcome doubts about the patient's condition.

2 Music therapy is a nonpharmacologic relaxation intervention that can be used to treat the patient anxiety that often occurs in the intensive care unit (ICU) environment. Lorazepam is an appropriate pharmacologic, not nonpharmacologic, intervention for anxiety. A sedation holiday is appropriate to conduct a neurologic exam that is often difficult due to the deep sedation required for intubation and mechanical ventilation. Range-of-motion exercises can help treat the physical manifestations of an injury, but are not relaxation interventions. Text Reference - p. 1601

The nurse is providing care to a patient in the intensive care unit (ICU) who is experiencing anxiety. Which nonpharmacologic relaxation intervention is appropriate for this patient? 1 Lorazepam 2 Music therapy 3 Sedation holiday 4 Range-of-motion exercises

2 Patients with preexisting dementia, such as Alzheimer's disease, are at an increased risk for developing delirium when receiving care in the intensive care unit (ICU). Diabetes mellitus, Parkinson's disease, and multiple sclerosis are not known risk factors for developing delirium. Text Reference - p. 1601

The nurse is providing care to a patient in the intensive care unit (ICU). The patient is currently sedated due to intubation and mechanical ventilation. Which finding in the patient's medical record would place this patient at an increased risk for delirium? 1 Diabetes mellitus 2 Alzheimer's disease 3 Parkinson's disease 4 Multiple sclerosi

3, 5 Medications that are appropriate to induce and maintain sleep for the patient in the intensive care unit (ICU) environment include zolpidem and tamazepam. Propofol is a medication that is used to induce sedation not sleep. Fentanyl and morphine are opioid medications used to treat pain and are not appropriate to induce and maintain sleep for this patient. Text Reference - p. 1601

The nurse is providing care to a patient in the intensive care unit (ICU). Which pharmacologic interventions are appropriate to induce and maintain sleep? Select all that apply. 1 Propofol 2 Fentanyl 3 Zolpidem 4 Morphine 5 Temazepam

1 Nonverbal communication is important when providing care to an unresponsive patient in the intensive care unit (ICU). High levels of procedure-related touch and lower levels of comfort-related touch often characterize the ICU environment. The nurse would encourage the patient's spouse to touch and talk with her husband even if he is unresponsive. There is no way of knowing how aware the patient is of the surrounding when sedated. While bringing items from home for comfort are important, this is not the most appropriate statement by the nurse. Telling the spouse to sit where she will not be in the way of providing care is not therapeutic. Text Reference - p. 1601

The nurse is providing care to an unresponsive patient in the intensive care unit (ICU). The patient's spouse is at the bedside and states, "I just want him to know that I am here with him." Which statement by the nurse is most appropriate? 1 "You should talk and touch you husband whenever you visit." 2 "Your husband is so sedated he is not aware of his surroundings." 3 "You can bring items from home to make your husband more comfortable." 4 "Please sit where you will not be in the way as we provide care to your husband."

4 The patient with a collapsed lung and multiple injuries sustained in a car accident would be admitted to the trauma intensive care unit (ICU). Patients with minor acute injuries or those who are not expected to recover from an illness is usually not admitted to an ICU. Therefore, the ICU is not used to manage a fractured wrist; to prolong the natural process of death such as the patient with stage IV metastatic breast cancer; or for the patient in a persistent coma or vegetative state. Text Reference - p. 1600

The nurse is providing care to several patients in the emergency department (ED). Which patient does the nurse anticipate will be admitted to the intensive care unit (ICU)? 1 A patient with a fractured wrist 2 A patient with stage IV metastatic breast cancer 3 A patient in a permanent vegetative state who has a urinary tract infection 4 The patient with a collapsed lung and multiple injuries sustained in a car accident

ANS: C Teach everyone to prevent dehydration by having adequate fluid intake during exercise or when in a hot and dry environment. Insensitive water loss increases in this type of environment. Heat causes vasodilation as well, also contributing to water loss. The other statements are not accurate.

The nurse is providing community education for clients at risk for dehydration. One client states, "We are not at risk because we live in a hot and dry climate." What is the nurse's best response? a. "You are still at risk but not as high a risk as those who live in hot and humid climates." b. "Any type of heat can cause peripheral vasoconstriction, which causes the body to lose water." c. "In a hot and dry environment, the body can lose an increased amount of water without your knowledge." d. "Even though you are not at risk, you should drink adequate fluids when you exercise."

ANS: B, E, F Daily temperatures, washing dishes in hot sudsy water or a dishwasher, and rinsing toothbrushes in liquid bleach or in the dishwasher are infection precautions for the immune compromised client. Clients at increased risk because of immune suppression need to wear a facemask when in large crowds or around ill people. Water need not be bottled but should not be used if it has been standing for longer than 15 minutes. This population is not restricted from pets but is only advised not to change pet litter boxes.

The nurse is providing health education to a client on immunosuppressant therapy. Which instructions does the nurse include in this client's teaching? (Select all that apply.) a. "Wear a facemask at all times." b. "Take your temperature once a day." c. "Drink only bottled water." d. "Avoid any contact with pets." e. "Wash dishes with hot sudsy water." f. "Rinse your toothbrush in liquid laundry bleach."

4 The nurse should advise the caregiver to avoid external rotation of the patient's hip; this movement can be avoided by properly positioning the patient and by the use of specialized mattresses and beds. Simple maneuvers such as arm circles, knee bends and quadriceps setting should be performed, because they maintain the muscle tone in the upper and lower extremities of the patient. Text Reference - p. 1624

The nurse is teaching the patient's caregiver about receiving positive pressure ventilation. What movements should the nurse tell the caregiver to avoid doing to the patient? 1 Arm circles 2 Knee bends 3 Quadriceps setting 4 External rotation of the hip

The client is one (1) day postoperative coronary artery bypass surgery. The client complains of chest pain. Which intervention should the nurse implement first? 2. Assess the client's chest dressing and vital signs.

The nurse must always assess the client to determine if the chest pain that is occurring is expected postoperatively or if it is a complication of the surgery.

The client is exhibiting ventricular tachycardia. Which intervention should the nurse implement first? 3. Assess the client's apical pulse and blood pressure

The nurse must assess the apical pulse and blood pressure to determine if the client is in cardiac arrest and then treat as ventricular defibrillation. If the client's heart is beating, the nurse would then administer lidocaine.

The client diagnosed with pericarditis is complaining of increased pain. Which intervention should the nurse implement first? 3. Assess the client for cardiac complications.

The nurse must assess the client to determine if the pain is expected pain secondary to pericarditis or if the pain is indicative of a complication that requires intervention from the health-care provider.

The client shows ventricular fibrillation on the telemetry at the nurse's station. Which action should the telemetry nurse implement first? 3. Call a STAT code.

The nurse must call a code that activates the crash cart being brought to the room and a team of health-care providers that will care for the client according to an established protocol.

The nurse enters the room of the client diagnosed with congestive heart failure. The client is lying in bed gasping for breath, is cool and clammy, and has buccal cyanosis. Which intervention would the nurse implement first? 4. Assist the client to a sitting position.

The nurse must first put the client in a sitting position to decrease the workload of the heart by decreasing venous return and maximizing lung expansion. Then, the nurse could take vital signs and check the pulse oximeter and then sponge the client's forehead.

The client is scheduled for a right femoral cardiac catheterization. Which nursing intervention should the nurse implement after the procedure? 2. Assess the client's neurovascular status.

The nurse must make sure that blood is circulating to the right leg so the client should be assessed for pulses, paresthesia, paralysis, coldness, and pallor.

2, 3, 4 When enteral nutrition is contraindicated in patients, the primary health care provider would prescribe parenteral nutrition to provide adequate nutrients to the patients. Patients with gastrointestinal disorders such as pancreatitis, paralytic ileus, and severe diarrhea will receive parenteral nutrition. Enteral nutrition is not contraindicated in the patient with arthritis and hypothyroidism. Text Reference - p. 1600

The nurse reviews medical records of several patient and concludes that which patients are appropriate candidates for parenteral nutrition? Select all that apply. 1 A patient with arthritis 2 A patient with pancreatitis 3 A patient with paralytic ileus 4 A patient with severe diarrhea 5 A patient with hypothyroidism

The client diagnosed with a DVT is on a heparin (an anticoagulant) drip at 1400 units per hour, and Coumadin (warfarin sodium; also an anticoagulant) 5 mg twice a day. Which intervention should the nurse implement first? 1. Check the PTT and PT/INR.

The nurse should check the laboratory values pertaining to the medications before administering the medications.

The nurse is teaching a community group about preventing rheumatic fever. What information should the nurse include? A. Prompt recognition and treatment of streptococcal pharyngitis

The nurse should emphasize the need for prompt and adequate treatment of streptococcal pharyngitis infection, which can lead to the complication of rheumatic fever

A stable patient with acute decompensated heart failure (ADHF) suddenly becomes dyspneic. Before positioning the patient on the bedside, what should the nurse assess first? d. Blood pressure

The nurse should evaluate the blood pressure before dangling the patient on the bedside because the blood pressure can decrease as blood pools in the periphery and preload decreases.

A 54-year-old male patient who had bladder surgery 2 days ago develops acute decompensated heart failure (ADHF) with severe dyspnea. Which action by the nurse would be indicated first? C Assist the patient to a sitting position with arms on the overbed table.

The nurse should place the patient with ADHF in a high Fowler's position with the feet horizontal in the bed or dangling at the bedside.

The client diagnosed with a myocardial infarction is on bed rest. The unlicensed nursing assistant is encouraging the client to move the legs. Which action should the nurse implement? 3. Praise the nursing assistant for encouraging the client to move legs

The nurse should praise and encourage assistants to participate in the client's care. Clients on bed rest are at risk for deep vein thrombosis, and moving the legs will help prevent that.

2, 3 Providing culturally competent care to critically ill patients and caregivers is challenging. The nurse who wants to provide culturally competent care to critically ill patients in the intensive care unit (ICU) should ask the patient or family members who is wanted in the room at the time of death. The nurse should also ask the patient or family members about cultural traditions regarding death and dying. The nurse should not prioritize cultural needs over physiologic needs. Often physiologic needs are the priority in the ICU. The nurse should not assume that the patient follows cultural customs for the documented ethnicity. The nurse should not tell the family that last rites are not possible and should advocate for the patient to receive last rites if this is the patient's wish. Text Reference - p. 1602

The nurse wants to provide culturally competent care to patients requiring care in the intensive care unit (ICU). Which actions by the nurse are appropriate? Select all that apply. 1 Prioritizing cultural needs over physiologic needs 2 Asking the patient who he or she wants in the room at the time of death 3 Asking the family about cultural traditions regarding death and dying 4 Assuming that the patient follows cultural customs for the documented ethnicity 5 Telling the family members it is not possible for last rites to be administered

2 Tidal volume is the volume of gas delivered to a patient during each ventilator breath. The number of breaths the ventilator delivers per minute is called the respiratory rate. The positive pressure used to augment the patient's inspiratory pressure is called pressure support. The positive pressure applied at the end of expiration of ventilator breaths is called positive end-expiratory pressure. Text Reference - p. 1619

The nurse working in a critical care unit understands that tidal volume is an important setting in a mechanical ventilator. Which statement appropriately describes tidal volume? 1 Number of breaths the ventilator delivers per minute 2 Volume of gas delivered to patient during each ventilator breath 3 Positive pressure used to augment patient's inspiratory pressure 4 Positive pressure applied at the end of expiration of ventilator breaths

Decreased urine output Increased HR

The patient has decreased oxygenation & impaired tissue perfusion. Which clinical manifestations are evidence of one of the non-progressive or compensatory stages of shock?

3 The most commonly used mechanical circulatory assist device is the IABP, and it is used to decrease ventricular workload, increase myocardial perfusion, and augment circulation. Cardiopulmonary bypass provides circulation during open heart surgery. It is not used as an assist device after surgery. ICG is a noninvasive method to obtain cardiac output and assess thoracic fluid status. CVP measurement is an invasive measurement of right ventricular preload and reflects fluid volume problems. Text Reference - p. 1611

The patient has developed cardiogenic shock after a left anterior descending myocardial infection. Which circulatory assist device should the nurse expect to use for this patient? 1 Cardiopulmonary bypass 2 Impedance cardiography (ICG) 3 Intraaortic balloon pump (IABP) 4 Central venous pressure (CVP) measuremen

What collaborative and nursing care of this patient should be done to improve cardiac output and the quality of life? (Select all that apply.) A. Decrease preload and afterload. C. Control heart failure by enhancing myocardial contractility.

The patient is experiencing dilated cardiomyopathy. To improve cardiac output and quality of life, drug, nutrition, and cardiac rehabilitation will be focused on controlling heart failure by decreasing preload and afterload and improving cardiac output, which will improve the quality of life.

After receiving change-of-shift report on a heart failure unit, which patient should the nurse assess first? a. A patient who is cool and clammy, with new-onset confusion and restlessness

The patient who has "wet-cold" clinical manifestations of heart failure is perfusing inadequately and needs rapid assessment and changes in management.

After receiving the following information about four patients during change-of-shift report, which patient should the nurse assess first? b. Patient who has just returned to the unit after balloon valvuloplasty

The patient who has just arrived after balloon valvuloplasty will need assessment for complications such as bleeding and hypotension.

The patient had a history of rheumatic fever and has been diagnosed with mitral valve stenosis. The patient is planning to have a biologic valve replacement. What protective mechanisms should the nurse teach the patient about using after the valve replacement? B. Antibiotic prophylaxis for dental care

The patient will need to use antibiotic prophylaxis for dental care to prevent endocarditis.

An outpatient who has chronic heart failure returns to the clinic after 2 weeks of therapy with metoprolol (Toprol XL). Which assessment finding is most important for the nurse to report to the health care provider? c. Blood pressure (BP) of 88/42 mm Hg

The patient's BP indicates that the dose of metoprolol may need to be decreased because of hypotension.

Two days after an acute myocardial infarction (MI), a patient complains of stabbing chest pain that increases with a deep breath. Which action will the nurse take first? a. Auscultate the heart sounds.

The patient's clinical manifestations and history are consistent with pericarditis, and the first action by the nurse should be to listen for a pericardial friction rub.

A patient with rheumatic fever has subcutaneous nodules, erythema marginatum, and polyarthritis. Based on these findings, which nursing diagnosis would be most appropriate? b. Activity intolerance related to arthralgia

The patient's joint pain will lead to difficulty with activity

After receiving change-of-shift report on a heart failure unit, which patient should the nurse assess first? b. Patient who is taking digoxin and has a potassium level of 3.1 mEq/L

The patient's low potassium level increases the risk for digoxin toxicity and potentially fatal dysrhythmias

During a visit to a 78-year-old with chronic heart failure, the home care nurse finds that the patient has ankle edema, a 2-kg weight gain over the past 2 days, and complains of "feeling too tired to get out of bed." Based on these data, the best nursing diagnosis for the patient is a. activity intolerance related to fatigue.

The patient's statement supports the diagnosis of activity intolerance.

3 Critical care nurses provide care for patients with acute problems who are unstable. Patients with acute problems who are stable are often cared for on a medical-surgical unit versus the critical care unit. Patients with chronic problems who are stable may be cared for in the community setting. Patients with chronic problems who are unstable may require care in a rehabilitation or medical-surgical setting. Text Reference - p. 1598

The seasoned nurse is orienting a novice nurse to the critical care unit. When teaching the novice nurse about critical care nursing, which statement is most appropriate by the seasoned nurse? 1 "We care for patients with acute problems who are stable." 2 "We care for patients with chronic problems who are stable." 3 "We care for patients with acute problems who are unstable." 4 "We care for patients with chronic problems who are unstable."

compensated shock. When the body is able to maintain blood pressure and perfusion in spite of failure of the cardiovascular system, it is called compensated shock. Compensating mechanisms include increased pulse and breathing, and constriction of peripheral blood vessels.

The severity of shock is classified into three categories. When the body is able to maintain blood pressure and perfusion, it is called:

The nurse is preparing to administer warfarin (Coumadin), an oral anticoagulant, to a client with a mechanical valve replacement. The client's International Normalized Ratio (INR) is 2.7. Which action should the nurse implement? 1. Administer the medication as ordered.

The therapeutic range for most clients' INR is 2-3, but for a client with a mechanical valve replacement it is 2-3.5. The medication should be given as ordered and not withheld.

all of the above. All causes of shock can be categorized into three major types. Hypovolemic shock is from blood and fluid loss. Cardiogenic shock is from failure of the heart to pump efficiently. Neurogenic shock is caused by uncontrolled dilation of the blood vessels.

The three major types of shock include: cardiogenic or pump failure. hypovolemice or fluid failure. neurogenic or container failure. all of the above.

A patient with a diagnosis of heart failure has been started on a nitroglycerin patch by his primary care provider. What should this patient be taught to avoid? B Drugs to treat erectile dysfunction

The use of erectile drugs concurrent with nitrates creates a risk of severe hypotension and possibly death.

A patient with a diagnosis of heart failure has been started on a nitroglycerin patch by his primary care provider. What should this patient be taught to avoid? b. Drugs to treat erectile dysfunction

The use of erectile drugs concurrent with nitrates creates a risk of severe hypotension and possibly death.

The client diagnosed with essential hypertension asks the nurse, "Why do I have high blood pressure?" Which response by the nurse would be most appropriate? "There is no specific cause for hypertension, but there are many known risk factors."

There is no known cause for essential hypertension, but many factors, both modi- fiable (obesity, smoking, diet) and nonmodifiable (family history, age, gender) are risk factors for essential hypertension.

The client with coronary artery disease asks the nurse, "Why do I get chest pain?" Which statement would be the most appropriate response by the nurse? 1. "Chest pain is caused by decreased oxygen to the heart muscle."

This is a correct statement presented in layman's terms. When the coronary arteries cannot supply adequate oxygen to the heart muscle, there is chest pain.

The client receiving low molecular weight heparin (LMWH) subcutaneously to prevent DVT following hip replacement surgery complains to the nurse that there are small purple hemorrhaged areas on the upper abdomen. Which action should the nurse implement? 3. Explain that this results from the medication.

This is not hemorrhaging, and the client should be reassured that this is a side effect of the medication.

The nurse is teaching a class on atherosclerosis. Which statement describes the scientific rationale as to why diabetes is a risk factor for developing atherosclerosis? 3. Diabetes speeds the atherosclerotic process by thickening the basement membrane of both large and small vessels.

This is the scientific rationale why diabetes mellitus is a modifiable risk factor for atherosclerosis.

4 Pulmonary vascular resistance (PVR) is calculated using the pulmonary artery mean pressure (PAMR) minus the pulmonary artery wedge pressure (PAWP), multiplying by 80 and dividing by the cardiac output (CO). Systolic and diastolic blood pressures are used to determine mean arterial pressure (MAP). Stroke volume (SV) and right ventricular ejection fraction (RVEF) are used to determine right ventricular end-diastolic volume (RVEDV). Mean arterial pressure (MAP), central venous pressure (CVP), and cardiac output (CO) are used to determine systemic vascular resistance (SVR). Text Reference - p. 1603

To determine a patient's peripheral vascular resistance (PVR) what hemodynamic parameters should the nurse use? 1 Systolic and diastolic blood pressures 2 Stroke volume and right ventricular ejection fraction 3 Mean arterial pressure, central venous pressure, and cardiac output 4 Pulmonary artery mean pressure, pulmonary artery wedge pressure, and cardiac output

3 To obtain an accurate central venous pressure (CVP) reading, the nurse should ensure that the patient is supine and in horizontal position so the zero reference point at the level of the right atrium (also known as the phlebostatic axis) is level with the zero mark on the manometer. If a patient is unable to lie flat, the manometer should be positioned so the zero reference is at the level of the right atrium and the degree of head elevation noted so there is consistency across measurements. Right or left side-lying and supine positions with the head of the bed elevated 45 degrees are all incorrect for obtaining a CVP reading. Text Reference - p. 1605

To obtain an accurate central venous pressure reading with a central venous catheter, a nurse should place the patient in what position? 1 Left side-lying 2 Right side-lying 3 Supine and horizontal 4 Supine with head of bed elevated 45 degrees

The client diagnosed with peripheral vascular disease is overweight, has smoked two (2) packs of cigarettes a day for 20 years, and sits behind a desk all day. What is the strongest factor in the development of atherosclerotic lesions? 4. Smoking cigarettes.

Tobacco use is the strongest factor in the development of atherosclerotic lesions. Nicotine decreases blood flow to the extremities and increases heart rate and blood pressure. It also increases the risk of clot formation by increasing the aggregation of platelets.

Hypovolemic

Total body fluid decreased (in all fluid compartments)

Obstructive

Total body fluid volume not affected

avoid covering the patient as not to overheat. The emergency care steps for shock include: maintain an open airway and assess the respiratory rate. If the patient is breathing adequately, apply high-concentration oxygen by nonrebreather mask. Assist ventilations or perform CPR if necessary; control any external bleeding; apply and inflate the pneumatic anti-shock garment (PASG) if approved or ordered by your local medical direction; if there is no possibility of spine injury, elevate the legs 8 to 12 inches; splint any suspected bone injuries or joint injuries; prevent loss of body heat by covering the patient with a blanket; transport the patient immediately; and if the patient is conscious, speak calmly and reassuringly throughout assessment, care, and transport.

Treatment for hypoperfusion includes all of the following EXCEPT:

o

Treatment involves drug therapy and/or synchronized cardioversion for a stable patient and rapid defibrillation for an unstable patient.

Which admission order written by the health care provider for a patient admitted with infective endocarditis (IE) and a fever would be a priority for the nurse to implement? b. Order blood cultures drawn from two sites.

Treatment of the IE with antibiotics should be started as quickly as possible, but it is essential to obtain blood cultures before initiating antibiotic therapy to obtain accurate sensitivity results.

Which cardiac enzyme would the nurse expect to elevate first in a client diagnosed with a myocardial infarction? 3. Troponin.

Troponin is the enzyme that elevates within 1 to 2 hours.

Which cardiac enzyme would the nurse expect to elevate first in a client diagnosed with a myocardial infarction?

Troponin. - elvated within 1-2 hours - CK-MB within 12-24 - LDH within 24-36

ST-segment

Typical changes seen in ischemia include ST-segment depression and/or T-wave inversion.ST-segment elevation and pathologic Q wave may be seen on the ECG of a patient with MI.ST-segment elevation is common with myocardial injury.

How is PVST treated? (6)

Vagal stimulation IV adenosine IV β-adrenergic blockers Calcium channel blockers Amiodarone DC cardioversion

The client with varicose veins asks the nurse, "What caused me to have these?" Which statement by the nurse would be most appropriate? 1. "You have incompetent valves in your legs."

Varicose veins are irregular, tortuous veins with incompetent valves that do not allow the venous blood to ascend the saphenous vein.

Which client would be most at risk for developing varicose veins? 1. A Caucasian female who is a nurse.

Varicose veins are more common in white females in occupations that involve prolonged standing.

exercise stress test.

Various protocols are used to evaluate the effect of exercise on CV function.

The nurse is teaching a class on venous insufficiency. The nurse would identify which condition as the most serious complication of chronic venous insufficiency? Venous ulcerations.

Venous ulcerations are the most serious complication of chronic venous insufficiency. It is very difficult for these ulcerations to heal, and often clients must be seen in wound care clinics for treatment.

The client is in ventricular fibrillation. Which interventions should the nurse implement? Select all that apply. 1. Start cardiopulmonary resuscitation. 3. Prepare to defibrillate the client. 4. Bring the crash cart to the bedside. 5. Prepare to administer the antidysrhythmic amiodarone IVP.

Ventricular fibrillation indicates the client does not have a heartbeat. Therefore CPR should be instituted. Defibrillation is the treatment of choice for ventricular fibrillation. The crash cart has the defibrillator and is used when performing advanced cardiopulmonary resuscitation. Amiodarone is an antidysrhythmic that is used in ventricular dysrhythmias.

Cardioversion is attempted for a 64-year-old man with atrial flutter and a rapid ventricular response. After the nurse delivers 50 joules by synchronized cardioversion, the patient develops ventricular fibrillation. Which action should the nurse take immediately?

Ventricular fibrillation produces no effective cardiac contractions or cardiac output. If during synchronized cardioversion the patient becomes pulseless or the rhythm deteriorates to ventricular fibrillation, the nurse should turn the synchronizer switch off and initiate defibrillation. Fluids, additional assessment, or treatment of pain alone will not restore an effective heart rhythm.

Colloid Hetastarch (Hespan) - Mechanism of Action

Volume expander Can exert osmotic effect for up to 36hrs Is at least as effective as albumin

The nurse is teaching the client recently diagnosed with essential hypertension. Which instruction should the nurse provide when discussing exercise? Walk at least 30 minutes a day on flat surfaces.

Walking 30 to 45 minutes a day will help in reducing blood pressure, weight, and stress and will increase a feeling of overall wellbeing.

Which instruction should be included when a client diagnosed with peripheral arterial disease is being discharged? 3. Instruct the client to walk daily for at least 30 minutes.

Walking promotes the development of collateral circulation to ischemic tissue and slows the process of atherosclerosis.

Cachexia

Weakness & wasting of the body due to severe chronic illness

2 Arterial pressure-based cardiac output (APCO) monitoring uses the arterial waveform characteristics along with patient demographic data including gender, age, height, and weight to calculate stroke volume (SV). Basal metabolic rate (BMR), body mass index (BMI), blood pressure, and heart rate are not used to calculate stroke volume with this measuring device. Heart rate is used to calculate continuous cardiac output (COO) and continuous cardiac index (CCI). Text Reference - p. 1607

What data are used to calculate stroke volume (SV) for a patient with arterial pressure-based cardiac output (APCO) monitoring? 1 Height, BMR, age, gender 2 Gender, age, height, weight 3 BMI, BMR, blood pressure, heart rate 4 Age, gender, blood pressure, heart rate

Reduced skin integrity

What factor increases an older adult's risk for distributive (septic) shock?

37.5 The formula to calculate the rapid shallow breathing index is f/VT. Therefore, rapid shallow breathing index = 15/0.4 = 37.5 breaths/min/L. Text Reference - p. 1626

What is the rapid shallow breathing index of a patient whose spontaneous respiratory rate (f) is 15 breaths/min and spontaneous tidal volume (VT) is 0.4 L? Record your answer using one decimal place. ___________________breaths/min/L

1 Both negative pressure ventilation and positive pressure ventilation involve passive expiration. Negative pressure ventilation does not require an artificial airway. Positive pressure ventilation may require invasive mechanical ventilation through an artificial airway. Positive pressure ventilation is primarily used with acutely ill patients; negative pressure ventilation is not routinely used for acutely ill patients. In positive pressure ventilation, intrathoracic pressure is raised during lung inflation but in negative pressure ventilation, intrathoracic pressure is decreased during lung inflation. Text Reference - p. 1618

What is the similarity between negative pressure ventilation and positive pressure ventilation? 1 Both involve passive expiration. 2 Both require an artificial airway. 3 Both are routinely used for acutely ill patients. 4 Both raise intrathoracic pressure during lung inflation.

75 Rapid shallow breathing index is equal to respiratory rate/tidal volume; therefore, 30/0.4 = 75. Text Reference - p. 1626

What will be the rapid shallow breathing index for a patient with a tidal volume of 0.4 L and respiratory rate of 30/minute? Record your answer using a whole number. ___ L

Extravasation

When administering norepinephrine (Levophed) what does the nurse monitor for in the patient?

2 While caring for a patient requiring mechanical ventilation, the registered nurses (RNs) must administer sedatives by themselves; they should not delegate this task to unlicensed assistive personnel (UAP). UAP can be tasked with obtaining vital signs, measuring urine output, and performing bedside glucose tests. Text Reference - p. 1625

When caring for a critically ill patient on mechanical ventilation, what task must the registered nurses (RNs) perform by themselves and not delegate to unlicensed assistive personnel (UAP)? 1 Obtaining vital signs 2 Administering sedatives 3 Measuring urine output 4 Performing bedside glucose test

The nurse is discussing the importance of exercise with the client diagnosed with coronary artery disease. Which intervention should the nurse implement? 3. Do not walk if it is less than 40F.

When it is cold outside, vasoconstriction occurs, and this will decrease oxygen to the heart muscle. Therefore, the client should not exercise when it is cold outside.

3 A dynamic response test is performed every 8 to 12 hours, as well as when the system is opened to air or the accuracy of the measurements is questioned. It involves activating the fast flush and checking that the equipment reproduces a distortion-free signal. A square wave indicates a normal response and requires no further action. This waveform does not indicate that the line needs to be flushed. Nor does it indicate that zero needs to be reconfirmed or that the wrist needs to be repositioned. Text Reference - p. 1605

When performing a dynamic response test, the nurse observes the following tracing. What action should the nurse perform based on this tracing?

Which medical treatment would be prescribed for the client with an AAA less than 3 cm? 1. Ultrasound every six (6) months.

When the aneurysm is small (5-6 cm) an abdominal sonogram will be done every six (6) months until the aneurysm reaches a size at which surgery to prevent rupture is of more benefit than possible complications of an abdominal aortic aneurysm repair.

Frank-Starling Mechanism

Whenever there is stretching of the heart or the filling increases, the contraction force will increase

Packed red cells

Which IV therapy results in the greatest increase in oxygen-carrying capacity for a patient with hypovolemic shock?

1 Prior to the family entering the intensive care unit (ICU) to visit a family member who is critically ill the nurse should provide the family with a description of what to expect regarding the patient's appearance and the equipment that is being used to provide care to their family member. The nurse should accompany the family into the patient's room; it is not appropriate for the nurse to ask the physician to do this, nor is it appropriate for the family to enter the room alone. Although it is important to instruct the family on what to expect, the family should be encouraged to touch and speak to the patient. Text Reference - p. 1602

Which action by the nurse is most appropriate when bringing a family member of a critically ill patient into the intensive care unit (ICU) for the first time? 1 Give a description of what to expect 2 Ask the physician to accompany the family 3 Allow the family member to enter the room alone 4 Instruct the family not to touch and speak to the patient

1, 2, 3, 4 Caregivers play a valuable role in the intensive care unit (ICU) patient's recovery because they provide the patient with support. They are also a link to the patient's personal life, help the patient with activities of daily living, and function as the decision maker during the hospitalization. Although the caregiver may make financial decision for the patient during the hospitalization, this is not an important role the caregiver plays in the patient's recovery. Text Reference - p. 1601

Which are the reasons why caregivers play a valuable role in the intensive care unit (ICU) patient's recovery? Select all that apply. 1 They provide loving support for the patient. 2 They provide a link to the patient's personal life. 3 They help the patient with activities of daily living. 4 They function as the patient's decision-maker during the hospitalization. 5 They make the financial decision for the patient during the hospitalization.

3 Systemic vascular resistance is an index of left ventricular afterload. Central venous pressure is an index of preload. Pulmonary arterial pressure and peripheral vascular resistance are indices of right ventricular afterload. Text Reference - p. 1604

Which hemodynamic value should the nurse use to determine a patient's left ventricular afterload? 1 Central venous pressure 2 Pulmonary arterial pressure 3 Systemic vascular resistance 4 Peripheral vascular resistance

-Renin -Antidiuretic hormone (ADH) -Epinephrine -Aldosterone

Which hormones are released in response to decreased mean arterial pressure (MAP)?

3, 4 Suctioning is preceded by a thorough assessment and hyperoxygenation for 30 seconds. Sterile, not clean, gloves are necessary and it is not necessary to administer a bronchodilator. Instillation of normal saline into the ET tube is not an accepted standard practice. Text Reference - p. 1616

Which interventions should the nurse perform before suctioning a patient who has an endotracheal (ET) tube using open-suction technique? Select all that apply. 1 Put on clean gloves. 2 Administer a bronchodilator. 3 Perform a cardiopulmonary assessment. 4 Hyperoxygenate the patient for 30 seconds. 5 Insert a few drops of normal saline into the ET to break up secretions.

1 Patients with preexisting dementia, such as Alzheimer's disease, are at an increased risk for developing delirium when receiving care in the intensive care unit (ICU). Diabetes mellitus, Parkinson's disease, and multiple sclerosis are not known risk factors for developing delirium. Text Reference - p. 1601

Which is the recommendation from the American Association of Critical Care Nursing (AACN) regarding family visitation in the intensive care unit (ICU)? 1 Individualized visitation 2 Visitation on the evening shift 3 Visitation as prescribed by the physician 4 Hourly visitations occurring on each shift

1 In a patient with a head injury, positive pressure ventilation decreases the venous return because of the increase in intrathoracic pressure. Increased intrathoracic pressure causes jugular vein distension rather than compression. Positive pressure ventilation increases the cerebral volume. A decrease in venous return causes an increase in intracranial pressure. Text Reference - p. 1623

Which neurologic complication may occur in a patient with a head injury who is on positive pressure ventilation? 1 Decrease in venous return 2 Compression of jugular vein 3 Reduction of cerebral volume 4 Reduction of intracranial pressure

A patient with a recent diagnosis of heart failure has been prescribed furosemide (Lasix) in an effort to physiologically do what for the patient? a. Reduce preload. b. Decrease afterload. c. Increase contractility. d. Promote vasodilation.

a. Reduce preload. Diuretics such as furosemide are used in the treatment of HF to mobilize edematous fluid, reduce pulmonary venous pressure, and reduce preload. They do not directly influence afterload, contractility, or vessel tone.

Perfusion Perfusion is the supply of oxygen to and removal of wastes from the cells and tissues of the body as a result of the flow of blood through the capillaries.

______________ is the supply of oxygen to and removal of wastes from the cells and tissues of the body as a result of the flow of blood through the capillaries.

• Studies that assist in diagnosis of shock include _

_a serum lactate, base deficit, 12-lead ECG, continuous cardiac monitoring, chest x-ray, continuous pulse oximetry, and hemodynamic monitoring.

• Absolute hypovolemia results when _

_fluid is lost through hemorrhage, gastrointestinal (GI) loss (e.g., vomiting, diarrhea), fistula drainage, diabetes insipidus, or diuresis.

• Relative hypovolemia results when _

_fluid volume moves out of the vascular space into extravascular space, such as with sepsis and burns.

Severely malnourished patient Patient with paralytic ileus Patient with kidney disease Patient with large wounds

Which patients are at risk for shock related to fluid shifts?

4 The electronic or teleICU assists the bedside ICU team by monitoring the patient from a remote location using informatics. The ICU, CCU, and PICU are traditional critical care units. Text Reference - p. 1599

Which type of critical care unit uses informatics to monitor a critically ill patient from a remote location? 1 Intensive care unit (ICU) 2 Coronary care unit (CCU) 3 Pediatric intensive care unit (PICU) 4 Electronic intensive care unit (teleICU)

pelvic fracture. Use of the PASG is contraindicated in eviscerations, chest injuries, and late stage pregnancy due to increased pressure and altered blood supply.

While the use of the PASG is controversial, all of the following are contraindications to using the device EXCEPT:

Shock

Widespread abnormal cellular metabolism that occurs when oxygenated & tissue perfusion needs are not met to the level necessary to maintain cell function. -Whole body response that occurs when too little oxygen is delivered to the tissues. -Any problem that impairs oxygen delivery to tissues & organs can start the syndrome of shock & lead to a life-threatening emergency.

Sepsis

Widespread infection that triggers a whole-body inflammatory response

• SIRS is characterized by _

_generalized inflammation in organs remote from the initial insult and can be triggered by mechanical tissue trauma (e.g., burns, crush injuries), abscess formation, ischemic or necrotic tissue (e.g., pancreatitis, myocardial infarction), microbial invasion, and global and regional perfusion deficits.

Cardiac Catheterization Nursing education

Withhold food & fluids for 6-12 hours before the procedure. Check for sensitivity to contrast media. Explain procedure to pt.

The key factor in describing any type of shock is_

_ inadequate tissue perfusion.

• Collaborative care for patients with MODS focuses on _

_(1) prevention and treatment of infection, (2) maintenance of tissue oxygenation, (3) nutritional and metabolic support, and (4) appropriate support of individual failing organs.

What are some precipitating factors of obstructive shock?

_Abdominal compartment syndrome, pulmonary embolism

What are some precipitating factors of Cardiogenic shock?

_Acute MI, Ventricular dysrhythmias

What are some precipitating factors of hypovolemic shock?

_Burns, hemhorrage, ruptured spleen, severe vomiting and diarrhea, ascites

Problems related to SIRS and MODS include:_

_Cardiovascular changes, neurologic dysfunction, acute renal failure, DIC, GI dysfunction, and liver dysfunction are common.

In SEPTIC shock, bacterial endotoxins cause vascular changes that result in_

_DECREASED systemic vascular resistance with INCREASED cardiac output.

Which shock is characterized by increased systemic vascular resistance (SVR), decreased CO, and decreased pulmonary artery wedge pressure (PAWP)?

_Hypovolemic shock

What are some precipitating factors of Neurogenic shock?

_Spinal cord injury, epidural block, severe pain

What are some precipitating factors of Septic shock?

_UTI, pneumonia, immunosuppression

A patient with chronic HF and atrial fibrillation is treated with a digitalis glycoside and a loop diuretic. To prevent possible complications of this combination of drugs, what does the nurse need to do (select all that apply)? a. Monitor serum potassium levels b. teach the patient how to take a pulse rate. c. keep an accurate measure of intake and output d. Teach the patient about dietary restriction or potassium e. Withhold digitalis and notify health care provider if heart rate is irregular

a & b Rationale: Hypokalemia, which can be caused by the use of potassium-depleting diuretics (e.g., thiazides, loop diuretics), is one of the most common causes of digitalis toxicity. Low serum levels of potassium enhance the action of digitalis, causing a therapeutic dose to achieve toxic levels. Hypokalemia can also precipitate dysrhythmias. Monitoring the serum potassium levels of patients receiving digitalis preparations and potassium-depleting diuretics is essential. Patients taking digitalis preparations should be taught how to measure their pulse rate because bradycardia and atrioventricular blocks are late signs of digitalis toxicity. In addition, patients should know what pulse rate would necessitate a call to the health care provider.

A 70-year-old patient in the ICU has become agitated and inattentive since his heart surgery. The nurse knows that this ICU psychosis frequently occurs in individuals with pre-existing dementia, history of alcohol abuse, and severe disease. What interventions should the nurse provide this patient to improve the patient's cognition *(select all that apply)*? a Improve oxygenation. b Provide a small amount of beer. c Have the family stay with the patient. d Enable the patient to sleep on a schedule with dim lights. e Decrease sensory overload by conversing away from patient's room.

a Improve oxygenation. d Enable the patient to sleep on a schedule with dim lights. e Decrease sensory overload by conversing away from patient's room. ICU psychosis is from delirium in most ICU patients. Improving oxygenation, enabling the patient to sleep, and decreasing sensory overload along with orientation is all helpful in improving the patient's cognition. The beer may or may not be allowed for this patient, and the nurse should not assume that it will help. Having a family member stay with the patient to reorient the patient is helpful, but the family group may increase sensory overload with conversations not involving the patient.

When caring for a patient in acute septic shock, what should the nurse anticipate? a Infusing large amounts of IV fluids b Administering osmotic and/or loop diuretics c Administering IV diphenhydramine (Benadryl) d Assisting with insertion of a ventricular assist device (VAD)

a Infusing large amounts of IV fluids Septic shock is characterized by a decreased circulating blood volume. Volume expansion with the administration of IV fluids is the cornerstone of therapy. The administration of diuretics is inappropriate. VADs are useful for cardiogenic shock not septic shock. Diphenhydramine (Benadryl) may be used for anaphylactic shock but would not be helpful with septic shock.

ectopic focus of accessory pathways outside of normal conduction pathway result in

a dysrhythmia, which replaces the normal sinus rhythm.

inside the cardiac cell concentrations are

a high concentration of potassium and a low concentration of sodium inside the cell.

Progressive tissue hypoxia leading to anaerobic metabolism and metabolic acidosis is characteristic of the progressive stage of shock. Identify what changes occur in the following tissues to cause this increasing tissue hypoxia. a) renal b) Lung c) Capillaries d) Cardiac

a) renal: Renin-angiotensin activation causes arteriolar constriction, decreasing perfusion b) Lung: Vasoconstriction of the pulmonary arterioles decreases the blood flow to pulmonary capillaries, and a ventilation-perfusion mismatch occurs. Areas of the lung that are oxygenated are not perfused by blood because of the decreased blood flow, resulting in additional hypoxemia and decreased oxygen for cells. c) Capillaries: Increased capillary permeability and profound vasoconstriction cause increased hydrostatic pressure with shift of fluid to interstitial spaces and decreased circulating blood volume. d) Cardiac: Decreased myocardial perfusion occurs as the heart fails, leading to dysrhythmias and myocardial ischemia, further decreasing CO and oxygen delivery to cells.

A patient admitted with heart failure appears very anxious and complains of shortness of breath. Which nursing actions would be appropriate to alleviate this patient's anxiety (select all that apply)? a. Administer ordered morphine sulfate. b. Position patient in a semi-Fowler's position. c. Position patient on left side with head of bed flat. d. Instruct patient on the use of relaxation techniques. e. Use a calm, reassuring approach while talking to patient.

a, b, d, & e Morphine sulfate reduces anxiety and may assist in reducing dyspnea. The patient should be positioned in semi-Fowler's position to improve ventilation that will reduce anxiety. Relaxation techniques and a calm reassuring approach will also serve to reduce anxiety.

The patient has heart failure (HF) with an ejection fraction of less than 40%. What core measures should the nurse expect to include in the plan of care for this patient (select all that apply)? a. Left ventricular function is documented. b. Controlling dysrhythmias will eliminate HF. c. Prescription for digoxin (Lanoxin) at discharge d. Prescription for angiotensin-converting enzyme (ACE) inhibitor at discharge e. Education materials about activity, medications, weight monitoring, and what to do if symptoms worsen

a, d, & e The Joint Commission has identified these three core measures for heart failure patients. Although controlling dysrhythmias will improve CO and workload, it will not eliminate HF. Prescribing digoxin for all HF patients is no longer done because there are newer effective drugs and digoxin toxicity occurs easily related to electrolyte levels and the therapeutic range must be maintained.

Match the following precipitating factors with their related types of shock. 1. Neurogenic 2. cardiogenic 3. anaphylactic 4. septic 5. hypovolemic 6. obstructive a. Acute MI b. insect bite c. burns d. severe pain e. Ventricular dysrhythmias f. Abdominal compartment syndrome g. Hemorrhage h. UTI i. Spinal cord injury j. Ruptured spleen k. Pneumonia l. Severe vomiting and diarrhea m. Ascites n. Pulmonary embolism o. Epidural block p. vaccines

a. Acute MI: cardiogenic b. insect bite: anaphylactic c. burns: hypovolemic d. severe pain: Neurogenic e. Ventricular dysrhythmias: cardiogenic f. Abdominal compartment syndrome: obstructive g. Hemorrhage: hypovolemic h. UTI: septic i. Spinal cord injury: neurogenic j. Ruptured spleen: hypovolemic k. Pneumonia: septic l. Severe vomiting and diarrhea: hypovolemic m. Ascites: hypovolemic n. Pulmonary embolism: obstructive o. Epidural block: neurogenic p. vaccines: anaphylactic

Identify 4 drugs and their action that are used in treatment of cardiogenic shock that are not generally used for other types of shock.

a. Diuretics (Lasix) - to decrease the workload of the heart by decreasing fluid volume and reducing preload b. Milrinone (Primacor) - increases cardiac contractility and output and decreases preload and afterload by directly relaxing vascular smooth muscles. c. Nitroglycerin (Nitrol, Tridil) - primarily dilates veins, reducing preload d. Nitroprusside (Nipride) - acts as a potent vasodilator of veins and arteries and may increase or decrease CO, depending on the extent of preload and afterload reduction

The patient with chronic heart failure is being discharged from the hospital. What information should the nurse emphasize in the patient's discharge teaching to prevent progression of the disease to ADHF? a. Take medications as prescribed. b. Use oxygen when feeling short of breath. c. Only ask the physician's office questions. d. Encourage most activity in the morning when rested.

a. Take medications as prescribed. The goal for the patient with chronic HF is to avoid exacerbations and hospitalization. Taking the medications as prescribed along with nondrug therapies such as alternating activity with rest will help the patient meet this goal. If the patient needs to use oxygen at home, it will probably be used all the time or with activity to prevent respiratory acidosis. Many HF patients are monitored by a care manager or in a transitional program to assess the patient for medication effectiveness and monitor for patient deterioration and encourage the patient. This nurse manager can be asked questions or can contact the health care provider if there is evidence of worsening HF.

Identify 2 medical therapies that are specific to each of the following types of shock. a. cardiogenic b. hypovolemic c. septic d. anaphylactic

a. cardiogenic - restore coronary blood flow with thrombolytic therapy, angioplasty, emergency revascularization - increase CO with inotropic agents - reduce workload by dilating coronary arteries, decreasing preload and afterload - use of circulatory assist devices such as an intraaortic balloon pump b. hypovolemic - fluid and blood replacement, control bleeding with pressure or surgery c. septic - fluid resuscitation, antimicrobial agents, inotropic agents with vasopressors d. anaphylactic - Epinephrine, inhaled bronchodilators, colloid fluid replacement, diphenhydramine, corticosteroids

The nurse recognizes that primary manifestations of systolic failure include: a. decreased EF and increased PAWP b. decreased PAWP and increased EF. c. decreased pulmonary hypertension associated with normal EF d. decreased afterload and decreased left ventricular end-diastolic pressure

a. decreased EF and increased PAWP Rationale: Systolic heart failure results in systolic failure in the left ventricle (LV). The LV loses its ability to generate enough pressure to eject blood forward through the aorta. This results in increased pulmonary artery wedge pressure (PAWP). The hallmark of systolic failure is a decrease in the left ventricular ejection fraction (EF).

The Holter monitor continuously records the ECG while the patient is...

ambulatory and performing daily activities. The patient keeps a diary and records activities and any symptoms. Events in the diary are correlated with any dysrhythmias on ECG.

release of vasoactive mediators-vasodilation and increased capillary permeability

anaphylactic shock

The nurse would recognize which clinical manifestation as suggestive of sepsis? a Sudden diuresis unrelated to drug therapy b Hyperglycemia in the absence of diabetes c Respiratory rate of seven breaths per minute d Bradycardia with sudden increase in blood pressure

b Hyperglycemia in the absence of diabetes Hyperglycemia in patients with no history of diabetes is a diagnostic criterion for sepsis. Oliguria, not diuresis, typically accompanies sepsis along with tachypnea and tachycardia.

A 50-year-old woman with a suspected brain tumor is scheduled for a computed tomography (CT) scan with contrast media. The nurse notifies the physician that the patient reported an allergy to shellfish. Which response by the physician should the nurse question? a Infuse IV diphenhydramine prior to the procedure. b Administer lorazepam (Ativan) before the procedure. c Complete the CT scan without the use of contrast media. d Premedicate with hydrocortisone sodium succinate (Solu-Cortef).

b Administer lorazepam (Ativan) before the procedure. An individual with an allergy to shellfish is at an increased risk to develop anaphylactic shock if contrast media is injected for a CT scan. To prevent anaphylactic shock, the nurse should always confirm the patient's allergies before diagnostic procedures (e.g., CT scan with contrast media). Appropriate interventions may include cancelling the procedure, completing the procedure without contrast media, or premedication with diphenhydramine or hydrocortisone. IV fluids may be given to promote renal clearance of the contrast media and prevent renal toxicity and acute kidney injury. The use of an antianxiety agent such as lorazepam would not be effective in preventing an allergic reaction to the contrast media.

When caring for a critically ill patient who is being mechanically ventilated, the nurse will astutely monitor for which clinical manifestation of multiple organ dysfunction syndrome (MODS)? a Increased serum albumin b Decreased respiratory compliance c Increased gastrointestinal (GI) motility d Decreased blood urea nitrogen (BUN)/creatinine ratio

b Decreased respiratory compliance Clinical manifestations of MODS include symptoms of respiratory distress, signs and symptoms of decreased renal perfusion, decreased serum albumin and prealbumin, decreased GI motility, acute neurologic changes, myocardial dysfunction, disseminated intravascular coagulation (DIC), and changes in glucose metabolism.

A 64-year-old woman is admitted to the emergency department vomiting bright red blood. The patient's vital signs are blood pressure 78/58 mm Hg, pulse 124 beats/minute, respirations 28 breaths/minute, and temperature 97.2° F (36.2° C). Which physician order should the nurse complete first? a Obtain a 12-lead ECG and arterial blood gases. b Rapidly administer 1000 mL normal saline solution IV. c Administer norepinephrine (Levophed) by continuous IV infusion. d Carefully insert a nasogastric tube and an indwelling bladder catheter.

b Rapidly administer 1000 mL normal saline solution IV. Isotonic crystalloids, such as normal saline solution, should be used in the initial resuscitation of hypovolemic shock. Vasopressor drugs (e.g., norepinephrine) may be considered if the patient does not respond to fluid resuscitation and blood products. Other orders (e.g., insertion of nasogastric tube and indwelling bladder catheter and obtaining the diagnostic studies) can be initiated after fluid resuscitation is initiated.

A 64-year-old male patient admitted to the critical care unit for gastrointestinal hemorrhage complains of feeling tense and nervous. He appears restless with an increase in blood pressure and pulse. If the physical assessment shows no other changes, it is most important for the critical care nurse to take which action? a Administer prescribed IV dose of lorazepam (Ativan). b Stay with the patient and encourage expression of concerns. c Ask a family member to remain at the bedside with the patient. d Teach the patient how to use guided imagery to reduce anxiety.

b Stay with the patient and encourage expression of concerns. Anxiety is a common problem for critically ill patients. The nurse should first stay with the patient and encourage the patient to express concerns and needs. After expression of feelings, the nurse should determine the appropriate intervention if needed (e.g., lorazepam, guided imagery, family presence) and closely monitor the patient's hemodynamic parameters.

Patients with a heart transplantation are at risk for which complications in the first year after transplantation (select all that apply)? a. cancer b. infection c. rejection d. vasculopathy e. sudden cardiac death

b, c, & e Rationale: A variety of complications can occur after heart transplantation. In the first year after transplantation, the major causes of death are acute rejection and infection. Heart transplant recipients also are at risk for sudden cardiac death. Later, malignancy (especially lymphoma) and cardiac vasculopathy (accelerated CAD) are major causes of death.

The nurse determines that a large amount of crystalloid fluids administered to a patient in septic shock is effective when hemodynamic monitoring reveals a. CO of 2.6L/min b. CVP of 15 mm Hg c. PAWP of 4 mm Hg d. HR of 106 beats/min

b. CVP of 15 mm Hg The endpoint of fluid resuscitation in septic and hypovolemic shock is a CVP of 15 mm Hg or a PAWP of 10-12 mm Hg. The CO is too low, and the heart rate is too high, to indicated adequate fluid replacement.

A patient with a diagnosis of heart failure has been started on a nitroglycerin patch by his primary care provider. What should this patient be taught to avoid? a. High-potassium foods b. Drugs to treat erectile dysfunction c. Nonsteroidal antiinflammatory drugs d. Over-the-counter H2 -receptor blockers

b. Drugs to treat erectile dysfunction The use of erectile drugs concurrent with nitrates creates a risk of severe hypotension and possibly death. High-potassium foods, NSAIDs, and H2-receptor blockers do not pose a risk in combination with nitrates.

What should the nurse recognize as an indication for the use of dopamine (Intropin) in the care of a patient with heart failure? a. Acute anxiety b. Hypotension and tachycardia c. Peripheral edema and weight gain d. Paroxysmal nocturnal dyspnea (PND)

b. Hypotension and tachycardia Dopamine is a β-adrenergic agonist whose inotropic action is used for treatment of severe heart failure accompanied by hemodynamic instability. Such a state may be indicated by tachycardia accompanied by hypotension. PND, anxiety, edema, and weight gain are common signs and symptoms of heart failure, but these do not necessarily warrant the use of dopamine.

An abnormal finding that the nurse would expect to find in early, compensated shock is a. metabolic acidosis b. increased serum sodium c. decreased blood glucose d. increased serum potassium

b. increased serum sodium In early compensated shock, activation of the renin angiotensin system stimulates the release of aldosterone, which causes sodium reabsorption and potassium excretion by the kidney, elevating serum sodium levels and decreasing serum potassium levels. Blood glucose levels are elevated during the compensated stage of shock in response to catecholamine stimulation of the liver, which releases glycogen stores in the form of glucose. Metabolic acidosis does not occur until the progressive stage of shock, when compensatory mechanisms become ineffective and anaerobic cellular metabolism causes lactic acid production.

When caring for a patient in cardiogenic shock, the nurse recognizes that the metabolic demands of turning and moving the patient exceed the oxygen supply when hemodynamic monitoring reveals a change in a. SvO2 from 62% to 54% b. SV from 52 to 68 mL/beat c.CO from 4.2 L/min to 4.8 L/min d. SVR from 1300 dyne/sec/cm to 1120 dyne/sec/cm

b. SvO2 from 62% to 54%, A decreased mixed venous oxygen saturation (SvO2) indicates that the patient has used the venous oxygen reserve and is at greater risk for anaerobic metabolism. The SvO2 decreases when more O2 is used by the cells, as in activity or hypermetabolism. All the other values indicate an improvement in the patient's condition.

Which hematologic problem most significantly increases the risks associated with pulmonary artery (PA) catheter insertion? a Leukocytosis b Hypovolemia c Hemolytic anemia d Thrombocytopenia

d Thrombocytopenia PA catheter insertion carries a significant risk of bleeding, a fact that is exacerbated when the patient has low levels of platelets. Leukocytosis, hypovolemia, and anemia are less likely to directly increase the risks associated with PA insertion.

The nurse is preparing to administer digoxin to a patient with heart failure. In preparation, laboratory results are reviewed with the following findings: sodium 139 mEq/L, potassium 5.6 mEq/L, chloride 103 mEq/L, and glucose 106 mg/dL. What should the nurse do next? a. Withhold the daily dose until the following day. b. Withhold the dose and report the potassium level. c. Give the digoxin with a salty snack, such as crackers. d. Give the digoxin with extra fluids to dilute the sodium level.

b. Withhold the dose and report the potassium level. The normal potassium level is 3.5 to 5.0 mEq/L. The patient is hyperkalemic, which makes the patient more prone to digoxin toxicity. For this reason, the nurse should withhold the dose and report the potassium level. The physician may order the digoxin to be given once the potassium level has been treated and decreases to within normal range.

A patient with a gunshot wound to the abdomen is being treated for hypovolemic and septic shock. To monitor the patient for early organ damage associated with MODS, what is most important for the nurse to assess? a. urine output b. breath sounds c. peripheral circulation d. central venous pressure

b. breath sounds Generally, the first organ system affected by mediator-induced injury in MODS is the respiratory system. Adventitious sounds and areas absent of breath sounds will be present. Other organ damage also occurs but the lungs usually go first.

A patient with severe trauma has been treated for hypovolemic shock. The nurse recognizes that the patient is in the irreversible stage of shock when assessment findings include. a. a lactic acidosis with a pH of 3.32 b. marked hypotension and refractory hypoxemia c. unresponsiveness that responds only to painful stimuli d. profound vasoconstriction with absent peripheral pulses.

b. marked hypotension and refractory hypoxemia During both the compensated and progressive stages of shock, the sympathetic nervous system is activated in an attempt to maintain CO and SVR. In the irreversible stage of shock, and sympathetic nervous system can no longer compensate to maintain homeostasis, and a loss of vasomotor tone leading to profound hypotension affects perfusion to all vital organs, causing increased cellular hypoxia, metabolic acidosis, and cellular death.

You are caring for a patient with ADHF who is receiving IV dobutamine (Dobutrex). You know that this drug is ordered because it (select all that apply): a. incerases SVR b. produces diuresis c. improves contractility d. dilates renal blood vessels e. works on the B1-receptors in the heart.

c & e Rationale: Dobutamine (Dobutrex) has a positive chronotropic effect and increases heart rate and improves contractility. It is a selective β-adrenergic agonist and works primarily on the β1-adrenergic receptors in the heart. It is frequently used in the short-term management of acute decompensated heart failure (ADHF).

A patient's localized infection has progressed to the point where septic shock is now suspected. What medication is an appropriate treatment modality for this patient? a Insulin infusion b IV administration of epinephrine c Aggressive IV crystalloid fluid resuscitation d Administration of nitrates and β-adrenergic blockers

c Aggressive IV crystalloid fluid resuscitation Patients in septic shock require large amounts of crystalloid fluid replacement. Nitrates and β-adrenergic blockers are most often used in the treatment of patients in cardiogenic shock. Epinephrine is indicated in anaphylactic shock, and insulin infusion is not normally necessary in the treatment of septic shock (but can be).

The critical care nurse is caring for a 55-year-old man who has a catheter in the right radial artery that is being used for continuous arterial blood pressure monitoring following his abdominal aortic aneurysm surgery. Which observation by the nurse would require an emergency intervention? a Calculated mean arterial pressure is 74 mm Hg. b Patient's head of bed elevation is at 30 degrees. c Capillary refill time in the right hand is 5 seconds. d Pressure bag attached to the arterial line is inflated to 270 mm Hg.

c Capillary refill time in the right hand is 5 seconds. Neurovascular status distal to the arterial insertion site is monitored hourly. If arterial flow is compromised, the limb will be cool and pale, with capillary refill time longer than 3 seconds. Symptoms of neurologic impairment include paresthesia, pain, or paralysis. Neurovascular impairment can result in loss of a limb and is an emergency. The pressure bag should be inflated to 300 mm Hg. Normal range for mean arterial pressure is 70 to 105 mm Hg. The backrest elevation may be up to 45 degrees unless the patient has orthostatic changes.

The patient has developed cardiogenic shock after a left anterior descending myocardial infection. Which circulatory-assist device should the nurse expect to use for this patient? a Cardiopulmonary bypass b Impedance cardiography (ICG) c Intraaortic balloon pump (IABP) d Central venous pressure (CVP) measurement

c Intraaortic balloon pump (IABP) The most commonly used mechanical circulatory-assist device is the intraaortic balloon pump (IABP), and it is used to decrease ventricular workload, increase myocardial perfusion, and augment circulation. Cardiopulmonary bypass provides circulation during open heart surgery. It is not used as an assist device after surgery. ICG is a noninvasive method to obtain cardiac output and assess thoracic fluid status. CVP measurement is an invasive measurement of right ventricular preload and reflects fluid volume problems.

The nurse is caring for a 29-year-old man who was admitted a week ago with multiple rib fractures, a pulmonary contusion, and a left femur fracture from a motor vehicle crash. After the attending physician tells the family that the patient has developed sepsis, the family members have many questions. Which information should the nurse include in explaining the early stage of sepsis? a Antibiotics are not useful once an infection has progressed to sepsis. bWeaning the patient away from the ventilator is the top priority in sepsis. c Large amounts of IV fluid are required in sepsis to fill dilated blood vessels. d The patient has recovered from sepsis if he has warm skin and ruddy cheeks.

c Large amounts of IV fluid are required in sepsis to fill dilated blood vessels. Patients with sepsis may be normovolemic but because of acute vasodilation, relative hypovolemia and hypotension occur. Patients in septic shock require large amounts of fluid replacement and may require frequent fluid boluses to maintain circulation. Antibiotics are an important component of therapy for patients with septic shock. They should be started after cultures (e.g., blood, urine) are obtained and within the first hour of septic shock. Oxygenating the tissues is the top priority in sepsis, so efforts to wean septic patients from mechanical ventilation halt until sepsis is resolving. Addititonal respiratory support may be needed during sepsis. Although cool and clammy skin is present in other early shock states, the patient in early septic shock may feel warm and flushed because of a hyperdynamic state.

The nurse is caring for a 72-year-old man in cardiogenic shock after an acute myocardial infarction. Which clinical manifestations would be of most concern to the nurse? a Restlessness, heart rate of 124 beats/minute, and hypoactive bowel sounds b Mean arterial pressure of 54 mm Hg, increased jaundice, and cold, clammy skin c PaO2 of 38 mm Hg, serum lactate level of 46.5 mcg/dL, and bleeding from puncture sites d Agitation, respiratory rate of 32 breaths/minute, and serum creatinine level of 2.6 mg/dL

c PaO2 of 38 mm Hg, serum lactate level of 46.5 mcg/dL, and bleeding from puncture sites Severe hypoxemia, lactic acidosis, and bleeding are clinical manifestations of the irreversible state of shock. Recovery from this stage is not likely because of multiple organ system failure. Restlessness, tachycardia, and hypoactive bowel sounds are clinical manifestations that occur during the compensatory stage of shock. Decreased mean arterial pressure, jaundice, cold/ clammy skin, agitation, tachypnea, and increased serum creatinine are clinical manifestations of the progressive stage of shock.

The nurse suspects sepsis as a cause when the laboratory test results of a patient in shock include. a) hypokalemia b) Increased BUN c) thrombocytopenia d) decreased hemoglobin

c) thrombocytopenia When sepsis is the cause of shock, the endotoxins stimulate a cascade of inflammatory responses starts with the release of TNF and IL-1, which stimulate other inflammatory mediators that increase neutrophil and platelet aggregation and adhesion to the endothelium. There is an increase in coagulation, inflammation, fibrinolysis, and platelet-activating factor causes formation of microthrombi wnr vessel obstruction. The process does not happen to other types of shock until the late stages.

What is the priority nursing responsibility in the prevention of shock? a. frequently monitoring all patients' vital signs b. using aseptic technique for all invasive procedures c. being aware of the potential for shock in all patients at risk d. teaching patients health promotion activities to prevent shock

c. being aware of the potential for shock in all patients at risk Prevention of shock necessitates identification of persons through baseline nursing assessments with frequent ongoing assessments to monitor and detect changes in patients at risk. Frequently monitoring all patients' vital signs is not necessary. Aseptic technique for all invasive procedures should always be implemented but will not prevent all types of shock. Health-promotion activities that reduce the risk for precipitating conditions such as CAD or anaphylaxis may help prevent shock in some select cases.

When administering any vasoactive drug during the treatment of shock, the nurse should know that what is the goal of the therapy? a. increasing urine output to 50 mL/hr b. constriction of vessels to maintain BP c. maintaining a MAP of at least 60 mmHg d. dilating vessels to improve tissue perfusion

c. maintain a MAP of a least 60 mm Hg Vasoactive drugs are those that can either dilate constrict blood vessels, and both are used in various staged of shock treatment. When using either vasodilators or vasoconstrictors, it is important to maintain a MAP of at least 60 mm Hg so that adequate perfusion is maintained.

A patient in the progressive stage of shock had rapid, deep respirations. The nurse determines that the patient's hyperventilation is compensating for metabolic acidosis when the patient;s arterial blood gas (ABG) results include: a. pH 7.42, PaO2 80 mmHg b, pH 7.44, PaO2 69 mmHg c. pH 7.38, PaCO2 30 mmHg d. pH 7.32, PaCO2 48 mmHg

c. pH 7.38, PaCO2 30 mmHg If metabolic acidosis is compensated, the pH will be within the normal range, and if the patient is hyperventilating, to blow off CO2 to reduce the acid load of the blood, PaCO2 will be decreased.

To establish hemodynamic monitoring for a patient, the nurse zeroes the: a. cardiac output monitoring system to the level of the left ventricle b. pressure monitoring system to the level of the catheter tip located in the patient c. pressure monitoring system to the level of the atrium, identified as the phlebostatic axis d. pressure monitoring system to the level of the atrium, identified as the midclavicular line

c. pressure monitoring system to the level of the atrium, identified as the phlebostatic axis

leading cause of death from acute MI

cardiogenic shock

Stimulation of the vagus nerve does 2 things:

causes a decreased rate of firing of the SA node and slowed impulse conduction of the AV node.

Two types of systems are used for telemetry monitoring.

centralized and non centralized. systems have the capability of detecting and storing data.

When a cell or groups of cells are stimulated, the cell membrane Fig. 36-1 describes the phases of the cardiac action potential.

changes its permeability.

If artifact is presents...

check the connections in the equipment. You may need to replace the electrodes if the conductive gel has dried out.

The nurse is assisting in the care of several patients in the critical care unit. Which patient is at greatest risk for developing multiple organ dysfunction syndrome (MODS)? a 22-year-old patient with systemic lupus erythematosus who is admitted with a pelvic fracture after a motor vehicle accident b 48-year-old patient with lung cancer who is admitted for syndrome of inappropriate antidiuretic hormone and hyponatremia c 65-year-old patient with coronary artery disease, dyslipidemia, and primary hypertension who is admitted for unstable angina d 82-year-old patient with type 2 diabetes mellitus and chronic kidney disease who is admitted for peritonitis related to a peritoneal dialysis catheter infection

d 82-year-old patient with type 2 diabetes mellitus and chronic kidney disease who is admitted for peritonitis related to a peritoneal dialysis catheter infection A patient with peritonitis is at high risk for developing sepsis. In addition, a patient with diabetes is at high risk for infections and impaired healing. Sepsis and septic shock are the most common causes of MODS. Individuals at greatest risk for developing MODS are older adults and persons with significant tissue injury or preexisting disease. MODS can be initiated by any severe injury or disease process that activates a massive systemic inflammatory response.

A 68-year-old male patient diagnosed with sepsis is orally intubated on mechanical ventilation. Which action is most important for the nurse to take? a Use the open-suctioning technique. b Administer morphine for discomfort. c Limit noise and cluster care activities. d Elevate the head of the bed 30 degrees.

d Elevate the head of the bed 30 degrees. The two major complications of endotracheal intubation are unplanned extubation and aspiration. To prevent aspiration all intubated patients and patients receiving enteral feedings must have the head of the bed (HOB) elevated a minimum of 30 to 45 degrees unless medically contraindicated. Closed-suction technique is preferred over the open-suction technique because oxygenation and ventilation are maintained during suctioning, and exposure to secretions is reduced. The nurse should provide comfort measures such as morphine to relieve anxiety and pain associated with intubation. To promote rest and sleep the nurse should limit noise and cluster activities.

What laboratory finding fits with a medical diagnosis of cardiogenic shock? a Decreased liver enzymes b Increased white blood cells c Decreased red blood cells, hemoglobin, and hematocrit d Increased blood urea nitrogen (BUN) and serum creatinine levels

d Increased blood urea nitrogen (BUN) and serum creatinine levels The renal hypoperfusion that accompanies cardiogenic shock results in increased BUN and creatinine levels. Impaired perfusion of the liver results in increased liver enzymes, while white blood cell levels do not typically increase in cardiogenic shock. Red blood cell indices are typically normal because of relative hypovolemia.

A patient with acute pancreatitis is experiencing hypovolemic shock. Which of the following initial orders for the patient will the nurse implement first? a. Start 1000 mL of normal saline at 500 mL/hr b. Obtain blood cultures before starting IV antibiotics. c. Draw blood for hematology and coagulation factors. d. Administer high flow oxygen (100%) with a nonrebreather bag

d. Administer high flow O2 (100%) with a nonrebreather bag. In every type of shock there is a deficiency of oxygen to the cells, and high-flow oxygen therapy is indicated. Fluids could be started next, blood cultures done before any antibiotic therapy, and lab specimens then can be drawn.

The nurse is administering a dose of digoxin (Lanoxin) to a patient with heart failure (HF). The nurse would become concerned with the possibility of digitalis toxicity if the patient reported which symptom(s)? a. Muscle aches b. Constipation c. Pounding headache d. Anorexia and nausea

d. Anorexia and nausea Anorexia, nausea, vomiting, blurred or yellow vision, and cardiac dysrhythmias are all signs of digitalis toxicity. The nurse would become concerned and notify the health care provider if the patient exhibited any of these symptoms.

A male patient with a long-standing history of heart failure has recently qualified for hospice care. What measure should the nurse now prioritize when providing care for this patient? a. Taper the patient off his current medications. b. Continue education for the patient and his family. c. Pursue experimental therapies or surgical options. d. Choose interventions to promote comfort and prevent suffering.

d. Choose interventions to promote comfort and prevent suffering. The central focus of hospice care is the promotion of comfort and the prevention of suffering. Patient education should continue, but providing comfort is paramount. Medications should be continued unless they are not tolerated. Experimental therapies and surgeries are not commonly used in the care of hospice patients.

classic sign of shock

drop in BP

The most accurate assessment parameters for the nurse to use to determine adequate tissue perfusion in the patient with MODS are: a. BP, P, RR b. breath sounds, BP, body temp. c. Pulse pressure, LOC, and pupillary response d. LOC, urine output, and skin color/temp

d. LOC, urine output, and skin color/temp

During administration of IV norepinephrine, what should the nurse assess the patient for? a. hypotension b. marked diuresis c. metabolic alkalosis d. decreased tissue perfusion

d. decreased tissue perfusion As a vasopressor, norepinephrine may cause severe vasoconstriction, which would further decrease tissue perfusion, especially if fluid replacement in inadequate. Vasopressors generally cause hypertension, reflex bradycardia, and decreased urine output because of decreased renal blood flow; they do not directly affect acid-base balance.

Which patient manifestations confirm the development of MODS? a. urine output of 30 mL/hr, BUN of 45 mg/dL, and WBC count of 1120/uL b. upper GI bleeding, GCS score, Hct of 25% c. respiratory rate of 45, PaCo2 of 60 mmHg, and chest x-ray with bilateral patchy infiltrates d. elevated serum amylase and lipase, serum creatinine 3.8 mg/dL, platelet count 15,000/mcL

d. elevated serum amylase and lipase, serum creatinine 3.8 mg/dL, platelet count 15,000/mcL Pancreatic, renal, hematologic failure.

A patient in shock has a nursing diagnosis of fear related to severity of condition and perceived threat of death as manifested by verbalization of anxiety about condition and fear of death. What is an appropriate nursing intervention for the patient? a, administer antianxiety agents b, allow caregivers to visit as much as possible c. call a member of the clergy to visit as much as possible d. inform the patient of the current plan of care and its rationale

d. inform the patient of the current plan of care and its rationale Although some patients in shock may be treated with antianxiety and sedative drugs to control anxiety and apprehension, the nurse should always acknowledge the patient's feelings, explain procedures before they are carried out, and inform the patient of the plan of care and its rationale. Members of clergy should be called only if the patient requests or agrees to a visit, and whereas visits by family my have a therapeutic effect, family visits may increase stress in others.

The waveforms on the ECG represent electrical activity produced by the movement of ions across the membranes of myocardial cells, representing

depolarization and repolarization.

QRS complex

depolarization of the ventricles (ventricular contraction), and the QRS interval represents the time it takes for depolarization.

•Pulseless electrical activity (PEA)

describes a situation in which electrical activity is observed on the ECG, but there is no mechanical activity of the ventricles and the patient has no pulse. PEA requires immediate treatment consisting of CPR with initiation of ACLS measures.

If the patient becomes symptomatic (e.g., chest pain)... 2 steps

determination of the rhythm by cardiac monitoring is a high priority. Activate the emergency medical services (EMS) system.

Dysrhythmias result from

disorders of impulse formation, conduction of impulses, or both.

use of telemetry can help rapidly diagnose

dysrhythmias, ischemia, or infarction. .

The property of myocardial tissue that allows it to be depolarized by a stimulus is called

excitability (conducts from one cell to another).

•Sinus bradycardia

has a normal sinus rhythm, but the SA node fires at a rate less than 60 beats/min. It may be a normal clinical condition. Treatment is only indicated in those with symptoms.

two things you must protect patient against when giving large amounts of fluids

hypothermia and coagulopathies

Patient-monitoring guidelines for patients with suspected ACS

include continuous, multilead ECG, and ST-segment monitoring.

Noncardiovascular causes of syncope

include hypoglycemia, hysteria, seizure, stroke, and transient ischemic attack.

example of relative hypovolemia during burn when fluid leaks into extravascular space because of

increased capillary permeability

using what for fluid resuscitation can cause lactate levels to rise cuz liver cannot convert lactate to bicarbonate

lactated ringer's

Changes in a 12-lead ECG

may occur with damage such as ischemia or infarction, electrolyte imbalance, dysrhythmias, or drug toxicity.

abdominal compartment syndrome causes

obstructive shock

PR interval

represents the time period for the impulse to spread through the atria, AV node, bundle of His, and Purkinje fibers.

•Asystole

represents the total absence of ventricular electrical activity with the patient being unresponsive, pulseless, and apneic. Asystole requires immediate treatment consisting of CPR with initiation of ACLS measures.

Dysrhythmias

result from disorders of impulse formation, conduction of impulses or both.

The autonomic nervous system plays an important role in

the rate of impulse formation, the speed of conduction, and the strength of cardiac contraction.

The components of the autonomic nervous system that affect the heart are

the vagus nerve fibers of the parasympathetic nervous system and nerve fibers of the sympathetic nervous system.

Saturated Fat

use sparingly. Butter, animal fat, cream cheese

Synchronized cardioversion

using a defibrillator is the therapy of choice for the patient with hemodynamically unstable ventricular or supraventricular tachydysrhythmias.

Dysrhythmias occur as the result of

various abnormalities and disease states. The cause of a dysrhythmia influences the patient's treatment.

if a patient has persistent hypotension after fluid resuscitation what 2 things could be added together or one or other

vasopressor (norepi, dopamine) or inotope (dobutamine)

Prolonged PSVT

with HR greater than 180 beats/min may decrease cardiac output (CO), resulting in hypotension, dyspnea, and angina.

Treatment Measures for Chronic Heart Failure

~ Correction of reversible causes ~ Surgical repair of a ventricular defect or an improperly functioning valve ~ Pharmacologic & non-pharmacologic control of afterload stresses ~ Modification of activities & lifestyle consistent with the functional limitations of a reduced cardiac reserve ~ Use of medications to improve cardiac function & limit excessive compensatory mechanisms

Causes of Cardiogenic Shock

~ Damage to the heart from myocardial infarction ~ Ineffective pumping caused by cardiac arrhythmias ~ Ventricular septal defect ~ Ventricular aneurysm ~ Acute disruption of valve function ~ Problems associated with open heart surgery

Signs & Symptoms of Right-Sided Heart Failure

~ Edema in lower limbs ~ Weight gain ~ Liver congestion/hepatomegaly ~ Pain in the upper right quadrant ~ Digestion altered ~ External jugular veins distended - can be visibly seen as larger

Treatment of Cardiogenic Shock

~ Improving cardiac output ~ Reducing the workload & oxygen needs of the myocardium ~ Increasing coronary perfusion ~ Affecting peripheral vascular resistance (has to be lowered) ~ Monitor fluid volume - has to decrease ~ Vasodilators, fibrinolytic therapy, CABG (coronary artery bypass grafting)

Systolic Dysfunction

~ Myocardial contractility decrease, results in decrease in ejection fraction to less than 40% ~ EDV increase, ventricles dilate ~ Accumulation of blood in atria & the venous system ~ Causing pulmonary, peripheral edema ~ Systolic dysfunction results form impaired contractile heart performance (ischemic heart disease, cardiomyopathy), valvular insufficiency, hypertension, valve stenosis ~ Commonly heart failure associated with impaired systolic function

Mechanical Aids & Surgical Therapy

~ Resynchronization - proper conduction ~ Implantable cardioverter defibrillators ventricular re-modeling ~ VAD (ventricular assist device) ~ Heart transplantation ~ Oxygen therapy ~ Pharmacologic Treatment - Diuretics ~ ACE Inhibitors (Angiotensin Converting Enzyme) ~ B-adrenergic Receptor Blocking Drugs ~ Vasodilators

Dysrhythmias can be caused by

• Acid-base imbalances • Alcohol • Caffeine, tobacco • Connective tissue disorders • Drug effects (e.g., antidysrhythmia drugs, stimulants, β-adrenergic blockers) or toxicity * Electrolyte disturbances can cause dysrhythmias and, if not treated, can lead to life-threatening dysrhythmias Electrolyte imbalances (e.g., hyperkalemia, hypocalcemia) • Electric shock • Emotional crisis • Herbal supplements (e.g., areca nut, wahoo root bark, yerba maté) • Hypoxia • Metabolic conditions (e.g., thyroid dysfunction) • Near-drowning • Sepsis, shock • Toxins Cardiac Conditions= • Accessory pathways • Cardiomyopathy • Conduction defects • Heart failure • Myocardial ischemia, infarction • Valve disease * Tachycardias may be the result of fever and may cause a decrease in cardiac output (CO) and hypotension.

Dysrhythmias assessment findings...

• Irregular rate and rhythm; tachycardia, bradycardia • Decreased/increased blood pressure • Decreased O2 saturation • Chest, neck, shoulder, back, jaw, or arm pain • Dizziness, syncope • Dyspnea • Extreme restlessness, anxiety • Decreased level of consciousness, confusion • Feeling of impending doom • Numbness, tingling of arms • Weakness and fatigue • Cold, clammy skin • Diminished peripheral pulses • Diaphoresis • Pallor • Palpitations • Nausea and vomiting


Set pelajaran terkait

CIS 193 - 70-412 - Active Directory - Infrastructure Testout Practice Questions

View Set

Mental Health Ch. 12_Schizophrenia and Spectrum disorders

View Set

mother baby unit 1 eaq questions

View Set

Chapter 7 Lifespan Development Quiz and Test

View Set

(1) AZ-500 Manage identity and access in Azure Active Directory

View Set

Ch 15: Nursing Care of a Family During Labor and Birth

View Set

International Finance Chapter 16

View Set

CAASPP Testing 8th Grade Math Practice Scoring Guide Answers

View Set